You are on page 1of 424

UNAS NOVEMBER 2020

1. A 21-year-old G1 P0 patient presents to your office with vaginal bleeding at approximately 8


weeks' gestation by her last menstrual period. Her examination is begin with a 9-week-sized
uterus, a closed cervical os, and a small amount of blood within the vaginal vault. You order a
complete pelvic ultrasound that shows an intrauterine gestational sac containing a fetus
measuring approximately 7 weeks' gestation. Doppler sonography is unable to demonstrate any
fetal heartbeat. You decide to perform a suction D&C. When giving informed consent, you discuss
the risk most commonly encountered in this operation.
Which of the following is the most common risk associated with suction D&C?
a. Infection
b. Uterine perforation
c. Uterovaginal bleeding
d. Damage to the bladder
e. Need for future surgery

2. Mrs. A 34 years old G3 P2 A0 36 weeks of gestation reffered from district hospital. She felt
contraction since 5 hours ago and water broke since one day ago. She was having antenatal care
regularly at midwives since 6 months of gestation. Since 2 months ago she said that she cannot
tolerate with heat and easy to get sweat. Physical findings shows BP 130/90; PR 110x/min; RR
8x/min, 37.6 C. Conjungtiva not pale. Thyroid gland do not enlarge. Heart and lung are normal.
Slight edema at lower extremities. Fundal height 29 cm, head presentation, FHR 144bpm.
Laboratory findings show CBC 12.0/36%/l0,900/230 000 Random plasma glucose 120mg/dL.
Urinalysis shows E O-l; L 3-5; Nitrit (-); Bacterial (+), LEA (-), protein (-), keton (-)
What is the most potential perioperative problem of this patient?
a. Intra-uterine infection
b. Gestational diabetes
c. Hyperthyroidism
d. Anemia
E. Urinary tract infection

3. Mrs. B, 37-years-old came to your office at 10 weeks of gestation from her last menstrual period.
She brought laboratory examination and found lgM CMV negative and IgG CMV positive. Which
of the following is the next best step in managing this patient?
a. No further testing
b. Offer patient to consume valacyclovir 8 g/day
c. Check lg G avidity
d. Repeat serological test for Ig M CMV in 2 weeks
e. Offer the patient to perform amniocentesis at 22 weeks gestational age
4. A 32-year-old gravida 2 Para 1 has been transferred from a midwifery-led unit for lack of progress
in labour at 4 cm. Her previous baby weighed 3100 g and was a normal delivery at 38 weeks
gestation. On admission, her observations are normal and the cardiotocography (CTG) was
reassuring. The midwife who examined her has diagnosed a complete breech presentation and
this is confirmed on scan. The woman is very keen to have a vaginal delivery and decision has
been taken to allow labour to continue. After 2 hours, there is no progress in labour and the CTG
has become suspicious.
What is the most appropriate action?
a. Discuss External Cephalic Version with the mother
b. Perform fetal blood sampling
c. Continue observation for one hour
d. Augment labor with oxytocin
e. Advice emergency caesarean section

5. A 25 year old lady come with abnormal pap smear result. She underwent colposcopy examination
and the result is a acetowhite lesion with punctation and atypical vessels. Biopsy result confirms
CIN I with HPV DNA test positive.
What do you suggest for patient?
a. Repeat cytology in 12 months
b. Repeat cytology in 6 months
c. LEEP procedure
d. Re-evaluation of HPV DNA
e. Cold knife conization

6. Mrs. D 39 years old G4 P3 34 weeks of gestation arrives at delivery ward with severe dyspnoe.
She looks really anxious, her vital signs show: BP 180/110 mmHg; PR 110x/min; RR 26x/min;
36.4°C, SpO2 95%. Conjunctiva not pale. Heart; normal heart sound, no murmur or gallop. Lung:
Vesicular with rales and no wheezing. Fundal height 30cm, head presentation, FHR 170bpm, no
contraction Laboratory findings: CBC Hb 10;Ht 32;L 12,000; Platelet 120,000
What is the mechanism that can induce this condition?
a. Community acquired pneumonia
b. Right heart failure
c. Immune response
d. High oncotic pressure
e. Diastolic dysfunction

7. Mrs A, 26 years-old, G1P0A0, according to her last LMP is 34 weeks pregnant, came for her first
antenatal care. She said that she had 20 kg of weight gain during her pregnancy with swelling
ankles for the past 4 weeks. She never took any iron or vitamin supplementation. From the
physical findings BP 145/95 mmHg, HR 86x/min, RR 20x/min, BMI 35 kg/m2. Ultrasound
examination confirmed twins in breech presentation. Results from urinalysis were as follows:
color cloudy yellow, specific gravity 1.013, albumin 2+, RBC 0-1, WBC 2-5, negative bacteriaI
count.
What is the most likely diagnosis?
a. Acute fatty necrosis of the liver
b. Chronic hypertension
c. Pyelonephritis
d. Renal disease
e. Preeclampsia

8. Mrs. A 34 years old G3 P2 A0 36 weeks of gestation referred from district hospital. She felt
contraction since 5 hours ago and water broke since one day ago. She was having antenatal care
regularly at midwives since 6 months of gestation. Since 2 months ago she said that she cannot
tolerate with heat and easy to got sweat. Physical findings shows BP 130/90; PR 110x/min; RR
18x/min; 37.6 C. Conjunctiva not pale. Thyroid gland do not enlarge. Heart and lung are normal.
Slight oedema at lower extremities. Fundal height 29cm. head presentation, FHR 144 bpm.
Laboratory findings show CBC 12.0/36%/10,900/230,000. Random plasma glucose 120mg/dl.
Urinalysis shows E 0-1; L 3-5; Nitrit (-); Bacterial (+). LEA (-), protein (-), keton (-)
What laboratory evaluation should be done next?
a. Hba1c
b. Cervical swab
c. Peripheral blood smear
d. TSH and FT4
e. Urinary Culture

9. Three weeks after delivery, a 29-year-old primipara, who is breast-feeding twin girls, presents to
the clinic, complaining of a tender right breast mass. On physical examination, you find a 5-cm
fluctuant, swollen, reddened mass in her right breast that is exquisitely tender to the touch.
Axillary lymph nodes on the ipsilateral side are enlarged and tender.
What is the most appropriate next step in the management of this patient?
a. Excisional biopsy of the mass
b. Incision and drainage of the mass plus oral antibiotics for the mother
c. Have the patient continue to breast-feed on the other side
d. Needle aspiration of the mass
e. Intravenous antibiotic therapy for the mother and infants

10. A 36 year old G2P1 presents to the antenatal clinic. She had an emergency caesarean section for
sudden onset hypertension and placental abruption at 30 weeks in her previous pregnancy. She
is currently 20 weeks of gestation and enquires about further plan of fetal monitoring in this
pregnancy.
What is the most appropriate advice?
a. Serial scans from 28 weeks
b. Serial scans starting from 24 weeks
c. Serial cardiotocography monitoring from 28 weeks
d. Uterine artery Doppler at 22 weeks
e. No extra monitoring is required

11. A 28-year-old G2P1 is seen for her first prenatal visit at 16 weeks' gestation by menstrual
history. Her first child was born at 32 weeks spontaneously. She is worried this pregnancy also
will be ended with spontaneous preterm birth.
What is the next appropriate management for the patient?
a. Treat asymptomatic bacterial vaginosis
b. Give tocolytics for inhibitinq preterm labor
c. Schedule for cervical cerclage
d. Progesterone prophylaxis
e. Intervention modifiable risk factors for preterm

12. A 28 women present to your clinic with feeling of fullness in the vagina. On examination you
find a bluish cystic mass came from the right lateral forniks sized 6 cm. The bulge is not tender
on palpation. What is the most probable diagnosis of this condition?
a. Vaginal inclusion cyst
b. Gartner's duct cyst
c. Bartholin cyst
d. Skene duct cyst
e. Endometriosis cyst

13. What is the implantation of a placenta in which there is a defect in the fibrinoid layer at the
implantation site, allowing the placental villi to invade and penetrate into but not through the
myometrium called?
a. Placenta percreta
b. Placental infarct
c. Placenta previa
d. Placenta accreta
e. Placenta increta

14. A patient returns for a postoperative check up 2 weeks after a total abdominal hysterectomy for
fibroids. She is distressed because she is having continuous leakage of urine from the vagina.
Her leakage is essentially continues and worsens with coughing, laughing, or movement. Given
her history and physical, you perform both a methylene blue dye test, which is negative.
What is the next management step for this case?
a. Cystoscopy
b. Indigo carmine test
c. lVP
d. Urinalysis
e. Retrograde Pyelography
15. Mrs E, 32 yo referred from midwife with antepartum haemorrhage. She is G3P2 term
pregnancy. On examination her blood pressure is 160/100 mmHg, HR 100 bpm. She looks
anemic, not icteric. Obstetrical examinations reveal contraction 4-5x/10 minutes, FHR 170 bpm,
head presentation 3/5. After thorough examination it is concluded that there is a placental
abruption with retroplacental hematoma size 6x5 cm. This patient is planned to do caesarean
section.
If during operation the uterus is couvelaire but with good contraction, how would you manage
that condition?
a. Ascending uterine artery ligation
b. Sub total hysterectomy
c. Perform prophylactic b-lynch suture
d. Uterotonic and observation
e. Hypogastric artery ligation

16. Mrs. 32-year old, P0, comes to your outpatient clinic due to her prolonged menstrual duration.
She reports her menstrual duration until 14 days and using 10 pads per day. She feels fatigue
easily. On physical examination, you palpate an irregularly enlarged uterus, non tender with
firm contour. Cervix appears to be hyperemic without mass appearance or other abnormalities.
What is the cause of necrotic and degenerative process in fibroids?
a. Limited blood supply within tumors
b. Mitotic activity
c. Cytogenetic mutations
d. Hyperperfusion
e. Chromosomal defects

17. A 30 years old patient came with complaint of infertility. Her husband is a 33-year-old who has
had a semen analysis, which was reported as normal. On further history, the patient reports
that her periods have been quite irregular over the last year and that she has not had period in
the last 3 months. She also reports of weight gain and abnormal hair growth. From ultrasound,
we found multiple follicles size 2-8 mm in both ovaries.
Which of following correlate to this condition?
a. Positive clomiphene citrate challenge test
b. Midluteal progesterone level of 4 ng/ml (ovulasi : 10, PCO tdk terjadi ovulasi)
c. Serum AMH level 2,6 ng/ml (N: 1, PCOS: > 3)
d. Day 3 FSH level 20 IU
e. Follicle antral basal count of 12

18. A 21 years old G1P0 patient presents to your office with vaginal bleeding at approximately 6
weeks’ gestation by her last menstrual period. Her examination is benign with a 6-week-sized
uterus, a closed cervical os, and a small amount of blood within the vaginal vault. You order a
complete pelvic ultrasound that shows as follow:
The gestational sac diameter 12 mm.
What is the most likely diagnosis?
a. Interstitial pregnancy
b. Embryonic demise
c. Threatened abortion
d. Normal intrauterine pregnancy
e. Blighted ovum

19. What is total iron need during pregnancy?


a. 500 mg
b. 1000 mg
c. 2000 mg
d. 1500 mg
e. 750 mg

20. Which of the following is the indication for anti-viral treatment in pregnant women with
hepatitis B?
a. HBV DNA 200.000 copies/mL
b. AntiHBe positive
c. HBsAg positive
d. Anti HBs positive
e. HBeAg positive, HBV DNA 2000 copies/mL

21. Which of the following is high risk factor for preeclampsia?


a. First pregnancy
b. Age > 40 years
c. BMI > 35 kg/m2
d. Diabetes mellitus
e. Family history of preeclampsia
22. A 60-year-old P2 presents to the urogynecology clinic with complaints of urinary incontinence.
She has urinary urgency and can’t make it to the bathroom before leaking a large amount or
urine. She urinates 12 times during the day and gets up two to three times per night to urinate.
What is the next examination plan for this case?
a. Voiding diary
b. Pelvic floor ultrasound
c. Pad test
d. Urinalysis
e. Urodynamic study

23. A 29 years old woman with a positive pregnancy test presents with a good history of tissue
expulsion vaginally passing tissue per vagina. A transvaginal ultrasound scan shows an empty
uterus with an endometrial thickness of 11 mm.
Regarding her diagnosis, you consider that :
a. She should be offered medical management of miscarriage
b. She should be offered a hysteroscopy
c. A laparoscopy should be performed to exclude an ectopic pregnancy
d. She has had a complete miscarriage and needs no further treatment
e. She has had pregnancy of unknown location and needs further investigations

24. Mrs. D 39 years old G4P3 34 weeks of gestation arrives at delivery ward with severe dyspnoe.
She looks really anxious. Her vital sign show: BP 180/110 mmHg, PR 110x/min, RR 26x/min, 36,4
oc Sp O2 95%. Conjunctiva not pale
Heart: normal heart sound, no murmur or gallop
Lung: Vesiculer with rales and no wheezing
Fundal height 30 cm, head presentation, FHR 170 bpm, no contraction
Laboratory findings: CBC Hb 10;Ht 32;L 12.000;Platelet 120.000
Whats is the most likely diagnosis of this patient?
a. Acute respiratory distress syndrome
b. Acute pulmonary edema
c. Cor pulmonale
d. Chronic obstructive pulmonary disease
e. Pneumonia

25. Mrs. N, 37 years old with chief complain of infertility for 6 years with history of severe
dysmenorrhea. From histerosalphingography, both tube were non patent. Pelvic ultrasound
found bilateral cystic mass with internal echo sized 50 and 60 mm in diameter. Her husband’s
sperm examination was within normal limit.
Which of the following is true regarding low ovarian reserve in endometriosis?
a. Ovary with endometrioma has a higher response rate to gonadotropin
b. Low ovarian reserve in endometriosis only happen after surgery
c. Ovulation rate in ovary with endometrioma is higher compared to ovary without
endometrioma
d. There is higher density of follicle in ovary with endometrioma
e. Loss of ovarian stromal appearance and fibrosis are present in ovarian cortex with
endometrioma

26. A 28 years old G2P1 is seen for her first prenatal visit at 16 weeks’ gestation by menstrual
history. Her first child was born at 32 weeks spontaneously. She is worried this pregnancy also
will be ended with spontaneous preterm birth.
What is the most accurate examination that can be done at 16 weeks to predict the risk of
preterm birth?
a. Urinary test to exclude urinary tract infection schedule routine antenatal care in 4 weeks
b. Measure cervical length (< 20 mgg)
c. Fibronectin examination (muncul bila sudah terjadi preterm labor)
d. Vaginal swab to exclude bacterial vaginosis
e. IGFBP-1 examination (PPROM)

27. A 28 years old woman is hoping to become pregnant soon. She is worried about her history of
acute pelvic inflammatory disease (PID) when in college 8 years ago.
Which of the following is the pathogenesis of infertility due to history of PID?
a. Ovulatory dysfunction due to pelvic inflammatory disease
b. Inflammatory response followed with fibrosis to protein released by chlamydia
c. Inflammation that hinders spermatozoa movement inside the uterus
d. Inflammation of the endometrium
e. Direct destruction of tubal mucosa by chlamydia

28. A 36 years old patient, P0, present to your clinic for fertility work up. She had been married for
2 years with regular intercourse. Her menstrual cycle is normal. Her general status was normal.
Vaginal examination revealed normal findings.
Which of the following examination that is not included in basic work up in the patient?
a. Endometrial dating according to Noyes criteria
b. Mid luteal progesterone examination
c. Semen analysis
d. Ultrasonography
e. Hysterosalpingography

29. Mrs 21 years old came to outpatient clinic with primary amenorrhea. She has a complaint of
impaired sense of smell. She is diagnosed with Kallman syndrome. She has a family history of
diabetes, currently her BMI is 29 kg/m2 with waist circumference of 90 cm.
Which of the following drugs can be used for ovulation induction in this condition?
a. Letrozole 1x2,5 mg for 5 days
b. Human menopausal gonadotropin injection
c. Clomiphene citrate 1x50 mg combined with metformin 2x500 mg
d. Clomiphene citrate 1x50 mg for 5 days
e. Metformin 2x500 mg

30. 32 years old woman presents to your office. She complained about her sexual problems of pain
during sexual intercourse since giving spontaneous birth of her second child 6 month ago. She
feels a normal desire to engaged in sexual intercourse. She is now still breastfeed the baby and
she is combine oral contraception pills.
Whats is the pathophysiology of sexual dysfunction in breastfeeding women?
a. Low oxytocin levels
b. Stress
c. Fatigue
d. Low progesterone levels
e. Low estrogen levels

31. A 28 years old patient, P0, present to your clinic for fertility work up. She had been married for 2
years with regular intercourse. Her menstrual cycle is normal. Her general status was normal.
Vaginal examination revealed normal findings. The following month she came back with the
result of hysterosalpingography (see the picture below)

What will be your next step?


a. Plan for IVF
b. Gives clomiphene citrate and plan for natural conception
c. Schedule operative laparoscopy
d. Order semen analysis
e. Gives clomiphene citrate and plan for intrauterine insemination

32. For patient with congenital heart disease. What is the most common adverse cardiovascular
event encountered in pregnancy?
a. Heart failure
b. Arrhythmia
c. Thromboembolic event
d. Heart axis changes
e. Cerebrovascular hemorrhage

33. Mrs. N, 37 years old with chief complain of infertility for 6 years with history of severe
dysmenorrhea. From hysterosalpingography, both tubes were non-patent. Pelvic ultrasound
found bilateral cystic mass with internal echo sized 50 and 60 mm in diameter. Her husband’s
sperm examination was within normal limit. Her AMH level was 0,9 ng/ml.
Which of the following might contribute to a diminished ovarian reserve in endometriosis?
a. Inflammation that lead to fibrosis of ovarian cortex
b. Low rate of follicle recruitment
c. Toxic effect of hormonal therapy
d. Using suture rather than bipolar coagulation during ovarian cystectomy
e. Removal of deep infiltrating endometriosis lesions

34. A 34-year old women with primary infertility 3 years, oligomenorrhea, and body mass index
(BMI) of 26. Day 23 progesterone level result was 5 ng/ml. Transvaginal ultrasound shows
multiple small follicle size 5-8 mm in both ovaries. HSG shows bilateral patent tubes. Her
partner’s semen analysis shows a volume of 3 ml, Ph of 7, and a sperm count of 20 million/ml.
According to the current International Guidelines, which of the following medication is
considered to be the first line of therapy for ovulation induction?
a. Gonadotropin injection 75 IU/day
b. Clomiphene citrate 50mg/day combined with metformin 2x500 mg
c. Clomiphene citrate starting at dose 50mg/day for 5 days
d. Letrozole 1x2,5 mg
e. Metformin 2x500 mg

35. A 50-year-old lady presents to gynaecology outpatient clinic with postmenopausal bleeding. A
thorough clinical workup reveals an endometrial thickness of 7mm with an irregular, echogenic,
thickly septated ovarian mass on her left ovary.
What type of tumor is responsible for this lady’s clinical presentation?
a. Dysgerminoma
b. Epithelial adenocarcinoma of the ovaries
c. Endodermal sinus tumour
d. Embryonal carcinoma
e. Granulosa-theca cell tumour

36. A 27-year-old woman presents to your office with a positive home pregnancy test and a 3-day
history of vaginal bleeding. She is concerned that she may be having a miscarriage. On
examination, the uterine fundus is at the level of umbilicus. By her last period, she should be
around 8 weeks gestation. On pelvic examination, there is a moderate amount of blood and
vesicle-like tissue in the vaginal vault, and the cervix is closed. The lab then calls you to say that
her serum B-HCG result is greater than 1,000,000 mIU/ML.
Which of the following is the best next step in this patient’s evaluation?
a. Schedule a follow-up visit in 2 to 4 weeks to recheck B-HCG level
b. Surgical intervention (suction curettage)
c. Complete pelvic ultrasound
d. Determination of RH status
e. Methotrexate administration

37. A 36 years of G2P1 presents to the antenatal clinic. She had an emergency caesarean section for
sudden onset hypertension and placental abruption at 30 weeks in her previous pregnancy. She
is currently 20 weeks of gestation and enquiries about further plan of fetal monitoring in this
pregnancy. What is the most appropriate advice?
A. Uterine artery doppler at 22 weeks
b. Serial scans from 28 weeks
c. Serial scans starting from 24 weeks
d. No extra monitoring in required
e. Serial cardiotocography monitoring from 28 weeks

38. A 30-year-old P0 obese woman in noted to have irregular menses and hirsutism. On ultrasound
examination, there are many small follicles in both ovaries. She was diagnosed with PCOS and
receive combined oral contraception for menstrual regulation. She noticed a decreased
hirsutism after takin COC, what is the most probable mechanism?
a. Suppression of prolactin secretion
b. Suppression of HPO axis
c. Suppression of androgen receptor in the peripheral tissue
d. Increased level of SHBG
e. Resumption of ovulation

39. A 25 years-old women G1 20 weeks of gestational age came to outpatient clinics with a mass in
perineum sized 1 cm flesh-coloured and cauliflower like appearance. She also feels itchy and
discomfort during sexual intercourse. On speculum examination we can see a small verrucous
mass sized 0,5 cm on vaginal side wall.
What is the recommended treatment for this patient?
a. Podophyllum resin
b. Surgical excision
c. Trichloroacetic acid
d. Fluorouracil
e. Imiquimod

40. A 38 years old multi gravid woman complains of the painless loss of urine, beginning
immediately with coughing, laughing, lifting, or straining. Cessation of the activity do not stop
the urine loss. She also complains of frequency with small-voided volume of urine, but no
urgency.
This history is most suggestive of
a. Fistula
b. Overflow incontinence
c. Stress incontinence
d. Urge incontinence
e. Mixed incontinence

41. A 45-years-old woman presents of your office for consultation-regarding her symptoms of
menopause. She stopped having periods 13 months ago after TAH-BSO operation and is having
severe hot flushes. The hot flushes are causing her considerable stress.
Which of the following is an absolute contraindication for hormonal therapy?
a. Migraine
b. Impairment of liver function
c. Coronary heart disease
d. Endometriosis
e. Diabetes mellitus

42. A 26- years-old woman complains of recurrent bouts of bacterial vaginosis (BV) despite
successful initial treatment. She does no douche or smoke and has been in a monogamous
relationship or 6 years. Recurrence of BV after initial treatment is common (up to 30 percent).
Which can be frustrating of the patient?
a. Use of acidiying vaginal gels
b. Treatment of long continuous antibiotic
c. No intervention consistently decrease recurrence
d. Probiotics and reintroduction of lactobacilli
e. Treatment of male partners

43. A 23-year-old G1 32 weeks is being admitted to the hospital because of preterm contraction. The
patient complaint regular contraction. Antenatal care was done regularly in PHC. No remarkable
abnormality was found during ANC. BMI before pregnancy was 30 kg/m 2, weight gain during
pregnancy is 14 kg. Abdominal examination showed FUT 36 cm, regular contractions, fetal
heartbeats 154 bpm. Speculum examination showed closed ostium uteri externa. An ultrasound
shows the estimated fetal weight 2400 g, AFI 30 cm, no fetal morphology abnormalities,
placenta implanted in anterior corpus, cervical length 1.8 cm, funnelling positive. Laboratory
results were Hb 10,7 g/dL, HT 33%, Leucocyte 13.500, Thrombocyte 315.000 MCV 82 MCH 30.
What is the most appropriate next step in the management of this patient?
a. Schedule for OGTT test
b. Give intravenous iron
c. Give MgSO4 for neuroprotection
d. Give antibiotic prophylaxis
e. Schedule for cervical cerclage

44. An 18-years-old G1 at 30 4/7 weeks presents for her scheduled obstetric (OB) appointment. A
28- week ultrasound showed the fetus to be in he 13 th percentile for estimated fetal weight. The
patient denies any complaints today. Fetal movement is active. BMI before pregnancy was 19.6
kg/m2 , gestational weight gain 7 kg. Vital sign are normal. Abdominal examination today shows
a gravid uterus measuring 27cm. Fetal heart tones (FHTs) are in the 140s. Laboratory results
were Hb 10,1 g/dl, HT 30%, Leucocyte 10.500, Thrombocyte 215.000, MCV 78, MCH 29. OGTT
result was fasting 92 mg/dl and 2 hours after 75 glucose 148 mg/dl.
What is the appropriate next step in the management of this patient?
a. Schedule for serum ferritin test
b. Give element iron 100-200 mg per oral
c. Admit patient to the hospital for lung maturation
d. Perform fetal growth ultrasound
e. Schedule for a biophysical profile (BPP)

45. A 32 years-old woman has a pelvic ultrasound that bilateral 5cm “kissing” ovarian cyst in the
pouch of Douglas. Both of which contain diffuse, low-level echoes giving a solid ‘ground-glass’
appearance. She reports severe dysmenorrhea and dyspareunia.
Which of the following condition that can be found associated with this finding?
a. Normal level of Ca 125
b. Increased level of He4
c. Adenomyosis in the posterior uterus
d. Fifty percent risk of malignant transformation
e. Increased level of serum AMH

46. A 35-year-old grand multipara has had a major postpartum haemorrhage (PPH) following a
normal delivery. Mechanicals and pharmacological measure have failed to control the bleeding.
Examination has confirmed that there are no retained placenta tissue in the uterine cavity and
absence of trauma to genital tract. What is the most appropriate first-line surgical
management?
a. Postpartum hysterectomy
b. B-Lynch or modified compression sutures
c. Selective arterial embolization
d. Ligation of internal iliac artery
e. Balloon tamponade

47. A 22 year old unbooked primigravida presents to the Emergency Department at 26 weeks of
gestation with a history of spontaneous painless bleeding of about 500 ml.
What is the best investigation to secure a diagnosis?
a. Transabdominal scan
b. Transvaginal scan
c. CT scan
d. MRI scan
e. CTG

48. Mrs B, 37-years- old come to your office at 32 weeks of gestation according to her last menstrual
period. She has no ultrasound examination before and did not do her routine antenatal care.
The vital sign is within normal limit. She has body mass index 19 kg/m2. During physical
examination, the uterine fundal height is 22 cm. From ultrasound examination, the fetus has
biometric values that correlate with 30 weeks fetus.
Which of the following is the next best step in managing this patient?
a. Repeat sonography for fetal growth in 2 weeks
b. Doppler velocimetry evaluation every 3 days
c. Evaluate maternal status and comorbidities
d. Consider deliver the baby
e. Antenatal care routinely for the next 2 weeks

49. A 32 year old primiparous woman 32 weeks gestation arrives at emergency room. She was
referred by midwives due to high blood pressure. She is fully alert with BP 160/110 mmHg; PR
98 x/m; RR 18 x/min: afebris. Sclera look icteric with pale conjunctiva. Heart and lung are
normal. Fundal height 32 cm, head presentation, FHR 144 bpm, with no contraction.
Her laboratory findings show: CBC Hb 9; Ht 30; L 15.000; platelet 75.000. SGOT/SGPT 80/72.
LDH 720. Albumin 2.5 g/dL. Urinalysis: protein +++. Peripheral blood smear shows anisocytosis
pattern. What is the cause of patient’s anemia?
a. Thalassemia
b. Iron deficiency
c. B12 deficiency
d. Auto immune
e. Microangiopathy

50. A 32-year-old woman presents to your office. She complained about her sexual problems of pain
during sexual intercourse since giving spontaneous birth of her second child 6 months ago. She
feels a normal desire to engaged in sexual intercourse. She is now still breastfeed the baby and
she is in combine oral contraceptive pills.
What is the most common cause of this problem?
a. Infection
b. Combine Oral contraceptive pills
c. Fatigue
d. Episiotomy scar
e. Breastfeeding

51. A 35-year-old woman, G4P3, at 37 weeks gestation presented in hospital with a ten-day history
of low extremities oedema, with idiopathic hypertension for 1 year. At presentation, she had a
blood pressure of 170/100 mmHg. Laboratory findings were normal except urinalysis (protein
2+). She was diagnosed with superimposed severe preeclampsia. It was decided to deliver the
fetus by means of a C-section by indication transverse lie. Blood pressure measurement was
150/100 mmHg. She lost consciousness for 30 seconds five hours after operation. The
laboratory studies gave the following results: serum aspartate aminotransaminase (AST), 225
IU/L, serum alanine aminotransaminase (ALT), 140 IU/L; serum lactate dehydrogenase (LDH),
1017 IU/L; Serum urea and creatine were normal; Haemoglobin, 10.6 mg/dL; platelet count, 50
x 103 µ/ml. A brain computed tomography (CT) scan was performed on patient which revealed
the left frontal lobe lacunar infarction. The patient was transferred to intensive care unit.
What is the management after, for this case
a. Magnesium Sulphate
b. Fresh-frozen plasma and thrombocytes concentrates
c. Anti-platelets
d. Corticosteroid
e. Anti-oxidant

52. A 26-years-old woman, G1P0A0 was admitted to ER because she lost her consciousness around 1
hour ago. According to her husband, she is 36 weeks pregnant. She performed antenatal care at
scheduled time, and never missed one. Her husband said, she never had any hypertension or
any other disease before. Three days prior hospitalization, she had severe nausea and vomiting.
Physical examination reveals, BP 120/80 mmHg, pulse rate 87 x/min, RR 18 x/min, Temperature
36.50C. You notice there is an icteric sclera. Other physical examination was remarkable.
Obstetrical examination reveals no fetal heartbeat was detected.
Laboratory examination reveals CBC 10.2/29.9/8900/263.000; Ur/Cr 18/0,8; AST/ALT 458/878;
RBG 32; Urinalysis was within normal limit.
What is the best next management in this case?
a. Emergency Caesarean section
b. Injections of 40% Dextrose
c. Abdominal ultrasound
d. Induction of labour
e. Whole Blood transfusion

53. A 18-year-old adolescent female complains of not having started her menses. Her breast
development is Tanner stage II, Pubic hair development was stage I. From vaginal examination
found a small uterus and normal vagina and vulva. Which of the following describes the most
likely diagnosis?
a. Late onset congenital hyperplasia
b. Complete androgen insensitivity syndrome
c. Partial androgen insensitivity syndrome
d. Turner syndrome
e. Polycystic ovarian syndrome

54. Numerous physiologic changes develop over the course of pregnancy; however the greatest
impact on potentially compromised cardiovascular system is:
a. Decreased SVR in the first trimester, and increase after 32 weeks
b. The red cell mass rises by 40% (5-20% increase in red blood cell mass occurs from 8-10 weeks
gestation till the end of pregnancy)
c. Increased 30% total plasma volume in 28 weeks
d. Cardiac output increases to 30-50% above pre-pregnancy levels by the end of the third
trimester
e. Increase in cardiac output occurs by 24 weeks Cardiac output is increased as early as the fifth week

55. A 26-year-old G2P1 (no live child) is seen for her first prenatal visit at 18 weeks’ gestation by
menstrual history. Her first child was born at 28 weeks spontaneously, the baby was died after
hospitalized for 1 month in NICU. She is worried this pregnancy also will be ended with
spontaneous preterm birth.
What is the most accurate examination that can be done at 18 weeks to predict the risk of
preterm birth?
a. Cell-free fetal DNA
b. PAMG-1 examination
c. Fibronectin examination
d. Measure cervical length
e. IGFBP-1 examination

56. A 30 years old multiparous woman has rapid delivery soon after arriving in emergency room.
After delivery the placenta she is noted to have heavy vaginal bleeding. Help has been
summoned. Abdominal examination demonstrated the fundus was soft.
What is the most appropriate next step?
a. Uterine packing
b. Suture the laceration
c. Balloon tamponade
d. Misoprostol administration
e. Intravenous access for fluid resuscitation
57. A 18-year-old adolescent female complains of not having started her menses. Her breast
development is Tanner stage 1, pubic hair development was stage 1. From ultrasound
examination found small uterus and ovaries.
Which of the following is a long term risk if this patient left untreated?
a. Risk of gonadoblastoma
b. Risk of pulmonary infection
c. Risk of endometrial carcinoma
d. Risk of fractures
e. Risk of cervical carcinoma

58. 25-year-old women in her first pregnancy is noted to have prolonged first and second stages of
labour. She was induced at 38 weeks pregnancy. The baby was delivered by forceps. After
delivery the placenta is noted to have heavy vaginal bleeding. Abdominal examination
demonstrates a relaxed uterus. What should we do if the fundus not firm after placental
delivery.
a. Fundal massage
b. Misoprostol (PGE1)
c. Carborprost (Hemabate, PGF2-alpha)
d. Methylergonovine (Methergine)
e. Dinoprostone-prostaglandin E2

59. Mrs. D 39 years old G4P3 34 weeks of gestation arrives at delivery ward. She looks really
anxious, her vital sign show : BP 180/110mmHg, PR 110x/min, RR 26x/min, 36.4°C, SpO2 95%.
Conjunctiva not pale
Heart: Normal heart sound, no murmur or gallop
Lung: vesicular with rales and no wheezing
Fundal height 30 cm, head presentation, FHR 170bpm, no contraction
Laboratory findings: CBC Hb 10, Ht 32;L 12,000; Platelet 120,000
What is the primary treatment for this patient?
a. Increase myometrial contractility
b. Put her into ventilator machine
c. Reduces heart rate
d. Increases systemic vascular resistance
e. Reduces pre-load

60. Which of the following is NOT appropriate test in first trimester for screening preeclampsia?
a. PIGF (10-12 mgg)
b. Mean arterial pressure (16 mgg)
c. sFLT-1 (10-12 mgg)
d. PI uterine artery (diatas 20 mgg)
e. PAPP-A
61. A 28-years-old G1 at 26 weeks present for her scheduled obstetric appointment. You ordered
OGTT examination that shows fasting blood glucose 102 mg/dL and 2 hours after 75 g oral
glucose 185 mg/dL. Her gestational weight gain during pregnancy is 12 kg. Her BMI before
pregnancy was 26 kg/m2.
What is the most likely diagnosis?
a. Impaired glucose test (2JPP yang tnggi)
b. Diabetes mellitus type 2
c. Gestational diabetes
d. Normal OGTT
e. Diabetes mellitus type 1

62. A 30-year-old P0 obese woman is noted to have irregular menses and hirsutism. On ultrasound
examination, there are many small follicles in both ovaries.
Which of the following is consistent with polycystic ovarian syndrome?
a. BMD showing osteopenia
b. Finding of a 9-cm right ovarian mass
c. Elevated level of AMH produced by theca cells
d. Positive progestin challenge test
e. Elevated 17-hydroxyprogesterone level

63. A 45 years old women presents to your office for consultation regarding her symptoms of
menopause. She stopped having period 13 months ago after TAH-BSO operation and is having
severe hot flushes. The hot flushes are causing her considerable stress.
What is the management option for this patient?
a. Estrogen only pills
b. Monophasic combined oral contraceptive pills
c. Biphasic combined oral contraceptive pills
d. Progestin only pills
e. Sequential estrogen-progestin therapy

64. Mrs. 21 years old came to outpatient clinic with primary amenorrhea. She has a complaint of
impaired sense of smell.
Which of the following can be found during additional examination?
a. Absent of uterus
b. Pubic hair tanner stage 3
c. Breast tanner stage 3
d. LH level 0,9 IU/L
e. FSH level of 30 IU/L

65. Which of the following is the most common cause of recurrent pregnancy loss?
a. Endocrine factor
b. Genetic factors
c. Autoimmune disease
d. Unexplained
e. Anatomic factors

66 when she defecate, and disappear when laying down. There were no difficulty in voiding and
defecation. No urinary leakage during coughing and sneezing. She is not sexually active. If on the
Pelvic Organ Prolapse Quantification examination result showing below, what is the diagnosis of this
patient?
Aa Ba C
+3 +4 +6

GH Pb TVL
5 2 8

Ap Bp D
+2 +2 +4

a. Uterine prolapse grade 3, cystocele grade 2, rectocele grade 2


b. Uterine prolapse grade 4, cystocele grade 3, rectocele grade 2
c. Uterine prolapse grade 4, cystocele grade 3, rectocele grade 3
d. Uterine prolapse grade 2, cystocele grade 2, rectocele grade 1
e. Uterine prolapse grade 3, cystocele grade 3, rectocele grade 2

67. A 25 years-old women G1 20 weeks of gestational age came to outpatient clinics with a mass in
perineum sized 1 cm flesh-colored and cauliflower like appearance. She also feel itchy and
discomfort during sexual intercourse. On speculum examination we can see a small verrucous
mass sized 0,5 cm on vaginal side wall.
What is the most probable cause of this condition?
a. HPV type 16, 18
b. HPV type 6 and 11
c. Molluscum contagiosum
d. Herpes simplex virus
e. Syphilis

68. A 35-year-old woman, G3P2, presents to labor and delivery (L&D) at 33-week gestation referred
by midwife with BP 180/110 mmHg. BP on arrival is 170/105 mmHg. Urine protein is 1+ on
dipstick. Patient had history of high blood pressure in her previous pregnancy. ANC was done in
midwife. Blood pressure at first trimester was 130-145/90-95 mmHg, urine protein was negative
on dipstick. No antihypertension drug was given. The patient denies any complaints today.
What is the most likely diagnosis of the patient?
a. Preeclampsia with severe feature
b. Chronic hypertension
c. Gestational hypertension
d. Superimposed preeclampsia
e. Preeclampsia

69. Mrs. N, 37 years old with chief complain of infertility for 2 years with history of severe
dysmenorrhea. From hysterosalpingography, both tubes were patent. Pelvic ultrasound found
bilateral cystic mass with internal echo sized 25 and 40 mm in diameter, her antral follicle count
was 12. Her husband’s sperm examination was within normal limit.
What is the next appropriate management?
a. Give GnRH analog for 3 months continue with IUI
b. Perform laparoscopy cystectomy and adhesiolysis
c. Offer her IVF
d. Offer her IUI
e. Give Dienogest 1x2 mg for 6 months

70. A 23-years-old G1 32 weeks is being admitted to the hospital because of preterm contraction.
The patient complaint regular contraction. Antenatal care was done regularly in PHC. No
remarkable abnormality was found during ANC. BMI before pregnancy was 30 kg/m 2, weight
gain during pregnancy is 14 kg. Abdominal examination showed FUT 36 cm, regular contraction,
fetal heartbeat 154 bpm. Speculum examination showed closed ostium uteri externa. An
ultrasound shows the estimate fetal weight 2400 g, AFI 30 cm, no fetal morphology
abnormalities, placenta implanted in anterior corpus, cervical length 1,8 cm, funneling positive.
Laboratory results were Hb 10,7 g/dL, Ht 33%, leucocyte 13.500, thrombocyte 315.000, MCV 82,
and MCH 30.
What is the most likely etiology of preterm contraction in this case?
a. Excessive gestational weight gain
b. Maternal anemia
c. Maternal obesity
d. Bacterial vaginosis
e. Uterine overdistention

71. Mrs. 32-years-old, P0, comes to your outpatient clinics due to her prolonged menstrual duration.
She reports her menstrual duration until 14 days and using 10 pads per day. She feel fatigue
easily, she denies to experience dysmenorrhea. On physical examination, you palpate an
irregularly enlarged uterus, non-tender with firm contour. Cervix appears to be hyperemic
without mass appearance or other abnormalities.
What is the most likely diagnosis?
a. AUB-M
b. AUB-P
c. AUB-L
d. AUB-C
e. AUB-A
72. What is the common cause of heart failure during pregnancy and the puerperium?
a. Viral myocarditis
b. Valvular heart disease
c. Chronic hypertension with severe preeclampsia
d. Obesity
e. Pulmonary Artery Hypertension

73. A 18 years old girl come to the emergency room with an abruptly pelvic pain for 2 days. She had
already started felt pain on and off for the past 2 weeks especially when she was moving. She
has also nausea and vomiting. She has no fever. She has regular menstrual cycle. From the
physical examination an intense of low abdominal pain was marked even more on palpation.
There was a muscle defense, on the right lower abdomen quadrant. She tends to bend her
abdomen a bit on the bed. On abdominal ultrasound reveal a mass measuring 12 cm in the
largest diameter on her pelvic, characterized with multiple hyper echogenic interfaces in a cystic
mass in the right ovary, no blood flow could be detected in the cyst except only in the one pole
near the uterus. Uterus ante flexed within normal limit. Left ovary within normal limit. There is
no fluid in pelvic cavity. What would be the working diagnosis?
a. Ovarian fibroma
b. Ruptured cyst
c. Functional cyst
d. Endometriosis
e. A torsion cyst

74. Your patient is a 13-year-old adolescent girl who present with cyclic pelvic pain. She has never
had a menstrual cycle. She denies any history of intercourse. She is afebrile and her vital signs
are stable. On physical examination, she has age-appropriate breast and pubic hair
development and normal external genitalia. There is no bulge seen in the introitus of vagina.
You obtain a trans abdominal ultrasound, which reveals a hematocolpos and hematometra.
What is the most likely diagnosis?
a. Imperforate hymen
b. MRKH
c. Longitudinal vaginal septum
d. Vaginal agenesis
e. Transverse vaginal septum

75. A 30 years old patient come with complaint of infertility. Her husband is a 33-year-old who has
had a semen analysis, which was reported as normal. On further history, the patient reports
that her periods have been quite irregular over the last year and that she has not had period in
the last 3 months. She also reports of weight gain and abnormal hair growth. From ultrasound,
we found multiple follicles size 2-8 mm in both ovaries. Which of the following might contribute
to this condition?
a. High level of vitamin D
b. Low level of LH
c. Increased visceral fat deposition
d. Increased sensitivity towards insulin
e. Low level of androgen from the ovary

76. A 28 women present to your clinic with feeling of fullness in the vagina. On examination you find
a bluish cystic mass came from the right lateral forniks sized 6 cm. The bulge is not tender on
palpation.
What is the cause of this condition?
a. Remnant of Wollfian duct
b. Blockage of the glands opening
c. Infection
d. Endometriosis
e. Remnant of Mullerian duct

77. Patient 65 years old, P4 came to outpatient clinic with chief complain of frequent urination.
Since 6 months ago she feels the urge to void every hour and also she has to wake up 3-4 times
in the night to void. She never leaks urine. She doesn’t feel any pain during urination and no
blood in the urine. She already came to general practitioner and got antibiotics for 7 days but
the symptoms remained.
What is the most useful supporting examination in this patient?
a. Gynecology ultrasound
b. Bladder diary
c. Urine culture
d. Pelvic floor ultrasound
e. Urodynamic evaluation

78. A 26-years-old G2P1 (no live child) is seen for her first prenatal visit at 18 weeks’ gestation by
menstrual history. She had history of preterm birth at 28 weeks and her first child was died after
hospitalized for 1 month in NICU. She is worried this pregnancy also will be ended with
spontaneous preterm birth. Trans vaginal ultrasound scan shows her cervical length is 30mm,
and there is amniotic sludge.
What is the most accurate intervention to prevent the risk of preterm birth?
a. Cervical cerclage should be performed to prevent preterm birth
b. She should be offered fibronectin examination
c. Evaluate the cervical length at 22 weeks gestational age
d. Antibiotic and progesterone should be given
e. She should be offered for Anti-Phospholipid Syndrome screening

79. Mrs. N, 37 years old had just undergone laparoscopic procedure. Her chief complaint was
infertility for 6 years with history of severe dysmenorrhea. From hysterosalpingography, both
tubes were non-patent. Pelvic ultrasound found bilateral cystic mass with internal echo sized 50
and 60 mm
in diameter. Her husband sperm examination was within normal limit. From laparoscopy
findings, normal sized uterus with adhesion of posterior part to colon and Douglas pouch was
also severely adhered. Both of ovaries ware enlarged approximately 50 – 60 mm with severe
adhesion at the right and left adnexa. After adhesiolysis, both of tubes could be identified and
were blocked. Chocolate fluid was spilled from the cysts. The pathology of the cysts was
endometriosis cyst.
Which one is the TRUE statement?
a. GnRH agonist 1 time prior to surgery will give better results
b. Laparoscopic procedures should be done if infertility problem already more than 3 years
c. The patient should directly undergo surgery to solve her pain and infertility problem
d. Laparoscopic cystectomy was done because the cyst have diameter 50 and 60 mm
e. Male factor and ovarian reserve should be measured prior to decision of surgery

80. A parity 3 40-year-old woman complains of cyclical heavy and painful menstrual bleeding. On
examination she is found to have an enlarged globular uterus and a trans vaginal sonography
revealed diffuse adenomyosis. She has completed her family and currently relies on condoms
for contraception. She smokes 10 cigarettes per day but is otherwise fit and well.
Which treatment would you consider most appropriate? Choose the single best answer.
a. LNG-IUS
b. COCP
c. Endometrial ablation
d. Hysterectomy
e. GnRH analogue

81. Which is one is correct regarding Eisenmenger Syndrome?


a. Physiologic changes during pregnancy such as high systemic vascular resistance are attributed
to worsening hypoxia
b. Caesarean section can reduce mortality rate
c. Patients should be counseled to do antenatal care in secondary or tertiary hospital
d. Most of patients with Eisenmenger syndrome died in intrapartum
e. May be a common cause of pulmonary hypertension due to chronic overperfusion of the
pulmonary vasculature

82. An 18-year-old G1 at 30 4/7 weeks presents for her scheduled obstetric (OB) appointment. A 28-
week ultrasound showed the fetus to be in the 13 th percentile for estimated fetal weight. The
patient denies any complaint today. Fetal movement is active. BMI before pregnancy was 19,2
kg/m2, gestational weight gain is 7 kg. Vital sign are normal. Abdominal examination today
shows a gravid uterus measuring 27 cm. Fetal heart tones (FHTs) are in the 140s. Laboratory
results were Hb 10,1 g/dl, Ht 30%, Leucocyte 10.500, Thrombocyte 215.000, MCV 78, MCH 28.
OGTT result was fasting 92 mg/dl and 2 hours after 75 glucose 148 mg/dl. What is the most
likely predisposition factor for this case?
a. Maternal anemia
b. Inadequate maternal nutrition
c. Genetic abnormalities
d. Placental insufficiency
e. Pre-gestational diabetes mellitus

83. A patient present to you with pain and swelling in the vulva. On examination you find a reddish
bulge on the vaginal introitus at 4 o’clock positions sized 3 cm, cystic and pain on palpation.
Which of the following is the most common causative organism for formation of this condition?
a. Bacterial vaginosis
b. Trichomonas vaginalis
c. Staphylococcus aureus
d. Streptococcus epidermidis
e. Neisseria gonorrhoeae

84. A 18-year-old nulligravid woman presents to the student health clinic with a 4-week history of
yellow vaginal discharge. She also reports vulvar itching and irritation. She is sexually active and
monogamous with her boyfriend. They use condoms inconsistently. On physical examination,
she is found to be nontoxic and afebrile. On genitourinary examination, vulvar and vaginal
erythema is noted along with a yellow, frothy, malodorous discharge with a pH of 6,5. The
cervix appears to have erythematous punctuations. There is no cervical, uterine, or adnexal
tenderness. The addition of 10% KOH to the vaginal discharge does not produce an amine odor.
Wet prep microscopic examination of the vaginal swabs is performed. What would you expect
to see under microscopy?
a. Branching hyphae
b. Multinucleated giant cells
c. Scant WBC
d. Epithelial cells covered with bacteria
e. Flagellated, motile organisms

85. A 45 years old woman presents to your office for consultation regarding her symptoms of
menopause. She stopped having period 13 months ago after BSO operation and is having severe
hot flushes. The hot flushes are causing her considerable stress. Which of the following
medication that you will give for hormonal therapy?
a. Sequential estrogen-progestin therapy
b. Biphasic combined oral contraception
c. Monophasic combined oral contraception
d. Estrogen only therapy
e. Triphasic combined oral contraception

86. A 28-year-old G1 at 26 weeks present for her scheduled obstetric appointment. You ordered
OGTT examination that shows fasting blood glucose 102 mg/dl and 2 hours after 75 g oral glucose
185 mg/dl. Her gestational weight gain during pregnancy is 12 kg. Her BMI before pregnancy was
26 kg/m2.
What is the appropriate next step in the management of this patient?
a. Schedule fetal growth ultrasound
b. Give metformin 3x500 mg orally
c. Admit to hospital for fetal monitoring
d. Schedule nonstress test (NSTs)
e. Advise insulin in order to lowering blood glucose

87. A 34-year old woman with primary infertility 3 years, oligomenorrhea and a body mass index
(BMI) of 26. Day 23 progesterone level result was 5 ng/ml. Transvaginal ultrasound shows
multiple small follicle size 5-8 mm in both ovary. HSG shows bilateral patent tubes. Her partner’s
semen analysis show a volume of 3 ml, pH of 7 and a sperm count of 20 million/ml. What is the
most appropriate step of management?
a. Start ovulation induction using gonadotropin
b. Measure serum testosterone level
c. Examine TSH and prolactin
d. Give Aromatase inhibitor
e. Measure FSH, LH and estradiol

88. A 32 year old primiparous woman 32 weeks gestation arrives at emergency room. She was
referred by midwives due to high blood pressure. She is fully alert with BP 160/110mmHg; PR
98x/m, RR 18x/m: afebris. Sclera look icteric with pale conjunctiva. Heart and lung are normal.
Fundal height 32 cm, head presentation, FHR 144 bpm, with no contraction.
Her laboratory findings show: CBC Hb 9; Ht 30; L 15,000; platelet 75.000. SGOT/SGPT 80/72. LDH
720. Albumin 2.5 g/dl. Urinalysis: protein +++.
Which of the following is appropriate?
a. Immediately perform Cesarean section
b. Conservative until term gestation
c. Conservative until 34 weeks
d. Labour induction by misoprostol
e. Labour induction after complete lung maturation

89. Mrs. 32-year old, P0, comes to your outpatient clinic due to her prolonged menstrual duration.
She reports her menstrual duration until 14 days and using 10 pads per day. She feels fatigue
easily. On physical examination, you palpate an irregularly enlarged uterus, non tender with firm
contour. Cervix appears to be hyperemic without mass appearance or other abnormalities.
By which mechanism does fibroid creates a hyperestrogenic environment requisites for their
growth?
a. Fibroid contains higher level of cytochrome P450 aromatase, which allows for conversion of
androgens to estrogen
b. Fibroid converts more estradiol to estrone
c. All of the above
d. Increase adipose conversion of androgens and estrogen
e. Fibroid cells contain less density of estrogen receptors compared with normal myometrium

90. What type of cyst that frequently associated with ovarian torsion?
a. Ovarian fibroma
b. Follicular cyst
c. Endometrioma
d. Dermoid cyst
e. Mucinous cyst

91. A primigravida at 36 weeks gestation is measuring large for dates. Ultrasound shows AC > 97 th
centile. GTT performed shows poorly controlled gestational diabetes.
What is the immediate management plan?
a. Start sliding scale and deliver
b. Start oral hypoglycaemics agent
c. Start induction
d. Observe for 1 week
e. Give steroids

92. A 28-year-old G2P2001 at 40+3d WGA is seen in obstetric outpatient clinic in your hospital for
routine antenatal care. She is experiencing contractions occasionally and is sensing of mildly
pressure in her birth canal, but does not feel like she is in labor as she experienced in her first
delivery. Her first child was born 3 years ago at 41 WGA following an induction, resulting in a
normal spontaneous vaginal delivery. She asked you if an induction for this pregnancy is
needed. You performed a physical and cervical examinations and discuss options available.
Which of the following cervical examinations is the most favorable for induction of labour?
a. Cervix soft, mid-position, 3 cm dilated, 50% effaced with -3 station
b. Cervix is firm, posterior, no dilatation, 0% effacement, -1 station
c. Cervix is soft, mid-position, 3 cm dilated, 50% effacement, -2 station
d. Cervix is soft, anterior, 4 cm dilated, 80% effacement, -1 station
e. Cervix is medium, posterior, 2 cm dilated, 30% effacement, -2 station

93. Which of the following is true regarding COVID-19 in pregnancy?


a. No anti-viral was approved by FDA for pregnant women
b. Preterm delivery and preeclampsia are increased in pregnant women with COVID-19
c. Recommendation mode of delivery in pregnant women with COVID-19 is caesarean section
d. Evidence showed that pregnancy offers an altered immunity scenario which may allow severe
COVID-19 disease
e. Vertical transmission is unlikely because placenta has low expression of ACE2 receptor

94. A 89-year-old female patient with multiple, serious medical comorbidities presents to discuss
option for treatment of her high-grade prolapse. The prolapse is externalized and becoming
ulcerated from friction against her undergarments. Her main priority is to “fix or get rid of this
thing”, but her primary care provider has cautioned against a lengthy of open abdominal
procedure.
She is not interested in future intercourse. What can you offer this patient?
a. Total colpocleisis
b. Ring pessary
c. Partial colpocleisis
d. Manchester Fothergyl operation
e. Sacrospinous hysteropexy

95. A 27-year-old woman presents to your office with a positive home pregnancy test and a 3-day
history of vaginal bleeding. She is concerned that she may be having a miscarriage. On
examination, the uterine fundus is at the level of umbilicus. By her last period, she should be
around 8 weeks gestation. On pelvic examination, there is a moderate amount of blood and
vesicle-like tissue in the vaginal vault, and the cervix is closed. The lab then calls you to say that
her serum β-hCG results is greater than 1,000,000 mIU/mL. The patient undergoes an
uncomplicated suction D&C. The pathology reports is available the next day and is consistent
with a complete molar gestation. What is the best next step in the care of this patient’s
condition?
a. Repeat pelvic imaging
b. Chemotherapy
c. Radiation therapy
d. Surveillance of serum β-hCG
e. No further follow-up is required

96. How much calcium intake daily is recommended by for low risk pregnant women?
a. 300 mg
b. 1000 mg
c. 1500 mg
d. 1300 mg
e. 500 mg

97. A patient present to you with pain and swelling in the vulva. On examination you find a reddish
bulge on the vaginal introitus at 4 o'clock positions sized 3 cm, cystic and pain on palpation.
What treatment of choice for this condition?
a. Excision of the glands
b. Incision and drainage
c. Marsupialization
d. Cystectomy
e. Antibiotic for 7 day

98. A 45 years old present for evaluation because her primary care physician has diagnosed her with
pelvic organ prolapse while performing annual care. She denies any pelvic pressure, bulge, or
difficulty with urination. Her only medical comorbidity is obesity. For asymptomatic grade 1 pelvic
organ prolapse. What do you recommend?
a. Laser vaginal rejuvenation
b. Reconstructive surgery
c. Pelvic floor muscle exercises
d. Pessary
e. Weight loss program

99. A 32 years-old woman comes to your clinics due to shortness of breath, that worsen since 2 days
ago. On history taking, she told you that she had ever diagnosed of having significant mitral
stenosis. She is 33 weeks pregnant. The fetus is size-date appropriate. She has had a recent
echocardiography showing ejection fraction of 54% with moderate-severe pulmonary
hypertension.
What is the best management for this patient currently?
a. Induction of labor
b. Lung maturation and C section
c. Perform emergency C section
d. Conservative management until term pregnancy
e. Second stage acceleration

100.A 25-year old woman underwent a uterine curettage after a miscarriage and has had no menses
since. Pregnancy test was negative. Intrauterine adhesions were suspected. Which of the
following is associated with asherman syndrome?
a. Presence of monthly LH surge
b. Associated with low estradiol levels
c. Associated with a positive estrogen-progestin test
d. It is associated with class 1 anovulation
e. Associated with low cortisol levels

UNAS AGUSTUS 2020

1. A 34-year old women with primary infertility 3 years, oligomenorrhea and a body mass index
(BMI) of 26. Day 23 progesterone level result was 5 ng/ml. Transvaginal ultrasound shows
multiple small follicle size 5-8 mm in both ovary. HSG shows bilateral patent tubes. Her
partner’s semen analysis show a volume of 3 ml, pH of 7 and a sperm count of 20 million/ml.
What is the most appropriate step of management?
A. Examine TSH and prolactin
B. Measure FSH, LH and estradiol
C. Measure serum testosterone level
D. Give aromatase inhibitor
E. Start ovulation induction using gonadotropin

2. A 34-year old women with primary infertility 3 years, oligomenorrhea and a body mass index
(BMI) of 26. Day 23 progesterone level result was 5 ng/ml. Transvaginal ultrasound shows
multiple small follicle size 5-8 mm in both ovary. HSG shows bilateral patent tubes. Her
partner’s semen analysis show a volume of 3 ml, pH of 7 and a sperm count of 20 million/ml.
According to the current International Guidelines, which of the following medication is
considered to be the first line of therapy for ovulation induction?
A. Letrozole 1 x 2,5 mg
B. Clomiphene citrate starting at dose 50 mg/ day for 5 days
C. Clomiphene citrate 50 mg/day combined with metformin 2x500 mg
D. Metformin 2x500 mg
E. Gonadotropin injection 75IU/day

3. A 18-year-old adolescent female complains of not having started her menses. Her breast
development is Tanner stage IV, Pubic hair development was stage I. From vaginal
examination found a blind vaginal pouch and no uterus and cervix. Which of the following
describes the most likely diagnosis?
A. Partial androgen insensitivity syndrome
B. Complete androgen insensitivity syndrome
C. Kallman syndrome
D. Turner syndrome
E. Polycystic ovarian syndrome

4. A 18-year-old adolescent female complains of not having started her menses. Her breast
development is Tanner stage IV, Pubic hair development was stage I. From vaginal
examination found a blind vaginal pouch and no uterus and cervix. From ultrasound
examination found no uterus and there was difficulty in identifying the gonads. What is the
next plan?
A. Prolactin measurement
B. Kariotyping
C. FSH and LH examination
D. FSH, LH and E2 examination
E. TSH, fT4 examination

5. A 30 years old patient came with complaint of infertility. Her husband is a 33-year-old who
has had a semen analysis, which was reported as normal. On further history, the patient
reports that her periods have been quiet irregular over the last year and that she has not had
period in the last 3 months. She also reports insomnia, vaginal dryness, and decreased libido.
What is the most likely diagnosis for this patient based on her history
a. Polycystic ovarian syndrome
b. Primary ovarian insufficiency
c. Endometriosis
d. Kallmann syndrome
e. Spontaneous pregnancy

6. A 30 years old patient came with complaint of infertility. Her husband is a 33-year-old who
has had a semen analysis, which was reported as normal. On further history, the patient
reports that her periods have been quiet irregular over the last year and that she has not had
period in the last 3 months. She also reports insomnia, vaginal dryness, and decreased libido.
Which of the following condition that corresponds to the above possible diagnosis?
a. Day 3 FSH level 40 IU -> (N: < 7,0)
b. Serum AMH level 2,6 ng/ml -> (N: 1 – 2,6 ng/ml)
c. Positive Clomiphene citrate challenge test
d. Midluteal progesterone level of 15 ng/ml -> (N: 10-15 ng/ml)
e. Follicle antral basal count of 12 -> (N: 6-10)

7. Mrs. N, 37 years old with chief complain of infertility for 6 years with history of severe
dysmenorrhea. From hysterosalpingography, both tubes were non-patent. Pelvic ultrasound
found bilateral cystic mass with internal echo sized 50 and 60 mm in diameter. Her husband
sperm examination was within normal limit. What is the next appropriate management?
a. Offer her IUI
b. Perform laparoscopy cystectomy and adhesiolysis
c. Give GnRH analog for 3 months continue with IUI
d. Give Dienogest 1x2mg for 6 months
e. Offer her IVF

8. Mrs. N, 37 years old with chief complain of infertility for 6 years with history of severe
dysmenorrhea. From hysterosalpingography, both tubes were non-patent. Pelvic ultrasound
found bilateral cystic mass with internal echo sized 50 and 60 mm in diameter. Her husband
sperm examination was within normal limit. Her AMH level was 0,9 ng/ml. What is the
reason for performing surgery in subfertile patient with bilateral endometrioma and
diminished ovarian reserve?
a. Removal of endometrioma
b. Ablation of all endometriosis lesion
c. To prevent infection in endometrioma
d. To improve access for follicle aspiration
e. Removal of deep infiltrating endometriosis lesions

9. Mrs. N, 37 years old with chief complain of infertility for 6 years with history of severe
dysmenorrhea. From hysterosalpingography, both tubes were non-patent. Pelvic ultrasound
found bilateral cystic mass with internal echo sized 50 and 60 mm in diameter. Her husband
sperm examination was within normal limit. Which of the following is true regarding low
ovarian reserve in endometriosis?
a. Ovulation rate in ovary with endometrioma is higher compared to ovary without
endometrioma
b. There is a higher density of follicle in ovary with endometrioma
c. Ovary with endometrioma has a higher response rate to gonadotropin
d. Loss of ovarian stromal appearance and fibrosis are present in ovarian cortex with
endometrioma
e. Low ovarian reserve in endometriosis only happen after surgery

10. A 27-year-old woman presents to your office with a positive home pregnancy test and a 3-
day history of vaginal bleeding. She is concerned that she may be having a miscarriage. On
examination, the uterine fundus is at the level of the umbilicus. By her last period, she should
be around 8 weeks gestation. On pelvic examination, there is a moderate amount of blood
and vesicle-like tissue in the vaginal vault, and the cervix is closed. The lab then calls you to
say that her serum β-hCG result is greater than 1,000,000 mIU/mL. Which of the following is
the best next step in this patient’s evaluation?
a. Complete pelvic ultrasound
b. Determination of Rh status
c. Surgical intervention (suction curettage)
d. Methotrexate administration
e. Schedule a follow-up visit in 2 to 4 weeks to recheck a β-hCG level

11. A 27-year-old woman presents to your office with a positive home pregnancy test and a 3-
day history of vaginal bleeding. She is concerned that she may be having a miscarriage. On
examination, the uterine fundus is at the level of the umbilicus. By her last period, she should
be around 8 weeks gestation. On pelvic examination, there is a moderate amount of blood
and vesicle-like tissue in the vaginal vault, and the cervix is closed. The lab then calls you to
say that her serum β-hCG result is greater than 1,000,000 mIU/mL. The patient undergoes an
uncomplicated suction D&C. The pathology report is available the next day and is consistent
with a complete molar gestation. What is the best next step in the care of this patient’s
condition.
a. Repeat pelvic imaging
b. Radiation therapy
c. Chemotherapy
d. Surveillance of serum β-hCG
e. No further follow-up is required

12. A 7-year-old girl presents to her pediatrician with her parents who are concerned about her
early sexual development. She is developing breasts, axillary hair, and pubic hair, and they
are noticing body odor. A thorough clinical workup reveals the child has an irregular,
echogenic,
thickly septated ovarian mass on her left ovary. What type of tumor is responsible for this
child's clinical presentation?
a. Dysgerminoma (Malignant, produce LDH, grow rapidly, pelvic pain, rarely bilateral,
b. Embryonal carcinoma (produce AFP & hCG,
c. Sertoli-Leydig cell tumor (produce testosterone & androgen,
d. Endodermal sinus tumor (Produce AFP,
e. Granulosa-theca cell tumor (low grade malignancies, produce estrogen, “coffee-bean”
nuclei, “Call-Exner bodies”)

13. A 36 years old patient, P0, presents to your clinic for fertility workup. She had been married
for 2 years with regular intercourse. Her menstrual cycle is normal. Her general status was
normal. Vaginal examination revealed normal findings. Which of the following examination
that is not included in basic workup in the patient?
a. Hysterosalpingography
b. Ultrasonography
c. Semen analysis
d. Endometrial dating according to Noyes criteria
e. Mid luteal progesterone examination
14. A 36 years old patient, P0, presents to your clinic for fertility workup. She had been married
for 2 years with regular intercourse. Her menstrual cycle is normal. Her general status was
normal. Vaginal examination revealed normal findings. The following month she came back
with the result of hysterosalpingography (see the picture below)

What will be your next plan ?


a. Repeat HSG next month
b. Schedule diagnostic laparoscopy
c. Gives clomiphene citrate and plan for natural conception
d. Gives clomiphene citrate and plan for intrauterine insemination
e. Plan for IVF

15. A 52-year-old woman presents to your office. She complained about her sexual problems of
low self esteem, and difficulties of initiating sexual intercourse, vaginal dryness and pain
during intercourse. She has the history of 3 Full term normal vaginal delivery and she had
already menopause and she has no history of hereditary disease. She underwent the lab
investigation such RBG – 129 mg/dl; Hb 10.6 mg/dl, Urea 21; Creatinine 0.5, Chest x-ray and
pelvic ultrasound studies showed no abnormalities. What is your diagnosis
a. Sexual desire disorder -> kurang hasrat untuk memulai
b. Genital arousal disorder -> pengen coitus terus menerus
c. Vaginismus -> penis gak bisa masuk karena vagina spasme
d. Orgasmic disorder -> gak nyampe orgasme
e. All of above -> semuanya wes

16. A 52-year-old woman presents to your office. She complained about her sexual problems of
low self esteem, and difficulties of initiating sexual intercourse, vaginal dryness and pain
during intercourse. She has the history of 3 Full term normal vaginal delivery and she had
already menopause and she has no history of hereditary disease. She underwent the lab
investigation such RBG – 129 mg/dl; Hb 10.6 mg/dl, Urea 21; Creatinine 0.5, Chest x-ray and
pelvic ultrasound studies showed no abnormalities. The most possible cause of sexual
disorders of this patient is
a. Menopause
b. Multiparity
c. Alcohol uses
d. Sexual abuse
e. Pain

17. Mrs. 32-year old, P0, comes to your outpatient clinic due to her prolonged menstrual
duration. She reports her menstrual duration until 14 days and using 10 pads per day. She
feels fatigue easily. On physical examination, you palpate an irregularly enlarged uterus, non
tender with firm contour. Cervix appears to be hyperemic without mass appearance or other
abnormalities. By which mechanism does fibroid creates a hyperestrogenic environment
requisites for their growth?
a. Fibroid contains higher level of cytochrome P450 aromatase, which allows for conversion
of androgens to estrogen.
b. Fibroid converts more estradiol to estrone
c. Fibroid cells contain less density of estrogen receptors compared with normal myometrium
d. Increased adipose conversion of androgens to estrogen
e. All of the above

18. Mrs. 32-year old, P0, comes to your outpatient clinic due to her prolonged menstrual
duration. She reports her menstrual duration until 14 days and using 10 pads per day. She
feels fatigue easily. On physical examination, you palpate an irregularly enlarged uterus, non
tender with firm contour. Cervix appears to be hyperemic without mass appearance or other
abnormalities. What is the cause of necrotic and degenerative process in fibroids?
a. Mitotic activity
b. Limited blood supply within tumors
c. Chromosomal defects
d. Hyperperfusion
e. Cytogenetic mutations

19. A 45-year-old presents for evaluation because her primary care physician has diagnosed her
with pelvic organ prolapse while performing annual care. She denies any pelvic pressure,
bulge, or difficulty with urination. Her only medical comorbidity is obesity. For asymptomatic
grade 1 pelvic organ prolapse, what do you recommend?
a. Conservative management with pelvic floor muscle exercises and weight loss
b. Colpocleisis obliterative procedure
c. Gellhorn pessary
d. Round ligament suspension
e. Hysterectomy

20. A 62-year-old G2 P2 presents to the urogynecology clinic with complaints of urinary


incontinence. She has urinary urgency and can’t make it to the bathroom before leaking a
large amount of urine. She gets up two to three times per night to urinate. A urinalysis and
urine culture done 1 week ago at her PCP’s office are both negative. What is the most likely
diagnosis and appropriate treatment option for this type of urinary incontinence?
a. Stress incontinence, mid-urethral sling
b. Urgency incontinence, oxybutynin (anticholinergic medication)
c. Overflow incontinence, oxybutynin (anticholinergic medication)
d. Urinary fistula, surgical repair
e. Functional incontinence, bladder suspension

21. Your patient is a 13-year-old adolescent girl who presents with cyclic pelvic pain. She has
never had a menstrual cycle. She denies any history of intercourse. She is afebrile and her
vital signs are stable. On physical examination, she has age-appropriate breast and pubic hair
development and normal external genitalia. However, you are unable to locate a vaginal
introitus. Instead, there is a tense bulge where the introitus would be expected. You obtain a
transabdominal ultrasound, which reveals a hematocolpos and hematometra. What is the
most likely diagnosis?
a. Transverse vaginal septum
b. Longitudinal vaginal septum
c. Imperforate hymen
d. Vaginal atresia
e. Bicornuate uterus

22. An 18-year-old nulligravid woman presents to the student health clinic with a 4-week history
of yellow vaginal discharge. She also reports vulvar itching and irritation. She is sexually
active and monogamous with her boyfriend. They use condoms inconsistently. On physical
examination, she is found to be nontoxic and afebrile. On genitourinary examination, vulvar
and vaginal erythema is noted along with a yellow, frothy, malodorous discharge with a pH
of
6.5. The cervix appears to have erythematous punctuations. There is no cervical, uterine, or
adnexal tenderness. The addition of 10% KOH to the vaginal discharge does not produce an
amine odor. Wet prep microscopic examination of the vaginal swabs is performed. What
would you expect to see under microscopy?
a. Branching hyphae
b. Multinucleated giant cells
c. Scant WBC
d. Flagellated, motile organisms
e. Epithelial cells covered with bacteria

23. A 89-year-old female patient with multiple, serious medical comorbidities presents to discuss
options for treatment of her high-grade prolapse. The prolapse is externalized and becoming
ulcerated from friction against her undergarments. She cannot tolerate a pessary. Her main
priority is to “fix or get rid of this thing,” but her primary care provider has cautioned against
a lengthy or open abdominal procedure. She is not interested in future intercourse. What
can you offer this patient?
a. Nothing can be done
b. Open abdominal sacral colpopexy
c. Robot-assisted laparoscopic sacral colpopexy
d. Hysterectomy with anterior and posterior colporrhaphy, vault suspension.
e. Colpocleisis

24. A patient returns for a postoperative checkup 2 weeks after a total abdominal hyserectomy
for fibroids. She is distressed because she is having continous leakage of urine from the
vagina. Her leakage is essentially continous and worsens with coughing, laughing, or
movement. Given her history and physical, you perform both a methylene blue dye test,
which is negative and an indigo carmine test, which is positive. The most likely diagnosis is:
a. Rectovaginal fistula
b. Uretro vaginal fistula -> metilen blue positif, indigo (+)
c. Vesico vaginal fistula -> metilen blue positif, indigo (-)
d. Uretero vagina fistula -> metilen negative, indigo (+)
e. Impossible to distinguish

25. A 38 years old multi gravid woman complains of the painless loss of urine, beginning
immediately with coughing, laughing, lifting, or straining. Immediate cessation of the activity
stops the urine loss after only a few drops. This history is most suggestive of
a. Fistula
b. Stress incontinence
c. Urge incontinence
d. Urethral diverticulum
e. UTI

26. A 25 year old lady come with abnormal pap smear result. She underwent colposcopy
examination and the result is a acetowhite lesion with punctation and atypical vessels.
Biopsy result confirms CIN I with HPV DNA test positve. What do you suggest for patient ?
a. LEEP procedure
b. Reevaluation of HPV DNA
c. Cold knife conization
d. Repeat cytology in 12 months
e. Repeat cytology in 6 months

27. A 45 years old woman presents to your office for consultation regarding her symptoms of
menopause. She stopped having periods 13 months ago after TAH-BSO operation and is
having severe hot flushes. The hot flushes are causing her considerable stress. What should
you tell her regarding the psychological symptoms of the climacteric?
a. They are not related to her changing levels of estrogen and progesterone.
b. They commonly include insomnia, irritability, frustration, and malaise.
c. They are related to a drop in gonadotropin levels.
d. They are not affected by environmental factors.
e. They are primarily a reaction to the cessation of menstrual flow

28. A 45 years old woman presents to your office for consultation regarding her symptoms of
menopause. She stopped having periods 13 months ago after TAH-BSO operation and is
having severe hot flushes. The hot flushes are causing her considerable stress. Which of the
following is an absolute contraindication for hormonal therapy?
a. Diabetes mellitus
b. Coronary heart disease
c. Endometriosis
d. Impairment of liver function
e. Migraine

29. A 45 years old woman presents to your office for consultation regarding her symptoms of
menopause. She stopped having periods 13 months ago after BSO operation and is having
severe hot flushes. The hot flushes are causing her considerable stress. Which of the
following medication that you will give for hormonal therapy?
a. Estrogen only therapy
b. Biphasic combined oral contraception
c. Monophasic combined oral contraception
d. Triphasic combined oral contraception
e. Sequential estrogen-progestin therapy

30. A 25 years-old women G1 20 weeks of gestational age came to outpatient clinics with a mass
in perineum sized 1 cm flesh-colored and cauliflower like appearance. She also feels itchy
and discomfort during sexual intercourse. On speculum examination we can see a small
verrucous mass sized 0,5 cm on vaginal side wall. What is the most probable cause of this
condition?
a. Herpes simplex virus
b. HPV type 6 and 11
c. HPV type 16,18 -> Ca cervix
d. Syphilis
e. Molluscum contagiosum

31. A 25 years-old women G1 20 weeks of gestational age came to outpatient clinics with a mass
in perineum sized 1 cm flesh-colored and cauliflower like appearance. She also feels itchy
and discomfort during sexual intercourse. On speculum examination we can see a small
verrucous mass sized 0,5 cm on vaginal side wall. What is the recommended treatment for
this patient?
a. Podophyllum resin
b. Fluorouracil
c. Trichloroacetic acid
d. Imiquimod
e. Surgical excision
32. A patient present to you with pain and swelling in the vulva. On examination you find a
reddish bulge on the vaginal introitus at 4 o’clock positions sized 3 cm, cystic and pain on
palpation. Which of the following is the most common causative organism for formation of
this condition? Bartholin
a. Trichomonas Vaginalis
b. Bacterial vaginosis
c. Streptococcus epidermidis
d. Neisseria gonorrhoeae
e. Staphylococcus aureus

33. A patient present to you with pain and swelling in the vulva. On examination you find a
reddish bulge on the vaginal introitus at 4 o’clock positions sized 3 cm, cystic and pain on
palpation. What treatment of choice for this condition?
a. Antibiotic for 7 days -> Bartolinitis (tanpa massa kistik, demam)
b. Incision and drainage -> abses Bartolin (kistik dan nyeri)
c. Marsupialization -> kista Bartolin (kistik saja) dan abses berulang
d. Cystectomy
e. Excision of the glands

34. A 28 women present to your clinic with feeling of fullness in the vagina. On examination you
find a bluish cystic mass came from the right lateral forniks sized 6 cm. The bulge is not
tender on palpation. What is the most probable diagnosis of this condition?
a. Gartner’s duct cyst -> forniks vagina
b. Skene duct cyst -> dekat uretra
c. Bartholin cyst -> jam 4 dan jam 8
d. Vaginal inclusion cyst
e. Endometriosis cyst

35. A 28 women present to your clinic with feeling of fullness in the vagina. On examination you
find a bluish cystic mass came from the right lateral forniks sized 6 cm. The bulge is not
tender on palpation. What is the cause of this condition?
a. Blockage of the glands opening
b. Infection
c. Endometriosis
d. Remnant of Mullerian duct
e. Remnant of Wollfian duct

36. Mrs 21 years old came to outpatient clinic with primary amenorrhea. She has a complaint of
impaired sense of smell. Which of the following can be found during additional examination?
a. Breast tanner stage 3
b. Pubic hair tanner stage 3
c. Absent of uterus
d. FSH level of 30 IU/L
e. LH level 0.9 IU/L

37. Mrs 21 years old came to outpatient clinic with primary amenorrhea. She has a complaint of
impaired sense of smell. She is diagnosed with Kallman syndrome. She has a family history of
diabetes, currently her BMI is 29 kg/m2 with waist circumference of 90 cm. Which of the
following drugs can be used for ovulation induction in this condition?
a. Letrozole 1x2.5 mg for 5 days
b. Clomiphene citrate 1x50 mg for 5 days
c. Metformin 2x500 mg
d. Clomiphene citrate 1x50 mg combined with metformin 2x500 mg
e. Human menopausal gonadotropin injection

38. Mrs. 32-year old, P0, comes to your outpatient clinic due to her prolonged menstrual
duration. She reports her menstrual duration until 14 days and using 10 pads per day. She
feels fatigue easily, she denies to experience dysmenorrhea. On physical examination, you
palpate an irregularly enlarged uterus, non-tender with firm contour. Cervix appears to be
hyperemic without mass appearance or other abnormalities. What is the most likely
diagnosis?
a. AUB-P
b. AUB-A
c. AUB-L
d. AUB-M
e. AUB-C

39. Mrs. N, 37 years old had just undergone laparoscopic procedures. Her chief complaint was
infertility for 6 years with history of severe dysmenorrhea. From hysterosalpingography, both
tubes were non-patent. Pelvic ultrasound found bilateral cystic mass with internal echo sized
50 and 60 mm in diameter. Her husband sperm examination was within normal limit. From
laparoscopy findings, normal size uterus with adhesion of posterior part to colon and
Douglas pouch was also severely adhered. Both of ovaries were enlarged approximately 50 –
60 mm with severe adhesion at the right and left adnexa. After adhesiolysis, both of tubes
could be identified and were blocked. Chocolate fluid was spilled from the cysts. The
pathology of the cysts was endometriosis cyst. Which one is the TRUE statement ?
a. The patient should directly undergo surgery to solve her pain and infertility problem
b. Laparoscopic procedures should be done if infertility problem already more than 3 years
c. Laparoscopic cystectomy was done because the cyst have diameter 50 and 60 mm
d. GnRH agonist 1 time prior to surgery will give better results
e. Male factor and ovarian reserve should be measured prior to decision of surgery
40. A 25-year-old woman underwent a uterine curettage after a miscarriage and has had no
menses since. Pregnancy test was negative. Intrauterine adhesions were suspected. Which of
the following is associated with asherman syndrome?
a. Associated with low cortisol levels
b. Associated with low estradiol levels
c. It is associated with class 1 anovulation
d. Presence of monthly LH surge
e. Associated with a positive estrogen-progestin test

41. A 30-year-old P0 obese woman is noted to have irregular menses and hirsutism. On
ultrasound examination, there are many small follicles in both ovaries. Which of the following
is consistent with polycystic ovarian syndrome?
a. BMD showing osteopenia
b. Finding of a 9-cm right ovarian mass
c. Elevated 17-hydroxyprogesterone level
d. Elevated level of AMH produced by theca cells (harusnya granulosa)
e. Positive progestin challenge test

42. A 30-year-old P0 obese woman is noted to have irregular menses and hirsutism. On
ultrasound examination, there are many small follicles in both ovaries. She was diagnosed
with PCOS and receive combined oral contraception for menstrual regulation. She noticed a
decreased hirsutism after taking COC, what is the most probable mechanism?
a. Suppression of HPO axis
b. Increased level of SHBG
c. Resumption of ovulation
d. Suppression of prolactin secretion
e. Suppression of androgen receptor in the peripheral tissue

43. A 18 years old girl come to the emergency room with an abruptly pelvic pain for 2 days. She
had already started felt pain on and off for the past 2 weeks especially when she was moving.
She has also nausea and vomiting. She has no fever. She has regular menstrual cycle. From
the physical examination an intense of low abdominal pain was marked even more on
palpation. There was a muscle defence, on the right lower abdomen quadrant. She tends to
bend her abdomen a bit on the bed. On abdominal ultrasound reveal a mass measuring 12
cm in the largest diameter on her pelvic, characterized with multiple hyperechogenic
interfaces in a cystic mass in the right ovary, no blood flow could be detected in the cyst
except only in the one pole near the uterus. Uterus anteflexed within normal limit. Left ovary
within normal limit. There is no fluid in pelvic cavity. What would be the working diagnosis?
a. Functional cyst
b. Endometriosis
c. Ovarian fibroma
d. A torsion cyst
e. Ruptured cyst

44. What type of cyst that frequently associated with ovarian torsion?
a. Ovarian fibroma
b. Follicular cyst
c. Dermoid cyst
d. Endometrioma
e. Mucinous cyst

45. A parity 3 40-year-old woman complains of cyclical heavy and painful menstrual bleeding. On
examination she is found to have an enlarged globular uterus and a transvaginal sonography
revealed diffuse adenomyosis. She has completed her family and currently relies on condoms
for contraception. She smokes 10 cigarettes per day but is otherwise fit and well. Which
treatment would you consider most appropriate? Choose the single best answer.
a. LNG-IUS
b. COCP
c. Hysterectomy
d. GnRH analogue
e. Endometrial ablation

46. A 32-year-old woman has a pelvic ultrasound that bilateral 5 cm ‘kissing’ ovarian cysts in the
pouch of Douglas. Both of which contain diffuse, low-level echoes giving a solid ‘ground-
glass’ appearance. She reports severe dysmenorrhoea and dyspareunia. Which of the
following condition that can be found associated with this finding? Endometrioma
a. Fifty percent risk of malignant transformation
b. Normal level of Ca 125
c. Increased level of serum AMH
d. Increased level of He4
e. Adenomyosis in the posterior uterus

47. A 26-year-old woman complains of recurrent bouts of bacterial vaginosis (BV) despite
successful initial treatment. She does not douche or smoke and has been in a monogamous
relationship or 6 years. Recurrence of BV after initial treatment is common (up to 30
percent), which can be frustrating of the patient. Which of the following consistently
decreases recurrence rates and should be recommended to this patient?
a. Treatment of male partners
b. Use of acidiying vaginal gels
c. Probiotics and reintroduction of lactobacilli
d. No intervention consistently decreases recurrence
e. Treatment of long continous antibiotic
48. A 28-year-old woman is hoping to become pregnant soon. She is worried about her history of
acute pelvic inflammatory disease (PID) when in college 8 years ago. What is her
approximate risk of infertility due to this acute PID?
a. 15 percent
b. 25 percent
c. 35 percent
d. 55 percent
e. 75 percent

49. A 82 years old woman P6 came to outpatient clinic with chief complaint of bulging mass
protrudes from vagina since 3 months ago. The mass usually occurs during activity and also
when she defecate , and disappear when lying down. There were no difficulty in voiding and
defecation. No urinary leakage during coughing and sneezing. She is not sexually active. If on
the Pelvic Organ Prolapse Quantification examination result showing below, what is the
diagnosis of this patient?

Aa Ba C
+3 +4 +5
GH Pb TVL
5 2 8
Ap Bp D
0 0 +3

a. Uterine prolapse grade 2, cystocele grade 2, rectocele grade 1


b. Uterine prolapse grade 4, cystocele grade 3, rectocele grade 2
c. Uterine prolapse grade 3, cystocele grade 3, rectocele grade 2
d. Uterine prolapse grade 3, cystocele grade 2, rectocele grade 2
e. Uterine prolapse grade 4, cystocele grade 4, rectocele grade 3

50. Patient 65 years old, P4 came to outpatient clinic with chief complaint of frequent urination.
Since 6 months ago she feels the urge to void every hour and also she has to wake up 3-4
times in the night to void. She never leaks urine. She doesn’t feel any pain during urination
and no blood in the urine. She already came to general practitioner and got antibiotics for 7
days but the symptoms remained. What is the most useful supporting examination in this
patient?
a. Urine culture
b. Pelvic floor ultrasound
c. Gynecology ultrasound
d. Bladder diary
e. Urodynamic evaluation

51. A 26-year-old G2P1 (no live child) is seen for her first prenatal visit at 18 weeks’ gestation by
menstrual history. Her first child was born at 28 weeks spontaneously, the baby was died
after
hospitalized for 1 month in NICU. She is worried this pregnancy also will be ended with
spontaneous preterm birth. What is the most accurate examination that can be done at 18
weeks to predict the risk of preterm birth?
a. Cell-free fetal DNA -> sindrom, dicek Trimester 1
b. IGFBP-1 examination
c. PAMG-1 examination
d. Fibronectin examination
e. Measure cervical length

52. A 23 year-old G1 32 weeks is being admitted to the hospital because of preterm contraction.
The patient complaint regular contraction. Antenatal care was done regularly in PHC. No
remarkable abnormality was found during ANC. BMI before pregnancy was 30 kg/m 2, weight
gain during pregnancy is 14 kg. Abdominal examination showed FUT 36 cm, regular
contractions, fetal heart beats 154 bpm. Speculum examination showed closed ostium uteri
externa. An ultrasound shows the estimated fetal weight 2400 g, AFI 30 cm, no fetal
morphology abnormalities, placenta implanted in anterior corpus, cervical length 1.8 cm,
funneling positive. Laboratory results were Hb 10,7 g/dL, Ht 33%, Leucocyte 13.500,
Thrombocyte 315.000 MCV 82 MCH 30. What is the most likely etiology of preterm
contraction in this case?
a. Maternal obesity
b. Maternal anemia
c. Bacterial vaginosis
d. Uterine overdistention
e. Excessive gestational weight gain

53. A 23 year-old G1 32 weeks is being admitted to the hospital because of preterm contraction.
The patient complaint regular contraction. Antenatal care was done regularly in PHC. No
remarkable abnormality was found during ANC. BMI before pregnancy was 30 kg/m 2, weight
gain during pregnancy is 14 kg. Abdominal examination showed FUT 36 cm, regular
contractions, fetal heart beats 154 bpm. Speculum examination showed closed ostium uteri
externa. An ultrasound shows the estimated fetal weight 2400 g, AFI 30 cm, no fetal
morphology abnormalities, placenta implanted in anterior corpus, cervical length 1.8 cm,
funneling positive. Laboratory results were Hb 10,7 g/dL, Ht 33%, Leucocyte 13.500,
Thrombocyte 315.000 MCV 82 MCH 30. What is the most appropriate next step in the
management of this patient?
a. Give intravenous iron
b. Schedule for OGTT test
c. Give antibiotic prophylaxis
d. Schedule for cervical cerclage
e. Give MgSO4 for neuroprotection

54. A 18-year-old G1 at 30 4/7 weeks presents for her scheduled obstetric (OB) appointment. A
28-week ultrasound showed the fetus to be in the 13th percentile for estimated fetal weight.
The patient denies any complaints today. Fetal movement is active. BMI before pregnancy
was
19.6 kg/m2, gestational weight gain is 7 kg. Vital sign are normal. Abdominal examination
today shows a gravid uterus measuring 27 cm. Fetal heart tones (FHTs) are in the 140s.
Laboratory results were Hb 10,1 g/dL, Ht 30%, Leucocyte 10.500, Thrombocyte 215.000,
MCV 78, MCH
28. OGTT result was fasting 92 mg/dL and 2 hours after 75 glucose 148 mg/dL. What is the
appropriate next step in the management of this patient? (MCV range: 85-95. MCH range:
27- 33,2)
a. Schedule for serum ferritin test
b. Perform fetal growth ultrasound
c. Schedule for a biophysical profile (BPP)
d. Give elemental iron 100-200 mg per oral
e. Admit patient to the hospital for lung maturation

55. A 18-year-old G1 at 30 4/7 weeks presents for her scheduled obstetric (OB) appointment. A
28-week ultrasound showed the fetus to be in the 13th percentile for estimated fetal weight.
The patient denies any complaints today. Fetal movement is active. BMI before pregnancy
was
19.6 kg/m2, gestational weight gain is 7 kg. Vital sign are normal. Abdominal examination
today shows a gravid uterus measuring 27 cm. Fetal heart tones (FHTs) are in the 140s.
Laboratory results were Hb 10,1 g/dL, Ht 30%, Leucocyte 10.500, Thrombocyte 215.000,
MCV 78, MCH
28. OGTT result was fasting 92 mg/dL and 2 hours after 75 glucose 148 mg/dL. What is the
most likely predisposition factor for this case?
a. Maternal anemia
b. Genetic abnormalities
c. Placental insufficiency
d. Inadequate maternal nutrition
e. Pre-gestational diabetes mellitus

56. A 35-year-old woman, G3P2, presents to labor and delivery (L&D) at 33-week gestation
referred by midwife with BP 180/110 mmHg. BP on arrival is 170/105 mmHg. Urine protein is
1+ on dipstick. Patient had history of high blood pressure in her previous pregnancy. ANC
was done in midwife. Blood pressure at first trimester was 130-145/90-95 mmHg, urine
protein was negative on dipstick. No antihypertension drug was given. The patient denies
any complaints today. What is the most likely diagnosis of the patient:
a. Preeclampsia
b. Chronic hypertension
c. Gestational hypertension
d. Superimposed preeclampsia
e. Preeclampsia with severe feature
57. A 29 year old woman with a positive pregnancy test presents with a good history of tissue
expulsion vaginally passing tissue per vagina. A transvaginal ultrasound scan shows an empty
uterus with an endometrial thickness of 11 mm. Regarding her diagnosis, you consider that :
a. She has had a complete miscarriage and needs no further treatment
b. She has had a pregnancy of unknown location and needs further investigations
c. She should be offered a hysteroscopy
d. She should be offered medical management of miscarriage
e. A laparoscopy should be performed to exclude an ectopic pregnancy

58. Mrs A, 26-years-old, G1P0A0, according to her last LMP is 34 weeks pregnant, came for her
first antenatal care. She said that she had 20 kg of weight gain during her pregnancy with
swelling ankles for the past 4 weeks. She never took any iron or vitamin supplementation.
From the physical findings, BP 145/95 mmHg, HR 86x/min, RR 20x/min, BMI 35 kg/m2.
Ultrasound examination confirmed twins in breech presentation. Results from urinalysis
were as follows: color cloudy yellow, spesific gravity 1.013, albumin 2+, RBC 0 - 1, WBC 2-5,
negative bacterial count. What is the most likely diagnosis?
A. Acute fatty necrosis of the liver
B. Chronic hypertension
C. Preeclampsia
D. Renal disease
E. Pyelonephritis

59. Mrs. B, 37-years-old came to your office at 32 weeks of gestation according to her last
menstrual period. She has no ultrasound examination before and did not do her routine
antenatal care. The vital sign is within normal limit. She has body mass index 19 kg/m2.
During physical examination, the uterine fundal height is 22 cm. From ultrasound
examination, the fetus has biometric values that correlate with 30 weeks fetus. Which of the
following is the next best step in managing this patient?
A. Antenatal care routinely for the next 2 weeks
B. Evaluate maternal status and comorbidities
C. Consider deliver the baby
D. Repeat sonography for fetal growth in 2 weeks
E. Doppler velocimetry evaluation every 3 days

60. Mrs E, 32 yo referred from midwife with antepartum hemorrhage. She is G3P2 term
pregnancy. On examination her blood pressure is 160/100 mmHg, HR 100 bpm. She looks
anemic, not icteric. Obstetrical examinations reveal contraction 4-5x/10 minutes, FHR 170
bpm, head presentation 3/5. After thorough examination it is concluded that there is a
placental abruption with retroplacental hematoma size 6x5 cm. This patient is planned to do
cesarean section. If during operation the uterus is couvelaire but with good contraction, how
would you manage that condition?
A. Perform prophylactic b-lynch suture
B. Ascending uterine artery ligation
C. Hypogastric artery ligation
D. Sub total hysterectomy
E. Uterotonic and observation

61. Mrs E, 32 yo referred from midwife with antepartum hemorrhage. She is G3P2 term
pregnancy. On examination her blood pressure is 160/100 mmHg, HR 100 bpm. She looks
anemic, not icteric. Obstetrical examinations reveal contraction 4-5x/10 minutes, FHR 170
bpm, head presentation 3/5. After thorough examination it is concluded that there is a
placental abruption with retroplacental hematoma size 6x5 cm. This patient is planned to do
cesarean section. Postoperative period is very crucial in this patient. Which of the following is
not included as a parameter needed to be evaluated in early warning system.
A. Blood pressure
B. Heart rate
C. Urine production
D. Central venous pressure
E. All of the above

62. A 26-years-old woman, G1P0A0 was admitted to ER because she lost her consciousness
around 1 hour ago. According to her husband, she is 36 weeks pregnant. She performed
antenatal care at scheduled time, and never missed one. Her husband said, she never had
any hypertension or any other disease before. Three days prior hospitalization, she had
severe nausea and vomiting. Physical examination reveals, BP 120/80 mmHg, pulse rate 87
x/min, RR 18x/min, Temperature 36.50C. You notice there is an icteric sclera. Other physical
examination was remarkable. Obstetrical examination reveals no fetal heartbeat was
detected. Laboratory examination reveals CBC 10.2/29.9/8900/263.000; Ur/Cr 18/0,8;
AST/ALT 458/878; RBG 32; Urinalysis was within normal limit. What is the best next
management in this case?
A. Abdominal ultrasound
B. Induction of labor
C. Emergency Caesarean section
D. Whole Blood transfusion
E. Injections of 40% Dextrose

63. Which of the following is not included in clinical characteristics that increase the risk for
acute fatty liver of pregnancy?
A. Nulliparity
B. Female fetus
C. Male fetus
D. Twin gestation
E. Third trimester
64. What is the underlying pathophysiology of intrahepatic cholestasis of pregnancy?
A. Acute hepatocellular destruction
B. Incomplete clearance of bile acids
C. Microvascular thrombus accumulation
D. Eosinophil infiltration of the liver
E. Hepatocellular injury

65. A 17-year-old G1P0 woman presents at 25 weeks’ gestation complaining of headache for the
past 36 hours. She has had regular prenatal visits going back to her first prenatal visit at 8
weeks’ gestation. A 20- week ultrasound redated her pregnancy by 2 weeks as it was 15 days
earlier than her LMP dating. She has a BP of 155/104 mm Hg. You review her medical record
and determine that she does not have chronic hypertension. The patient denies having RUQ
pain but because of your high suspicion of severe preeclampsia you order a CBC, liver
enzymes, renal function test, and a 24-hour urine protein collection. Her laboratory test
results reveal a normal platelet count and liver enzymes but a slightly elevated creatinine
and proteinuria of 550 mg in 24 hours. Her headache has resolved after a dose of
acetaminophen. What is the next best step in her management?
A. Give her a prescription for labetalol and have her follow-up in clinic in 2 weeks
B. (a) plus bed rest
C. Hospitalization for further evaluation and treatment
D. Immediate delivery
E. Begin induction of labor

66. A 17-year-old G1P0 woman presents at 25 weeks’ gestation complaining of headache for the
past 36 hours. She has had regular prenatal visits going back to her first prenatal visit at 8
weeks’ gestation. A 20- week ultrasound redated her pregnancy by 2 weeks as it was 15 days
earlier than her LMP dating. She has a BP of 155/104 mm Hg. Over the next 12 hours, her
BP’s rise above 160 mmHg on several occasions, most notably to 174/102 mmHg 2 hours
after admission and to 168/96 mmHg 9 hours after admission. Her headache does not return
and she has no RUQ pain or visual symptoms. A set of repeat laboratory test results are
unchanged and by increasing her labetalol dose to 400 mg TID, her BP’s decrease to 140s–
150s/70–90 mmHg. She is also started on magnesium sulfate. What change in physical or
laboratory examination do you observe that would indicate delivery?
A. Another BP of 174/102 mmHg
B. Headache returning
C. Double vision
D. Platelets of 108
E. AST of 265

67. A 35-year-old woman, G4P3, at 37 weeks gestation presented in hospital with a ten-day
history of low extremities edema, with idiopathic hypertension for 1 year. At presentation,
she had a blood pressure of 170/100 mmHg. Laboratory findings were normal except
urinalysis (protein 2+). She was diagnosed with superimposed severe preeclampsia. It was
decided to deliver the
fetus by means of a C-section by indication transverse lie. Blood pressure measurement was
150/100 mmHg. She lost consciousness for 30 seconds five hours after operation. The
laboratory studies gave the following results: serum aspartate aminotransaminase (AST),
225 IU/L; serum alanine aminotransaminase (ALT), 140 IU/L; serum lactate dehydrogenase
(LDH), 1017 IU/L; serum urea and creatine were normal; hemoglobin, 10.6 mg/dL; platelet
count, 50 × 103 μ/mL. A brain computed tomography (CT) scan was performed on patient
which revealed the left frontal lobe lacunar infarction. The patient was transferred to
intensive care unit. What is the most appropriate diagnosis.
A. DIC
B. Acute Fatty Liver in Pregnancy
C. HELLP Syndrome
D. Severe puerpural infection
E. Thrombotic thrombositopenic Purpura

68. A 35-year-old woman, G4P3, at 37 weeks gestation presented in hospital with a ten-day
history of low extremities edema, with idiopathic hypertension for 1 year. At presentation,
she had a blood pressure of 170/100 mmHg. Laboratory findings were normal except
urinalysis (protein 2+). She was diagnosed with superimposed severe preeclampsia. It was
decided to deliver the fetus by means of a C-section by indication transverse lie. Blood
pressure measurement was 150/100 mmHg. She lost consciousness for 30 seconds five
hours after operation. The laboratory studies gave the following results: serum aspartate
aminotransaminase (AST), 225 IU/L; serum alanine aminotransaminase (ALT), 140 IU/L;
serum lactate dehydrogenase (LDH), 1017 IU/L; serum urea and creatine were normal;
hemoglobin, 10.6 mg/dL; platelet count, 50 × 103 μ/mL. A brain computed tomography (CT)
scan was performed on patient which revealed the left frontal lobe lacunar infarction. The
patient was transferred to intensive care unit. What is the best management after, for this
case.
A. Fresh-frozen plasma and trombocytes concentrates
B. Anti-platelets
C. Anti-oxidant
D. Corticosteroid
E. Magnesium sufate

69. A 35-year-old woman, G4P3, at 37 weeks gestation presented in hospital with a ten-day
history of low extremities edema, with idiopathic hypertension for 1 year. At presentation,
she had a blood pressure of 170/100 mmHg. Laboratory findings were normal except
urinalysis (protein 2+). She was diagnosed with superimposed severe preeclampsia. It was
decided to deliver the fetus by means of a C-section by indication transverse lie. Blood
pressure measurement was 150/100 mmHg. She lost consciousness for 30 seconds five
hours after operation. The laboratory studies gave the following results: serum aspartate
aminotransaminase (AST), 225 IU/L; serum alanine aminotransaminase (ALT), 140 IU/L;
serum lactate dehydrogenase (LDH), 1017 IU/L; serum urea and creatine were normal;
hemoglobin, 10.6 mg/dL; platelet count, 50 × 103 μ/mL. A brain computed tomography
(CT) scan was performed on patient
which revealed the left frontal lobe lacunar infarction. The patient was transferred to intensive
care unit. During twelve hours observation showed urine production was 100 ml.
A. Immediately giving diuretics bolus iv.
B. Immediately giving diuretics maintained by syringe-pump.
C. Check albumin level, giving diuretic justified after hipoalbuminemia condition had been
confirmed.
D. Renal failure due to micro thrombopathy suspected, heparin provision could be
considered
E. Immediately step on fluid rescucitation

70. A primigravida at 36 weeks gestation is measuring large for dates. Ultrasound shows AC > 97
th centile. GTT performed shows poorly controlled gestational diabetes. What is the
immediate management plan?
A. Give steroids
B. Start induction
C. Start hypoglycaemics agent
D. Wait and watch
E. Start sliding scale and deliver

71. A 36 year old G2P1 presents to the antenatal clinic. She had an emergency caesarean section
for sudden onset hypertension and placental abruption at 30 weeks in her previous
pregnancy. She is currently 20 weeks of gestation and enquires about further plan of fetal
monitoring in this pregnancy. What is the most appropriate advice?
A. No extra monitoring is required
B. Uterine artery Doppler at 22 weeks
C. Serial scans starting from 24 weeks
D. Serial cardiotocograph monitoring from 28 weeks
E. Serial scans from 28 weeks

72. A 34 year old pregnant lady, G2P1 has been diagnosed with ductal carcinoma of the right
breast (Stage1). She is currently 22 weeks pregnant. What is the initial treatment of choice
for her?
A. Termination of pregnancy
B. Local mastectomy with reconstruction
C. Local mastectomy without reconstruction
D. Local radiotheraphy
E. Single dose chemotheraphy with trastuzumab

73. A triple test is performed for Down’s screening at 16 weeks in a 40-year old woman. The
result suggests a high risk of trisomy 21. What would the results typically show?
A. Reduced AFP, reduced estriol, increased β-hCG
B. Increased AFP, reduced estriol, increased β-hCG
C. Reduced AFP, increased estriol, increased β-hCG
D. Reduced AFP, increased estriol, reduced β-hCG
E. Increased AFP, increased estriol, increased β-hCG

74. Women with one or more previous caesarean section scars and an anterior placenta are at of
placenta accreta. Which test has been shown in recent research to provide the highest
sensitivity and specificity for antenatal diagnosis of placenta accreta?
A. Colour Doppler
B. 3D Power Doppler
C. Contrast CT
D. Gadolinium Contrast MRI
E. Grey Scale Ultrasound

75. A 22 year old unbooked primigravida presents to the Emergency Department at 26 weeks of
gestation with a history of spontaneous painless bleeding af about 500 ml. What is the best
investigation to secure a diagnosis?
A. MRI scan
B. Transabdominal scan
C. CTG
D. Transvaginal scan
E. CT scan

76. A primigravida at 35 weeks of gestation presents with pain in the right hypochondrium and
right side of her back. There is no history of nausea or vomiting, hypertension, urinary
symptoms and bowel problems. Vital signs: pulse-106, temperature 38.1, BP 128/75.
Abdominal examination is unremarkable. Chest is clear. Fetal monitoring is normal. Urine
shows 2+ leucocytes and 1+ blood. White cell count 16 x 10 9/L. What is the most likely
diagnosis?
A. Appendicitis
B. Cholecystitis
C. Pyelonephritis
D. Abruption
E. Right basal pneumonitis

77. A 25-year-old G1P0 presents to the emergency room with vaginal bleeding. Her last normal
menstrual period was 6 weeks earlier. She reports that she is sexually active with male
partners and does not use any hormonal or barrier methods for contraception. On arrival,
her temperature is 37°C, blood pressure is 115/80, pulse is 75 beats per minute, respiratory
rate is 16 breaths per minute, and she has 100% oxygen saturation on room air. A pelvic
examination reveals a small amount of dark blood in the vagina. The external cervical os
appears 1 to 2 cm dilated. Her uterus is mildly enlarged, anteverted, and nontender. A urine
pregnancy test is positive. A pelvic ultrasound is obtained and shows an intrauterine
gestational sac with a yolk sac. No fetal pole or cardiac motion is seen. Bilateral adnexa are
normal. What is her diagnosis?
A. Incomplete abortion
B. Threatened abortion
C. Ectopic pregnancy
D. Missed abortion
E. Inevitable abortion

78. During a routine return OB visit, an 18-year-old G1P0 patient at 23 weeks gestational age
undergoes a urinalysis. The dipstick done by the nurse indicates the presence of trace
glucosuria. All other parameters of the urine test are normal. Which of the following is the
most likely etiology of the increased sugar detected in the urine?
A. The patient has diabetes
B. The patient has a urine infection
C. The patient’s urinalysis is consistent with normal pregnancy
D. The patient’s urine sample is contaminated
E. The patient has kidney disease

79. A maternal fetal medicine specialist is consulted and performs an indepth sonogram. The
sonogram indicates that the fetuses are both male, and the placenta appears to be
diamniotic and monochorionic. Twin B is noted to have oligohydramnios and to be much
smaller than twin A. In this clinical picture, all of the following are concerns for twin A except.
A. Congestive heart failure
B. Anemia
C. Hypervolemia
D. Polycythemia
E. Hydramnion

80. You are called in to evaluate the heart of a 19-year-old primigravida at term. Listening
carefully to the heart, you determine that there is a split S1, normal S2, S3 easily audible with
a 2/6 systolic ejection murmur greater during inspiration, and a soft diastolic murmur. You
immediately recognize that
A. The presence of the S3 is abnormal
B. The systolic ejection murmur is unusual in a pregnant woman at term
C. Diastolic murmurs are rare in pregnant women
D. The combination of a prominent S3 and soft diastolic murmur is a significant abnormality
E. All findings recorded are normal changes in pregnancy

81. A 25-year-old woman in her first pregnancy is noted to have prolonged first and second
stages of labour. She was induced at 38 weeks’ pregnancy. The baby was delivered by
forceps. After delivery the placenta she is noted to have heavy vaginal bleeding.
Abdominal examination
demonstrates a relaxed uterus. What should we do if the fundus not firm after placental
delivery.
A. Methylergonovine (Methergine)
B. Carboprost (Hemabate, PGF2-alpha)
C. Fundal Massage
D. Misoprostol (PGE1)
E. Dinoprostone-prostaglandin E2

82. A 30-year-old multiparous woman has rapid delivery soon after arriving in emergency room.
After delivery the placenta she is noted to have heavy vaginal bleeding. Help has been
summoned. Abdominal examination demonstrates the fundus was soft. What is the most
appropriate next step?
A. Intravenous access for fluid resuscitation
B. Uterine packing
C. Balloon tamponade
D. Suture the laceration
E. Misoprostol administration

83. A 30-year-old multiparous woman has rapid delivery soon after arriving in emergency room.
After delivery the placenta she is noted to have heavy vaginal bleeding. Help has been
summoned. Abdominal examination demonstrates the fundus was soft. After use of a 20
units of oxytocin in 1000 mL of crystalloid solution to increase the tone of her uterus stop the
bleeding; however, you continue to notice a massive bleeding from the vagina. What is the
most appropriate next step in the evaluation of this patient’s bleeding?
A. Perform a bedside ultrasound for retained products of conception
B. Perform a bedside ultrasound to look for blood in the abdomen significant for uterine
rupture
C. Perform a manual exploration of the uterine fundus and exploration for retained clots or
products
D. Examine the perineum and vaginal for laceration during delivery
E. Consult interventional radiology for uterine artery embolization

84. A 32-years-old woman comes to your clinics due to shortness of breath, that worsen since 2
days ago. On history taking, she told you that she had ever diagnosed of having significant
mitral stenosis. She is 33 weeks pregnant. The fetus is size-date appropriate. She has had a
recent echocardiography showing ejection fraction of 54% with moderate-severe pulmonary
hypertension. What is the best management for this patient currently?
A. Perform emergency C section
B. Lung maturation and C section
C. Conservative management until term pregnancy
D. Second stage acceleration
E. Induction of labor

85. What is the most common cause of heart failure during pregnancy and the puerperium?
A. Chronic hypertension with severe preeclampsia
B. Viral myocarditis
C. Obesity
D. Valvular heart disease
E. Pulmonary Artery Hypertension

86. For patients with congenital heart disease, what is the most common adverse cardiovascular
event encountered in pregnancy?
A. Heart failure
B. Arrhythmia
C. Thromboembolic event
D. Cerebrovascular hemorrhage
E. Heart axis changes

87. A 24-year-old patient, P2, has just delivered vaginally an infant weighing 3000 g after a
spontaneous uncomplicated VBAC. Her prior obstetric history was a low uterine segment
transverse cesarean section for breech. She has had no problems during the pregnancy and
labor. The placenta delivers spontaneously. There is immediate vaginal bleeding of greater
than 500 cc. Although all of the following can be the cause for postpartum hemorrhage,
which is the most frequent cause of immediate hemorrhage as seen in this patient?
A. Uterine atony
B. Coagulopathies
C. Uterine rupture
D. Retained placental fragments
E. Vaginal and/or cervical lacerations

88. A 22-year-old G1A0 at 10 weeks presents for her scheduled obstetric (OB) appointment.
Laboratory examination showed hemoglobin 11.2 g/dL, hematocrit 34%, MCV 86 fl MCH 32
pg. Which of the following shows iron deficiency anemia:
A. Low ferritin, normal serum iron
B. Low ferritin, serum iron and transferrin
C. Normal hemoglobin level, low ferritin and serum iron
D. Microcytic hypochromic, low serum iron and transferrin saturation
E. Microcytic hyperchromic, low serum iron and transferrin saturation

89. What is total iron need during pregnancy?


A. 500 mg
B. 750 mg
C. 1000 mg
D. 1500 mg
E. 2000 mg

90. How much iron is recommended by WHO for pregnant women in Indonesia? (prevalence of
anemia in Indonesia based on RISKESDAS 48%)
A. 27 mg
B. 30 mg
C. 60 mg
D. 100 mg
E. 200 mg

91. A 22-year-old primiparous woman presents for her first prenatal evaluation. On physical
examination you hear a grade 3/6 pansystolic murmur. Which is the most common CHD in
pregnancy that would cause that type of murmur?
A. Aortic stenosis
B. Pulmonary stenosis
C. Atrial septal defect (ASD)
D. Patent ductus arteriosus (PDA)
E. Ventricular septal defect (VSD)

92. Numerous physiologic changes develop over the course of pregnancy; however the greatest
impact on potentially compromised cardiovascular system is:
A. The red cell mass rises by 40%
B. Increased 30% total plasma volume in 28 weeks
C. Increase in cardiac output occurs by 24 weeks
D. Decreased SVR in the first trimester, and increase after 32 weeks
E. Cardiac output increases to 30-50% above pre-pregnancy levels by the end of the third
trimester

93. A 28-year-old G1 at 26 weeks present for her scheduled obstetric appointment. You ordered
OGTT examination that shows fasting blood glucose 102 mg/dL and 2 hours after 75 g oral
glucose 185 mg/dL. Her gestational weight gain during pregnancy is 12 kg. Her BMI before
pregnancy was 26 kg/m2. What is the most likely diagnosis?
A. Normal OGTT
B. Gestational diabetes
C. Impaired glucose test
D. Diabetes melitus type 1
E. Diabetes mellitus type 2
94. A 28-year-old G1 at 26 weeks present for her scheduled obstetric appointment. You ordered
OGTT examination that shows fasting blood glucose 102 mg/dL and 2 hours after 75 g oral
glucose 185 mg/dL. Her gestational weight gain during pregnancy is 12 kg. Her BMI before
pregnancy was 26 kg/m2. What is the appropriate next step in the management of this
patient?
A. Schedule nonstress test (NSTs)
B. Schedule fetal growth ultrasound
C. Admit to hospital for fetal monitoring
D. Advise insulin in order to lowering blood glucose
E. Schedule routine antenatal care in 4 weeks

95. A 28-year-old G1 at 26 weeks present for her scheduled obstetric appointment. You ordered
OGTT examination that shows fasting blood glucose 102 mg/dL and 2 hours after 75 g oral
glucose 185 mg/dL. Her gestational weight gain during pregnancy is 12 kg. Her BMI before
pregnancy was 26 kg/m2. What is the most likely fetal consequences cause by patient’s
condition?
A. Spina bifida
B. Fetal hypoxia
C. Intrauterine growth restriction
D. Fetal large for gestational age
E. Congenital valvular heart disease

96. A 28-year-old G2P1 is seen for her first prenatal visit at 16 weeks’ gestation by menstrual
history. Her first child was born at 32 weeks spontaneously. She is worried this pregnancy
also will be ended with spontaneous preterm birth. What is the most accurate examination
that can be done at 16 weeks to predict the risk of preterm birth?
A. IGFBP-1 examination
B. Fibronectin examination
C. Measure cervical length
D. Vaginal swab to exclude bacterial vaginosis
E. Urinary test to exclude urinary tract infection

97. What is the next appropriate management for the patient?


A. Progesterone prophylaxis
B. Schedule for cervical cerclage
C. Treat asymptomatic bacterial vaginosis
D. Give tocolytics for inhibiting preterm labor
E. Intervention modifiable risk factors for preterm

98. What is the implantation of a placenta in which there is a defect in the fibrinoid layer at the
implantation site, allowing the placental villi to invade and penetrate into but not through
the myometrium called?
A. Placenta accreta
B. Placenta increta
C. Placenta percreta
D. Placental infarct
E. Placenta previa

99. A 21-year-old G1 P0 patient presents to your office with vaginal bleeding at approximately 8
weeks’ gestation by her last menstrual period. Her examination is benign with a 9-week-sized
uterus, a closed cervical os, and a small amount of blood within the vaginal vault. You order a
complete pelvic ultrasound that shows an intrauterine gestational sac containing a fetus
measuring approximately 7 weeks’ gestation. Doppler sonography is unable to demonstrate
any fetal heartbeat. What is the most likely diagnosis?
A. Ectopic pregnancy
B. Complete abortion
C. Embryonic demise
D. Incomplete abortion
E. Threatened abortion

100. A 21-year-old G1 P0 patient presents to your office with vaginal bleeding at approximately 8
weeks’ gestation by her last menstrual period. Her examination is benign with a 9-week-
sized uterus, a closed cervical os, and a small amount of blood within the vaginal vault. You
order a complete pelvic ultrasound that shows an intrauterine gestational sac containing a
fetus measuring approximately 7 weeks’ gestation. Doppler sonography is unable to
demonstrate any fetal heartbeat. You decide to perform a suction D&C. When giving
informed consent, you discuss the risk most commonly encountered in this operation.
Which of the following is the most common risk associated with suction D&C?
A. Infection
B. Uterine perforation
C. Damage to the bladder
D. Uterovaginal bleeding
E. Need for future surgery

Sampun nggih.. matur


nuwun..
UNAS MEI 2020

1. A 28-year old G1 at 26 weeks present for her scheduled obstetric appointment. You ordered OGTT
examination that shows fasting blood glucose 102 mg/dl and 2 hours after 75 g oral glucose 185
mg/dl. Her gestational weight gain during pregnancy is 12 kg. Her BMI before pregnancy was 26
kg/m.
What is appropriate next step in the management of this patient?
a. Schedule routine antenatal care in 4 weeks
b. Schedule NST
c. Advise insulin order to lowering blood glucose
d. Admit to hospital for fetal monitoring
e. Schedule fetal growth ultrasound

2. A 18-year old young woman presents to you with a complaint of amenorrhea. She notes that she
has never had menstrual period, but that she has mild cyclic abdominal bloating. She is sexually
active, but she complains of painful sexual intercourse. Her past medical and surgical history is
unremarkable. On physical examination, you note normal appearing axillary and pubic hair. Her
breast development is normal. Pelvic examination reveals normal appearing external genitalia,
and a shortened vagina ending in a blind pouch. From further examination it was found that
uterus cannot be visualized, but both of ovaries were normal. What is the most likely diagnosis?
a. Mullerian agenesis
b. Androgen insensitivity syndrome
c. Gonadal dysgenesis
d. Imperforate hymen
e. Transverse vaginal septum

3. A 21- year old G1 P0 patient presents to your office with vaginal bleeding at approximately 8
weeks gestation by her last menstrual period. Her examination is benign with a 9 weeks sized
uterus, a closed cervical os, and a small amount of blood within the vaginal vault. You order a
complete pelvic ultrasound that shows an intrauterine gestational sac containing a fetus
measuring approximately 7 weeks gestation. Doppler sonography is unable to demonstrate any
fetal heartbeat.
Which of the following is the common chromosomal abnormality found in tissue from first
trimester spontaneous abortion?
a. Tetraploidy
b. Sex-chromosome polysomy
c. Sex-chromosome monosomy
d. Autosomal trisomy
e. Triploidy
4. A women P2 came to outpatient clinic due to inability to control defecation since 3 months age
after delivering her 3rd child with vacuum extraction. On examination you identify perineal wound
break with external anal sphincter totally torn, but internal sphincter and anal mucosa were still
intact. What is the best method to repair external anal sphincter?
3A: end to end, benang polyglactin 2.0 atau polydiaxanone 3.0
3B: end to end atau overlapping, benang polyglactin 2.0 atau polydiaxanone 3.0
3C: eksterna dijahit end to end atau overlapping, interna dijahit interrupted (matras horizontal); benang
interna polyglactin 3.0 atau polydiaxanone 3.0
4: mukosa rectum jahitan interrupted simpul intralumen atau jahitan kontinyu; benang polyglactin 3.0
Otot perineum: Polyglactin 2.0, jelujur tidak terkunci, atau interrupted
Mukosa vagina + kulit perineum: Polyglactin 3.0 atau polyglycolic 2.0, jelujur tidak terkunci, interrupted,
subkutikuler
a. End to end using polyglycolic acid 2-0
b. Overlapping using Polydioxanone 3-0
c. Overlapping using chromic gut 2-0
d. Overlapping using polyglycolic acid 2-0
e. End to end using chromic gut 2-0

5. Mrs. E, 32 y.o referred from midwife with antepartum hemorrhage. She is G3P2 term pregnancy.
On examination her blood pressure is 160/100 mmHg, HR 100 bpm. She looks anemic, not icteric.
Obstetrical examination reveal contraction 4-5x/10 minutes. FHR 170 bpm, head presentation
3/5. After through examination it is concluded that there is a placental abruption with
retroplacental hematoma size 6x5 cm. This patient is planned to do cesarean section. Post
operative period is very crucial in this patient. Which of the following is not included as a
parameter needed to be evaluated in early warning system?
a. All of the above
b. Heart rate
c. Central venous pressure
d. Blood pressure
e. Urine production

6. Which if the following is true regarding low ovarian reserve in endometriosis?


a. Ovary with endometrioma has a higher response rate to gonadotropin
b. Low ovarian reserve in endometriosis only happen after surgery
c. There is a higher density of follicle in ovary with endometrioma
d. Loss of ovarian stromal appearance and fibrosis are presents in ovarian cortex with
endometrioma
e. Ovulation rate in ovary with endometrioma is higher compared to ovary without endometrioma

7. A 34-year old pregnant lady, G2P1 has been diagnosed with ductal carcinoma of the right breast
(stage 1). She in currently 22 weeks pregnant. What is the initial treatment if choice for her?
a. Termination of pregnancy
b. Local mastectomy without reconstruction
c. Local radiotherapy
d. Local mastectomy with reconstruction
e. Single dose chemotherapy with trastuzumab

8. A 29 year old woman with a positive pregnancy test presents with a good history of tissue
expulsion vaginally passing tissue per vagina. A transvaginal ultrasound scan shows an empty
uterus with endometrial thickness of 11 mm. Regarding her diagnosis, you consider that..
a. She should be offered medical management of miscarriage
b. A laparoscopy should be performed to exclude an ectopic pregnancy
c. She has had a pregnancy of unknown location and needs further investigations
d. She should be offered hysteroscopy
e. She has had a complete miscarriage and needs no further treatment

9. A 22 year old primiparous woman presents for her first prenatal evaluation. On physical
examination you hear a grade 3/6 pansystolic murmur. Which of the following mothers cardiac
abnormalities is the greatest risk for fetal structural congenital cardiac disease?
a. Tetralogy of fallot
b. ASD
c. Pulmonary stenosis
d. VSD
e. Aortic coarctation

10. A 17 year old G1P0 woman presents at 25 weeks gestation complaining of headache for the past
36 hours. She has had regular prenatal visit going back to her first prenatal visit at 8 weeks
gestation. A 20 weeks ultrasound redated her pregnancy by 2 weeks as it was 15 days earlier
than her LMP dating. She has a BP of 155/104 mmHg. You review her medical record and
determine that she does not have chronic hypertension. The patient denies having RUQ pain
but because of your suspicion of severe preeclampsia you order a CBC, liver enzymes, renal
function test, and 1 24 hour urine protein collection. Her laboratory test result reveal a
normal platelet count and
liver enzymes but a slightly elevated creatinine and proteinuria of 550 mg in 24 hours. Her
headache has resolved after a dose of acetaminophen.
Over the next 12 hours, her BP’s rise above 160 mmHg on several occasions, most notably to
174/102 mmHg 2 hours after admission and to 168/96 mmHg 9 hours after admission. Her
headache does not return and she has no RUQ pain of visual symptoms. A set of repeat
laboratory test result are uncharged and by increasing her labetalol dose to 400 mg TID, her
BP’s decrease to 140-150/70-90 mmHg. She is also started on magnesium sulfate. What change
in physical or laboratory examination do you observe that would indicate delivery?
a. Headache returning
b. Double vision
c. Platelets of 108
d. AST of 265
e. Another BP of 174/102 mmHg

11. A 46 years old woman experiences irregular vaginal bleeding of 3 months duration. You perform
an endometrial biopsy, which obtains copious tissue with a velvety, lobulated texture. The
pathologist report shows proliferation of glandular and stromal elements with dilated
endometrial glands, consistent with simple hyperplasia. Cytologic atypia is absent. She agreed
for a medical treatment, which of the following is the most appropriate?
a. LNG IUS
b. Norethisterone acetate 1x5 mg for 14 days on-off
c. Nomogestrel 1x25 mg for 14 days on-off
d. Combined oral contraception
e. MPA 1x25 mg for 14 days on-of

12. Mrs. N, 37 years old, with chief complain of infertility for 6 years with history of severe
dysmenorrhea. From hysterosalpingography, both tubes were non-patent. Pelvic ultrasound
found bilateral cystic mass with internal echo sized 50 and 60 mm in diameter. Her husband
sperm examination was within normal limit. Her AMH level was 0,9 mcg/ml. What is the reason
form performing in subfertile patient with bilateral endometrioma and diminished ovarian
reserve?
a. Removal of endometrioma
b. To improve access for follicle aspiration
c. Removal of deep infiltrating endometriosis lesion
d. Ablation of all endometrial lesion
e. To prevent infection in endometrioma

13. A 22 years old primiparous woman presents for her first prenatal evaluation. On physical
examination you hear a grade 3/6 pansystolic murmur. Numerous physiologic changes develop
over the course of pregnancy. However the greatest impact on potentially compromised
cardiovascular system is..
a. A cardiac output increases to 30-50% above pre pregnancy levels by the end of third trimester
b. Increased 30% total plasma volume in 28 weeks
c. Increased in cardiac output occurs by 24 weeks
d. Decreased SVR in the first trimester and increase after 32 weeks
e. The red cell mass rises by 40%

14. A primigravida at 35 weeks of gestation presents with pain in the right hypochondrium and right
side of her back. There is no history of nausea and vomiting, hypertension, urinary symptoms
and bowel problems. Vital signs : pulse 106, temperature 38,1, BP 128/75. Abdominal
examination is unremarkable. Chest is clear. Fetal monitoring is normal. Urine shows 2+
leucocytes and 1+ blood, white cells count 16 x 1000. What is the most likely diagnosis?
a. Abruption
b. Right basal pneumonitis
c. Appendicitis
d. Cholecystitis
e. Pyelonephritis

15. A 25 years old G1P0 presents to the emergency room with vaginal bleeding. Her LMP was 6
weeks earlier. She reports that she is sexually active with male partners and does not use any
hormonal or barrier methods of contraception. On arrival, her temperature is 37 C, BP 115/80
mmHg, pulse is 75 bpm, RR 16 and she has 100% oxygen saturation on room air.
A pelvic examination reveal small amount of dark blood in the vagina, The external cervical os
appears 1 to 2 cm dilated. Her uterus is mildly enlarged, anteverted, and non tender. A urine
pregnancy test is positive. A pelvic ultrasound is obtained and shows an intrauterine gestational
sac with a yolk sac. No fetal pole or cardiac motion is seen. Bilateral adnexa are normal. What her
diagnosis?
a. Inevitable abortion
b. Ectopic pregnancy
c. Threatened abortion
d. Missed abortion
e. Incomplete abortion

16. A 21- year old G1 P0 patient presents to your office with vaginal bleeding at approximately 8
weeks gestation by her last menstrual period. Her examination is benign with a 9 weeks sized
uterus, a closed cervical os, and a small amount of blood within the vaginal vault. You order a
complete pelvic ultrasound that shows an intrauterine gestational sac containing a fetus
measuring approximately 7 weeks gestation. Doppler sonography is unable to demonstrate any
fetal heartbeat. You decide to perform a suction D&C. When giving informed consent, you discuss
the risk most commonly encountered in this operation, Which of the following is the most
common risk associated with suction D&C?
a. Damage to the bladder
b. Infection
c. Uterovaginal bleeding
d. Need for future surgery
e. Uterine perforation

17. What is the implantation of a placenta in which there is a defect in the fibrinoid layer at the
implantation site, allowing the placental villi to invade and penetrate into but not through the
myometrium called?
a. Placenta percreta
b. Placenta accreta
c. Placental infarct
d. Placenta previa
e. Placenta increta

18. All EXCEPT which of the following are clinical characteristics that increase the risk for acute fatty
liver of pregnancy?
a. Third trimester
b. Female fetus
c. Nulliparity
d. Male fetus
e. Twin gestation

19. During a routine return OB visit, an 18 years old G1P0 patient at 23 weeks gestational age
undergoes a urinalysis. The dipstick done by the nurse indicates the presence of trace
glucosuria. All other parameters of the urine test are normal. Which of the following is the most
likely etiology of the increased sugar detected in the urine?
a. The patient’s urine sample is contaminated
b. The patients has a urine infection
c. The patient has kidney disease
d. The patient’s urinalysis is consistent with normal pregnancy
e. The patient has diabetes

20. A 30 years old multiparous woman has rapid delivery soon after arriving in emergency room.
After delivery the placenta she is noted to have heavy vaginal bleeding. Help has been
summoned. Abdominal examination demonstrates the fundus was soft. What is the most
appropriate next step?
a. Balloon tamponade
b. Suture the laceration
c. Uterine packing
d. Misoprostol administration
e. Intravenous access for fluid resuscitation

21. A 52 years old woman presents to your office. She complained about her sexual problems of low
self esteem and difficulties of initiating sexual intercourse, vaginal dryness and pain during
intercourse. She has the history of 3 full term normal vaginal delivery and she had menopause
and she has no history of hereditary disease. She underwent the lab investigation such RBG :
129mg/dl, Hb 10,6, urea 21, creatinine 0,5. Chest X ray and pelvic ultrasound studies showed no
abnormalities. What is your diagnosis?
a. Orgasmic disorder
b. Genital arousal disorder
c. All of above
d. Sexual desire disorder
e. Vaginismus

22. A 32 years old woman comes to your clinics due to shortness of breath, that worsen since 2 days
ago. On history taking, she told you she had ever diagnosed of having significant mitral stenosis.
She is in 33 weeks pregnant. The fetus in size date appropriate, She has had a recent
echocardiography showing ejection fraction of 54% with moderate – severe pulmonary
hypertension. What is the best management for this patient currently?
a. Conservative management until term pregnancy
b. Second stage acceleration
c. Induction of labor
d. Lung maturation and C section
e. Perform emergency C section

23. A 18 years old adolescent female complains of not having started her menses. Her breast
development is tanner stage IV. Pubic hair development was stage I. From vaginal examination
found a blind vaginal pouch and no uterus and cervix. Which of the following management will
be appropriate to this condition?
Ada payudara besar (Tanner IV) tapi gak ada bulu pubis (Tanner I) -> complete AIS
a. Vaginal reconstruction surgery
b. Give estrogen – progestin sequential
c. Give combined oral contraception
d. Give progestin 14 days on-off
e. Laparoscopy gonad removal

24. A 22 years old G1A0 at 10 weeks presents for her scheduled obstetric appointment. Laboratory
examination showed Hb 11,2, Hematocrit 34%, MCV 86 fl MCH 32 pg. How much iron is
recommended by WHO for pregnant women in Indonesia? (prevalence of anemia in Indonesia
based on RISKESDAS 48%)
a. 100 mg
b. 27 mg
c. 60 mg
d. 30 mg
e. 200 mg
25. A 34 years old female, Para 1, presented to our clinic with secondary amenorrhea and severe,
progressive hirsutism. On clinical examination she was noted to have severe hirsutism and male
pattern scalp bolding. Her BMI was 30 kg/m2. Laboratory results showed an elevated total
testosterone (T) level of 140 mcg/dl (reference value in our lab is 0-80) and androstenedione of
272 mcg/dl (reference value of 30-250). CT of the abdomen and pelvis showed normal adrenal
glands. Pelvic ultrasound of the pelvis demonstrated mildly prominent ovaries, containing
numerous small follicles around the periphery. Your diagnosis according to ASRM/ESHRE
definition, based on two of the following criteria :
a. Polycystic ovaries on ultrasound, hirsutism, obesity
b. Polycystic ovaries on ultrasound, amenorrhea, hirsutism
c. Polycystic ovaries on ultrasound, oligo-or amenorrhea or evidence of hyperandrogenism
d. Polycystic ovaries on ultrasound, amenorrhea, obesity
e. Presence of hyperandrogenism, ovarian dysfunction and exclusive of related disorders

26. A 26 years old woman, G1P0A0 was admitted to ER because she lost her consciousness around 1
hour ago. According to her husband, she is 36 weeks pregnant, She performed antenatal care at
scheduled time and never missed one. Her husband said, she never had any hypertension or any
other disease before. Three days prior hospitalization, she had severe nausea and vomiting,
Physical examination reveals BP 120/80 mmHg, Pulse rate 87 bpm, RR 18. Temperature 36,5.
You notice there is an icteric sclera. Other physical examination was remarkable. Obstetrical
examination reveals no fetal heartbeat was detected. Laboratory examination reveals CBC
10,2/29,9/8900/263.000, Ur/Cr 18/0,8, AST/ALT 458/878, RBG 32, Urinalysis was within normal
limit. What is the best next management in this case?
a. Injection of 40% dextrose
b. Induction of labor
c. Emergency caesarean section
d. Whole blood transfusion
e. Abdominal ultrasound

27. A maternal fetal medicine specialist is consulted and performs an indepth sonogram. The
sonogram indicates that the fetuses are both male and the placenta appears to be diamniotic
and monochorionic. Twin B is noted to have oligohydramnios and to be much smaller than twin
A. In this clinical picture, all of the following are concerns for twin A except?
a. Hydramnion
b. Anemia
c. Polycythemia
d. Congestive heart failure
e. Hypervolemia

28. A 27 years old woman presents to your office with a positive home pregnancy test and a 3 day
history of vaginal bleeding. She is concerned that she may be having a miscarriage. On
examination, the uterine fundus is at the level of the umbilicus. By her last period, she should be
around 8 weeks
gestation. On pelvic examination, there is a moderate amount of blood an vesicle like tissue in
the vaginal vault, and the cervix is closed. The lab then calls you to say that her serum B-HCG
result us greater than 1.000.000 mIU/ml. During further visit, you meet with her in your office
about 3 months after the index visit. Which of the following interventions is most important to
emphasize during her follow up period?
a. Prophylactic antibiotic use during surveillance
b. Await pregnancy attempt for 2 years
c. No further pregnancies are recommended
d. Reliable contraception during surveillance
e. Prophylactic chemotherapy to decrease the risk of persistent and recurrent disease

29. Mrs. A, 26 years old, G1P0A0, according her LMP is 34 weeks pregnant, came for her first
antenatal care. She said that she had 20 kg of weight gain during her pregnancy with swelling
ankles for the past 4 weeks. She never took any iron or vitamin supplementation. From the
physical findings, BP 145/95, HR 88bpm, RR 20, BMI 35 kg/m2. Ultrasound exam confirmed twins
in breech presentation. Result from urinalysis were as follows L color cloudy yellow, specific
gravity 1.013, albumin 2+, RBC 0-1, WBC 2-5, negative bacterial count. What is the most likely
diagnosis?
a. Preeclampsia
b. Renal disease
c. Acute fatty necrosis of the liver
d. Pyelonephritis
e. Chronic hypertension

30. A women P2 came to outpatient clinic due to inability to control defecation since 3 months age
after delivering her 3rd child with vacuum extraction. On examination you identify perineal wound
break with external anal sphincter totally torn, but internal sphincter and anal mucosa were still
intact. What is the proper management given to this patient post sphincter and perineal repair?
a. Analgesic suppository, IV antibiotic, high fiber diet
b. Analgesic suppository, IV antibiotic, laxative agent
c. Oral analgesic, oral antibiotic, liquid diet
d. Oral analgesic, oral antibiotic, stool softener
e. Analgesic suppository, oral antibiotic, stool softener

31. A 24 years old patient, P2, has just delivered vaginally an infant weighing 3000 g after a
spontaneous uncomplicated VBAC. Her prior obstetric history was a low uterine segment
transverse cesarean section for breech. She has had no problems during the pregnancy and labor.
The placenta delivers spontaneously. There is immediate vaginal bleeding of greater than 500cc.
Although all of the following can be the cause for postpartum hemorrhage. Which is the most
frequent cause of immediate hemorrhage as seen in this patient?
a. Uterine atony
b. Vaginal and/or cervical lacerations
c. Coagulopathies
d. Retained placental fragments
e. Uterine rupture

32. A 34 years old women with primary infertility 3 years, oligomenorrhea and a BMI of 26. Day 23
progesterone level result was 5mcg/ml. Transvaginal ultrasound shows multiple small follicle size
5-8 mm in both ovary. HSG shows bilateral patent tubes. Her partner’s semen analysis show a
volume of 3 ml, PH of 7 and a sperm count of 20 million/ml. What is the most appropriate step of
management?
a. Start ovulation induction using gonadotropin
b. Examine TSH and prolactin
c. Give aromatase inhibitor
d. Measure serum testosterone level
e. Measure FSH, LH and estradiol

33. A 35 years old P3 with a positive high risk HPV on DNA testing and a pap smear showing high
grade squamous intraepithelial lesion of the cervix (CIN III) has an inadequate colposcopy. What
is the best next step for this patient?
a. Repeat pap smear after 3 months
b. Proceed to simple hysterectomy
c. Cryotherapy
d. Punch biopsy
e. Cone biopsy

34. Patient 65 years old, P4 came to outpatient clinic with chief complaint of frequent urination.
Since 6 months ago she feels the urge to void every hour and also she has to wake up 3-4 times
in the night to void. She never leaks urine. She doesn’t feel any pain during urination and no
blood in the urine. She already came to general practitioner and got antibiotics for 7 days but
the symptoms remained. What is the most useful supporting examination in this patient?
a. Pelvic floor ultrasound
b. Bladder diary
c. Urine culture
d. Gynecology ultrasound
e. Urodynamic evaluation

35. A 25 years old women, para 1, 6 weeks after delivery. She want to use contraception. She is
asking about the oral contraceptive pills. If the woman missed 3 pills on day 18 of the pack (for 28
day pill packs) and had unprotected intercourse 3 days ago, what is the management plan for this
women? a. Take the most recent missed pill as soon as possible and discard other missed pills.
Continue taking the remaining pills and start the new pack without the hormone-free interval. No
need
to take emergency contraception pills, use additional contraceptive protection for 7 days.
b. Take the most recent missed pills as soon as possible and discard other missed pills. Continue
taking the remaining pills and start the new pack without the hormone-free interval. No
additional contraceptive protection is needed
c. Take the most recent missed pills as soon as possible and discard other missed pills. Continue
taking the remaining pills as usual, take emergency contraception pills and no additional
contraceptive protection is needed
d. Take the most recent missed pills as soon as possible and discard other missed pills. Continue
taking the remaining pills as usual
e. Take the most recent missed pills as soon as possible and discard other missed pills. Continue
taking the remaining pills and start the new pack without the hormone free interval.

36. A 28-year old G1 at 26 weeks present for her scheduled obstetric appointment. You ordered
OGTT examination that shows fasting blood glucose 102 mg/dl and 2 hours after 75 g oral
glucose 185 mg/dl. Her gestational weight gain during pregnancy is 12 kg. Her BMI before
pregnancy was 26 kg/m. What is the most likely fetal consequences cause by patient’s
condition?
a. Congenital valvular heart disease
b. Spina bifida
c. Fetal hypoxia
d. Intrauterine growth restriction
e. Fetal large for gestational age

37. A 18-year old young woman presents to you with a complaint of amenorrhea. She notes that she
has never had menstrual period, but that she has mild cyclic abdominal bloating. She is sexually
active, but she complains of painful sexual intercourse. Her past medical and surgical history is
unremarkable. On physical examination, you note normal appearing axillary and pubic hair. Her
breast development is normal. Pelvic examination reveals normal appearing external genitalia,
and a shortened vagina ending in a blind pouch. Which of the following tests would be your first
step in determining the diagnosis?
Ada susu besar, bulu normal  MRKH
a. Karyotype
b. Pelvic ultrasound -> MRKH
c. Diagnostic laparoscopy
d. Serum FSH, E2
e. Serum FSH

38. Mrs. E, 32 y.o referred from midwife with antepartum hemorrhage. She is G3P2 term pregnancy.
On examination her blood pressure is 160/100 mmHg, HR 100 bpm. She looks anemic, not
icteric. Obstetrical examination reveal contraction 4-5x/10 minutes. FHR 170 bpm, head
presentation 3/5. After through examination it is concluded that there is a placental abruption
with retroplacental hematoma size 6x5 cm. This patient is planned to do cesarean section. If
during operation the uterus is Couvelaire but with good contraction, how would you manage
that condition?
a. Hypogastric artery ligation
b. Uterotonic and observation
c. Sub total hysterectomy
d. Perform prophylactic b-lynch procedure
e. Ascending uterine artery ligation

39. A 82 years old woman P6 came to outpatient clinic with chief complaint of bulging mass
protrudes from vagina since 3 months ago. The mass usually occurs during activity and also
when she defecate, and disappear when lying down. There were no difficulty in voiding and
defecation. No urinary leakage during coughing and sneezing. She is not sexually active. On the
pelvic organ prolapse quantification examination results showing below.
If the patient choose to use pessary instead of surgery, how to choose the right size of the
pessary?

Aa Ba C
+3 +4 +5
GH Pb TVL
5 2 8
Ap Bp D
0 0 +3

a. The intermediate size that do not fall off, but doesn’t cause pain and obstruction of
urination and defecation
b. The smallest size that do not fall off and doesn’t cause pain and obstruction of urination and
defecation
c. The biggest size that do not fall off, but doesn’t cause pain and obstruction of urination and
defecation
d. By measuring the total vaginal length
e. By measuring the genital hiatus of the patient

40. A 22 years old primiparous woman presents for her first prenatal evaluation. On physical
examination you hear a grade 3/6 pansystolic murmur. Which is the most common CHD in
pregnancy that would cause that type of murmur?
a. Atrial septal defect (ASD)
b. Aortic stenosis
c. Ventricular septal defect (VSD)
d. Patent ductus arteriosus (PDA)
e. Pulmonary stenosis

41. A 22 years old unbooked primigravida presents to the emergency department at 26 weeks of
gestation with a history of spontaneous painless bleeding of about 500 ml. What is the best
investigation to secure a diagnosis?
a. CTG
b. CT scan
c. Transvaginal scan
d. Transabdominal scan
e. MRI scan

42. Which of the following is true regarding injury related to trocar insertion in laparoscopy?
a. Hasson technique has the lowest injury creating a small umbilical incision under direct
visualization to enter the abdominal cavity followed by the introduction of a blunt trocar.
b. Open umbilical entry has the lowest rate of injury
c. Closed entry using veress needle has the highest rate of injury
d. All techniques have similar rate of injury
e. Optional trocar insertion has the lowest rate of injury

43. A 16 years old girl presents with primary amenorrhea. Her breast development is Tanner stage 2.
Pubic and axillary hair show stage I development. The girl appear otherwise well. Ultrasound
shows the presence of normal uterus and tubes. Her height is 140 cm. Structural abnormalities
that can happened in this syndrome :
a. Deletion of long arms
b. Ring chromosomes
c. Deletion of short arms
d. All of the answers
e. Isochromosome

44. A women P2 came to outpatient clinic due to inability to control defecation since 3 months age
after delivering her 3rd child with vacuum extraction. On examination you identify perineal wound
break with external anal sphincter totally torn, but internal sphincter and anal mucosa were still
intact. What is the most likely diagnosis of this patient?
a. Chronic perineal rupture grade IIIC
b. Chronic perineal rupture grade IIIB
c. Chronic total perineal rupture
d. Chronis perineal rupture grade IIIA
e. Chronic perineal rupture grade II

45. Fetal growth restriction is associated with all of the following EXCEPT:
a. Antiphospholipid antibody syndrome
b. Inherited thrombophilias
c. Social deprivation
d. Immunosuppressive drugs
e. Infertility
46. A 30 years old multiparous woman has rapid delivery soon after arriving in emergency room.
After delivery the placenta she is noted to have heavy vaginal bleeding. Help has been
summoned. Abdominal examination demonstrates the fundus was soft. After use of a 20 units
oxytocin in 1000cc of crystalloid solution to increase the tone of her uterus stop the bleeding.
However you continue to notice a massive bleeding from the vagina. What is the most
appropriate next step in the evaluation of this patients bleeding?
a. Perform a manual exploration of the uterine fundus and exploration for retained clots or
products
b. Consult interventional radiology for uterine artery embolization
c. Perform a bedside ultrasound for retained products of conception
d. Examine the perineum and vaginal for laceration during delivery
e. Perform a bedside ultrasound to look for blood in the abdomen significant for uterine

47. A 30 years old patient with complain of infertility. Her husband 33 yo who has had semen
analysis which was reported as normal. On further history, the patient reports that her period
have been quiet irregular over the last year and that she has not had period in the last 6 months.
She also reports insomnia, vaginal dryness and decreased libido. What is the most likely diagnosis
for this patient based on her history?
a. Polycystic ovarian syndrome
b. Endometriosis
c. Spontaneous pregnancy
d. Kallmann syndrome
e. Primary ovarian insufficiency

48. A 30 years old multiparous woman has rapid delivery soon after arriving in emergency room.
After delivery the placenta she is noted to have heavy vaginal bleeding. Help has been
summoned. Abdominal examination demonstrates the fundus was soft. Labor induction and
augmentation are NOT associated with of the following risk?
a. The increased risk for cesarean delivery undergoing induction is related with cervical
favorability (Bishop score)
b. Women whose labor is managed with amniotomy have lower incidence of chorioamnionitis
compared with those in spontaneous labor
c. Amniotic fluid embolism in a laboring patient receiving oxytocin can be occurred
d. Post partum hemorrhage from uterine atony is more common in women undergoing induction
or augmentation
e. The uterine rupture risk is increased threefold for women in spontaneous labor with uterine scar

49. A 21- year old G1 P0 patient presents to your office with vaginal bleeding at approximately 8
weeks gestation by her last menstrual period. Her examination is benign with a 9 weeks sized
uterus, a closed cervical os, and a small amount of blood within the vaginal vault. You order a
complete pelvic ultrasound that shows an intrauterine gestational sac containing a fetus
measuring
approximately 7 weeks gestation. Doppler sonography is unable to demonstrate any fetal
heartbeat. What is the most likely diagnosis?
a. Incomplete abortion
b. Complete abortion
c. Threatened abortion
d. Embryonic demise
e. Ectopic pregnancy

50. A 22 years old G1 A0 at 10 weeks presents for her scheduled obstetric appointment. Lab
examination showed hemoglobin 11,2, hematocrit 34%, MCV 86, MCH 32. What is total iron
need during pregnancy?
a. 750 mg
b. 500 mg
c. 1500 mg
d. 2000 mg
e. 1000 mg

51. A women 28 years old came to outpatient clinic referred by obgyn specialist due to continuous
leakage of urine since 2 weeks ago. She underwent cesarean section due to dystocia on second
stage of labor. The baby’s weight was 4200 gr. On examination the cervix was torn at II o’clock
position until anterior fornix, but the hole was not seen clearly. What is the next to confirm
diagnosis in this case?
a. Intravenous pyelography
b. Ultrasound
c. Blue dye test
d. Consult to urologist
e. Indigo carmine test

52. A 25 years old woman in her first pregnancy is noted to have prolonged first and second stages
of labor. She was induced at 38 weeks pregnancy. The baby was delivered by forceps. After
delivery the placenta she is noted to have heavy vaginal bleeding. Abdominal examination
demonstrates a relaxed uterus. What should we do if the fundus not firm after placental
delivery?
a. Fundal massage
b. Dinoprostone-prostaglandinE2
c. Methylergonovine (Methergine)
d. Misoprostol (PGE1)
e. Carboprost (Hemabate, PGF2-alpha)

53. A 16 years old girl presents with primary amenorrhea. Her breast development is Tanner stage 2.
Pubic and axillary hair show stage I development. The girl appear otherwise well. Ultrasound
shows the presence of normal uterus and tubes. Her height is 140 cm. What karyotype that
shows that syndrome?
a. 45, X -> sindrom Turner
b. 69, XXX -> partial mola
c. 47, XXY -> sindrom Klinefelter
d. 47, XX + 13 -> Trisomi 13 (sindrom Patau)
e. 47, XXX -> triple X sindrom

54. A 30 years old patient with complain of infertility. Her husband 33 yo who has had semen
analysis which was reported as normal. On further history, the patient reports that her period
have been quiet irregular over the last year and that she has not had period in the last 6 months.
She also reports insomnia, vaginal dryness and decreased libido. If the diagnosis was confirmed,
what is her best option to achieve pregnancy? Assesment: POI
a. Minimal stimulation
b. High dose gonadotropin IVF
c. Clomiphene citrate – IUI
d. IVF with oocyte donation
e. Gonadotropin – IUI

55. A 36 years old patient, P0, presents to your clinic for fertility workup. She had been married for 2
years with regular intercourse. Her menstrual cycle is normal. Her general status is normal.
Vaginal examination revealed normal findings. Which of the following examination that is not
included in basic workup in the patient above?
a. Hysterosalphyngography
b. Ultrasonography
c. Mid luteal progesterone examination
d. Semen analysis
e. Endometrial dating with Noyes Criteria

56. A primigravida at 36 weeks gestation is measuring large dates. Ultrasound shows ac > 97th
centile. GTT performed shows poorly controlled gestational diabetes. What is the immediate
management plan?
a. Start insulin therapy
b. Wait and watch
c. Start sliding scale and deliver
d. Start induction
e. Give steroids

57. A 32 year old woman has a pelvic ultrasound that bilateral 5 cm ‘kissing’ ovarian cyst in the
pouch of douglas. Both of which contain diffuse, low-level echoes giving a solid ‘ground-glass’
appearance. She report severe dysmenorrhea and dyspareunia. Which of the following condition
that can be found associated with this finding
a. Increased level of He4
b. Adenomyosis in the posterior uterus
c. Fifty percent risk of malignant transformation
d. Increased level of serum AMH
e. Normal level of CA 125

58. A women 28 years old came to outpatient clinic referred by obgyn specialist due to continuous
leakage of urine since 2 weeks ago. She underwent cesarean section due to dystocia on second
stage of labor. The baby’s weight was 4200 gr. On examination the cervix was torn at II o’clock
position until anterior fornix, but the hole was not seen clearly. What is the best management of
this case at this time? Fistula vesicovaginal
a. Trans-vesical fistula repair 3 month from now
b. Transabdominal fistula repair as soon as possible
c. Transvaginal fistula repair as soon as possible
d. Put indwelling transurethral catheter, evaluate 3 months post caesarean section
e. Antibiotics for 7 days continue with transvaginal fistula

59. What is underlying pathophysiology of intrahepatic cholestasis of pregnancy?


a. Acute hepatocellular destruction
b. Incomplete clearance of bile acids
c. Microvascular thrombus accumulation
d. Hepatocellular injury
e. Eosinophil infiltration of the liver

60. Women with one or more previous caesarean section scars and an anterior placenta are at
placenta accreta. Which of test has been shown in recent research to provide the highest
sensitivity and specificity for antenatal diagnosis of placenta accreta.
a. Gadolinium contras MRI
b. 3D power doppler
c. Color doppler
d. Contrast CT
e. Grey scale ultrasound

61. With abdominal entry using the veress needle, what threshold or initial abdominal is used
to reassure the surgeon regarding correct intraperitoneal needle placement ?
a. < 3 mm Hg
b. < 25 mm Hg
c. < 15 mm Hg
d. < 8 mm Hg
e. < 20 mm Hg

62. A 45 years old woman present to your office for consultation regarding her symptoms of
menopause. She stopped having periods 13 months ago after TAH-BSO operation and is having
severe hot flushes. The hot flushes are causing her considerable stress. Which of the following
medication that you will give for hormonal therapy?
a. Sequential estrogen-progestin therapy
b. Estrogen only therapy
c. Monophasic combined oral contraception
d. Biphasic combined oral contraception
e. Triphasic combined oral contraception

63. A 28 year old G2P1 is seen for her first prenatal visit at 16 weeks gestation by menstrual history.
Her first child was born at 32 weeks spontaneously. She is worried this pregnancy also will be
ended with spontaneous preterm birth. What is the next appropriate management for the
patient ?
a. Progesterone prophylaxis
b. Give tocolytics for inhibiting preterm labor
c. Schedule for cervical cerclage
d. Intervention modifiable risk factors for preterm
e. Treat asymptomatic bacterial vaginosis

64. A women 28 years old came to outpatient clinic referred by obgyn specialist due to continuous
leakage of urine since 2 weeks ago. She underwent cesarean section due to dystocia on second
stage of labor. The baby’s weight was 4200 gr. On examination the cervix was torn at II o’clock
position until anterior fornix, but the hole was not seen clearly. What is the criteria of simple
vesicovaginal fistula?
a. Size < 4cm
b. Size < 2cm
c. Size < 2,5 cm
d. Size < 3cm
e. Size < 1,5 cm

65. Mrs. 32 years old, P0, comes to your outpatient clinic due to her prolonged menstrual duration.
She reports her menstrual duration until 14 days and using 10 pads per day. She feels fatigue
easily. On physical exam, you palpate an irregularly enlarged uterus, non tender with firm
contour. Cervix appears to be hyperemic without mas appearance or other abnormalities. By
which mechanism does fibroid creates a hyperestrogenic environment requisites for their
growth?
a. Fibroid converts more estradiol to estrone
b. Fibroid cells contain less density of estrogen receptors compared with normal myometrium
c. Fibroid contains higher level of cytochrome P450 aromatase which allows for conversion of
androgens to estrogen
d. Increased adipose conversion of androgens to estrogen
e. All of the above
66. A 28 years old G2P1 is seen for her first prenatal visit at 16 weeks gestation by menstrual history.
Her first child was born at 32 weeks spontaneously. She is worried this pregnancy also will be
ended
with spontaneous preterm birth. What is the most accurate examination that can be done at 16
weeks to predict the risk of preterm birth?
a. Fibronectin examination -> di uk > 30 minggu, (+) preterm labour > 50 ng/ml
b. IGFBP-1 examination
c. Urinary test to exclude urinary tract infection
d. Measure cervical length
e. Vaginal swab to exclude bacterial vaginosis

67. A 25 years old women, para 1, 6 weeks after delivery. She want to use contraception. She is
asking about the oral contraceptive pills. The mechanism of action of oral contraceptive pills is :
a. Inducing endometrial atrophy
b. Inhibiting ovulation by suppression of serum FSH -> progestogen only, COC pill
c. Increasing cervical mucous hostility
d. Inducing endometritis
e. Inhibiting prolactin

68. A 35 years old P3 with a positive high risk HPV on DNA testing and a pap smear showing high
grade squamous intraepithelial lesion of the cervix (CIN III) has an inadequate colposcopy. Cone
biopsy shows squamous cell cancer that has invaded only 1 mm beyond the basement
membrane. There are no confluent tongues of tumor, and there is no evidence of lymphatic or
vascular invasion. The margins of the cone biopsy specimen are free of disease. How would you
classify or stage this patients disease?
a. Microinvasive cancer
b. Carcinoma of low malignant potential
c. Atypical squamous cells of undetermined significance
d. Invasive cancer, stage Ia
e. Carcinoma in situ

69. You are called in to evaluate the heart of a 19 years old primigravida at term. Listening carefully
to the heart, you determine that there is a split S1, normal S2, S3 easily audible with 2/6 systolic
ejection murmur greater during inspiration and a soft diastolic murmur. You immediately
recognize that:
a. All findings recorded are normal changes in pregnancy
b. The combination of a prominent S3 and soft diastolic murmur is a significant abnormality
c. Diastolic murmurs are rare in pregnant women
d. The presence of the S3 is abnormal
e. The systolic ejection murmur is unusual in a pregnant woman at term

70. Mrs. N, 37 years old, with chief complain of infertility for 6 years with history of severe
dysmenorrhea. From hysterosalpingography, both tubes were non-patent. Pelvic ultrasound
found bilateral cystic mass with internal echo sized 50 and 60 mm in diameter. Her husband
sperm examination was within normal limit. What is the next appropriate management?
a. Give dienogest 1x2 mg for 6 months
b. Give GnRH analog for 3 months continue with IUI
c. Perform laparoscopy cystectomy and adhesiolysis
d. Offer her IVF
e. Offer her IUI

71. A 28 years old woman is hoping to become pregnant soon, She is worried about her history of
acute PID when in college 8 years ago. What is her approximate risk of infertility due to this one
about acute PID?
a. 55 %
b. 35 %
c. 15%
d. 75%
e. 25 %

72. A 36 years old G2P1 presents to the antenatal clinic. She had an emergency cesarean section for
sudden onset hypertension and placental abruption at 30 weeks in her previous pregnancy. She is
currently 20 weeks of gestation and enquires about further plan of fetal monitoring in this
pregnancy. What is the most appropriate advice?
a. Serial scans starting from 24 weeks
b. Uterine artery doppler at 22 weeks
c. Serial cardiotocograph monitoring from 28 weeks
d. No extra monitoring is required
e. Serial scans from 28 weeks

73. A 27 years old woman presents to your office with a positive home pregnancy test and a 3 day
history of vaginal bleeding. She is concerned that she may be having a miscarriage. On
examination, the uterine fundus is at the level of the umbilicus. By her last period, she should be
around 8 weeks gestation. On pelvic examination, there is a moderate amount of blood an vesicle
like tissue in the vaginal vault, and the cervix is closed. The lab then calls you to say that her
serum B-HCG result us greater than 1.000.000 mIU/ml. Which following is the best next step in
this patient’s evaluation?
a. Schedule a follow up visit in 2-4 weeks to recheck B-HCGG level.
b. Determination of Rh status
c. Complete pelvic ultrasound
d. Methotrexate administration
e. Surgical intervention (suction curettage)

74. A 35 years old P3 with a positive high risk HPV on DNA testing and a pap smear showing high
grade squamous intraepithelial lesion of the cervix (CIN III) has an inadequate colposcopy. If the
histopathology result shows squamous cell cancer that has invaded only 1 mm beyond the
basement membrane. There are no confluent tongues of tumor, and there is no evidence of
lymphatic or vascular invasion. The margins of the cone biopsy specimen are free of disease. Of the
following , appropriate therapy for patient is.? Microinvasive cancer
a. Radical hysterectomy
b. External beam radiation
c. Simple hysterectomy with pelvic lymphadenectomy
d. Simple hysterectomy
e. Implantation of radioactive cesium

75. A 32 years old woman comes to your clinics due to shortness of breath, that worsen since 2 days
ago. On history taking, she told you she had ever diagnosed of having significant mitral stenosis.
She is in 33 weeks pregnant. The fetus in size date appropriate, She has had a recent
echocardiography showing ejection fraction of 54% with moderate – severe pulmonary
hypertension. What is the most common cause of heart failure during pregnancy and the
puerperium?
a. Viral myocarditis
b. Pulmonary artery hypertension
c. Valvular heart disease
d. Chronic hypertension with severe preeclampsia
e. Obesity

76. A 35 years old woman, G4P3M at 37 weeks gestation presented in hospital with a ten-day
history of low extremities edema, with idiopathic hypertension for 1 year. At presentation, she
had a BP 170/100 mmHg. Laboratory findings were normal except urinalysis (protein 2+). She
was diagnosed with superimposed severe preeclampsia. It was decided to deliver the fetus by
means of a C-section by indication transverse lie. BP measurement was 150/100 mmHg. She lost
consciousness for 30 seconds five hours after operation.
The laboratory studies gave the following results: serum aspartate aminotransaminase (AST)
225 IU/L, serum alanin aminotransaminase (ALT) 140 IU/L, serum lactate dehydrogenase (LDH)
1017 IU/L, serum urea and creatinine were normal, Hb 10,6, platelet count 50 x 1000. A brain CT
scan was performed on patient which revealed the left frontal lobe lacunar infarction. The
patient was transferred to intensive care unit. Twelve hours observation showed urine
production was 100 ml.
a. Immediately step on fluid resuscitation
b. Immediately giving diuretics bolus iv
c. Immediately giving diuretics maintained by syringe pump
d. Renal failure due to micro thrombopathy suspected, heparin provision could be considered
e. Check albumin level, giving diuretic justified after hypoalbuminemia condition had been
distinguished

77. A 34 years old women with primary infertility 3 years, oligomenorrhea and a BMI of 26. Day 23
progesterone level result was 5mcg/ml. Transvaginal ultrasound shows multiple small follicle size
5-8 mm in both ovary. HSG shows bilateral patent tubes. Her partner’s semen analysis show a
volume of 3 ml, PH of 7 and a sperm count of 20 million/ml. According to the current
international guidelines, which of the following medication is considered to be the first line of
therapy for ovulation induction?
a. Letrozole 1x2,5 mg
b. Clomiphene citrate starting dose 50 mg/day for 5 days
c. Gonadotropin injection 75 IU/day
d. Metformin 2x500 mg
e. Clomiphene citrate 50 mg/day combined with metformin 2x500 mg

78. A 18-year-old adolescent female complains of not having started ger menses. Her breast
development is tanner stage IV. Public hair development was stage I. from vaginal examination
found a blind vaginal pouch and no uterus and cervix. Which of the following describes the most
likely diagnosis?
a. Polycystic ovarian syndrome
b. Kallman syndrome
c. Turner syndrome
d. Complete androgen insensitivity syndrome
e. Partial androgen insensitivity syndrome

79. A 35-year-old P2 is 36 weeks pregnant. Clinically there is a suspicion of left calf DVT. CTG
is normal. What is the next step in the immediate management?
a. Prophylactic dose of tinzaparin
b. Plan delivery
c. Thrombophilia screen
d. FBC, coagulation screen, LFTs
e. Therapeutic dose of tinzaparin

80. A parity 3, 40 years old woman complains of cyclical heavy and painful menstrual bleeding.
On examination she is found to have an enlarged globular uterus and a transvaginal
sonography revealed diffuse adenomyosis. She has completed her family and currently
relies on condoms for contraception. She smokes 1 cigarette per day but is otherwise fit and
well. Which treatment would you consider most appropriate?
a. LNG-IUS
b. Combined oral contraceptive pill (COCP)
c. Endometrial ablation
d. GnRH analogue
e. Hysterectomy

81. A 36-year-old, gravida 5, para 3 at 29 weeks’ gestation by last menstrual period present
complaining of shortness of breath while she was doing her daily routine activity. Based on
NYHA functional classification of patient is?
a. Class 1 -> sesak saat aktivitas berat
b. Class 2 -> sesak aktivitas sedang, daily routine
c. Class 3 -> sesak aktivitas ringan
d. Class 4 -> sesak saat istirahat
e. Class 5

82. Patient 65 years old, P4 came to outpatient clinic with chief complaint of frequent urination.
Since 6 months ago she feels the urge to void every hour and also she has to wake up 3-4
times in the night to void. She never leaks urine. She doesn’t feel any pain during urination
and no bleed in the urine. She already came to general practitioner and got antibiotics for 7
days but the symptoms remained. What is the most useful supporting diagnosis of this
patient?
a. Painful bladder syndrome
b. Overactive bladder
c. Urge incontinence
d. Pyelonephritis
e. Stress urinary incontinence

83. A 27-years old nulligravida presents with 6 months of amenorrhea and is diagnosed
with hyperprolactinemia. Laboratory tests measuring which of the following should also
be obtained?
a. Insulin-like growth factor I
b. Thyroid-stimulating hormone
c. Anti-Mullerian hormone
d. Total testosterone
e. 24-hour urinary free cortisol

84. A 38 year old woman, G4P3, at 37 weeks gestation presented in hospital with a ten day history
of low extremities edema, with idiopathic hypertension for 1 year. At presentation, she had a
blood pressure of 170/100mmHg. Laboratory findings were normal except urinalysis (protein
2+). She was diagnosed with superimposed severe preeclampsia. It was decided to deliver the
fetus by means of a C-section by indication transverse lie. Blood pressure measurement was
150/100 mmHg. She lost consciousness for 30 seconds five hours after operation.
The laboratory studies gave the following result: serum aspartate aminotransferase (AST) 225
IU/L; serum alanine aminotransaminase (ALT) 140 IU/L; serum lactate dehydrogenase (LDH)
1017 IU/L; serum urea and creatinine were normal; hemoglobin 10,8 mg/dL; platelet count 50 x
103 u/ml. A brain computed tomography (CT) scan was performed on patient which revealed
the left frontal lobe lacunar infarction. What is the most appropriate diagnosis?
A. DIC
B. HELLP Syndrome
C. Severe puerperal infection
D. Thrombotic thrombositopenic purpura
E. Acute fatty liver in pregnancy
85. A 34 year old female, para 1, presented to our clinic with secondary amenorrhea and severe
progressive hirsutism. On clinical examination she was noted to have severe hirsutism and male-
pattern scalp bolding. Her BMI was 30 kg/m2. Laboratory result showed an elevated total
testosterone (T) level of 140 ng/dL (reference value in our laboratory is 0-80ng/dL) and
androsetenedione of 272 ng/dL (reference value of 30-250 ng/dL). CT of the abdomen and
pelvis showed normal adrenal glands. Pelvic ultrasound of the pelvis demonstrated mildly
prominent ovaries, containing numerous small follicles around the periphery.
What is your most probable diagnosis?
A. Multicystic ovary
B. Congenital adrenal
hyperplasia C. Polycystic ovary
syndrome
D. Microadenoma pituitary
E. Hyperprolactinemia
85. A triple test is performed for Down’s screening at 16 weeks in a 40 year old woman. The result
suggest a high risk of trisomy 21.
Which would the results typically show?
A. Reduced AFP, reduced estriol, increased β-hCG
B. Increased AFP, reduced estriol, increased β-hCG
C. Increased AFP, increased estriol, increased β-hCG
D. Reduced AFP, increased estriol, increased β-hCG
E. Reduced AFP, increased estriol, reduced β-hCG
86. A 35 year old woman, G4P3, at 37 weeks gestation presented in hospital with a ten day history
of low extremities edema, with idiopathic hypertension for 1 year. At presentation, she had a
blood pressure of 170/100mmHg. Laboratory findings were normal except urinalysis (protein
2+). She was diagnosed with superimposed severe preeclampsia. It was decided to deliver the
fetus by means of a C-section by indication transverse lie. Blood pressure measurement was
150/100 mmHg. She lost consciousness for 30 seconds five hours after operation.
The laboratory studies gave the following result: serum aspartate aminotransferase (AST)
225 IU/L; serum alanine aminotransaminase (ALT) 140 IU/L; serum lactate dehydrogenase
(LDH) 1017 IU/L; serum urea and creatinine were normal; hemoglobin 10,8 mg/dL; platelet
count 50 x 103 u/ml. A brain computed tomography (CT) scan was performed on patient
which revealed the left frontal lobe lacunar infarction. The patient was transferred to
intensive care unit.
What is the best management for this case?
A. Anti oxidant
B. Magnesium sulfate
C. Fresh frozen plasma and thrombocytes concentrates
D. Corticosteroid
E. Anti platelets

87. A 32 years old woman comes to your clinics due to shortness of breath, that worsen since 2
days ago. On history taking, she told you that she had ever diagnosed of having significant
mitral stenosis. She is 33 weeks pregnant. The fetus is size-date appropriate. She has had a
recent echocardiography showing ejection fraction of 54% with moderate-severe pulmonary
hypertension.
For patient with congenital heart disease, what is the most common adverse cardiovascular
event encountered in pregnancy?
A. Thromboembolic event
B. Heart failure
C. Cerebrovascular hemorrhage
D. Arrythmia
E. Heart axis change

88. A 16 year old girl presents with primary amenorrhea. Her breast development is Tanner stage
2. Pubic and axillary hair show stage I development. The girl appears otherwise well.
Ultrasound show the presence of a normal uterus and tubes. Her height is 140 cm.
From these clinical syndromes, what diagnose suitable for that girl?
A. Patau syndrome
B. Edward syndrome
C. Turner syndrome
D. Down syndrome
E. Klinefelter syndrome
89. A 30 years old patient come with complaint of infertility. Her husband is a 33 year old who
has had a semen analysis, which was reported as normal. On further history, the patient
reports that her periods have been quiet irregular over the last year and that she has not had
period in the last 6 months. She also reports insomnia, vaginal dryness, and decreased libido.
Which of the following condition that corresponds to the above possible diagnosis?
A. Positive clomiphene citrate challenge test
B. Day 3 FSH level 10 IU
C. Serum AMH level 1,2 pmol/ml
D. Midluteal progesterone level of 15 ng/ml
E. Follicle antral basal count of 10
90. A 17 year old G1P0 woman presents at 25 weeks gestation complaining headache for the
past 36 hours. She has had regular prenatal visits going back to her first prenatal visit at 8
weeks’ gestation. A 20 week ultrasound redated her pregnancy by 2 weeks as it was 15 days
earlier than her LMP dating. She has a BP of 155/104 mmHg.
You review her medical record and determine that she does not have chronic hypertension.
The patient denies having RUQ pain but because of your high suspicion of severe
preeclampsia you order a CBC, liver enzyme, renal function test, and a 24 hour urine protein
collection. Her laboratory result reveal a normal platelet count and liver enzymes but a
slightly elevated creatinine and proteinuria of 550 mg in 24 hours. Her headache has
resolved after a dose of acetaminophen.
What is the next best step in her management?
A. Bed rest
B. Give her prescription for labetalol and have her follow up in clinic in 2 weeks
C. Begin induction of labor
D. Hospitalization for further evaluation and treatment
E. Immediate delivery
91. 28-year-old G1 at 26 weeks present for her scheduled obstetric appointment. You ordered
OGTT examination that shows fasting blood glucose 102 mg/dl and 2 hours after 75 g oral
glucose 185 mg/dL. Her gestational weight gain during pregnancy is 12 kg. Her BMI before
pregnancy was 26 kg/m2.
What is the most likely diagnosis?
A. Normal OGTT
B. Diabetes mellitus type 2
C. Impaired glucose test
D. Gestational diabetes
E. Diabetes mellitus type 1
92. A 27 year old woman presents to your office with a positive home pregnancy test and a 3-day
history of vaginal bleeding. She is concerned that she may be having a miscarriage. On
examination, the uterine fundus is at the level of the umbilicus. By her last period, she should
be around 8 weeks gestation. On pelvic examination, there is a moderate amount of blood
and vesicle-like tissue in the vaginal vault, and the cervix is closed. The lab then calls you to
say that her serum β-hCG result is greater than 1,000,000mIU/mL.
The patient undergoes an uncomplicated suction D&C. The pathology report is available the
next day and is consistent with a complete molar gestation.
What is the best next step in the care of this patient’s condition?
A. Repeat pelvic imaging
B. Chemotherapy
C. Surveillance of serum β-hCG
D. No further follow up is required
E. Radiation therapy
93. A 45 years old woman presents to your office for consultation regarding her syptoms of
menopause. She stopped having periods 13 months ago after TAH-BSO operation and is
having severe hot flushes. The hot flushes are causing her considerable stress.
Which of the following is an absolute contraindication for hormonal therapy?
A. Coronary heart disease
B. Diabetes mellitus
C. Impairment of liver function
D. Endometriosis
E. Migraine
94. A 22 year old G1A0 at 10 weeks presents for her scheduled obstetric (OB) appointment.
Laboratory examination showed hemoglobin 11,2 g/dL, hematocrit 34%, MCV 86 fl MCHC 32
pg.
Which of the following shows iron deficiency anemia:
A. Microcytic hyperchromic, low serum iron and transferrin saturation
B. Normal hemoglobin level, low ferritin, and serum ion
C. Low ferritin, normal serum iron
D. Low ferritin, serum iron, and transferrin
E. Microcytic hypochromic, low serum iron, and transferrin saturation
95. A 26 year old woman complain of reccurent bouts of bacterial vaginosis (BV) despite
successful initial treatment. She does not douche or smoke and has been in a monogamous
relationship or 6 years. Recurrence of BV after initial treatment is common (up to 30
percent), which can be frustrating of the patient. Which of the following consistently
decreases recurrence rates and should be recommended to this patient?
A. No intervention consistently decreases recurrence
B. Treatment of long continuous antibiotics
C. Use of acidiying vaginal gels
D. Probiotics and reintroduction of lactobacilli
E. Treatment of male partners

96. A 18 years old adolescent female complains of not having started her menses. Her breast
development is Tanner stage IV. Pubic hair development was stage I. From vaginal
examinataion found a blind vaginal pouch and no uterus and cervix. From ultrasound
examination found no uterus and there was difficulty in identifying the gonads. What is the
next plan?
A. FSH and LH examination
B. Kariotyping
C. FSH, LH, and E2 examinataion
D. TSH, fT4 examination
E. Prolactin measurement

97. A 27 year old nullogravida presents with 6 motnhs of amenorrhea and is diagnosed with
hyperprolactinemia.
Laboratory tests measuring which of the following should also be obtained?
A. Total testosterone
B. 24 hour urinary free cortisol
C. Anti Mullerian Hormone
D. Insuline like growth factor I
E. Thyroid stimulating hormone

98. A 62 years old woman P6 come to outpatient clinic with chief complaint of bulging mass
protrude from vagina since 3 months ago. The mass usually occurs during activity and when
she defecate, and disappear when lying down. There were no difficulty in voiding and
defecation. No urinary leakage during coughing and sneezing. She is not sexually active.
Based on the Pelvic Organ Prolapse Quantification examination result showing below.

Aa Ba C
+3 +4 +5
GH Pb TVL
5 2 8
Ap Bp D
0 0 +3
A. Uterine prolapse grade 3, cystocele grade 3, rectocele grade 2
B. Uterine prolapse grade 2, cystocele grade 2, rectocele grade 1
C. Uterine prolapse grade 4, cystocele grade 4, rectocele grade 3
D. Uterine prolapse grade 3, cystocele grade 2, rectocele grade 2
E. Uterine prolapse grade 4, cystocele grade 3, rectocele grade

2 Sampun inggih. Matur nuwun…

UNAS NOVEMBER 2019

1. The following statement is true regarding the physiological adaptations to pregnancy ?


a. The increase in internal heart rate contributes to an increase in cardiac output during
pregnancy
b. Mean arterial blood pressure falls because of a rising in systemic vascular resistance
c. Anatomical and physiological chases in the tungs allow a pregnant woman to withstand
hypoxia better than a non pregnant woman
d. The lower bicarbonate levels in pregnant woman reflect a state of metabolic acidosis
e. TSH (thyrotropin) levels fall in the first trimester but returns fastly to normal by term.

2. A 29 year old woman with a positive pregnancy test presents with a good history of passing
tissue per vagina. A transvaginal ultrasound scan show an empty uterus with an endometrial
thickness of 11 mm Regarding her diagnosis, you consider that
a. She has had a complete mismanage and needs no further treatment
b. She has a pregnancy of unknown location and needs further investigations
c. She should be offered a hysteroscopy
d. She should be offered medical management of miscarriage
e. A laparoscopy should be performed to exclude an ectopic pregnancy

3. A – 7 year – old girl presents to her pediatrician what her parents who are concerned about her
early sexual development. She is developing breasts, axillary her, and pubic her and they are
noticing body odor A thorough clinical workup reveals the child has an irregular, echogenic,
thickly septate ovarian mass on her left ovary. What type of tumor is reasonable for this child’s
clinical presentation ?
a. Dysgerminoma
b. Embryonal carcinoma
c. Sertoli leydig cell tumor
d. Endodermal sinus tumor
e. Granulosa theca cell tumor
4. Which of the following is true about the screening and diagnosis of spina bifida?
a. A maternal serum alpha-fetal protein (MSAFP) of 2-5 multiples of the median (MoM) is
diagnostic of an NTD
b. First-trimester ultrasound screening for NTDs primarily involves identification of a normal
posterior fossa during aneuploidy screening to rule out the presence of a Arnold-Chiari II
malformation
c. Magnetic resonance imaging (MRI) has proven to be a better diagnostic tool for NTDs than
ultrasound
d. The banana sign is indicative of ventriculomegaly, which is present in most cases of open
NTDs
e. Diagnosis of lesion level requires three-dimensional ultrasound or MRI modalities

5. On prenatal ultrasound, which of the following feature characterize gastroschizis ?


a. The abdominal wall defect is superior to cord insertion
b. Ectopic cords is present
c. The abdominal wall defect is lateral to cord insertion
d. The abdominal wall defect is lower than cord insertion
e. The bladder cannot be visualized

6. A 39 years old female G2P1A0, 15 weeks pregnant presents to your clinic for having routine ANC
On Physical examination, you found her fundal height equals umbilical point. You performed
ultrasound and saw a multilocular hypoechoic mass sized 10 cm (in diameter) in her left adnexa
No free fluid in her abdomen and pelvis. What is your consideration in this case ?
a. The incidence of adnexal masses in pregnancy is 1%
b. The incidence of ovarian cancers in pregnancy is between 1 : 1000
c. The most common type of benign ovarian cyst in pregnancy is a mature teratoma
d. The most common histopathological subtype for malignant ovarian tumor in pregnancy is
epithelial ovarian tumor
e. The resolution rate of adnexal masses in the second trimester of pregnancy is 60-70%

7. The patient does not believe that she has ovarian cyst during pregnancy. She really concern
about the possibility of malignant cyst. What will you inform regarding this to her ?
a. The most common mode of presentation of an adnexal mass is pain
b. The sensitivity of detection of ovarian cysts on clinical examination alone is less than 5%
c. The size of ovarian cyst that should prompt investigation for malignancy is 10 cm
d. The validated sensitivity and specificity of IOTA rules on US evaluation of an ovarian cyst is
sensitivity : 78%, specificity: 87%
e. The sensitivity and specificity of MRI in the diagnosis of a malignancy is 100 and 94%
respectively
8. You have cheked her Ca125 serum level and the result was 350 u/ml You performed Per
laparotomy because her frozen section result revealed malignancy cyst After 1 week the
pathology result comes with serous papillary carcinoma of left ovary. She is planned for
chemotherapy. What will you inform to her regarding chemotherapy for ovarian cancer during
pregnancy ?
a. In a patient with ovarian cancer in pregnancy receiving chemotherapy the delivery should be
planned at completion of chemotherapy
b. Chemotherapy use in pregnancy is generally considered safe beyond 20 weeks of gestation
c. CNS and neural tube complications occur during the week 8-12 weeks in pregnancy
d. This percentage of patients receiving chemotherapy in pregnancy who develop major
congenital malformations is 30-40%
e. Cardiovascular defects are common congenital malformations in platin based chemotherapy
regimens

A 28-year woman, G1 36 weeks of gestational age, went to your clinic to do routine antenatal care.
During ultrasound, the doctor told that she will be expecting baby boy with estimated lethal Weight
2500 g, however, amniotic fluid considered to be less than normal. Then you asked the patient to
drink minimal of 2 L of water a day and get herself another ultrasound within 3 days to evaluate the
amniotic fluid

9. Oligohydramnios is defined as which of the following ?


A. Amniotic fluid index < 5 cm
B. Single deepest pocket < 2 cm
C. Amniotic fluid index < 90m percentile
D. All of the above
E. None of the above

10. Amniotic fluid volume is a balance between production and reception. What is the primary
mechanism of fluid reception ?
A. Fetal breathing
B. Fetal swallowing
C. Absorption across fetal skin
D. Absorption by fetal kidneys
E. Filtration by fetal kidneys

11. In a normal fetus at term, what is the daily volume of fetal urine that contributes to the amount
of amniotic fluid present ?
A. 200 mL
B. 250 mL
C. 500 mL
D. 750 mL
E. 1000 mL
Mrs. A, 26 years-old, G1P0A0, according to her last LMP is 34 weeks pregnant, came for her first
antenatal care. She admit to have 20 kg weight gain during pregnancy with swelling ankles for the
past 4 weeks. She never took any iron or vitamin supplementation. From the physical findings, BP
145/95 mmHg, HR 86x/min, RR 20x/min, BMI 35 kg/m 3 Ultrasound examination confirmed twins in
breach presentation. Results from urinalysis were as follows color cloudy yellow, specific gravity 1
,013, albumin 2+, RBC 0 – 1, WBC 2-5, bacteria negative

12. What is the most likely diagnosis


?
A. Acute fatty necrosis of the liver
B. Chronic hypertension
C. Preeclampsia
D. Renal disease
E. Pyelonephritis

13. Given the history if this patient, several more laboratory and diagnostics tests were obtained.
She was stable and the fetuses have reassuring heart rate tracings. Which of the following do
you expect to see in the test results ?
A. Chest X-ray to show decreased pulmonary vascular markings
B. Urine to show infection
C. Creatinine clearance to be increased above normal pregnancy levels
D. Serum uric acid to be increased
E. A decreased hematocrit

Mrs. B, 37 years-old came to your office at 32 weeks of gestation according to her last menstrual
period. She has no ultrasound examination before and didn’t get antenatal care routinely. The vital
sign is within normal limit. She has body mass index 19 kg/m2. During physical examination the
uterine fundal height is 22 cm. From ultrasound examination, the fetus has biometric values that
correlate with 30 weeks fetus.

14. Which of the following is the next best step in managing this patient ?
A. Antenatal care routinely for the next 2 weeks
B. Evaluate maternal status and comorbidities
C. Consider definer the baby
D. Repeat sonography for fetal growth in 2 weeks
E. Doppler velocimetry evaluation every 3 days

15. According to algorithm for management of fetal-growth restriction, you evaluate the Doppler
Velocimetry then find reversed and-diastolic flow and oligohydramnios. What is the appropriate
management at this time?
A. Regular fetal testing
B. Weekly evaluation of amniotic fluid
C. Consider corticosteroids for lung maturation
D. Deliver the baby
E. Reevaluate middle cerebral arteries and ductus venous

16. Fetal growth restriction is associated with all of the following, EXCEPT
A. Antiphospholipid antibody syndrome
B. Inherited thrombophilias
C. Infertility
D. Immunosuppressive drugs
E. Social deprivation

17. If during operation the uterus is couvelaire but with good contraction, how would you manage
that condition ?
A. Perform prophylactic b-lynch suture
B. Ascending uterine artery ligation
C. Hypogastric artery ligation
D. Sub total hysterectomy
E. Uterotonic and observation

18. Postoperative period is very crucial in this patient Which of the following is not included as a
parameter needed to be evaluated in early warning system..
A. Blood pressure
B. Heart rate
C. Urine production
D. Central venous pressure
E. All of the above

A 26-years-old woman, G1P0A0 was admitted to emergency room because she lost her
consciousness around 1 hour ago. According to her husband, she is 36 weeks pregnant she
performed antenatal care at scheduled time, and never missed one. Her husband said, she never
had any hypertension or any other disease before. Three days prior hospitalization, she had severe
nausea and vomiting. Physical examination reveals, BP 120/80 mmHg, pulse rate 87 x/min, RR
18x/min, Temperature 36,5 OC. You notice there is an icteric sclera. Other physical examination was
remarkable. Obstetrical examination reveals no fetal heartbeat was detected. Laboratory
examination reveals CBC 10.2/29.9/8900/263.000; Ur/Cr 18/0, 8 AST/ALT 458/878; RBG 32;
Urinalysis was within normal limit.

19. What is the best next management in this case ?


A. Abdominal ultrasound
B. Induction of labor
C. Emergency Caesarean section
D. Whole Blood transfusion
E. Injections of 40% Dextrose
20. All EXCEPT which of the following clinical characteristic that increase the risk for acute fatty liver
of pregnancy ?
A. Null parity
B. Female fetus
C. Male fetus
D. Twin gestation
E. Third trimester

21. What is the underlying pathophysiology or intrahepatic cholestasis of pregnancy ?


A. Acute hepatocellular destruction
B. Incomplete clearance of bile acids
C. Microvascular thrombus accumulation
D. Eosinophil infiltration of the liver
E. Hepatocellular injury

A 17-year-old G1P0 woman presents at 25 weeks gestation complaining of headache for the past 36
hours. She has had regular prenatal visits going back to her first prenatal visit at 8 weeks gestation. A
20-week ultrasound related her pregnancy by 2 weeks as it was 15 days earlier than her LMP dating.
She has a BP of 155/104 mmHg.

22. You review her medical record and determine that she does not have chronic hypertension. The
patient denies having RUQ pain but because of your high suspicion of severe preeclampsia you
order a CBC, liver enzymes, renal function test and a 24 hour urine protein collection. Her
laboratory test results reveal a normal platelet count and liver enzymes but a slightly elevated
creatinine and proteinuria of 550 mg in 24 hours. Her headache has resolved after a dose of
acetaminophen. What is the next best step in her management ?
A. Give her a prescription for labetatol and have her follow-up in clinic in 2 weeks
B. (a) plus bed rest
C. Hospitalization for further evaluation and treatment
D. Immediate delivery
E. Begin induction of labor

23. Over the next 12 hours, her SPBPs rise above 160 mmHg on several occasions, most notably to
174/102 mmHg 2 hours after admission and to 168/96 mmHg 9 hours after admission. Her
headache does not return and she has no RUQ pain or visual symptoms. A set of repeat
laboratory test results are unchanged and by increasing her labetalol dose to 400 mm TID, her
BPs decrease to 140s-150s/70-90 mmHg. She is also started on magnesium sulfate. What change
in physical or laboratory examination do you observe that would indicate delivery ?
A. Another BP of 174/102 mmHg
B. Headache returning
C. Double vision
D. Platelets of 108
E. AST of 265

A 43-year-old woman, G4P3, at 37 weeks gestation presented in hospital with a ten day history of
low extremities edema, with idiopathic hypertension for 1 year. At presentation, she had a blood
pressure of 170/100 mmHg. Laboratory findings were normal except urinalysis (protein 2+). She was
diagnosed with superimposed severe preeclampsia. It was decided to deliver the fetus by means of a
C-section by indication transverse lie. Blood pressure measurement was 150/100 mmHg. She lost
consciousness for 30 seconds five hours after operation. The laboratory studies gave the following
result serum aspartate aminotransaminase (AST), 225 IU/L, serum alanine aminotransaminase (ALT),
140 IU/L. serum urea and creatine were normal, hemoglobin, 10.6 mg/dL, platelet count, 50 x 103
/ml. A brain computed tomography (CT) scan was performed on patient which revealed the left
frontal lobe lacunar infarction. The patient was transferred to intensive care unit.

24. What is the most appropriate diagnosis


A. DIC
B. Acute Fatty liver in Pregnancy
C. HELLP Syndrome
D. Severe puerperal infection
E. Thrombotic thrombositopenic purpura

25. What is the best management after for this case


A. Fresh-frozen plasma and trombocytes concentrates
B. Anti-platelets
C. Anti-oxidant
D. Corticosteroid
E. Magnesium sulfate

26. Twelve hours observation showed urine production was 100 ml


A. Immediately giving diuretics bolus-iv
B. Immediately giving diuretics maintained by syringe-pump
C. Check albumin level, giving diuretic justified after hipoalbuminemia condition had bean
distinguished
D. Renal failure due to micro thrombopathy suspected, heparin provision could be considered
E. Immediately step on fluid rescucitation

A 33-year-old woman, G1P0A0, came to hospital with major complaint, watery leakage. She was on
her 33 weeks of gestational age. Data from medical record showed that she came previously a week
ago, complaining vaginal swab has done.

27. In case above, what kind of examination should you performed for establishing diagnosis
A. Vaginal examination
B. Inspeculo
C. Blood test
D. Ultrasound
E. Simple urine test

28. You found on Leopold 1, hard, round with ballottement (+). Contraction was infrequent and
weak. A Shat was your next plan ?
A. Went for labour induction
B. Immediate C. Section
C. Tocolytic and corticosteroid prevision
D. Performed ultrasound
E. Performed external-version

29. Ultrasound examination showed that trans-cerebellar diameter was proper to gestational age.
Abdominal circumference was lower than 2.5 centile and amniotic fluid deepest pocket was 1.2
cm. What is the most likely diagnosis?
A. Growth restriction with olygohydramnion
B. Normal Growth with olygohydramnion
C. Growth restriction with normal amniotic fluid
D. Normal growth with normal amniotic fluid
E. Need another examination for establishing diagnosis

30. Lack of baby movement had been felt for two days, lethal heart rate was 146 bpm. What was
your next step ?
A. Termination of pregnancy
B. Giving oxygenation and left lateral position
C. Ensuring fetal welt-being by Manning criteria
D. Fetal lung maturation
E. Giving intravenous fluid rehydration

31. Cardiotocography, showed low variability with checkmark pattern and no desceleration. What
was your interpretation and the best management through ?
A. Category one continued for fetal lung maturation
B. Category two, intrauterine resuscitation for 24 hours and reevaluation after
C. Category two, went for Doppler velocymetry
D. Category three, went for Doppler ultrasound ultrasound exam
E. Category three, delivered the baby

32. A primigravida at 36 weeks gestation is measuring large for dates. Ultrasound shows AC > 97 th
canticle GTT performed shows poorly controlled gestational diabetes. What is the immediate
management plan?
A. Give steroids
B. Start induction
C. Start hypoglycaemics drugs
D. Wait and watch
E. Start sliding scale and deliver

33. A 36 year old G2P1 presents to the antenatal clinic. She had an emergency caesarean section for
sudden onset hypertension and placental abruption at 30 weeks in her previous pregnancy. She
is currently 20 weeks of gestation and enquires about further plan of fetal monitoring in this
pregnancy. What is the most appropriate advice?
A. No extra monitoring is required
B. Uterine artery Doppler at 22 weeks
C. Serial scans starting form 24 weeks
D. Serial cardiotocograph monitoring from 28 weeks
E. Serial scans from 28 weeks

34. A 34 year old pregnant lady, G2P1 has been diagnosed with ductal carcinoma of the right breast
(Stage 1). She is currently 22 weeks pregnant. What is the initial treatment of choice for her ?
A. Termination of pregnancy
B. Local mastectomy with reconstruction
C. Local mastectomy without reconstruction
D. Local radiotherapy
E. Single close chemotherapy with frastuzumab

35. A triple test is performed for down’s screening at 16 weeks in a 40 year old woman. The result
suggests a high risk of trisomy 21. What would the results typically show ?
A. Reduced AFP, reduced estriol, increased -hCG
B. Increased AFP, reduced estriol increased -hCG
C. Reduced AFP, increased estriol, increased -hCG
D. Reduced AFP, increased estriol, reduced -hCG
E. Increased AFP, increased estriot increased -hCG

36. Women with one or more previous caesarean section scars and an anterior placenta are at of
placenta accrete. Which test has been shown in recent research to provide the highest sensitivity
and specificity for antenatal diagnosis of placenta accrete?
A. Colour Doppler
B. 3D power Doppler
C. Contrast CT
D. Gadolinium contrast MRI
E. Grey scale ultrasound

37. A 22 year-old unhooked primigravida presents to the Emergency Department at 26 weeks of


gestation with a history of spontaneous painless bleeding at about 500 ml. What is the best
investigation to clinch a diagnosis ?
A. MRI scan
B. Transabdominal scan
C. CTG
D. Transvaginal scan
E. CT-scan

38. A primigravida at 35 weeks of gestation presents with pain in the right hypochondrium and right
side of her back. There is no history of nausea or vomiting. Hypertension, urinary symptoms and
bowel problems. Vital signs: pulse-106, temperature 38,1 BP 128/75. Abdominal examination is
unremarkable. Chest is clear. Fetal monitoring is normal. Urine shows 2+ leucocytes and 1+
blood. White cell count 16x10 lt. What is the most likely diagnosis ?
A. Appendicitis
B. Cholecystitis
C. Pyelonephritis
D. Abruption
E. Right basal pneumonilis

39. A 35-year-old P2 is 36 weeks pregnant clinically there is a suspicion of left calf DVT, CTG is
normal. What is the next step in the immediate management ?
A. Plan delivery
B. Therapeutic dose of tinzaparin
C. Prophylactic dose of tinzaparin
D. FBC, coagulation screen LFTs
E. Thrombophilia screen

40. A 24-year-old G2P1 woman at 39 weeks and 3 days is seen in clinic. She has been experiencing
more frequent contractions and thinks she miht be in labor. Her last pregnancy ended with a
caesarean delivery after a stage 1 arrest. There was no evidence of cephalopelvic disproportion.
Earlier in the course of her current pregnancy she had desired a scheduled repeat cesarean, but
now that she might be in labor she would like to try and delivery vaginally.
What would be a contra indication to a trial of labor after cesarean (TOLAC) ?
A. Prior classical hysterotomy
B. Prior Kerr hysterotomy
C. Small for gestational age fetus
D. Oligohydraminios
E. GBS + righter

41. A 25-year-old G1P0 presents to the emergency room with vaginal bleeding. Her last normal
menstrual period was 6 weeks earlier. She reports that she is sexually active with male partners
and does not use any hormonal of burner methods for contraception. On arrival her
temperature is 37oC. blood pressure is 115/80 pulse is 75 beats per minute respiratory rate is 15
breaths per minute, and she has 100% oxygen saturation on room air. A pelvic examination
reveals a small
amount of dark blood in the vagina. The external cervical as appears 1 to 2 cm dilated. Her
uterus is mildly enlarged, anteverted and nontender. A pelvic ultrasound is obtained and shows
an intrauterine gestational sac with a yolk sac. No fetal pole or cardiac motion is seen. Bilateral
adnexa are normal what is her diagnosis ?
A. Incomplete abortion
B. Threatened abortion
C. Ectopic pregnancy
D. Missed abortion
E. Inevitable abortion

42. A 45-year-old presents for evaluation because her primary care physician has diagnosed her with
pelvic organ prolapsed while performing annual care. She denies any pelvic pressure, bulge, or
difficulty with urination. Her only medical comorbidity is obesity. for asymptomatic grade 1
pelvic organ prolapsed, what do you recommend ?
A. Conservative management with pelvic floor muscle exercises and weight loss
B. Colpocleisis obliterative procedure
C. Gellhom pessary
D. Round ligament suspension
E. Hysterectomy

43. The majority of vulvar, vaginal and cervical cancers appear to have a common cause and usually
caused by
A. High risk types of herpes simplex virus (HSV) infection
B. High risk types of human papiloma virus (HPV) infection
C. Increased exposure to endogenous estrogen
D. Increased exposure to exogenous estrogen
E. Chronic bacterial and parasitic infection

44. Childhood neoplastic ovarian masses most commonly originate from


A. Gonadal epithelium
B. Gonadal stroma
C. Sex cords
D. Germ cells
E. Metastatic disease

45. During a routine return OB visit, an 18-years-old G1P0 patient at 23 weeks gestational age
undergoes a urinalysis. The dipstick done by the nurse indicates the presence of trace glucosuria.
All other parameters of the urine test are normal. Which of the following is the most likely
etiology of the increased sugar detected in the urine ?
A. The patient has diabetes
B. The patient has a urine infection
C. The patient’s urinalysis is consisten with normal pregnancy
D. The patient’s urine sample is contaminated
E. The patient has kidney disease

46. A maternal fetal medicine specialist is consulted and performs an in depth sonogram. The
sonogram indicates that the fetuses are both male and the placenta appears to be diamniotic
and monochronic. Twin B is noted to have oligohydramnios and to be much smaller than twin A.
in this clinical picture all of the following are concerns for twin A Except
A. Congestive heart failure
B. Anemia
C. Hypervolemia
D. Polycythemia
E. Hydramnion

47. you are called in to evaluate the heart of a 18-year-old primigravida at term. Listening carefully
to the heart, you determine that there is a split S1 normal S2, S3 easily audible with a 2/6 systolic
ejection murmur greater during inspiration, and a soft diastolic murmur. You immediately
recognize that
A. The presence of the S3 is abnormal
B. The systolic ejection murmur is unusual in a pregnant woman at term
C. Diastolic murmur are rare in pregnant women
D. The combination of a prominian and soft diastolic murmur is a significant abnormally
E. All findings recorded are normal changes in pregnancy

A 24-year-old G1P0 woman presents for prenatal care at 8 weeks by LMP. She has regular menses
every 28 to 30 days and you confirm her gestational age with an ultrasound today in the office. She
has no past medical or surgical history. She and her husband of 6 months planned the pregnancy
and they have both been reading about pregnancy and prenatal care. You discuss the prenatal tests
for the first visit as well as the plan throughout the rest of the pregnancy.

48. Your next patient is a 13-year-old adolescent girl who presents with cyclic pelvic pain. She has
never had a menstrual cycle. She denies any history of intercourse. She is afebrile and her vital
signs are stable. On physical examination, she has age-appropriate breast and pubic hair
development and normal external genitalia. However, you are unable to locate a vaginal
introitus instead, there is a tense bulge where the introitus would be expected you obtain a
transabdominal ultrasound, which reveals a hematocolpos and hematometra. What is the most
likely diagnosis ?
A. Transverse vaginal septum
B. Longitudinal vaginal septum
C. Imperforate hymen
D. Vaginal atresia (MRKH)
E. Bicornuate uterus
49. An 18-year-old nulligravid woman presents to the student health clinic with a 4 week history of
yellow vaginal discharge. She also reports vulval itching and irritation. She is sexually active and
monogamous with her boyfriend. They use condoms inconsistently on physical examination,
she is found to be nontoxic and afebrile On genitourinary examination, vulvar and vaginal
erytherna is noted along with a yellow, frothy malodocous discharge with a pH of 6,5. The cervix
appears to have erythematous punctuations. There is no cervical. Uterine of adnexal tenderness
The addition of 10% KOH to the vaginal discharge does not produce an amine odor Wet prep
microscopic examination ot the vaginal swabs is performed. What would you expect to see
under microscopy ?
A. Branching hyphae
B. Multinucleated giant cells
C. Scant WBC
D. Flagellated, motile organisms
E. Epithelial cells covered with bacteria

50. An 89-year-old female patient with multiple, serious medical comorbidities presents to discuss
options for treatment of her high-grade prolapsed. The prolapse is externalized and becoming
ulcerated from friction against her undergarments. She cannot tolerate a pessary. Her main
priority is to fix or get rid of this thing but her primary care provider has cautioned against a
lengthy of open abdominal procedure. She is not interested in future intercourse. What can you
offer this patient ?
A. Nothing can be done
B. Open abdominal sacral colpopexy
C. Robot assisted laparoscopic sacral colpopexy
D. Hysterectomy with anterior and posterior colporrhaphy
E. Colpocleisis

51. A patient returns for a postoperative checkup 2 weeks after a total abdominal hysterectomy for
fibroids. She is distressed because she is having continous leakage of urine from the vagina. Her
leakage is essentially continous and worsens with coughing, laughing, or movement. Given her
history and physical, you perform both a metthylene blue dye test, which is negative and an
indigo carnine test. Which is positive. The most likely diagnosis is:
A. Rectovaginal fistula
B. Uretro vaginal fistula
C. Vesico vaginal fistula
D. Uretero vagina fistula
E. Improssible to distinguish

52. A 32-year-old presents for an infertility workup She and her partner have been trying to conceive
for 2 years without success. She has regular menstruation though she mentions she has severe
cramping during her cycles. She also notes she experiences pelvic pain during sex. On
examination, she is a febrile and she and she experiences a great deal of pain during the pelvic
examination.
You do not note discharge on examination Which of the following tests is required for diagnosis
of the patient’s infertility ?
A. Ultrasound
B. -hCG level
C. Pap smear
D. Laparoscopy
E. Hysterosalpingogram

A 25-year-old woman in her first pregnancy is notes to have prolonged first and second stages of
labour. She was induced at 38 weeks’ pregnancy. The baby was delivered by forceps. After delivery
the placenta she is noted to have heavy vaginal bleeding. Abdominal examination demonstrates a
relaxed uterus.

53. What is the most likely cause of


bleeding A. Uterine atony
B. Uterine rupture
C. Retained placenta
D. Genital tract laceration
E. DIC

54. What should we do if the fundus not firm after placental delivery
A. Methylergonovine (Mehergine)
B. Carboprost (Hemabate, PGF2-alpha)
C. Fundal Massage
D. Miscoprostal (PGE1)
E. Dinoprostone-prostaglandin E2

A 30-years-old multiparous woman has rapid delivery soon after arriving in emergency room. After
delivery the placenta she is noted to have heavy vaginal bleeding. Help has been summoned.
Abdominal examination demonstrates the fundus was soft
55. What is the most appropriate next step?
A. Intravenous access for fluid resuscitations
B. Uterine packing
C. Balloon tamponade
D. Suture the laceration
E. Misoprostol administration

56. After use of a 20 units of oxytocin in 1000 mL of crystalloid solution to increase the tone of her
uterus stop the bleeding. However you continue to notice a massive bleeding form the vagina.
What is the most appropriate next step in the evaluation of this patient’s bleeding ?
A. Perform a bedside ultrasound for retained products of conception
B. Perform a bedside ultrasound to look for blood in the abdomen significant for uterine
rupture
C. Perform a manual exploration of the uterine fundus and exploration for retained clots or
products
D. Examine the perineum and vaginal for laceration during delivery
E. Consult interventional radiology for uterine artery embolization

57. Labor induction and augmentation are NOT associated with of the following risk ?
A. Postpartum hemorrhage from uterine atony is more common in going induction or
augmentation
B. Amniotic fluid embolism in a laboring patient receiving oxytocin can be occurred
C. The increased risk for cesarean delivery undergoing induction is related with cervical
favorability (Bishop Score)
D. The uterine rupture risk is increased threefold for women in spontaneous labor with
uterine scar
E. Women whose labor is managed with amniotomy have lower incidence of
chorioamnionitis compared with those in spontaneous labor

A 32-year-old woman comes to your clinics due to shortness of breath, that worsen since 2 days ago.
On history taking, she told you that she had ever diagnosed of having significant mitral stenosis. She
is 33 weeks pregnant. The fetus is size-date appropriate. She has had a recent echocardiography
showing ejection fraction of 54% with moderate-severe pulmonary hypertension

58. What is the best management for this patient currently ?


A. Perform emergency C section
B. Lung maturation and C section
C. Conservative management until term pregnancy
D. Second stage acceleration
E. Induction of labor

59. What is the most common cause of heart failure during pregnancy and the puerperium ?
A. Chronic hypertension with severe preeclampsia
B. Viral myocarditis
C. Obesity
D. Valvular heart disease
E. Pulmonary Artery Hypertension

60. For patients with congenital heart disease, what is the most common adverse cardiovascular
event encountered in pregnancy ?
A. Heart failure
B. Arrhythmia
C. Thromboembolic event
D. Cerebrovascular hemorrhage
E. Heart axis changes

61. A 55-year-old woman presents to your office for consultation regarding her symptoms of
menopause. She stopped having periods 8 months ago and is having severe hot flushes. The hot
flushes are causing her considerable stress. What should you tell her regarding the psychological
symptoms of the climacteric ?
A. They are not related to her changing levels of estrogen and progesterone
B. They commonly include insomnia, irritability, frustration, and malaise
C. They are related to a drop in gonadotropin levels
D. They are not affected by environmental factors
E. They are primarily a reaction to the cessation of menstrual flow

62. A 62-year-old woman present for annual examination. Her last spontaneous menstrual period
was 9 years ago and she has been reluctant to use postmenopausal hormone replacement
because of a strong family history of breast cancer. She now complains of diminished interest in
sexual activity. Which of the following is the most likely cause of her complaint ?
A. Decreased vaginal length
B. Decreased ovarian function
C. Alienation from her partner
D. Untreatable sexual dysfunction
E. Physiologic anorgasmia

A 49-year-old woman experiences irregular vaginal bleeding for 3 months duration. You performed
endometrial biopsy, which obtains copious tissue with a velvety, lobutated texture. The pathologist
report shows proliferation of glandular and stroma elements with dilated endometrial glands,
consistent with simple hyperplasia Cytologic atypia is absent
63. Which of the following is the best way to advise the patient ?
A. She should be treated to estrogen and progestin hormone therapy
B. The tissue will progress to cancer in approximately 10% of cases
C. The tissue may be weakly premaligmant and progress to cancer in approximately 1% of
cases
D. She requires a hysterectomy
E. No further therapy is needed

64. The result showing a condylomatous acetowhite lesion with punctuation and atypical vessels.
Biopsy result confirms CIN I with HPV DNA tes positive. What do you suggest for patient ?
A. LEEP procedure
B. Reevalutation of HPV DNA
C. Cold knife conization
D. Repeat cytology in 12 months
E. Repeat cytology in 6 months
UNAS MARET 2019

A 36-year-old G2PO at 26 weeks gestational age presents to the office with palpitations, anxiety, lack
of sleep, and a 5 kg weight loss over the past 2 weeks. She started to develop substernal chest pain
this morning. She has no significant medical history. On examination, she is afebrile with a heart rate
of 152 and a BP of 158/82. She appears anxious and her eyes are prominently open. Her thyroid is
enlarged but nontender. An EKG reveals sinus tachycardia without ST segment changes.

1. Which of the following is the appropriate immediate first line therapy?


a. Propanolol
b. Lorazepam
c. Hydrocortisone
d. Radioactive iodine
e. Propylthiouracil (PTU)

2. Which clinical symptom is not characteristic of mild thyrotoxicosis


a. Palpitation
b. Tachycardia
c. Thyromegaly
d. Cold intolerance
e. Failure to gain weight

A 24-year-old women at 32 weeks' gestation complains of shortness of breath during her pregnancy,
especially with physical exertion. She has no prior medical history. Her respiratory rate is 16x/m; her
lungs are clear to auscultation; and your office oxygen saturation monitor reveals her oxygen
saturation to be 98% on room air.

3. You reassure her that this sensation is normal and explain which of the following?
a. Pulmonary resistance increases during pregnancy,
b. Airway conductance is decreased during pregnancy.
c. Small amniotic fluid emboli are shed throughout pregnancy,
d. Maximal breathing capacity is not altered by pregnancy.
e. Because of enlarging uterus pushing up on the diaphragm, her vital capacity is decreased by
20%.

4. A 33-year-old G2P1 presents at 35 weeks' gestation with complaints of nausea and vomiting.
Laboratory evaluation reveals markedly decreased glucose level, elevated transaminase levels,
renal dysfunction, and coagulopathy. What is the possibility diagnosis in this patient ?
a. Acute hepatitis
b. Cirrhosis hepatis
c. Intrahepatic cholestasis
d. Acute fatty liver of pregnancy
e. Non alcoholic fatty liver disease
A 32-year-old lady, G2P1AO presented to delivery ward at 30 weeks gestation with worsening
abdominal pain for few hours. She had also had some vaginal bleeding within the past hour. Her
uterus was tender and firm to palpation. She was found to have low-amplitude, high frequency
uterine contractions, and the fetal heart rate tracing showed recurrent late decelerations and
reduced variability. Her blood pressure was 160/100 mmHg and she has had a +2 proteinuria. She
did her antenatal care in your hospital and ultrasound examination was performed 3 times with no
remarkable abnormalities.

5. The most likely diagnosis is:


a. Vasa previa
b. Preterm labor
c. Placenta previa
d. Placental abruption
e. Preterm Premature Rupture of Membrane (PPROM)

6. From obstetrical examination you found her cervix was unfavorable. Your next plan is to deliver
the baby by:
a. Vaginal delivery
b. Elective C-section
c. Emergency C-section
d. Operative vaginal delivery
e. Observation until the cervix was favorable

7. You are counseling a couple in your clinic who desire VBAC. Her baby is in a vertex presentation,
appropriate size for 37 weeks, and her previous low transverse procedure was for breech
presentation. You have to give inform consent about VBAC. In providing informed consent, in
which of the following ways do you explain the risk of uterine rupture?
a. Less than 1%
b. Between 2% and 5 %
c. Between 15-20%
d. Depend on the length of her labor
e. Depend on the location and proximity of the scar site to the placental implantation

A 17-years-old G2P1 woman with no prenatal care at 29 weeks' gestation presents with painful
contractions arid-pressure. Her cervix is 1 cm, 40% effaced, and breech at -station 2. There is no
evidence of ruptured membranes. Her contractions are every 4 minutes. FHR are 150 bpm with
accelerations. Maternal vital signs are temperature 36.8°C, pulse 96x/m, BP110/72 mmHg.
8. What should you do?
a. C-section
b. Performed fetal fibronectin
c. Begin tocolytic agents and corticosteroids
d. Observe the cervical changes and labor progress
e. Perform amniocentesis to rule out chorioamnionitis

9. What fetal complication is associated with the Nonsteroidal anti-inflamatory agent


Indomethacin as tocolytics agent?
a. Hydramnios
b. Achondroplasia
c. Pulmonary valve atresia
d. Bronchopulmonary dysplasia
e. Premature closure of the ductus arteriosus

10. The nonstress test (NST) has which of the following characteristics?
a. Low positive predictive value
b. Low specificity (with reactive NST)
c. Low false-positive rate (with nonreactive NST)
d. FHR reactivity depends on normal cardiac development
e. Acceleration without fetal movement should not be accepted

11. Corticosteroids administered to women at risk for preterm birth have been demonstrated to
decrease rates of neonatal respiratory distress if the birth is delayed for at least what amount of
time after the initiation of therapy?
a. 12 hours
b. 24 hours
c. 36 hours
d. 48 hours
e. 72 hours

12. A 24-year-old patient (Parity 2) has just delivered vaginally an infant weighing 4300g after a
spontaneous uncomplicated labor. Her prior obstetric history was a low uterine segment
transverse cesarean section for breech. She has had no problems during the pregnancy and
labor. The placenta delivers spontaneously. There is immediate vaginal bleeding of greater than
500 cc. Although all of the following can be the cause for postpartum hemorrhage, which is the
most frequent cause of immediate hemorrhage as seen in this patient?
a. Uterine atony
b. Coagulopathies
c. Uterine rupture
d. Retained placental fragments
e. Vaginal and or cervical lacerations

Mrs, X, 24 yo came to the ER with complaints of headaches since the last day of examinations
obtained expecting her first child, gestational age 32 weeks with blurred vision and denied
heartburn. On physical examination found BP 190/120 mmHg, pulse 90 x/m, breathing 16 x/m.
Leopold found the lower left back head, FHR 140 bpm, irregular contraction. Pelvic score of 1 was
found, pelvis size wide. Laboratory investigation; hemoglobin 11.5 g%, platelet 9000/mm3, LDH 510
iu/L, Proteinuria +2, ALT 10 u/L, AST 15 u/L

13. What is the best diagnosis for Mrs. X


a. HELLP syndrome
b. Severe preeclampsia
c. Chronic hypertension
d. Gestational hypertension
e. Superimposed preeclampsia

14. What is the most clinically effective antihypertensive agent for Mrs. X
a. ISDN
b. Atenolol
c. Nifedipines
d. Furosemide
e. Methyldopa

15. Which antihypertension drugs can cause fetal growth restriction?


a. Nifedipine
b. Atenolol
c. Hydralazine
d. Captopril
e. Methyldopa

16. A woman is being treated with magnesium sulphate. There is concern about magnesium toxicity.
What is the first sign of magnesium toxicity?
a. Bradycardia
b. Reduced consciousness
c. Respiratory depression
d. Decreased urine output
e. Loss of deep tendon reflexes

17. With two home pregnancy tests and ultrasound revealed 6-7 wga pregnancy. As her pregnancy
continues, you would expect her cardiac output to increase by which of the following
mechanisms:
a. An increased heart rate alone
b. A decrease in systemic vascular resistance
c. First an increase in stroke volume, then an increase heart rate
d. Cardiac output would not change significantly until the third trimester
e. An increase in systemic vascular resistance facilitated by elevated progesterone levels

18. Which of the following is true regarding the physiologic changes she might expect during her
pregnancy?
a. An overall decrease in the number of WBC and platelets
b. Gastric emptying and large bowel motility are increased in pregnancy
c. An increase in the tidal volume along with an increase in total lung capacity (TLC)
d. BUN and creatinine will decrease as a result of an increase in glomerular filtration rate (GFR)
e. Nausea and vomiting that should be treated aggressively with antiemetics and intravenous
hydration

19. A woman with a previous stillbirth and postpartum DVT is found to have lupus anticoagulant and
medium-titre Immunoglobulin M (IgM) anticardiolipin antibodies (aCL) on two occasions. In a
subsequent pregnancy:
a. Warfarin should be discontinued
b. She has an increase risk of miscarriage
c. She requires antibiotic prophylaxis to cover delivery
d. Low dose aspirin should be discontinued at 34 weeks
e. She dose not require postpartum heparin if she has vaginal delivery
A 28-year-oid P1 woman is being discharged from the hospital on postoperative day 4 after having
received a primary low transverse cesarean section for breech presentation, with an estimated
blood loss of 700 mL. Her pregnancy was otherwise uncomplicated and her hospital course was also
uncomplicated. Ten days after Cesarean section, the patient came complaining of abdominal pain
and fever. Fundal height 2 fingers below navel.
20. What is the most likely probable diagnosis of the patient?
a. Metritis
b. Mastitis
c. Typhoid fever
d. Urinary tract infection
e. Breast engorgement

21. Lower urinary tract symptoms with pyuria but a sterile urine culture are likely due to which
pathogen?
a. Candida
b. Escherichia coli
c. Proteus mirabilis
d. Klebsiella pneumoniae
e. Chlamydia trachomatis

22. A patient calls your clinic complaining of continued heavy vaginal bleeding. She had an
"uncomplicated" vaginal birth 2 weeks ago of her second child. What is the most likely diagnosis
from the following differentials?
a. Uterine atony
b. Uterine rupture
c. Coagulopathies
d. Vaginal lacerations
e. Retained placental fragments

A 29-year-old G2P1 woman came to outpatient clinic with obesity, a history of GDM in the prior
pregnancy, and a strong family history for type 2 diabetes mellitus (T2DM) presents at 7 weeks'
gestation. In her previous pregnancy, she required insulin therapy. She delivered at 39 weeks and
her baby boy weighed 4.300 gr

23. In addition to the routine prenatal laboratory tests, what other testing will you obtain at this
point?
a. HbAlC
b. No other testing needed
c. An ultrasound to estimate gestational age
d. Perform a glucose challenge test at first visit
e. Perform a glucose challenge test at 24 weeks' gestation

24. Laboratory test results return, and her fasting blood glucose is 145 mg/dL. An ultrasound reveals
the pregnancy to be 7 weeks and 2 days, consistent with LMP, Hemoglobin A 1c is 7,5%. Her
diagnosis is:
a. GDM (GDP dan GD2JPP meningkat dan HbA1C < 7)
b. T1DM (usia muda, insulin dependent)
c. T2DM (GDP/GD2JPP meningkat, HbA1C > 7)
d. no diabetes in pregnancy
e. Impaired glucose tolerance (2JPP meningkat)

25. A 41-year-old women present at 36 weeks of gestation in active labour. The ultrasound scan at
32 weeks showed low lying placenta. Immediately after rupture of the membrane, she started
bleeding vaginally with associated cardiotocography (CTG) of non reassuring. What is the most
likely diagnosis?
a. Vasa previa
b. Placenta previa
c. Placenta accreta
d. Abruptio placenta
e. Low lying placenta

A woman has a booking scan in 16 weeks gestation, which reveals a monochorionic diamniotic twin
pregnancy. She asks you about the risks regarding her pregnancy.

26. Regarding complications of twin pregnancy:


a. Caesarean section is the preferred route of delivery
b. With significant growth discordance, particularly when the first twin is the smaller
c. In twin-to-twin transfusion syndrome, the haemoglobin levels for both twins are often not
discordant
d. In a twin pregnancy with one fetal loss in the third trimester, in 90 per cent of cases the
remaining twin will be delivered within 72 h
e. Twin reversed arterial perfusion sequence is associated with high mortality in the recipient
twin due to prematurity and intra-uterine cardiac failure

27. The timing of separation of the embryo in monochorionic diamniotic is:


a. up to 4 days
b. 4-7 days
c. 7-14 days
d. > 14 days
e. > 28 days

28. Regarding twin to twin transfusion syndrome:


a. The donor develop hydrops
b. The recipient develops polyhydramnios
c. Quinterro classification is up to Quinterro IV
d. The perinatal mortality in twins reaches to 85%
e. Complicates up to 35 per cent of dichorionic multiple pregnancies

29. The most frequent twin pregnancy is:


a. Conjoined
twins b. Dizygotic
twins
c. Dichorionic diamniotic
d. Monochorionic diamniotic
e. Monochorionic monoamniotic

30. Which of the following statements regarding chorionocity is true? (William 23)
a. A dichorionic pregnancy is always dizygotic
b. Monochorionic membranes should have four layers
c. Monochorionic twins are always monozygotic
d. Determination of chorionicity is easiest in the second trimester
e. Complications in twin pregnancy is more frequent in dichorionic pregnancy

A 38 years old P6 lady is being operated for abdominal delivery. Unfortunately uterine contraction is
not good. The patient suffers for massive bleeding. The operator quickly decides to perform uterine
removal in order to stop the bleeding.

31. Which artery that should be blocked if the operator would like to stop the blood flows to the
uterine artery?
a. Pudenda artery
b. Abdominal aorta
c. Hypogastric artery
d. Common iliac artery
e. Uterine and ovarian arteries

32. The uterine artery:


a. Gives a branch to ovary
b. Runs at the back of the ureter
c. May anastomose with femoral artery
d. Gives myometrium vascularization only
e. Is a branch of the anterior division of the internal iliac artery .

33. The ovarian arteries:


a. Are crossed by the ureters
b. Arise just above the renal artery
c. Reach the ovary through round ligament
d. Reach the ovary through infundibulo-pelvic ligament
e. Anastomose with the descendent branch of uterine artery

A 22 years old female, Gl at 26 weeks gestation, presents to the office for her routine obstetrical
visit. Currently, she is complaining shortness of breath. She has no other complaints. On physical
examination: Blood pressure 100/70 mmHg, PR 90 bpm, RR 22x/m. No abnormality was found in
chest examinations. BGA results: pH 7,45 (7,35 - 7,45), pO2 103 mmHg (75,-100), pC02 28 mmHg (35
- 45),
HCO3 17 mEq/L (22 - 26), BE 2 mmol/L (-2 - 2), 02 sat 99% (>95%).

34. What is your explanation about the cause of symptom of this patient?
a. Cardiac output increases 20%
b. Maternal blood volume increases 50%
c. Decrease in stroke volume and blood viscosity
d. The uterus and the diaphragm becomes elevated
e. The heart is displaced upward, and somewhat to the right with rotation on its long axis.

35. What is the result of blood gas analysis (BGA) stated above?
a. Normal BGA
b. Metabolic acidosis
c. Metabolic alkalosis
d. Respiratory acidosis
e. Respiratory alkalosis

36. What is the most appropriate next management in this patient?


a. Ask patient to perform echocardiography examination
b. Ask patient to perform chest radiography examination
c. Admit the patient to the hospital and give non-rebreathing mask
d. Counsel the patient that this is physiologic changes during pregnancy
e. Terminate the pregnancy because the symptom will become worsen in 32 weeks pregnancy

37. What is ECG finding consider normal during pregnancy?


a. Specifics changes
b. Increased heart rate (15%)
c. Inverted T waves in lead II
d. 15 degree right axis deviation
e. Irreversible ST waves changes

A 32 years old female, Gl at 8 weeks gestation, presents to the office for her routine obstetrical visit.
She asks you about the nutrition demand during pregnancy. Her BMI is 24 kg/m2. No remarkable
past medical history is noted.

38. According to WHO Asian criteria, her BMI is classified as:


a. Normal
b. Underweight
c. Overweight
d. Obese type 2.
e. Obese type 2 ,

39. She ask you what is the optimal total weight gain during her pregnancy:
a. < 5 kg
b. 5-9 kg
c. 7-11,5 kg
d. 11,5-16 kg
e. 12,5-18 kg

40. What is the most likely risk of this patient?


a. Anemia
b. Congenital anomaly
c. Gestational diabetes
d. Spontaneous abortion
e. Post partum hemorrhage
41. A 29 year old woman with a positive pregnancy test presents with a good history of tissue
vaginally. A transvaginal ultrasound scan shows an empty uterus with an endometrial thickness
of 11 mm. Regarding her diagnosis, you consider that :
a. She has had a complete miscarriage and needs no further treatment
b. She has had a pregnancy of unknown location and needs further investigations
c. She should be offered a hysteroscopy
d. She should be offered medical management of miscarriage
e. A laparoscopy should be performed to exclude an ectopic pregnancy

42. Which of the following is true about the screening and diagnosis of spina bifida?
a. A maternal serum alpha-fetal protein (MSAFP) of 2.5 multiples of the median (MoM) is
diagnostic of an NTD.
b. First-trimester ultrasound screening for NTDs primarily involves identification of a normal
posterior fossa during aneuploidy screening to rule out the presence of a Arnoid Chiari II
malformation.
c. Magnetic resonance imaging (MRI) has proven to be a better diagnostic tool for NTDs than
ultrasound,
d. The banana sign is indicative of ventricutomegaly, which is present in most cases of open NTDs.
e. Diagnosis of lesion level requires three-dimensional ultrasound or MRI modalities.

43. On prenatal ultrasound, which of the following feature characterise gastroschisis?


a. The abdominal wall defect is superior to cord insertion,
b. Ectopia cordis is present.
c. The abdominal wall defect is lateral to cord insertion.
d. The abdominal wall defect is lower than cord insertion,
e. The bladder cannot be visualized.

44. A 39 years old female G2P1AO, 15 weeks pregnant presents to your clinic for having routine
ANC. On physical examination, you found her fundal height equals umbilical point. You
performed ultrasound and saw a multilocular hypoechoic mass sized 10 cm (in diameter) in her
left adnexa. No free fluid in her-abdomen and pelvis. What is your consideration in this case?
a. The incidence of adnexal masses in pregnancy is 1%
b. The incidence of ovarian cancers in pregnancy is between 1:1000
c. The most common type of benign ovarian cyst in pregnancy is a mature teratoma
d. The most common histopathological subtype for malignant ovarian tumor in pregnancy is
epithelial ovarian tumor
e. The resolution rate of adnexal masses in the second trimester of pregnancy is 60-70%

45. The patient does not believe that she has ovarian cyst during pregnancy. She is really concern
about the possibility of malignancy. Regarding this situation, what would you inform her?
a. The most common mode of presentation of an adnexal mass is pain
b. The sensitivity of detection of ovarian cysts on clinical examination alone is less than 5%.
c. The size of ovarian cyst that should prompt investigation for malignancy is 10 cm
d. The validated sensitivity and specificity of IOTA rules on ultrasound evaluation of an ovarian
cyst is sensitivity: 78%, specificity: 87%
e. The sensitivity and specificity of MRI in the diagnosis of a malignancy is 100 and 94%
respectively
46. You have checked her CA125 serum level and the result was 350 u/ml. You performed
conservative surgical staging by laparotomy because her frozen section revealed malignancy
cyst. After 1 week, pathology result serous papillary carcinoma of the left oyary. She is planned
for chemotherapy. What will you inform to her regarding chemotherapy for ovarian cancer
during pregnancy?
a. In a patient with ovarian cancer in pregnancy receiving chemotherapy the delivery should be
planned at completion of chemotherapy
b. Use of chemotherapy in pregnancy generally considered safe after 20 weeks of gestation
c. CNS and neural tube complications occur during the week 8-12 weeks in pregnancy
d. This percentage of patients receiving chemotherapy in pregnancy who develop major
congenital malformations is 30-40%
e. Cardiovascular defects are common congenital malformations in platin based
chemotherapy regimens

A 28 yo woman, Gl 36 weeks of gestational age, went to your clinic to do routine antenatal care.
During ultrasound, the doctor told her that she will be expecting baby boy with estimated fetal
weight 2500 g, however, amniotic fluid considered to be less than normal. Then you asked the
patient to drink minimal of 2L of water a day and get herself another ultrasound within 3 days to
evaluate the amniotic fluid.

47. Amniotic fluid volume is a balance between production and resorption. What is the primary
mechanism of fluid resorption?
a. Fetal breathing
b. Fetal swallowing
c. Absorption across fetal skin
d. Absorption by fetal kidneys
e. Filtration by fetal kidneys

48. In a normal fetus at term, what is the daily volume of fetal urine that contributes to the amount
of amniotic fluid present?
a. 200 ml
b. 250 ml
c. 500 ml
d. 750 ml
e. 1000 ml

Mrs. B, 37-years-old came to your office at 32 weeks of gestation according to her last menstrual
period. She has no ultrasound examination before and did not do her routine antenatal care. The
vital sign is within normal limit. She has body mass index 19 kg/m2. During physical examination, the
uterine fundal height is 22 cm. From ultrasound examination, the fetus has biometric values that
correlate with 30 weeks fetus. (suspek IUGR)

49. Which of the following is the next best step in managing this patient?
a. Antenatal care routinely for the next 2 weeks
b. Evaluate maternal status and comorbidities
c. Consider deliver the baby
d. Repeat sonography for fetal growth in 2 weeks
e. Doppler velocimetry evaluation every 3 days
50. According to algorithm for management of fetal-growth restriction, you evaluate the Doppler
Velocimetry then find reversed end-diastolic flow and oligohydramnios. What is the, appropriate
management at this time?
a. Regular fetal testing
b. Weekly evaluation of amniotic fluid
c. Consider corticosteroids for lung maturation
d. Deliver the baby
e. Reevaluate middle cerebral arteries and ductus venosus

A 26-years-old woman, GIPOAO was admitted to ER because she lost her consciousness around 1
hour ago. According to her husband, she is 36 weeks pregnant. She performed antenatal care at
scheduled time, and never missed one. Her husband said, she never had any hypertension or any
other disease before. Three days prior hospitalization, she had severe nausea and vomiting.
Physical examination reveals, BP 120/80 mmHg, pulse rate 87 x/min, RR 18x/min, Temperature
36.5°C. You notice there is an icteric sclera. Other physical examination was remarkable. Obstetrical
examination reveals no fetal heartbeat was detected.
Laboratory examination reveals CBC 10.2/29.9/8900/263.000; Ur/Cr 18/0,8; AST/ALT 458/878; RBG
32; Urinalysis was within normal limit.

51. What is the best next management in this case?


a. Abdominal ultrasound
b. Induction of labor
c. Emergency Caesarean section
d. Whole Blood transfusion
e. Injections of 40% Dextrose

52. What is the underlying pathophysiology of intrahepatic cholestasis of pregnancy?


a. Acute hepatocellular destruction
b. Incomplete clearance of bile acids
c. Microvascular thrombus accumulation
d. Eosinophil infiltration of the liver
e. Hepatocellular injury

A 33-year-old woman, G1POAO, came to hospital with major complaint, watery leakage. She was on
her 33 weeks of gestational age. Data from medical record showed that she came previously a week
ago, complaining vaginal discharge. Vaginal swab has been done.

53. In case above, what kind of examination should you performed for establishing diagnosis,
a. Vaginal examination
b. Inspeculo
c. Blood test
d. Ultrasound
e. Simple urine test

54. You found on Leopold 1, hard, round with ballotement (+). Contraction was infrequent and
weak. What was your next plan?
a. Went for labour induction
b. Immediate C-section
c. Tocolytic and corticosteroid provision
d. Performed ultrasound
e. Performed external version

55. Ultrasound examination showed that trans-cerebellar diameter was proper to gestational age
abdominal circumferrence was lower than 2.5 centile and amniotic fluid deepest pocket was 1.2
cm. What is the most likely diagnosis?
a. Growth restriction with oligohydramnios
b. Normal Growth with olygohydramnion
c. Growth restriction with normal amniotic fluid
d. Normal growth with normal amniotic fluid
e. Need another examination for establishing diagnosis.

56. Lack of baby movement had been felt for two days, fetal heart rate was 146 bpm. What was
your next step?
a. Termination of pregnancy
b. Giving oxygenation and left lateral position
c. Ensuring Fetal well-being by Manning criteria
d. Fetal lung maturation
e. Giving intravenous fluid rehidration.

57. Cardiotocography, showed low variability with checkmark pattern and no desceleration. What
was your interpretation and the best management through?
a. Category one, continued for fetal lung maturation
b. Category two, intrauterine resuscitation for 24 hours and reevaluation after
c. Category two, went for doppler veiocymetri
d. Category three, went for doppler velocymetry ultrasound exam.
e. Category three, delivered the baby

58. A patient wishes to consider pregnancy after treatment for her breast cancer. What is the most
important predictor of a good prognosis?
a. Young age
b. Herceptin positivity
c. Estrogen receptor positivity
d. BRCA gene positivity
e. Family history of treatable breast cancer

59. A primigravida at 36 weeks gestation is measuring large for dates. Ultrasound shows AC > 97 th
centile. GTT performed shows poorly controlled gestational diabetes. What is the immediate
management plan?
a. Give steroids
b. Start induction
c. Start hypoglycaemic drugs
d. Wait and watch
e. Start sliding scale and deliver the baby
60. During cesarean section, the uterine contraction suddenly became poor. No significant bleeding
was found. The vital sign was within normal limit. B Lynch procedure is then applied. The main
concept of B Lynch is
a. If bimanual compression of the exteriorized uterus decreases the uterine bleeding
b. If bimanual compression of the exteriorized uterus does not decrease the uterine bleeding.
c. If bimanual compression of the exteriorized uterus increases the uterine bleeding, d. If
bimanual compression of the exteriorized uterus decreases the uterine contraction,
d. If bimanual compression of the exteriorized uterus increases the uterine contraction.

A 27-year-old patient and her husband present to you with primary infertility. The patient reports
regular periods every 28 to 30 days. The patient has no significant medical history and does not take
any medications other than prenatal vitamins. Her husband is also in good health, is 30 year of age,
and has two children from a previous marriage. When you asked the patient how long they have
been trying to achieve a pregnancy, they tell you 6 months.

61. Your instructions to the couple are the following:


a. They will likely need IVF to achieve a pregnancy
b. They will likely need IUI cycles
c. Continue trying appropriately timed intercourse for 6 more months and return for follow up if
no pregnancy is achieved
d. Consider egg donor
e. Consider adoption

62. A 29 year old lady with 30 year old husband come with 2 years of primary infertility. Semen
analysis, HSG and an endocrine evaluation including FSH, E2, TSH, prolactin levels, and ovarian
reserve testing had been done. All of the tests come back normal. Your next recommendation,
a. Have 6 more months of timed intercourse and return for follow up visit if no pregnancy is
achieved
b. Clomiphene citrate with IUI
c. Gonadotropin injection
d. Human gonadotropin (hMG)
e. IVF

63. Characteristically, menopause begins with cycle irregularity that extends to 1 year after
permanent cessation of menses. The more correct, scientific terminology for this time is
menopausal transition, and it typically takes place over a span of how many years?
a. 1 to 2 years
b. 2 to 3
years c. 4 to 7
years
d. 5 to 10 years
e. 11 to 12 years

64. A 49-year-old woman had a radical hysterectomy and lymph node sampling for stage IB
squamous cell cancer of the cervix. A suprapubic catheter was placed at the time of surgery. She
is now 8 weeks postoperative and has not been able to void. She is also leaking urine with
activity, coughing, and sneezing. What is the most likely reason for voiding difficulty?
a. Spasm of the pelvic floor muscles
b. Outflow obstruction
c. Postoperative swelling around the bladder
d. Innervation to the lower urinary tract was transected
e. Overdistention of the bladder

A 46-year-old lady, Parity 2, presents to you with a complaint of LLQ pain, intermittent nausea,
abdominal pressure, and bloating. Her history is notable for mild obesity, right breast cancer, and
hypertension. Her family history is notable for premenopausal breast cancer in her mother and
maternal aunt. She had a pelvic ultrasound showing a left ovarian mass containing internal
septations and papillary projection. She has moderate ascites and her Ca-125 was 719.

65. Which of the following is associated with an increased risk of ovarian


cancer? a. History of breast cancer
b. Breastfeeding
c. Multiparity
d. Tubal ligation
e. Obesity

66. She underwent TAH, BSO, collection of pelvic washings, omentectomy, cytoreduction or
"debulking," and bilateral pelvic and para-aortic lymph node sampling. The mass had spread
beyond the ovary to the omentum, peritoneum, and bowel. She was found to have ascites, and
pelvic washings were positive. What stage of ovarian cancer does she have?
a. Stage-I
b. Stage-II
c. Stage-III
d. Stage-IV (hepar)
e. Stage-V

A couple presents because they have been trying to conceive for 18 months. During the interview
you learn that the man has fathered a child in a previous relationship and is in good health. The
woman is 28 and reports that she has had painful menses for the past 5 or 6 years.

67. You begin to suspect that she may have endometriosis. All of information below would increase
that suspicion EXCEPT:
a. She reports that a maternal cousin has a history of endometriosis
b. She has experienced dyspareunia with deep penetration for several years
c. Her ethnicity is Caucasian
d. She report the development of abnormal bleeding in the last year
e. Her menarche began at age 9

68. A 35 years old women with 2 children came to specialist clinics with the complaint of vaginal
discharge with her pap smear result showed : CIN 3 + HPV infection, she ask for the appropiate
treatment. What is the next diagnostic procedure should be offered to the patient ?
a. HPV DNA test
b. Liquid Based Cytology Papsmear
c. Ultrasound
d. VIA
e. Colposcopy
A 45 years-old women came to hospital with major complain chronic leucorrhea. She had been
treated by various antibiotics given by general practitioner and midwives. Recently she got bleeding
each after intercourse. General condition is unwell with anemic appearance. Blood pressure
70/palpable, pulse weak 120x/minute.

69. What is the best step for this patient?


a. Performed bimanual examination
b. Performed ultrasound examination
c. Giving oxygen and putting the IV line
d. Performed emergency curretage
e. Giving tranexamic acid

70. By first impression, what would the diagnosis would like to be?
a. Uterine sarcoma
b. Cervical polyps
c. Endometrial cancer
d. Uterine myoma
e. Cervical cancer

71. What is the etio-pathogenesis of the diagnosis above?


a. Malignant transformation of myocyte
b. Hyperestrogenic intra cavitary condition
c. Human Pappiloma Virus type 6 infection
d. Herpes simplex virus infection
e. Human Pappiloma Virus type 16 infection

A 46-year-old P2 obese woman is referred from her primary care physician because of increasingly
heavy and painful menses over the last 18 months. She has tried an oral contraceptive with some
improvement of her bleeding but no improvement in her pain. She reports no other history of pelvic
pain or abnormal bleeding in the past. She has never had any infections, “down there” On
examination, you note normal external genitalia, vagina, and cervix. However, her uterus is slightly
enlarged, mildly tender, and softer than you expected. She has no adnexal mass or tenderness.

72. You explain to your patient that you think she may have adenomyosis and that it is most /likely
causing her symptoms. She is reluctant for having surgery. Which of the following management
is appropriate?
a. Start combined oral contraception (usia muda)
b. Depo provera injection every month (gejala
nyeri) c. LNG-IUS
d. GnRH analog injection for 3 months (tidak mau hamil)
e. Oral NSAID

A 30 year old patient came with complaint of infertility. Her husband is a 33-year-old who has had a
semen analysis, which was reported as normal. On further history, the patient reports that her
periods have been quiet irregular over the last year and that she missed her period in the last 4
months. She also reports insomnia, vaginal dryness, and decreased libido.

73. What is the most likely diagnosis for this patient based on her history
a. Polycystic ovarian syndrome
b. Primary ovarian insufficiency
c. Endometriosis
d. Kallmann syndrome
e. Spontaneous pregnancy

74. Which of the following condition that corresponds to the above possible diagnosis?
a. Day 3 FSH level 40 IU
b. Serum AMH level 2,6 ng/ml
c. Positive Clomiphene citrate challenge test
d. Midluteal progesterone level of 18 ng/ml
e. Follicle antral basal count of 12

A 34-year old women with primary infertility 3 years, oligomenorrhea and a body mass index (BMI) of
26. Day 23 progesterone level result was 5 ng/ml. Transvaginal ultrasound shows multiple small
follicle size 5-8 mm in both ovary. HSG shows bilateral patent tubes. Her partner's semen analysis
show a volume of 3 ml, pH of 7 and a sperm count of 20 million/ml.

75. The patient later on given clomiphene citrate 1x50 mg for 5 days, but during follow up there
were no dominant follicle. Which of the following is appropriate?
a. We should increase the dose to 100 mg/day for 5 days
b. The patient corresponds to CC resistance and should undergo laparoscopy ovarian drilling
c. The patient corresponds to CC failure and should switch to gonadotropin injection
d. Give aromatase inhibitor
e. Combine metformin with CC
Resisten : dikasi CC 3 siklus (dosis maks 150 mg/hr)  tidak terbentuk folikel dominan > 18 mm
Gagal CC : diberikan 3-6 siklus terbentuk folikel dominan / ovulasi namun tidak hamil (saran tambah
metformin)

76. Which of the following condition that we can find in patient with CC resistance?
a. Body mass index 22
b. AMH level more than 8 ng/ml
c. LH level 7 mlU/ml
d. FSH level 5 mlU/ml
e. Estradiol level 45 pg/ml

A 20-year-old lady come to outpatient clinic never had menstruation. She had no history surgery or
medication. On physical examination reveal body mass index 20kg/m2, tanner stage M3P3. From
ultrasound uterus cannot be visualized. Both ovaries was visualized size 28x25 mm and 26x20 mm
with dominant follicle size 18 mm on the left ovary.

77. Which of the following is the most appropriate diagnosis?


a. Polycycstic ovary syndrome
b. Kallman syndrome
c. Premature ovarian insufficiency
d. MRKH syndrome
e. Hyperprolactinemia
78. Which characteristic that differentiate the above diagnosis with androgen insensitivity
syndrome? a. Presence of axillary and pubic hair
b. Elevated level of prolactin
c. Elevated level of FSH
d. Low level of AMH
e. Elevated level of testosterone

79. If the diagnosis is confirmed, what is her option to achieve pregnancy?


a. Conventional IVF
b. IVF-ICSI
c. Minimal stimulation IVF
d. Oocyte donation
e. Gestational surrogacy

80. A 7-year-old girl presents to her pediatrician with her parents who are concerned about her
early sexual development. She is developing breasts, axillary hair, and pubic hair, and they are
noticing body odor. A thorough clinical workup reveals the child has an irregular, echogenic,
thickly septated ovarian mass on her left ovary. What type of tumor is responsible for this child's
clinical presentation?
a. Dysgerminoma
b. Embryonal carcinoma
c. Sertoli-Leydig cell tumor
d. Endodermal sinus tumor
e. Granulosa-theca cell tumor

A 49-year-old woman experiences irregular vaginal bleeding for 3 months duration. You performed
endometrial biopsy, which obtains copious tissue with a velvety, lobulated texture. The pathologist
report shows proliferation of grandular and stromal elements with dilated endometrial glands,
consistent with simple hyperplasia. Cytologic atypia is absent.

81. Which of the following is the best way to advise the patient?
a. She should be treated to estrogen and progestin hormone therapy
b. The tissue will progress to cancer in approximately 10% of cases
c. The tissue may be weakly premalignant and progress to cancer in approximately 1% of cases
d. She requires a hysterectomy
e. No further therapy is needed

82. Which of the following factors is protective against endometrial hyperplasia?


a. Obesity
b. Tamoxifen
c. Oral contraceptive pills (OCPs)
d. Early menarche or late menopause
e. Unopposed exogenous estrogen therapy

A 36 years old patient, PO, presents to your clinic for fertility workup. She had been married for 2
years with regular intercourse. Her menstrual cycle is normal. Her general status was normal.
Vaginal examination revealed normal findings.
83. Which of the following examination that is not included in basic workup in the patient above?
a. Hysterosalpingography
b. Ultrasonography
c. Semen analysis
d. Mid luteal progesterone examination
e. Serum AMH

84. A 30 years old patient, P0 present to your clinic for fertilitiy workup. She had been married for 2
years with regular intercourse. Her menstrual cycle is normal. Her general status was normal.
Vaginal examination revealed normal findings. The following month she came back with the
result of hysterosalpingography (see the picture below)

What will be your next plan ?


a. Repeat HSG next month
b. Schedule diagnostic laparoscopy
c. Gives clomiphene citrate and plan for natural conception
d. Gives clomiphene citrate and plan for intrauterine insemination
e. Plan for IVF

85. A 68-year-old woman presents with vulvar pruritus since the previous year that has been
increasing over the last few months. She has tried antifungal medications, which seem to help,
but the symptoms keep recurring. She went through menopause at age 49 and has not been
sexually active for 10 years. She does not use any douching products and is not taking any
antibioticsl On physical examination, you note thin white epithelium of the labia minora with
multiple red oval-shaped erosions, varying in size from 0.5 to 1.5 cm. How would you proceed?
a. Culture the vagina and treat with high-dose antifungal
b. Wide local excision of the lesions
c. Cryotherapy to eradicate the
lesions d. Punch biopsy of the vulvar
lesions
e. Treat with moderate-high potency topical steroids

86. A 27-year-old woman presents to her gynecologist for evaluation of new vulvar lesions. She first
noticed several "bumps" on her outer labia 3 weeks ago. She is otherwise without any associated
symptoms. On pelvic examination there are six flesh-colored, nontender, 1- to 3-mm verrucous
papules. Which of the following is the most likely cause of the patient's vulvar lesions?
a. Human papillomavirus (HPV) serotype 16
b. Herpes simplex virus (HSV) type 1
c. Pox virus
d. Human papillomavirus (HPV) serotype 6
e. Haemophilus ducreyi

87. 45-year-old presents for evaluation because her primary care physician has diagnosed her with
pelvic organ prolapse while performing annual care. She denies any pelvic pressure, bulge, or
difficulty with urination. Her only medical comorbidity is obesity. For asymptomatic grade 1
pelvic organ prolapse, what do you recommend?
a. Conservative management with pelvic floor muscle exercises and weight loss
b. Colpocleisis obliterate procedure
c. Gellhorn pessary
d. Round ligament suspension
e. Hysterectomy

88. A 62-year-old G2 P2 presents to the urogynecology clinic with complaints of urinary


incontinence. She has urinary urgency and can't make it to the bathroom before leaking a large
amount of urine. She gets up two to three times per night to urinate. A urinalysis and urine
culture done 1 week ago at her PCP's office are both negative. What is the most likely diagnosis
and appropriate treatment option for this type of urinary incontinence?
a. Stress incontinence, mid-urethral sling
b. Urgency incontinence, oxybutynin (anticholinergic medication)
c. Overflow incontinence, oxybutynin (anticholinergic medication)
d. Urinary fistula, surgical repair
e. Functional incontinence, bladder suspension

89. Cervical cancer of squamos type identified only microscopically, invasion is limited to measured
stomal invasion with a maximum depth of 2 mm and no wider than 6 mm should be treated
with:
a. Observation
b. Tissue ablation
c. LEEP/LLETZ S
d. Simple total hysterectomy
e. Radical hysterectomy

90. The majority of vulvar, vaginal and cervical cancers appear to have a common cause and usually
caused by:
a. High risk types of herpes simplex virus (HSV) infection
b. High risk types of human papilloma virus (HPV) infection
c. Increased exposure to endogenous estrogen
d. Increased exposure to exogenous estrogen
e. Chronic bacterial and parasitic infection

91. Childhood neoplastic ovarian masses most commonly originate from:


a. Gonadal epithelium
b. Gonadal stroma
c. Sex cords
d. Germ cells
e. Metastatic disease
92. Your patient is a 13-year-old adolescent girl who presents with cyclic pelvic pain. She has never
had a menstrual cycle. She denies any history of intercourse. She is afebrile and her vital signs
are stable. On physical examination, she has age-appropriate breast and pubic hair development
and normal external genitalia. However, you are unable to locate a vaginal introitus. Instead,
there is a tense bulge where the introitus would be expected. You obtain a transabdominal
ultrasound, which reveals a hematocolpos and hematometra. What is the most likely diagnosis?
a. Transverse vaginal septum
b. Longitudinal vaginal
septum c. Imperforate hymen
d. Vaginal atresia
e. Bicornuate uterus

93. An 18-year-old nulligravid woman presents to the student health clinic with a 4-week history of
yellow vaginal discharge. She also reports vulvar itching and irritation. She is sexually active and
monogamous with her boyfriend. They use condoms inconsistently. On physical examination,
she is found to be nontoxic and afebrile. On genitourinary examination, vulvar and vaginal
erythema is noted along with a yellow, frothy, malodorous discharge with a pH of 6.5. The cervix
appears to have erythematous punctuations. There is no cervical, uterine, or adnexal
tenderness. The addition of 10% f KOH to the vaginal discharge does not produce an amine odor.
Wet prep microscopic examination of the vaginal swabs is performed. What would you expect to
see under microscopy?
a. Branching hyphae
b. Myltinucleated giant cells
c. Scant WBC
d. Flagellated, motile organisms
e. Epithelial cells covered with bacteria

94. An 89-year-old female patient with multiple, serious medical comorbidities presents to discuss
options for treatment of her high-grade prolapse. The prolapse is externalized and becoming
ulcerated from friction against her undergarments. She cannot tolerate a pessary. Her main
priority is to "fix or get rid of this thing," but her primary care provider has cautioned against a
lengthy or open abdominal procedure. She is not interested in future intercourse. What can you
offer this patient?
a. Nothing can be done
b. Open abdominal sacral colpopexy
c. Robot-assisted laparoscopic sacral colpopexy
d. Hysterectomy with anterior and posterior colporrhaphy, vault
suspension. e. Colpocleisis

95. A patient returns for a postoperative checkup 2 weeks after a total abdominal hyserectomy for
fibroids. She is distressed because she is having continous leakage of urine from the vagina. Her
leakage is essentially continous and worsens with coughing, laughing, or movement. Given her
history and physical, you perform both a metthylene blue dye test, which is negative and an
indigo carnine test, which is positive. The most likely diagnosis is:
a. Rectovaginal fistul
b. Uretro vaginal fistula
c. Vesico vaginal fistula
d. Uretero vagina fistula
e. Impossible to distinguish

96. A 38 years old multi gravid woman complains of the painless loss of urine, beginning
immediately with coughing, laughing, lifting, or straining. Immediate cessation of the activity
stops the urine loss after only a few drops. This history is most suggestive of
a. Fistula
b. Stress incontinence
c. Urge incontinence
d. Urethral diverticulum
e. UTI

A 30 years old woman, P1A1 had performed curettage on indication molar pregnancy, she came
with βhCG IU/dL after evacuation. The first βhCG level was 10,000,000 IU/dL. The histopathology
result was hydatidiform mole, poor differentiation.

97. The principles for follow-up of hydatidiform molar pregnancy :


a. Prevent pregnancy during the follow-up period - at a minimum, for 6 months
b. Measure serum chorionic gonadotropin levels every 4 weeks
c. Measure serum chorionic gonadotropin levels every 4 months
d. Measure serum chorionic gonadotropin levels every 3 weeks
e. Observation in selected, highly individualized patients may be an option, to perform
hysterectomy

98. Criteria for diagnosis of Gestational Trophoblastic Neoplasia or Postmolar Gestational


Trophoblastic Disease:
a. Plateau of serum phCG level (± 10%) for four measurements during a period of 3 weeks or
longer - days 1,7,14,21
b. Rise of serum phCG < 10% during 3 weekly consecutive measurements or longer, during a
period of 2 weeks or more - days 1,7,14
c. Rise of serum (3hCG < 10% during 2 weekly consecutive measurements or longer, during a
period of 4 weeks or more - days 1,7,14,21
d. The serum βhCG levels remains detectable for 3 months or less
e. The serum βhCG levels remains detectable for 12 months or less

99. A 53-year-old woman presents for counseling and management of her low-grade anterior wall
prolapse. She is only symptomatic on days when she has engaged in heavy lifting. She is morbidly
obese and would like to begin a formal weight loss program. She agrees to a trial with a pessary.
You choose a supportive pessary and counsel her regarding pessary insertion, removal, cleaning,
etc. Which of the following are factors associated with associated with good response for
pessary in uterine prolapse ?
a. Vaginal length 9 cm
b. Obesity
c. History of traumatic vaginal deliveries
d. Menopausal state
e. Large genital hiatus
100. A 25 year old lady come with abnormal pap smear result. She underwent colposcopy
examination and the result is a acetowhite lesion with punctation and atypical vessels. Biopsy
result confirms CIN I with HPV DNA test positve. What do you suggest for patient ?
a. LEEP procedure
b. Reevaluation of HPV DNA
c. Cold knife conization .
d. Repeat cytology in 12 months
e. Repeat cytology in 6

months Sampun nggih.. matur

nuwun..

UNAS JULI 2019

A 33-year-old woman, GIPOAO, came to hospital with major complaint, watery leakage. She was
on her 33 weeks of gestational age. Data from medical record showed that she came previously
a week ago, complaining vaginal discharge. Vaginal swab has been done.
1. In case above, what kind of examination should you performed for establishing diagnosis.
a. Vaginal examination
b. Speculum examination
c. Blood test
d. Ultrasound
e. Simple urine test

2. You found on Leopold I, hard, round with ballottement(+). Contraction was infrequent and
weak. A What was your next plan?
a. Went for labour induction
b. Immediate C-section
c. Tocolytic and corticosteroid provision
d. Performed ultrasound
e. Performed external version

3. Ultrasound examination showed that trans-cerebellar diameter was proper to gestational age,
abdominal circumference was lower than 2.5 centile and amniotic fluid deepest pocket was
1.2 cm. What is the most likely diagnosis?
a. Growth restriction with oligohydramnios
b. Normal Growth with olygohydramnios
c. Growth restriction with normal amniotic fluid
d. Normal growth with normal amniotic fluid
e. Need another examination for establishing diagnosis.
4. Cardiotocography, showed low variability with checkmark pattern and no acceleration. What
was your interpretation and the best management through?
a. Category one, continued for fetal lung maturation
b. Category two, intrauterine resuscitation for 24 hours and reevaluation after
c. Category two went for doppler velocymetri
d. Category three, went for doppler velocymetry ultrasound exam
e. Category three delivered the baby

A 28-year-old G3P1Al presents with a history of fatigue, mild palpitation and dyspnea upon
exertion. She was unable to tolerate her prenatal vitamins during pregnancy, because of nausea.
Examination reveals pallor and spooning of her nails. Vital signs are normal. There is no
lymphadenopathy or hepatosplenomegaly.

5. If her Hb level is 8.2 g/dl., MCV 70 L, MCH 25 picograms/red cell, MCHC 22 g/dL, RDW 16%,
peripheral blood smear result was microcytic and hypochromic red cells. The most likely
diagnosis is :
a. Iron deficiency anemia
b. Megaloblastic anemia
c. Haemolytic anemia
d. Aplastic anamia
e. Acute blood loss anemia

6. To provide accurate diagnostic, your next diagnostic tools is:


a. Platelet count
b. Hb electrophoresis
c. Reticulocyte count
d. Coombs test
e. Serum ferritin

7. If the symptoms were worsening to cardiac compromise, ie, symptomatic at rest with dyspnea,
chest pain, or presyncope, your advice is :
a. High fiber diet
b. Red cell tranfusion
c. Adjunct ascorbic acid
d. Oral iron supplementation
e. Parenteral iron replacement

8. A primi gravida aged 26 is admitted with threatened preterm labour at 30 weeks and seeks
counselling with regards to antenatal corticosteroids. What are the three recognized fetal
benefits associated with antenatal corticosteroid administration in the case of premature
delivery?
a. Reduced respiratory distress syndrome, reduced incidence of hypoglycemia, reduced
neonatal death rates
b. Reduced respiratory distress syndrome, reduced VII nerve damage, reduced incidence of
hypoglycemia
c. Reduced respiratory distress syndrome, reduce incidence of pneumothorax formation,
reduced retinal disease of prematurity
d. Reduced respiratory distress syndrome, reducedintra-ventricular haemorrhage reduced
neonatal death rate
e. Reduced respiratory distress syndrome, reduced intraventricular haemorrhage reduced
necrotizing enterocolitis rates

9. A 36-year-old para 3 woman attends day assessment unit a er a growth scan for large for
gestational age at 28 weeks' gestation. She had three previous normal vaginal deliveries and was
low risk at booking. She declined first trimester screening, but had normal dating and anomaly
scans. Growth scan showed polyhydramnios, small for gestation foetus with a double bubble
sign, and she was referred to the foetal medicine unit by the sonographers. Which one of the
aneuploidies is the foetus more likely to have?
a. Down syndrome
b. Edwards syndrome
c. Klinefelter syndrome
d. Patau syndrome
e. Turner syndrome

10. Which of the following imaging finding increased suspicion of placenta accreta?
a. A small intraplacental lakes .
b. A distance less than 5 mm between uterine serosa and retroplacental vessels
c. Heterogenous signal intensity within the placenta on MRIexamination
d. Presence of no intraplacental bands on T2-weighted imaging
e. Uterine bulging in ultrasound

11. Which method that can be used for PPH patient that we apply vertical sutures for the uterine
corpus and several transverse cervicoisthmic sutures ?
a. Pereira sutures
b. O'Leary sutures
c. Bakry Sutures
d. Hayman Sutures
e. Cho Sutures

A 30 years old patient came with complaint of infertility. Her husband is a 33-year-old who has
had a semen analysis, which was reported as normal. On further history, the patient reports that
her periods have been quiet irregular over the last year and that she has not had period in the
last 3 months. She also reports insomnia, vaginal dryness, and decreased libido
12. What is the most likely diagnosis for this patient based on her history
a. Polycystic ovarian syndrome
b. Primary ovarian insufficiency
c. Endometriosis
d. Kallmann syndrome
e. Spontaneous pregnancy

13. Which of the following condition that corresponds to the above possible diagnosis?
a. Day 3 FSH level 40 IU
b. Serum AMH level 2,6 ng/ml
c. Positive Clomiphene citrate challenge test
d. Midluteal progesterone level of 15 ng/ml
e. Follicle antral basal count of 12

14. If the diagnosis was confirmed, what is her best option to achieve pregnancy?
a. Clomiphene citrate-IUI
b. Gonadotropin-IUI
c. Minimal stimulation
d. IVF with Oocyte donation
e. High dose gonadotropin IVF

A 34-year old women with primary infertility 3 years, oligomenorrhea and a body mass index
(BMI) of 26. Day 23 progesterone level result was 5 ng/ml. Transvaginal ultrasound shows
multiple small follicle size 5-8 mm in both ovary. HSG shows bilateral patent tubes. Her
partner's semen analysis show a volume of 3 ml. pH of 7 and a sperm count of 20 million/ml.
15. What is the most appropriate step of management?
a. Examine TSH and prolactin
b. Measure FSH, LH and estradiol
c. Measure serum testosterone level
d. Give Aromatase inhibitor
e. Start ovulation induction using gonadotropin

16. The patient later on given clomiphene citrate 1x50 mg for 5 days, but during follow up there
were no dominant follicle. Which of the following is appropriate?
a. We should increase the dose to 100 mg/day for 5 days
b. The patient corresponds to CC resistance and should undergo laparoscopy ovarian
drilling
c. The patient corresponds to CC failure and should switch to FSH
d. Give aromatase inhibitor
e. Combine metformin with CC

Mrs. N. 37 years old with chief complain of infertility for 6 years with history of severe
dysmenorrhea. From hysterosalpingography, both tubes were non-patent. Pelvic ultrasound
found bilateral cystic mass with internal echo sized 50 and 60 mm in diameter. Her husband
sperm examination was within normal limit
17. What is the next appropriate management?
a. Offer her IUI
b. Perform laparoscopy cystectomy and adhesiolysis
c. Give GnRH analog for 3 months continue with IUI
d. Give Dienogest 1x2mg for 6 months
e. Offer her IVF

18. Her AMH level was 0.9 ng/ml. What is the reason for performing surgery in subfertile patient
with bilateral endometrioma and diminished ovarian reserve?
a. Removal of endometrioma
b. Ablation of all endometriosis lesion
c. To prevent infection in endometrioma
d. To improve access for follicle aspiration
e. Removal of deep infiltrating endometriosis lesions
19. Which of the following is true regarding low ovarian reserve in endometriosis?
a. Ovulation rate in ovary with endometrioma is higher compared to ovary without
endometrioma
b. There is a higher density of follicle in ovary with endometrioma
c. Ovary with endometrioma has a higher response rate to gonadotropin
d. Loss of ovarian stromal appearance and fibrosis are present in ovarian cortex with
endometrioma
e. Low ovarian reserve in endometriosis only happen after surgery

A 18-year-old adolescent female complains of not having started her menses. Her breast
development is Tanner stage IV. Pubic hair development was stage 1. From vaginal examination
found a blind vaginal pouch and no uterus and cervix
20. Which of the following describes the most likely diagnosis?
a. Partial androgen insensitivity syndrome  predominat male and female, ambigu
b. Complete androgen insensitivity syndrome  female ext genetalia
c. Kallman syndrome
d. Turner syndrome
e. Polycystic ovarian syndrome

21. From ultrasound examination found no uterus and there was difficulty in identifying the
gonads. What is the next plan?
a. Prolactin measurement
b. Kariotyping
c. FSH and LH examination
d. FSH, LH and E2 examination
e. TSH, FT4 examination

22. Which of the following management will be appropriate for this condition?
a. Give progestin 14 days on of
b. Give estrogen-progestin sequential
c. Give combined oral contraception
d. Vaginal reconstructive surgery
e. Laparoscopy gonad removal

An 18-year-old young woman presents to you with a complaint of amenorrhea. She notes that
she has never had a menstrual period, but that she has mild cyclic abdominal bloating. She is
sexually active, but she complains of painful sexual intercourse. Her past medical and surgical
history is unremarkable. On physical examination, you note normal appearing axillary and pubic
hair. Her breast development is normal. Pelvic examination reveals normal appearing external
genitalia, and a shortened vagina ending in a blind pouch.  MRKH
23. Which of the following tests would be your first step in determining the diagnosis?
a. Karyotype
b. Pelvic ultrasound
c. Serum FSH
d. Serum FSH, E2
e. Diagnostic laparoscopy

24. From further examination it was found that uterus cannot be visualized but both ovaries were
normal. What is the most likely diagnosis
a. Imperforate hymen
b. Transverse vaginal septum
c. Müllerian agenesis
d. Androgen insensitivity syndrome
e. Gonadal dysgenesis

25. Which additional organ system should you be evaluating in a patient with this disorder?
a. Pancreas and duodenum
b. Cerebral circulation
c. Olfactory system
d. Renal and urinary collecting system
e. Distal gastrointestinal tract

A 34 year old female para 1, presented to our clinic with secondary amenorrhea and severe.
Progressive hirsutism. On clinical examination she was noted to have severe hirsutism and male
pattern scalp balding. Her BMI was 30 kg/m2. Laboratory results showed an elevated total
testosterone (T) level of 140 ng/dL (reference value in our laboratory is 0-80 ng/dL) and
androstenedione of 272 g/dL (reference value of 30-250 ng/dL), CT of the abdomen and pelvis
showed normal adrenal glands. Pelvic ultrasound of the pelvis demonstrated mildly prominent
ovaries, containing numerous small follicles around the periphery.
26. What is your most probable diagnosis?
a. Multicystic ovary
b. Congenital adrenal hyperplasia
c. Polycystic ovary syndrome
d. Hyperprolactinemia
e. Microadenoma pituitary

27. Your diagnosis according to ASRM/ESHRE definition, based on two of the following criteria:
a. Polycystic ovaries on ultrasound, oligo-or amenorrhea, or evidence of
hyperandrogenism
b. Polycystic ovaries on ultrasound, amenorrhea, obesity
c. Polycystic ovaries on ultrasound, amenorrhea, hirsutism
d. Presence of hyperandrogenism, ovarian dysfunction and exclusion ofrelated disorders
e. Polycystic ovaries on ultrasound, hirsutism, obesity

A 27-year-old woman presents to your office with a positive home pregnancy test and a 3- day
history of vaginal bleeding. She is concerned that she may be having a miscarriage. On
examination, the uterine fundus is at the level of the umbilicus. By her last period, she should be
around 8 weeks gestation. On pelvic examination, there is a moderate amount of blood and
vesicle-like tissue in the vaginal vault, and the cervix is closed. The lab then calls you to say that
her serum B-hCG result is greater than 1,000,000 mU/mL.
28. Which of the following is the best next step in this patient's evaluation?
a. Complete pelvic ultrasound
b. Determination of Rh status
c. Surgical intervention (suction curettage)
d. Methotrexate administration
e. Schedule a follow-up visit in 2 to 4 weeks to recheck a B-hCG level
29. The patient undergoes an uncomplicated suction D&C. The pathology report is available the
next day and is consistent with a complete molar gestation. What is the best next step in the
care of this patient's condition.
a. Repeat pelvic imaging
b. Radiation therapy
c. Chemotherapy
d. Surveillance of serum B-hCG
e. No further follow-up is required

30. During further visit, you meet with her in your office about 3 months after the index visit.
Which of the following interventions is most important to emphasize during her follow- up
period?
a. No further pregnancies are recommended
b. Await pregnancy attempt for 2 years
c. Reliable contraception during surveillance
d. Prophylactic antibiotic use during surveillance
e. Prophylactic chemotherapy to decrease the risk of persistent andrecurrent disease

A 46 years old woman experiences irregular vaginal bleeding of 3 months duration. You perform
an endometrial biopsy, which obtains copious tissue with a velvety, lobulated texture. The
pathologist report shows proliferation of glandular and stromal elements with dilated
endometrial glands, consistent with simple hyperplasia. Cytologic atypia is absent.
31. Which of the following is the best way to advise the patient?
a. She should be treated to estrogen and progestin hormone therapy.
b. The tissue will progress to cancer in approximately 10% of cases.
c. The tissue may be weakly premalignant and progresses to cancer in approximately 1%
of cases.
d. She requires a hysterectomy.
e. No further therapy is needed.

32. She agreed for a medical treatment, which of the following is the most appropriate?
a. Norethsiterone acetate 1x5 mg for 14 days on-off.
b. MPA 1x2,5 mg for 14 days on-off
c. Nomegestrel 1x2,5 mg for 14 days on-off
d. LNG IUS
e. Combined oral contraception

33. A 7-year-old girl presents to her pediatrician with her parents who are concerned about her
early sexual development. She is developing breasts, axillary hair, and pubic hair, and they are
noticing body odor. A thorough clinical workup reveals the child has an irregular, echogenic,
thickly septated ovarian mass on her left ovary. What type of tumor is responsible for this
child's clinical presentation?
a. Dysgerminoma
b. Embryonal carcinoma
c. Sertoli-Leydig cell tumor
d. Endodermal sinus tumor
e. Granulosa-theca cell tumor
A 36 years old patient, Po, presents to your clinic for fertility workup. She had been married for 2
years with regular intercourse. Her menstrual cycle is normal. Her general status was normal.
Vaginal examination revealed normal findings.
34. Which of the following examination that is not included in basic workup in the patient above?
a. Hysterosalpingography
b. Ultrasonography
c. Semen analysis
d. Mid luteal progesterone examination
e. Serum AMH

35. A 30 years old patient, P0 present to your clinic for fertilitiy workup. She had been married for 2 years
with regular intercourse. Her menstrual cycle is normal. Her general status was normal. Vaginal
examination revealed normal findings. The following month she came back with the result of
hysterosalpingography (see the picture below)

What will be your next plan?


a. Repeat HSG next month
b. Schedule diagnostic laparoscopy
c. Gives clomiphene citrate and plan for natural conception
d. Gives clomiphene citrate and plan for intrauterine insemination
b. Plan for IVF

A 52-year-old woman presents to your office. She complained about her sexual problems of
lowself esteem, and difficulties of initiating sexual intercourse, vaginal dryness and pain during
intercourse. She has the history of 3 Full term normal vaginal delivery and she had already
menopause and she has no history of hereditary disease. She underwent the lab investigation
such RBG - 129 mg/dl; Hb 10.6 mg/dl, Urea 21: Creatinine 0.5, Chest x-ray and pelvic ultrasound
studies showed no abnormalities
36. What is your diagnosis
a. Sexual desire disorder  gangguan hasrat sexual yang menurun
b. Genital arousal disorder  mudah terangsang , mupeng
c. Vaginismus
d. Orgasmic disorder tidak bisa orgasme
e. All of above

37. The most possible cause of sexual disorders of this patient is


a. Menopause
b. Multiparity
c. Alcohol uses
d. Sexual abuse
e. Pain

Mrs. 32-year old, P0, comes to your outpatient clinic due to her prolonged menstrual duration.
She reports her menstrual duration until 14 days and using 10 pads per day. She feels fatigue
easily. On physical examination, you palpate an irregularly enlarged uterus, non tender with firm
contour. Cervix appears to be hyperemic without mass appearance or other abnormalities.
38. By which mechanism does fibroid creates a hyperestrogenic environment requisitesfor their
growth?
a. Fibroid contains higher level of cytochrome P450 aromatase, which allows for
conversion of androgens to estrogen.
b. Fibroid converts more estradiol to estrone X
c. Fibroid cells contain less density of estrogen receptors compared with normal
myometrium
d. Increased adipose conversion of androgens to estrogen
e. All of the above

39. What is the cause of necrotic and degenerative process in fibroids?


a. Mitotic activity
b. Limited blood supply within tumors
c. Chromosomal defects
d. Hyper perfusion
e. Cytogenetic mutations

40. A 45-year-old presents for evaluation because her primary care physician has diagnosed her
with pelvic organ prolapse while performing annual care. She denies any pelvic pressure,
bulge, or difficulty with urination. Her only medical comorbidity is obesity. For asymptomatic
grade 1 pelvic organ prolapse, what do you recommend?
a. Conservative management with pelvic floor muscle exercises and weight loss
b. Colpocleisis obliterative procedure
c. Gellhom pessary
d. Round ligament suspension
e. Hysterectomy

41. A 62-year-old G2 P2 presents to the urogynecology clinic with complaints of urinary


incontinence. She has urinary urgency and can't make it to the bathroom before leaking a
large amount of urine. She gets up two to three times per night to urinate. A urinalysis and
urine culture done 1 week ago at her PCP's office are both negative. What is the most likely
diagnosis and appropriate treatment option for this type of urinary incontinence?
a. Stress incontinence, mid-urethral sling
b. Urgency incontinence, oxybutynin (anticholinergic medication)
c. Overflow incontinence, oxybutynin (anticholinergic medication)
d. Urinary fistula, surgical repair
e. Functional incontinence, bladder suspension

42. Childhood neoplastic ovarian masses most commonly originate from:


a. Gonadal epithelium
b. Gonadal stroma
c. Sex cords
d. Germ cells
e. Metastatic disease

43. Your patient is a 13-year-old adolescent girl who presents with cyclic pelvic pain. She has
never had a menstrual cycle. She denies any history of intercourse. She is afebrile and her vital
signs are stable. On physical examination, she has age-appropriate breast and pubic hair
development and normal external genitalia. However, you are unable to locate a vaginal introitus.
Instead, there is a tense bulge where the introitus would be expected. You obtain a
transabdominal ultrasound, which reveals a hematocolpos and hematometra. What is the most
likely diagnosis?
a. Transverse vaginal septum
b. Longitudinal vaginal
septum c. Imperforate hymen
d. Vaginal atresia
e. Bicornuate uterus

44. An 18-year-old nulligravid woman presents to the student health clinic with a 4-week history
of yellow vaginal discharge. She also reports vulvar itching and irritation. She is sexually active
and monogamous with her boyfriend. They use condoms inconsistently On physical
examination, she is found to be nontoxic and afebrile. On genitourinary examination, vulvar
and vaginal erythema is noted along with a yellow, frothy, malodorous discharge with a pH of
6.5. The cervix appears to have erythematous punctuations. There is no cervical, uterine, or
adnexal tenderness. The addition of 10% KOH to the vaginal discharge does not produce an
amine odor. Wet prep microscopic examination of the vaginal swabs is performed. What
would you expect to see under microscopy?
a. Branching hyphae
b. Multinucleated giant cells
c. Scant WBC
d. Flagellated, motile organisms
e. Epithelial cells covered with bacteria

45. A 89-year-old female patient with multiple, serious medical comorbidities presents to discuss
options for treatment of her high-grade prolapse. The prolapse is externalized and becoming
ulcerated from friction against her undergarments. She cannot tolerate a pessary. Her main
priority is to "fix or get rid of this thing." but her primary care provider has cautioned against
a lengthy or open abdominal procedure. She is not interested in future intercourse. What can
you offer this patient?
a. Nothing can be done
b. Open abdominal sacral colpopexy
c. Robot-assisted laparoscopic sacral colpopexy
d. Hysterectomy with anterior and posterior colporthaphy, vault suspension
e. Colpocleisis

46. A patient returns for a postoperative checkup 2 weeks after a total abdominal hyserectomy
for fibroids. She is distressed because she is having continous leakage of urine from the
vagina. Her leakage is essentially continous and worsens with coughing, laughing, or
movement. Given her history and physical, you perform both a metthylene blue dye test,
which is negative and an indigo carine test, which is positive. The most likely diagnosis is:
a. Rectovaginal fistula
b. Uretro vaginal fistula
c. Vesico vaginal fistula
d. Uretero vagina fistula
e. Impossible to distinguish

47. A 38 years old multi gravid woman complains of the painless loss of urine, begining
immediately with coughing, laughing, lifting, or straining. Immediate cessation of the activity
stops the urine loss after only a few drops. This history is most suggestive of
a. Fistula
b. Stress incontinence
c. Urge incontinence
d. Urethral diverticulum
e. UTI

48. A 25 year old lady come with abnormal pap smear result. She underwent colposcopy
examination and the result is a acetowhite lesion with punctation and atypical vessels.
Biopsy result confirms СIN I with HPV DNA test positve. What do you suggest forpatient?
a. LEEP procedure
b. Reevaluation of HPV DNA
c. Cold knife conization
d. Repeat cytology in 12 months
e. Repeat cytology in 6 months

A 45 years old woman presents to your office for consultation regarding her symptoms of
menopause. She stopped having periods 13 months ago after TAH-BSO operation and is having
severe hot flushes. The hot flushes are causing her considerable stress.

49. What should you tell her regarding the psychological symptoms of the climacteric?
a. They are not related to her changing levels of estrogen and progesterone.
b. They commonly include insomnia, irritability, frustration, and malaise.
c. They are related to a drop in gonadotropin levels.
d. They are not affected by environmental factors.
e. They are primarily a reaction to the cessation of menstrual flow

50. Which of the following is an absolute contraindication for hormonal therapy?


a. Diabetes mellitus
b. Coronary heart disease
c. Endometriosis
d. Impairment of liver function
e. Migraine

51. Which of the following medication that you will give for hormonal therapy?
a. Estrogen only therapy
b. Biphasic combined oral contraception
c. Monophasic combined oral contraception
d. Triphasic combined oral contraception
e. Sequential estrogen-progestin therapy
Mrs. 24 years old comes to emergency room with complaints of shortness of breath since 3 days
ago. She is 38" weeks of gestation and this is her first pregnancy. On physical examination BP
130/90 HR 110 beats per minute, RR 28 times per minute. Chest examination reveals soft rales
on both lungs. cardiomegaly and systolic murmur
on the apex.
52. What further cardiac evaluation that is appropriate for this case to assess ventricle function?
a. Electrocardiogram
b. Echocardiography
b. Chest x ray
c. Troponin T
d. CKMB

53. Which of the following is included in classical criteria for peripartum cardiomyopathy?
a. Electrocardiogram showing atrial fibrillation
b. Development of cardiac failure within 5 months post partum  6 bln post partum
c. Ejection fraction less than 50 percent  EF PPCM : < 45 %
d. Presence of mitral regurgitation
e. Dilated atrial dimension on echocardiography

54. This patient later on diagnosed having peripartum cardiomyopathy and decided to reduce
her preload. What kind of medication that can achieve that goal?
a. Hydralazine
b. Digoxin
c. Enalapril
d. Furosemide
e. Amlodipine

55. Which of the following is not included in diagnostic criteria of peripartum cardiomyopathy
a. Development of cardiac failure in the last month of pregnancy
b. Absent of an identifiable cause of cardiac failure
c. Left ventricular systolic dysfunction
d. Decreased ejection fraction
e. Development of cardiac failure within 8 months of pregnancy

A 34-year-old lady, G3P2A0 presented to delivery ward at 30 weeks gestation with worsening
abdominal pain for few hours. She had also had some vaginal bleeding within the past hour. Her
uterus was tender and firm to palpation. She was found to havelow-amplitude, high-frequency
uterine contractions, and the fetal heart rate tracing showed reduced variability. Her blood.
pressure was 160/100 mmHg, pulse rate 106 bpm. Conjungtiva was pale. She has had a +2
proteinuria. She did her antenatal care in your hospital and ultrasound examination was
performed 3 times with no remarkable abnormalities.
56. The most likely diagnosis is :
a. Vasa previa
b. Preterm labor
c. Placenta previa
d. Placental abruption
e. Preterm Premature Rupture of Membrane (PPROM)
In this case, your ultrasound findings are posteriorly implanted placenta, retroplacental
hematoma, increased placental thickness and echogenicity and massive subchorionic collection
of blood.
57. The most likely complication that might be occur is:
a. Mild preeclampsia
b. Severe preeclampsia
c. Eclampsia
d. DIC
e. HELLP Syndrome

58. From obstetrical examination you found her cervix was unfavorable. Your next plan is to
deliver the baby by:
a. Vaginal delivery
b. Elective C-section
c. Emergency C-section
d. Operative vaginal delivery
e. Observation until the cervix was favorable

59. You are counseling a couple in your clinic who desire VBAC. Her baby is in a vertex
presentation, appropriate size for 37 weeks, and her previous low transverse procedure was
for breech presentation. You have to give inform consent about VBAC. In providing informed
consent, in which of the following ways do you explain the risk of uterine rupture?
a. Less than 1%
b. Between 2% and 5%
c. Between 15-20%
d. Depend on the length of her labor
e. Depend on the location and proximity of the scar site to the placental implantation

60. What is the most concerning complication related to multiple repeated cesarean deliveries?
a. Bladder injury
b. Placenta accreta
c. Organs adhesion
d. Prolonged wound recovery
e. Placenta previa

A 17-years-old G2P1 woman with no prenatal care at 29 weeks' gestation presents with painful
contractions and pressure. Her cervix is 1 cm, 40% effaced, and breech at-station 2. There is no
evidence of ruptured membranes. Her contractions are every 4 minutes. FHR are 150 bpm with
accelerations. Matemal vital signs are temperature 36.8°C, pulse 96x/m, BP 110/72 mmHg.
61. What should you do?
a. C-section
b. Performed fetal fibronectin
c. Begin tocolytic agents and corticosteroids
d. Observe the cervical changes and labor progress
e. Perform amniocentesis to rule out chorioamnionitis

62. What fetal complication is associated with the nonsteroidal anti-inflamatory agent
Indomethacin as tocolytics agent?
a. Hydramnios
b. Achondroplasia
c. Pulmonary valve atresia
d. Bronchopulmonary dysplasia
e. Premature closure of the ductus arteriosus

63. The nonstress test (NST) has which of the following characteristics?
a. Low positive predictive value
b. Low specificity (with reactive NST)
c. Low false-positive rate (with nonreactive NST)
d. FHR reactivity depends on normal cardiac development
e. Acceleration without fetal movement should not be accepted

64. Corticosteroids administered to women at risk for preterm birth have been demonstrated to
decrease rates of neonatal respiratory distress if the birth is delayed for at least what amount
of time after the initiation of therapy?
a. 12 hours
b. 24 hours
c. 36 hours
d. 48 hours
e. 72 hours

65. A 24-year-old patient (Parity 2) has just delivered vaginally an infant weighing 4300 g after a
spontaneous uncomplicated labor. Her prior obstetric history was a low uterine segment
transverse cesarean section for breech. She has had no problems during the pregnancy and
labor. The placenta delivers spontaneously. There is immediate vaginal bleeding of greater
than 500 cc. Although all of the following can be the cause for postpartum hemorrhage,
which is the most frequent cause of immediate hemorrhage as seen in this patient?
a. uterine atony
b. Coagulopathies
c. Uterine rupture
d. Retained placental fragments
e. Vaginal and/or cervical lacerations

Mrs. X, 24 yo came to the ER with complaints of headaches since the last day of examinations
obtained expecting her first child, gestational age 32 weeks with blurred vision and denied
heartburn. On physical examination found BP 190/120 mmHg, pulse 90 x/m, breathing 16 x/m.
Leopold found the lower left back head, FHR 140 bpm, irregular contraction. Pelvic score of 1 was
found, pelvis size wide. Laboratory investigation; hemoglobin 11.5 g%, platelets 490007 mm3,
LDH 754 iu/L, Proteinuria +2, ALT 60 u/L,
AST 75 u/L
66. What is the best diagnosis for Mrs. X
a. Preeclampsia
b. HELLP syndrome
c. Chronic hypertension
d. Gestational hypertension
e. Superimposed preeclampsia
67. What is the most clinically effective antihypertensive agent for Mrs. X
a. ISDN
b. Atenolol
c. Nifedipine
d. Furosemide
e. Methyldopa

68. Which antihypertension drugs can cause fetal growth restriction?


a. Nifedipine
b. Atenolol
c. Hydralazine
d. Captopril
e. Methyldopa

69. A woman is being treated with magnesium sulphate. There is concern about magnesium
toxicity. What is the first sign of magnesium toxicity?
a. Bradycardia
b. Reduced consciousness
c. Respiratory depression
d. Decreased urine output
e. Loss of deep tendon reflexes

70. With two home pregnancy tests and ultrasound revealed 6-7 wga pregnancy. As her
pregnancy continues, you would expect her cardiac output to increase by which of the following
mechanisms:
a. An increased heart rate alone
b. A decrease in systemic vascular resistance
c. first an increase in stroke volume, then an increase in heart rate
d. Cardiac output would not change significantly until the third trimester
e. An increase in systemic vascular resistance facilitated by elevated progesterone levels

71. Which of the following is true regarding the physiologic changes she might expect during her
pregnancy?
a. An overall decrease in the number of WBC and platelets
b. Gastric emptying and large bowel motility are increased in pregnancy
c. An increase in the tidal volume along with an increase in total lung capacity (TLC)
d. BUN and creatinine will decrease as a result of an increase in glomerular filtration
rate (GFR)
e. Nausea and vomiting that should be treated aggressively with antiemetics and
intravenous hydration

72. A woman with a previous stillbirth and postpartum DVT is found to have lupus anticoagulant
and medium-titre Immunoglobulin M (IgM) anticardiolipin antibodies (aCL) on two occasions.
In a subsequent pregnancy:
a. Warfarin should be discontinued
b. She has an increased risk of miscarriage
c. She requires antibiotic prophylaxis to cover delivery
d. Low dose aspirin should be discontinued at 34 weeks
e. She does not require postpartum heparin if she has vaginal delivery
A 28-year-old GIP1 woman is being discharged from the hospital on postoperative day 4 after
having received a primary low transverse cesarean section for breech presentation, with an
estimated blood loss of 700 ml. Her pregnancy was otherwise no complication and her hospital
course was also no complication. Ten days after Cesarean section, the patient came complaining
of abdominal pain and fever. Fundal height 2 fingers below navel

73. What is the most likely probable diagnosis of the patient?


a. Metritis
b. Mastitis
c. Typhoid fever
d. Urinary tract infection
e. Breast engorgement

74. A patient calls your clinic complaining of continued heavy vaginal bleeding. She had an
"uncomplicated" vaginal birth 2 weeks ago of her second child. What is the most likely
diagnosis from the following differentials?
a. Uterine atony
b. Uterine rupture
c. Coagulopathies
d. Vaginal lacerations
e. Retained placental fragments

A 29-year-old G2P1 woman came to outpatient clinic with obesity, a history of GDM in the prior
pregnancy, and a strong family history for type 2 diabetes mellitus (T2DM) presents at 7 weeks'
gestation. In her previous pregnancy, she required insulin therapy. She delivered at 39 weeks and
her baby boy weighed 4.300 g.

75. In addition to the routine prenatal laboratory tests, what other testing will you obtain at this
point?
a. HbA1c
b. No other testing needed
c. An ultrasound to estimate gestational age
d. Perform a glucose challenge test at first visit
e. Perform a glucose challenge test at 24 weeks' gestation

76. Laboratory test results return, and her fasting blood glucose is 145 mg/dL. An ultrasound
reveals the pregnancy to be 7 weeks and 2 days, consistent with LMP. Hemoglobin A1c is
7,5%. Her diagnosis is:
a. GDM
b. T1DM
c. T2DM
d. no diabetes in pregnancy
e. Impaired glucose tolerance

77. By which mechanism of transfer does glucose cross the placenta?


a. Carrier mediated active transport
b. Channels
c. Facilitated diffusion
d. Simple diffusion
e. Solvent drag

78. A 41-year-old women present at 36 weeks of gestation in active labour. The ultrasound scan
at 32 weeks showed low lying placenta. Immediately after rupture of the membrane, she
started bleedino vaginally with associated cardiotocography (CTG) of non reassuring. What is
the most likely diagnosis?
a. Vasa previa
b. Placenta previa
c. Placenta accreta
d. Abruptio placenta
e. Low lying placenta

A woman has a booking scan in 16 weeks gestation, which reveals a monochorionic diamniotic
twin pregnancy. She asks you about the risks regarding her pregnancy.
79. Regarding complications of twin pregnancy:
a. Caesarean section is the preferred route of delivery
b. With significant growth discordance, particularly when the first twin is the smalle
c. In twin-to-twin transfusion syndrome, the haemoglobin levels for bothtwins are often
not discordant
d. In a twin pregnancy with one fetal loss in the third trimester, in 90 per cent of cases the
remaining twin will be delivered within 72 h
e. Twin reversed arterial perfusion sequence is associated with high mortality in the
recipient twin due to prematurity and intra-uterine cardiac failure

80. The timing of separation of the embryo in monochorionic diamniotic is


a. up to 4 days
b. 4-7 days
c. 7-14 days
d. 14 days
e. 28 days

81. Regarding twin to twin transfusion syndrome:


a. The donor develop hydrops
b. The recipient develops polyhydramnios
c. Quinterro classification is up to Quinterro IV
d. The perinatal mortality in twins reaches to 85%
e. Complicates up to 35 per cent of dichorionic multiple pregnancies

82. The most frequent twin pregnancy is:


a. Conjoined twins
b. Dizygotic twins
c. Dichorionic diamniotic
d. Monochorionic diamniotic
e. Monochorionic monoamniotic

83. Which of the following statements regarding chorionocity is true?


a. A dichorionic pregnancy is always dizygotic
b. Monochorionic membranes have four layers
c. Monochorionic twins are always monozygotic
d. Determination of chorionicity is easiest in the second trimester
e. Complications in twin pregnancy is more frequent in dichorionic pregnancy

A 38 years old P6 lady is being operated for abdominal delivery. Unfortunately uterine
contraction is not good. The patient suffers for massive bleeding. The operator quickly decides to
perform uterine removal in order to stop the bleeding.
84. Which artery that should be blocked if the operator would like to stop the blood flows to the
uterine artery?
a. Pudenda artery
b. Abdominal aorta
c. Hypogastric artery
d. Common iliac artery
e. Uterine and ovarian arteries

85. The uterine artery:


a. Gives a branch to ovary
b. Runs at the back of the ureter
c. May anastomose with femoral artery
d. Gives myometrium vascularization only
e. Is a branch of the anterior division of the internal iliac artery

86. The ovarian arteries


a. Are crossed by the ureters
b. Arise just above the renal artery
c. Reach the ovary through round ligament
d. Reach the ovary through infundibulo-pelvic ligament
e. Anastomose with the descendent branch of uterine artery

A 22 years old female, Gl at 26 weeks gestation, presents to the office for her routine obstetrical
visit. Currently, she is complaining shortness of breath. She has no other complaints. On physical
examination: Blood pressure 100/70 mmHg, PR 90 bpm, RR 22x/m. No abnormality was found in
chest examinations. BGA results: pH 7,45 (7.35 - 7,45), pO2 103 mmHg (75 - 100), pCO2 28
mmHg (35-45), HCO3' 17 mEq/L (22-26), BE 2 mmol/L (-2-2), O2 sat 99% (>95%)

87. What is your explanation about the cause of symptom of this patient?
a. Cardiac output increases 20%
b. Maternal blood volume increases 50%
c. Decrease in stroke volume and blood viscosity
d. The uterus and the diaphragm becomes elevated
e. The heart is displaced upward, and somewhat to the right with rotation on its long axis.

88. What is the result of blood gas analysis (BGA) stated above?
a. Normal BGA
b. Metabolic acidosis
c. Metabolic alkalosis
d. Respiratory acidosis
e. Respiratory alkalosis

89. What is the most appropriate next management in this patient?


a. Ask patient to perform echocardiography examination
b. Ask patient to perform chest radiography examination
c. Admit the patient to the hospital and give non-rebreathing mask
d. Counsel the patient that this is physiologic changes during pregnancy
e. Terminate the pregnancy because the symptom will become worsen in 32 weeks
pregnancy

90. What is ECG finding consider normal during pregnancy?


a. Specific ST changes
b. Increased heart rate (15%)
c. Inverted T waves in lead II
d. 15 degree right axis deviation
e. Irreversible ST waves changes

A 32 years old female, G1 at 8 weeks gestation presents to the office for her routine obstetrical
visit. She asks you about the nutrition demand during pregnancy. Her BMI is 24 kg/m. No
remarkable past medical history is noted.

91. According to WHO Asian criteria, her BMI is classified as:


a. Normal
b. Underweight
c. Overweight
d. Obese type 1
b. Obese type 2

92. She ask you what is the optimal total weight gain during her pregnancy:
a. <5 kg
b. 5-9 kg
c. 7-11,5 kg
d. 11,5-16 kg
e. 12,5-18 kg

93. What is the most likely risk of this patient?


a. Anemia
b. Congenital anomaly
c. Gestational diabetes
d. Spontaneous abortion
e. Post partum haemorrhage

94. A 29 year old woman with a positive pregnancy test presents with a good history of tissue
expulsion vaginally. A transvaginal ultrasound scan shows an empty uterus with an
endometrial thickness of 11 mm. Regarding her diagnosis, you consider that:
a. She has had a complete miscarriage and needs no further treatment
b. She has had a pregnancy of unknown location and needs further investigations
c. She should be offered a hysteroscopy
d. She should be offered medical management of miscarriage
e. A laparoscopy should be performed to exclude an ectopic pregnancy

95. A 39 years old female G2P1AO, 15 weeks pregnant presents to your clinic for having routine
ANC. On physical examination, you found her fundal height equals umbilical point. You
performed ultrasound and saw a multilocular hypoechoic mass sized 10 cm (in diameter) in
her left adnexa. No free fluid in her abdomen and pelvis. What is your consideration in this
case?
a. The incidence of adnexal masses in pregnancy is 1%
b. The incidence of ovarian cancers in pregnancy is between 1:1000
c. The most common type of benign ovarian cyst in pregnancy is a mature teratoma
d. The most common histopathological subtype for malignant ovarian tumor in pregnancy is
epithelial ovarian tumor
e. The resolution rate of adnexal masses in the second trimester of pregnancy is 60- 70%

96. You have checked her CA125 serum level and the result was 350 /ml. You performed
conservative surgical staging by laparotomy because her frozen section revealed malignancy
cyst. After 1 week, pathology result serous papillary carcinoma of the left ovary. She is
planned for chemotherapy. What will you inform to her regarding chemotherapy for ovarian
cancer during pregnancy?
a. In a patient with ovarian cancer in pregnancy receiving chemotherapy the delivery
should be planned at completion of chemotherapy
b. Use of chemotherapy in pregnancy generally considered safe after 20 weeks of gestation
c. CNS and neural tube complications occur during the week 8-12 weeks in pregnancy
d. This percentage of patients receiving chemotherapy in pregnancy who develop major
congenital malformations is 30-40%
e. Cardiovascular defects are common congenital malformations in platin based
chemotherapy regimens

A 28 yo woman. Gl 36 weeks of gestational age, went to your clinic to do routine antenatal care.
During ultrasound, the doctor told her that she will be exnecting baby boy with estimated fetal
weight 2500 g, however, amniotic fluid considered to be less than normal. Then you asked the
patient to drink minimal of 2L of water a day and get herself another ultrasound within 3 days to
evaluate the amniotic fluid.

97. Amniotic fluid volume is a balance between production and resorption. What is the primary
mechanism of fluid resorption?
a. Fetal breathing
b. Fetal swallowing
c. Absorption across fetal skin
d. Absorption by fetal kidneys
e. Filtration by fetal kidneys

98. In a normal fetus at term, what is the daily volume of fetal urine that contributes to the
amount of amniotic fluid present?
a. 200 ml
b. 250 ml
c. 500 ml
d. 750 ml
e. 1000 ml.

Mrs. B, 37-years-old came to your office at 32 weeks of gestation according to her last menstrual
period. She has no ultrasound examination before and did not do her routine antenatal care. The
vital sign is within normal limit. She has body mass index 19 kg/m2. During physical examination
the uterine fundal height is 27 cm. From ultrasound examination, the fetus has biometric values
that correlate with 30 weeks fetus

99. Which of the following is the next best step in managing this patient?
a. Antenatal care routinely for the next 2 weeks
b. Evaluate maternal status and comorbidities
c. Consider deliver the baby
d. Repeat sonography for fetal growth in 2 weeks
e. Doppler velocimetry evaluation every 3 days

100. According to algorithm for management of fetal-growth restriction, you evaluate the
Doppler Velocimetry then find reversed end-diastolic flow and oligohydramnios. What is the
appropriate management at this time?
a. Regular fetal testing
b. Weekly evaluation of amniotic fluid
c. Consider corticosteroids for lung maturation
d. Deliver the baby
e. Reevaluate middle cerebral arteries and ductus venosus

UNAS NOVEMBER 2018

A woman has a booking scan in weeks gestation, which reveals a monochorionic diamniotic twin
pregnancy. She ask you about the risk regarding her pregnancy.
1. Regarding complications of twin pregnancy
a. Caesarean section is the preferred route of delivery
b. With significant growth discordance, particularly when the first twin is the smaller
c. In twin to twin transfusion syndrome, the haemoglobin levels for both twins are often
not discordant
d. In a twin pregnancy with one fetal loss in the third trimester, in 90 per cent of cases
the remaining twin will be delivered within 72 h
e. Twin reversed arterial perfusion sequence is associated with high mortality in
recipient twin due to prematurity and intra-uterine cardiac failure

2. The timing of separation of the embryo in monochorionic diamniotic is :


a. Up to 4
days b. 4-7 days
c. 7-14 days
d. > 14 days
e. > 28 days

3. Regarding twin to twin transfusion syndrome :


a. The donor develop hydrops
b. The recipient develops polyhydramnios
c. Quintero classification is up to IV
d. The perinatal mortality in twins reaches to 85 %
e. Complicates up to 35 per cent of dichorionic multiple pregnancies

4. The most frequent twin pregnancy is


a. Conjoined
twins b. Dizygotic
twins
c. Dichorionic diamniotic
d. Monochorionic diamniotic
e. Monochorionic monoamniotic

5. Which of the following statements regarding chorionicity is true?


a. A dichorionic pregnancy is always dizygotic
b. Monochorionic membranes have four layers
c. Monochorionic twins are always monozygotic
d. Determination of chorionicity is easiest in the second trimester
e. Complications in twin pregnancy is more frequent in dichorionic pregnancy

A 38 years old lady is being operated for abdominal delivery. Unfortunately uterine contraction is
not good. The patient suffers for massive bleeding. The operator quickly decides to perform uterine
removal in order to stop bleeding.
6. Which artery that should be blocked if the operator would like to stop the blood flows to the
uterine artery?
a. Pudenda artery
b. Abdominal artery
c. Hypogastric artery
d. Common iliac artery
e. Uterine and ovarian arteries

7. The uterine artery :


a. Gives a branch to ovary
b. Runs at the back of the ureter
c. May anastomose with femoral artery
d. Gives myometrium vascularization only
e. Is a branch of the anterior division of the internal iliac artery

8. The ovarian arteries :


a. Are crossed by the ureters
b. Arise just above the renal artery
c. Reach the ovary through round ligament
d. Reach the ovary through infundibulo-pelvic ligament
e. Anastomose with the descendent branch of uterine artery

A 22 years old female, G1 at 26 weeks gestation, presents to office for her routine obstetrical visit.
Currently, she is complaining shortness of breath. She has no other complaints. On physical
examination: blood pressure 100/70 mmHg, PR: 90 bpm, RR 22x/m. No abnormality was found in
chest examinations. BGA result pH 7.45 (7.35-7.45), pO2 103 mmHg (75-100), pCO2 28 mmHg (35-
45), HCO3 17 mEq/L (22-26), BE 2 mmol/L( -2 – 2), O2 sat 99 % (>95%)
9. Whats is your explanation about the cause of symptom of this patient?
a. Cardiac output increases 20%
b. Maternal blood volume increases 50%
c. Decrease in stroke volume and blood viscosity
d. The uterus and the diaphragm become elevated
e. The hearts is displaced upward, and somewhat to the right with rotation on its long
axis

10. What is the result of blood gas analysis (BGA) stated above?
a. Normal BGA
b. Metabolic acidosis
c. Metabolic alkalosis
d. Respiratory acidosis
e. Respiratory alkalosis

11. What is the most appropriate next management in this patient?


a. Ask patient to perform echocardiography examination
b. Ask patient to perform chest radiography examination
c. Admit the patient to the hospital and give non rebreathing mask
d. Counsel the patient that this physiologic changes during pregnancy
e. Terminate the pregnancy because the symptom will become worsen in 32 weeks
pregnancy

12. What is ECG finding consider normal during pregnancy?


a. Specific ST changes
b. Increased heart rate (15%)
c. Inverted T waved in lead II
d. 15 degree right axis deviation
e. Irreversible ST wave changes

A 32 years old female, G1 at 8 weeks gestation, presents to the office for her routine obstetrical visit.
She asks you about the nutrition demand during pregnancy. Her BMI is 24kg/m2. No remarkable
past medical history is noted.
13. According to WHO Asian criteria, her BMI is classified as
: a. Normal (18,5 – 22,9) ↑BB: 11,5-16 kg
b. Underweight (< 18,5) ↑BB: 12,5-18 kg
c. Overweight (23-27,5) ↑BB: 7-11,5 kg
d. Obese type 1 (>27,5) ↑BB: 5-9 kg
e. Obese type 2

14. She ask you what is the optimal total weight gain during her pregnancy (BB ideal + (usia
kehamilan x 0,35)):
a. < 5kg
b. 5-9 kg
c. 7-11,5 kg
d. 11,5-16 kg
e. 12,5- 18 kg

15. What is the most likely risk of this patient?


a. Anemia
b. Congenital anomaly
c. Gestational diabetes
d. Spontaneous abortion
e. Postpartum hemorhage

16. A 29 years old woman with a positive pregnancy test presents with a good history of tissue
expulsion vaginally. A transvaginal ultrasound scan shows an empty uterus with an
endometrial thickness of 11 mm. Regarding her diagnosis, you consider that :
a. She has had a complete miscarriage and needs no further treatment
b. She has had a pregnancy of unknown location and needs further investigations
c. She should be offered a hysteroscopy
d. She should be offered medical management of miscariage
e. A laparoscopy should be performed to exclude an ectopic pregnancy

17. Which of the following is true about the screening and diagnosis of spina bifida ?
a. A maternal serum alpha-fetal protein (MSAFP) of 2.5 multiples of the median (MoM)
is diagnostic of an NTD
b. First trimester ultrasound screening for NTDs primarily involves identification of a
normal posterior fossa during aneuploidy screening to rule out the presence of a
Arnold-Chiari II malformation
c. Magnetic resonance imaging (MRI) has proven to be a better diagnostic tool for NTDs
than ultrasound
d. The banana sign is indicative of ventriculomegaly, which is present in most cases of
open NTDs
e. Diagnosis of lesion level requires three-dimensional ultrasound or MRI modalities

18. On prenatal ultrasound, which of the following feature characterise gastrochisis?


a. The abdominal wall defect is superior to cord insertion
b. Ectopia cordis is present
c. The abdominal wall defect is lateral to cord insertion
d. The abdominal wall defect is lower than cord insertion
e. The bladder cannot be visualized

19. A 39 years old female G2P1A0, 15 weeks pregnant presents to your clinic for having routine
ANC. On physical examination, you found her fundal height equals umbilical point. You
performed ultrasound and saw a multilocular hypoechoic mass sized 10 cm ( in diameter) in
her left adnexa. No free fluid in her abdomen and pelvis. What is your consideration in this
case?
a. The incidence of adnexal masses in pregnancy is 1%
b. The incidence of ovarian cancers in pregnancy is between 1:1000
c. The most common type of benign ovarian cyst in pregnancy is a mature teratoma
d. The most common hystopatological subtype for malignant ovarian tumor in
pregnancy is epithelial ovarian tumor
e. The resolution rate of adnexal masses in the second trimester of pregnancy is 60-70%

20. The patient does not believe that she has ovarian cyst during pregnancy. She is really
concern about the possibility of malignancy. Regarding this situation, what would you inform
her?
a. The most common mode of presentation of an adnexal mass is pain
b. The sensitivity of detection of ovarian cysts on clinical examination alone is less than
5%
c. The size of ovarian cyst that should prompt investigation for malignancy is 10 cm
d. The validated sensitivity and specificity of IOTA rules on ultrasound evaluation of an
ovarian cyst is sensitivity : 78%, specificity : 87%
e. The sensitivity and specificity of MRI in the diagnosis of a malignancy is 100 and 94%
respectively

21. You have checked her CA125 serum level and the result was 350µ/ ml. You performed
conservative surgical staging by laparotomy because her frozen section revealed malignancy
cyst. After 1 week, pathology result serous papilary carcinoma of the left ovary. She is
planned
for chemoterapy. What will you inform to her regarding chemoterapy for ovarian cancer
during pregnancy?
a. in a patient with ovarian cancer in pregnancy receiving chemotherapy the delivery
should be planned at completion of chemotherapy
b. Use of chemotherapy in pregnancy generally considered safe after 20 weeks of
gestation
c. CNS and neural tube complication occur during the week 8-12 weeks in pregnancy
d. This percentage of patients receiving chemotherapy in pregnancy who develop major
congenital malformations is 30-40%
e. Cardiovascular defects are common congenital malformations in platin based
chemotherapy regimens

A 28 yr old woman, G1 36 weeks of gestational age, went to your clinic to do routine antenatal care.
During ultrasound, the doctor told her that she will be expecting baby boy with estimated fetal
weight 2500g, however, amniotic fluid considered to be less than normal. Then you asked the
patient to drink minimal of 2L water a day and get herself another ultrasound within 3 days to
evaluate the amniotic fluid.
22. Amniotic fluid volume is a balance between production and resorption. What is the
primary mechanism of fluid resorption?
a. Fetal breathing
b. Fetal swallowing
c. Absorption across fetal skin
d. Absorption by fetal kidneys
e. Filtration by fetal kidneys

23. In a normal fetus at term, what is the daily volume of fetal urine that contributes to the
amount of amniotic fluid present?
a. 200 mL
b. 250 mL
c. 500 mL
d. 750 mL
e. 1000 mL

Mrs. B, 37-yr old came to your office at 32 weeks of gestation according to her last menstrual
period. She has no ultrasound examination before and did not do her routine antenatal care. The
vital sign is within normal limit. She has body mass index 19 kg/m2. During physical examination, the
uterine fundal height is 22 cm. From ultrasound examination, the fetus has biometric values that
correlate with 30 weeks fetus.
24. Which of the following is the next best step in managing this patient?
a. Antenatal care routinely for the next 2
weeks b. Evaluate maternal status and
comorbidities
c. Consider deliver the baby
d. Repeat sonography for fetal growth in 2 weeks
e. Doppler velocimetry evaluation every 3 days

25. According to algorithm for management of fetal growth restriction, you evaluate the
Doppler Velocimetry then find a reserved end diastolic flow and oligohydramnios. What is
the appropriate management at this time?
a. Regular fetal testing
b. Weekly evaluation of amniotic fluid
c. Consider corticosteroids for fetal lung
maturation d. Deliver the baby
e. Reevaluate middle cerebral arteries and ductus venosus

A 26 years old woman, G1P0A0 was admitted to emergency room because she lost her
consciousness around 1 hour ago, according to the husband, she is 36 weeks pregnant. She
performed antenatal care at scheduled time, and never missed one. Her husband said, she never
had severe nausea and vomitting. Physical examination reveals BP 120/80 mHg, pulse rate 86 x/min,
RR 18 x/min, temperature 36.5 C. You notice there is an icteric sclera. Other physical examination
was remarkable. Obstetrical examination reveals no heart beat was detected.
Laboratory examination reveals CBC 10.2/29.9/8900/253.000, Ur/C 18/0,8, AST/ALT 458/878, RBG
32, urinalysis was within normal limit.

26. What is the best next management in this case?


a. Abdominal ultrasound
b. Induction of labor
c. Emergency cesarean section
d. Whole blood transfusion
e. Injection of 40 % dextrose

27. What is the underlying pathophysiology of intrahepatic cholestasis of pregnancy?


a. Acute hepatocellular destruction
b. Incomplete clearance of bile acids
c. Microvascular thrombus accumulation
d. Eosinophil infiltration of the liver
e. Hepatocellular injury

A 33 year old woman, G1P0A0, came to the hospital with major complaint, watery leakage. She was
on her 33 weeks of gestational age. Data from medical record showed that she came previously a
week ago, complaining vaginal discharge. Vaginal swab has done.
28. In case above, what kind of examination should you performed for establishing diagnosis?
a. Vaginal examination
b. Inspeculo
c. Blood test
d. Ultrasound
e. Simple urine test

29. You found on Leopold 1,hard, round, with ballottement +, contraction was infrequent and
weak. What was your plan?
a. Went to labor induction
b. Immediate C-section
c. Tocolytic and corticosteroid
provision d. Performed ultrasound
e. Performed external version

30. Ultrasound examination showed that trans-cerebellar diameter was proper to gestational
age, abdominal circumference was lower than 2.5 percentile and amniotic fluid deepest
pocket was 1.2 cm. what is the most likely diagnosis?
a. Growth restriction with oligohydramnion
b. Normal growth with oligohydramnion
c. Growth restriction with normal amniotic fluid
d. Normal growth with normal amniotic fluid
e. Need another examination for establishing diagnosis

31. Lack of baby movement had been felt for two days, fetal heart rate was 146 bpm. What
was your next step?
a. Termination of pregnancy
b. Giving oxygenation and left lateral position
c. Ensuring fetal well being by Manning criteria
d. Fetal lung maturation
e. Giving intravenous fluid rehidration

32. Cardiotocography showed low variability with checkmark pattern and no desceleration.
What was your interpretation and the best management through?
a. Category one, continued for fetal lung maturation
b. Category two, intrauterine resuscitation for 24 hours and reevaluation after
c. Category two, went for Doppler velocimetry
d. Category three, went for Doppler velocimetry ultrasound
exam e. Category three, delivered baby

33. A patient wishes to consider pregnancy after treatment for her breast cancer. What is the
most important predictor of a good prognosis?
a. Young age
b. Herceptin positivity
c. Estrogen receptor positivity
d. BRCA gene positivity
e. Family history of treatable breast cancer

34. A primigravida at 36 weeks gestation is measuring large for dates. Ultrasound shows
AC>97 th percentile. GTT performed shows poorly controlled gestational diabetes. Whats
is the immediate management plan?
a. Give steroids
b. Start induction
c. Start hypoglycaemic drugs
d. Wait and watch
e. Start sliding scale and deliver the baby

35. During Cesarean section, the uterine contraction suddenly became poor. No significant
bleeding was found. The vital sign was within normal limit. B-Lynch procedure is then
applied. The main concept of B-Lynch is:
a. If bimanual compression of the exteriorized uterus decrease the uterine bleeding
b. If bimanual compression of the exteriorized uterus does not decrease the uterine
bleeding
c. If bimanual compression of the exteriorized uterus increase the uterine bleeding
d. If bimanual compression of the exteriorized uterus decrease the uterine contraction
e. If bimanual compression of the exteriorized uterus increase the uterine contraction

A 27 yo patient and her husband present to you with primary infertility. The patient reports regular
period every 28-30 days. The patient has no significant medical history and does not take any
medications other than prenatal vitamins. Her husband is also in good health, is 30 yo, and has 2
children from a previous marriage. When you asked the patient how long they have been trying to
achieve a pregnancy, they tell you 6 month.
36. Your instruction to the couple are the following:
a. They will likely need IVF to achieve a pregnancy
b. They will likely need IUI cycles
c. Continue trying appropately timed intercourse for 6 more months and return for follow
up if no pregnancy is achieved
d. Consider egg donor
d. Consider adoption

37. A 29 yo lady with 30 yo husband come with 2 years of primary infertility. Semen analisis,
HSG and an endocrine evaluation including FSH, E2, TSH, prolactin levels, and ovarian
reserve testing had been done. All of the tests come back normal. Your next
recommendation:
a. have 6 more months of timed intercourse and return for follow up visit if no
pregnancy is achieved
b. Clomiphene citrate with IUI
c. Gonadotropin injection
d. Human gonadotropin (hMG)
e. IVF

38. Characteristically, menopause begins with cycle irregularity that extends to 1 year after
permanent cassation of menses. The more correct, scientiffic terminology for this time is
menopausal transition and it typically takes place over a span of how many years?
a. 1 to 2 years
b. 2 to 3
years c. 4 to 7
years
d. 5 to 10 years
e. 11 to 12 years

39. A 49-years woman had radical histerectomy and lymph node sampling for stage IB
squamous cell cancer cervix. A suprapubic catheter was placed at the time of surgery. She is
now 8 weeks postoperative and has not been able to void. She is also leaking urine with
activity, coughing, and sneezing. What is the most likely reason for voiding difficulty?
a. spasm of the pelvic floor muscle
b. outflow obstruction
c. postoperative swelling around the bladder
d. innervation of the lower urinary tract was transected
e. overdistention of the bladder

A 46 women yo, parity 2, complain of LLQ pain, intermitten nausea, abdominal pressure and
bloating. Her history is notable for mild obesity, right breast cancer and HT. Her family history:
premenopausal breast cancer in her mother and maternal aunt. She had a pelvic ultrasound a left
ovarian mass containing internal septation and papillary projection. She had moderate ascites. Ca
125 was 179.
40. Which of the following is associated with an increased of ovarian cancer risk?
a. history of breast cancer
b. breastfeeding
c. multiparity
d. tubal ligation
e. obesity

41. She underwent TAH, BSO, collection of pelvic washing, omentectomy, cytoreduction or
debulking and bilateral pelvic and paraaortic lymph node sampling. The mass had spread
beyond the ovary to omentum and bowel. She was found to have ascites, and pelvic
washing was positive. What stage of ovarian cancer?
a. st I
b. st II
c. st III
d. st IV
e. st V
A couple has been trying conceive 18 months. During interview the man has fathered a child in a
previous relationship. And is in a good health. The woman is 28 yo and had painful menses for the
past 5 or 6 years
42. You begin to suspect that she may have endometriosis. The below increase that suspect
except :
a. she report that maternal cousin has a history of endometriosis
b. she has experienced dyspareunia with deep penetration for several years
c. her ethnicity as Caucasian
d. she report the development of abnormal bleeding in the last years
e. her menarche began at age 9 years

43. A 35 yo woman with 2 children came to specialist clinics with the complain of vaginal
discharge with pap smear result showed : CIN 3 + HPV infection, she ask the appropriate
treatment.
What is the next diagnostic procedure should be offered to the patient
a. HPV DNA test
b. Liquid based cytology pap smear
c. Ultrasound
d. VIA
e. Colposcopy

A 45 yo women complaint : chronic leucorrhea. She had been treated by various AB by midwife.
Recently she get bleeding each after intercourse. General condition is anemic appearance. BP
70/palpation. HR 120x/m
44. What is the best step for this patient:
a. performed bimanual examination
b. ultrasound examination
c. giving oxygen and putting iv line
d. giving emergency curetage
e. giving tranexamic acid

45. By first impression what would the diagnosis would like to be?
a. uterine sarcoma
b. cervical polyp
c. endometrial cancer
d. uterine myoma
e. cervical cancer

46. What is the etio-pathogenesis of diagnosis above:


a. malignant transformation of myocite
b. hiperestrogenic intra cavity
c. HPV tipe 6
d. herpes simplex
virus e. HPV tipe 16

A 46 yo G2P2 obese woman referred from primary hospital because increasingly heavy and painful
menses over the last 18 months. She tried some oral contraceptive with some improvement of the
bleeding but no improvement with the pain. No other history of pelvic pain. Or abdominal bleeding.
No abnormal pap smear she never had any infection.,”down there”. On examination : normal
genitalia externa, vagina and cervix. But uterus slightly enlarge, mildly tender and softer than you
expected. no adnexal mass or tenderness.

47. You explain to your patient that you think she may have adenomyosis and that it is the
most likely causing her symptoms. She is reluctant for having surgery. Which of the
following management is appropriate?
a. start combined oral contraception
b. depoprovera injection every
months c. LNG IUS
d. GnRH analog injection for 3 months
e. oral NSAID

A 30 yo complaint infertility. Husband 33 yo, semen analisis : normal. On further history the patient
report that her cycle quiet irreguler over last year and that she missed her periode in the last 4
months. She also reports insomnia, vaginal dryness and decreased libido.
48. What the most likely diagnosed for this patient based on history :
a. Polycystic ovarian syndrome
b. primary ovarian insufficiency
c. endometriosis
d. kallman syndrome
e. spontaneus pregnancy

49. Which of the following condition that corresponds to the above possible
diagnosis? a. Day 3 FSH 40 IU
b. Serum AMH level 2,6 ng/ml
c. Positive CCCT
d. Midluteal progesterone level of 18ng/ml
e. Folicle antral basal count of 12

A 34 y.o women with primary infertility 3 years, oligomenorea and BMI 26. Day 23 progesteron level
result was 5ng/ml. TVS shows multiple small follicle size 508 mm in both ovary. HSG shows bilateral
patent tubes. Her partners semen analysis show a volume of 3ml (min: 1,5 ml), pH of 7 (>7,2) and
sperm count of 20 million/ml (15 juta), Motilitas : 32%, Morfologi normal : 4%, sperma: 39
juta/ejakulat, vitalitas : 58, viskositas: < 2 cm
50. The patient later given CC 1x50mg for 5 days, but during follow up there were no dominant
follicle. Which of the following is appropriate?
a. We should increase the dose to 100mg/day for 5 day
b. Patient correspond to CC resistent, should undergo laparoscopy drilling
c. Patient correspond to CC failure and should switch to gonadotropin inj
d. Give aromatase inhibitor
e. Combine metformin with CC

51. Which of the following conditions that we can find patient with CC resistant?
a. BMI 22
b. AMH >8 ng/ml
c. LH level 7 mIU/ml
d. FSH level 5 mIU/ml
e. Estradiol level 45 ng/ml

A 20 y.o lady come to outpatient clinic never had menstruation. No history of surgery or medication.
BMI 20, tanner M3P3. From USG uterus can not be visualized. Both ovaries size 28x25mm and
26x20mm with dominant follicle size 18mm on left ovary.
52. Which of the following is the most appropriate diagnosis?
a. PCO
b. Kallman syndrome
c. POI
d. MRKH
e. Hyperprolactinemia

53. Which characteristics that differentiate the above diagnosis with AIS?
a. Presence of axillary and pubic hair
b. Elevated level of prolactin
c. Elevated level of FSH
d. Low level of ANH
e. Elevated level of testosterone

54. If the diagnosis is confirmed, what is her option to achieve pregnancy?


a. Conventional IVF
b. IVF- ICSI
c. Minimal stimulation IVF
d. Oocyte donation
e. Gestational surrogacy

55. A 7 y.o girl presents to her pediatrician with her parents who are concerned about her early
sexual development. She is developing breast, axillary hair, pubic hair and body odor. A
through clinical workup reveals the child has an irregular, echogenic, thickly septated ovarian
mass on the her left ovary. What type of tumor is responsible for this child’s clinical
presentation?
a. Dysgerminoma
b. Embryonal carcinoma
c. sertoli leydig cell tumor
d. Endodermal sinus tumor
e. Granulosa theca cell tumor

A 49 y.o woman experiences irregular vaginal bleeding for 3 months duration. You performed
endometrial biopsy, which obtain copious tissue with a velvety, lobulated texture. The pathologist
report shows proliferation of glandular and stromal elements with dilated endometrial glands,
consistent with simple hyperplasia. Cytology atypia is absent.
56. Which of the following is the best way to advise the patient?
a. She should be treated to estrogen and progestin hormone therapy
b. The tissue will progress to cancer in approximately 10% of cases
c. The tissue may be weakly premalignant and progress to cancer in approximately 1%
of the cases
d. She requires hysterectomy
e. No further therapy is needed

57. Which of the following factors protective against endometrial hyperplasia?


a. Obesity
b. Tamoxifen
c. OCPs
d. Early menarche and late menopause
e. Unopposed exogenous estrogen therapy

A 36 y.o patient, P0, presents to your clinic for fertility workup. She had been married for 2 years
with regular intercourse. Her menstrual cycle is normal. Her general status was normal. Vaginal
examination revealed normal findings.
58. Which of the following examination that is not include in basic workup in the patient above?
a. HSG
b. USG
c. semen analysis
d. Midluteal progesterone
examination e. Serum AMH

59. A 30 years old patient, P0 present to your clinic for fertilitiy workup. She had been married
for 2 years with regular intercourse. Her menstrual cycle is normal. Her general status was
normal. Vaginal examination revealed normal findings. The following month she came back
with the result of hysterosalpingography (see the picture below)
What will your next plan?
b. Repeat HSG next month
c. Schedule diagnostic laparoscopy
d. Gives CC and plan for natural conceptions
e. Gives CC and plan for IUI
f. Plan for IVF

60. A 68 y.o woman present with valvular pruritus since the previous year that has been
increasing over the last few months. She has tried antifungal medication, which seems to
help, but the symptoms keep recurring. She went through menopause at age 49 and has not
been sexually active for 10 years. She does not use any douching products and is not taking
any antibiotics. On physical examination, you note thin white epithelium of the labia minora
with multiple red oval shaped erosions, varying in size from 0,5 to 1,5cm. How would you
proceed?
a. Culture the vagina and treat with high dose antifungal
b. Wide local excision to the lesions
c. Cryotherapy to eradicate the
lesions d. Punch biopsy of the vulvar
lesions
e. Treat with moderate high potency topical steroids

61. A 27 y.o woman presents to her gynecologists for evaluation of new vulvar lesions. She first
noticed several “bumps” on her outer labia 3 weeks ago. She is otherwise without any
associated symptoms. On pelvic examination there are six flesh colored, nontender, 1-3mm
verrucous papules. Which of the following is the most likely cause of the patients vulvar
lesions?
a. HPV serotype 16
b. HSV type 1
c. Pox virus
d. HPV serotype 6
e. Haemophillus ducreyi

62. A 45 year old present for evaluation because her primary care physician has diagnosed her
with pelvic organ prolapse while performing annual care. She denies any pelvic pressure
bulge or difficulty with urination. Her only medical comorbidity is obesity for asytomatic
grade 1 pelvic organ prolapse, what do you recommend
a. Conservative management with pelvic floor muscle exercise and weight loss
b. Colpocleisis obliterative procedure
c. Gellhorn pessary
d. Round ligament suspension
e. Hysterectomy

63. A 62 year old G2 P2 presents to the urogynecology clinic with complains of urinary
incontinence. She has urinary urgency and cant make it in bathroom before leaking a large
amount of urine. She gets up two to three times per night to urinate. A urinalysis and urine
culture done 1 weeks ago at her PCP’s Office are both negative. What is the most likely
diagnosis and appropriate treatment option for this type of urinary incontinence
a. Stress incontinence mid urethral sling
b. Urgency incontinence, oxybutynin (anticholinergic medication)
c. Overflow incontinence, oxybutynin (anticholinergic medication
d. Urinary fistula, surgical repair
e. Functional incontinence, bladder suspension

64. Cervical cancer of squamous type identified only microscopically, invasion is limited to
measured stromal invasion with a maximum depth of 2 mm and no wider than 6 mm should
be treated with
a. observation
b. tissue ablation
c. LEEP/LLETZ
d. Simple total hysterectomy
e. Radical Hysterectomy

65. The majority of vulvar, vaginal and cervical cancers appears to have a common cause and
usually caused by
a. High risk type of herpes simplex virus (HSV) infection
b. High risk types of human papilloma virus (HPV) infection
c. Increased exposure to endogenous estrogen
d. Increased exposure to exogenous estrogen
e. Chronic bacterial and parasitic infection

66. Childhood neoplastic ovarian masses most commonly originate from


a. gonad epithelium
b. gonadal stroma
c. sex cords
d. germ cells
e. metastatic disease

67. Your patient is a 13 year old adolescent girl who present with cyclic pelvic pain, she has
never had menstrual cyle. She denies any history of tntercourse. She is afebrile and her vital
sign are
stable. On physical examination. She has age appropriate breast and public hair development
and normal external genitalia. However, you are unable to locate a vaginal introitus instead
there is a tense bulge where the introitus would be expected. You obtain a transabdominal
ultrasound, which reveals a hematocolpos and hematometra, what is the most likely
diagnosis?
a. Transverse vaginal septum
b. Longitudinal vaginal
septum c. Imperforate hymen
d. Vaginal atreasia
e. Bicornuate uterus

68. A 18 year old nuligravid woman presents to the student health clinic with a 4 week history of
yellow vaginal discharge. She also report vulvar itching and irritation. She is sexually active
and monogamous with her boyfriend. They use condom inconsistently. On physical
examination she is found to be nontoxic and febrile. On genitourinary examination vulvar
and vaginal erythema is noted along with yellow, frothy, malodorous discharge with a PH of
6,5 the cervix appears to have erythematous punctuations. There is no cervical, uterine, or
adnexal tenderness. The addition of 10% KOH to the vaginal discharge does not produce an
amine odor wet pep microscopic examination of the vaginal swabs is performed. What
would you expert to see under microscopy?
a. Branching hyphae
b. Multinucleated giant cells
c. Scant WBC
d. Flagellated, motile organisms
e. Epithelial cell covered with bacteria

69. An 89 year old female patient with multiple, serious medical comorbidities present to discuss
options for treatment of her high grade prolapse is externalized and becoming ulcerated
from friction against her undergarments. She cannot tolerate a pessary. Her main priority Is
to “Fix of get of this thing”, but her primary care provider has cautioned against a lengthy or
open abdominal procedure. She is not interested in future intercourse. What can you offer
this patient
a. Nothing can be done
b. Open abdominal sacral colpopexy
c. Robot assited laparoscopic sacral colpopexy
d. Hysterectomy with anterior and posterior colporaphy, vault
suspension e. Colpocleisis

70. Patient returns for a postoperative check up 2 weeks after total abdominal hysterectomy for
fibroids. She is distressed because she is having continuous leakage of urine from the vagina.
Her leakage is essentially continuous and worsens with coughing, laughing, or movement.
Given her history and physical, you perfom both a methylene blue dye test, which is negative
and an indigo carnine test, which is positive. The most likely diagnosis is
a. Rectovaginal fistula
b. Uretro vaginal fistula
c. Vesico vagina fistula
d. Uretero vagina fistula
e. Impossible to distinguish

71. A 38 years old multi gravid woman complain of the painlees loss of urine. Beginning
immediately with coughing, laughing, lifting, or straining. Immediate cessation of the activity
stops the urine loss after only a few drops. This history is most suggestive of
a. Fistula
b. Stress incontinence
c. Urge incontinence
d. Urethral Diverticulum
e. UTI

A 30 years old woman, P1A1 had performed curettage on indication molar pregnancy, she came
with BhCG level 10.000iu/dl after evacuation. The first BhCG level was 1.000.000 IU/dl. The
histopathology result was hydatidiform mole, poor differentiation.
72. The principles for follow up of hydatidiform molar pregnancy:
a. Prevent pregnancy during the follow up period at minimum for 6 month
b. Measure serum chorionic gonadotropin level every 4 weeks
c. Measure serum chorionic gonadotropin level every 4 month
d. Measure serum chorionic gonadotropin level every 3 weeks
e. Observation in selected, highly individualized patients may be an option, to perform
hysterectomy

73. Criteria for diagnosis of gestasional trophoblastic neoplasia or postmolar gestasional


trophoblastic disease:
a. Plateau of serum BhCG level (+-10%) for four measurement during a period of 3 weeks
or longer days 1, 7, 14, 21
b. Rise of serum BhCG <10% during 3 weekly consecutive measurement or longer during
period 2 weeks or more-days 1,7,14
c. Rise of serum BhCG <10% during 2 weekly consecutive measurement or longer during
period 2 weeks or more-days 1,7,14, 21
d. The serum BhCG level remains detectable for 3 months or less
e. The serum BhCG level remains detectable for 12 months or less

74. A 53 years old woman presents for counseling and management of her low grade anterior
wall prolapse. She is only symptomatic on days when she has engaged in heavy lifting. She is
morbidly obese and would like to begin a formal weight loss program. She agrees to trial with
a pessary. You choose a supportive pessary and counsel her regarding pessary insertion,
removal, cleaning, etc. which of the following are factors associated with a good response for
pessary in uterine prolapse?
a. Vaginal length 9 cm
b. Obesity
c. History of traumatic vaginal deliveries
d. Menopausal state
e. Large genital hiatus

75. A 25 years old lady come with abnormal pap smear result. She underwent colposcopy
examination and the result is acetowhite lesion with punctuation and atypical vessels. Biopsy
result confirms CIN I with HPV DNA test positive. What do you suggest for patient?
a. LEEP procedure
b. Reevaluation of HPV DNA
c. Cold knife conization
d. Repeat cytology in 12 months
e. Repeat cytology in 6 months

A 36 years old G2P0 at 26 weeks gestational age presents to the office with palpitations, anxiety,
lack of sleep, and a 5 kg weight loss over the past 2 weeks. She started to develop substernal chest
pain this morning. She has no significant medical history. On examination, she is afebrile with heart
rate of 152 and BP of 158/82. She appears anxious and her eyes are prominently open. Her thyroid is
enlarged but nontender. An EKG reveals sinus tachycardia without ST segment changes.
76. Which of the following is the appropriate immediate first line
therapy? a. Propanolol
b. Lorazepam
c. Hydrocortisone
d. Radioactive iodine
e. Propylthiouracil (PTU)

77. Which clinical symptom is not characteristic of mild thyrotoxicosis


a. Palpitation
b. Tachycardia
c. Thyromegaly
d. Cold intolerance
e. Failure to gain weight

A 24 years old woman at 32 weeks gestation complains of shortness of breath during her pregnancy,
especially with physical exertion. She has no prior medical history. Her respiratory rate is 16x/m, her
lungs are clear to auscultation, and your office oxygen saturation monitor reveals her oxygen
saturation to be 98% on room air.
78. You reassure her that this sensation is normal and explain which of the following?
a. Pulmonary resistance increases during pregnancy
b. Airway conductance is decreased during pregnancy
c. Small amniotic fluid emboli are shed throughout
pregnancy d. Maximal breathing capacity is not altered by
pregnancy
e. Because of enlarging uterus pushing up on the diaphragm, her vital capacity is
decreased by 20%

79. A 33 years old G2P1 presents at 35 weeks gestations with complaints of nausea and
vomiting. Laboratory evaluation reveals markedly decreased glucose level, elevated
transaminase levels, renal dysfunction, and coagulopathy. What is the possibility diagnosis in
this patient?
a. Acute hepatitis
b. Cirrhosis hepatis
c. Intrahepatic cholestasis
d. Acute fatty liver of pregnancy
e. Non-alcoholic fatty liver disease

A 32 years old lady, G2P1A0 presented to delivery ward at 30 weeks gestation with worsening
abdominal pain for few hours. She had also had some vaginal bleeding within the past hour. Her
uterus was tender and firm to palpation. She was found to have low amplitude high frequency
uterine contractions, and the fetal heart rate tracing showed recurrent late decelerations and
reduced variability. Her blood pressure was 160/100 mmHg and she has had a +2 proteinuria. She
did her antenatal care in your hospital and ultrasound examination was performed 3 times with no
remarkable abnormalities.
80. The most likely diagnosis is:
a. Vasa previa
b. Preterm labor
c. Placenta previa
d. Placental abruption
e. Preterm premature rupture of membrane (PPROM)

81. From obstetrical examination you found her cervix was unfavorable. Your next plan is to
deliver the baby by:
a. Vaginal delivery
b. Elective C-section
c. Emergency C-section
d. Operative vaginal delivery
e. Observation until the cervix was favorable

82. You are counseling a couple in your clinic who desire VBAC. Her baby is in a vertex
presentation, appropriate size for 37 weeks, and her previous low transverse procedure was
for breech presentation. You have to give inform consent about VBAC. In providing informed
consent, in which of the following ways do you explain the risk of uterine rupture?
a. Less than 1%
b. Between 2% and 5%
c. Between 15-20%
d. Depend on the length of her labor
e. Depend on the location and proximity of the scar site to the placental implantation

A 17 years old G2P1 woman with no prenatal care at 29 weeks gestation presents with painful
contractions and pressure. Her cervix is 1 cm, 40% effaced, and breech at station 2. There is no
evidence of ruptured membranes. Her contractions are every 4 minutes. FHR are 150 bpm with
accelerations. Maternal vital signs are temperature 36,80 C, pulse 96 x/mnt, BP 110/72 mmHg
83. What should you do?
a. C-section
b. Performed fetal fibronectin
c. Begin tocolytic agents and corticosteroids
d. Observe the cervical changes and labor progress
e. Perform amniocentesis to rule out chorioamnionitis

84. What fetal complication is associated with the nonsteroidal anti-inflammatory agent
indomethacin as tocolytics agent?
a. Hydramnios
b. Achondroplasia
c. Pulmonary valve atresia
d. Bronchopulmonary dysplasia
e. Premature closure of the ductus arteriosus

85. The nonstress test (NST) has which of the following


characteristics? a. Low positive predictive value
b. Low specificity (with reactive NST)
c. Low false positive rate (within non-reactive NST)
d. FHR reactivity depends on normal cardiac development
e. Acceleration without fetal movement should not be accepted

86. Corticosteroids administered to women at risk for preterm birth have been demonstrated to
decrease rates of neonatal respiratory distress if the birth is delayed for at least what
amount of time after the initiation of therapy?
a. 12 hours
b. 24 hours
c. 36 hours
d. 48 hours
e. 72 hours
87. A 24 years old patient (parity 2) has just delivered vaginally an infant weight 4300 g after a
spontaneous uncomplicated labor. Her prior obstetric history was a low uterine segment
transverse cesarean section for breech. She has had no problem during the pregnancy and
labor. The placenta delivers spontaneously. There is immediate vaginal bleeding of greater
than 500 cc. Although all of the following can be the cause of postpartum hemorrhage,
which is the most frequent cause of immediate hemorrhage as seen in this patient?
a. Uterine atony
b. Coagulopathies
c. Uterine rupture
d. Retained placental fragments
e. Vaginal and/or cervical lacerations

Mrs. X, 24 yo., came to the ER with complaints of headaches since the last day of examinations
obtained expecting her first child, gestational age 32 weeks with blurred vision and denied
heartburn. On physical examination found BP 190/120 mmHg, pulse 90 x/m, breathing 16 x/m.
Leopold found the lower left back head, FHR 140 bpm, irregular contraction. Pelvic score of 1 was
found, pelvis size wide. Laboratory investigation; hemoglobin 11.5 g%, platelets 9000/mm3, LDH
510 iu/L, Proteinuria
+2, ALT 10 u/L, AST 15 u/L
88. What is the best diagnosis for Mrs.
X a. HELLP syndrome
b. Severe preeclampsia
c. Chronic hypertension
d. Gestational hypertension
e. Superimposed preeclampsia

89. What is the most clinically effective antihypertensive agent for Mrs. X
a. ISDN
b. Atenolol
c. Nifedipine
d. Furosemide
e. Methyldopa

90. Which antihypertension drugs can cause fetal growth restriction?


a. Nifedipine
b. Atenolol
c. Hydralazine
d. Captopril
e. Methyldopa

91. A woman is being treated with magnesium sulphate. There is concern about magnesium
toxicity. What is the first sign of magnesium toxicity?
a. Bradycardia
b. Reduced consciousness
c. Respiratory depression
d. Decreased urine output
e. Loss of deep tendon reflexes

92. With two home pregnancy tests and ultrasound revealed 6-7 wga pregnancy. As her
pregnancy continues, you would expect her cardiac output to increase by which of the
following mechanisms:
a. An increased heart rate alone
b. A decrease in systemic vascular resistance
c. First an increase in stroke volume, then an increase in heart rate
d. Cardiac output would not change significantly until the third trimester
e. An increase in systemic vascular resistance facilitated by elevated progesterone levels

93. Which of the following is true regarding the physiologic changes she might expect during her
pregnancy?
a. An overall decrease in the number of WBC and platelets
b. Gastric emptying and large bowel motility are increased in pregnancy
c. An increase in the tidal volume along with an increase in total lung capacity (TLC)
d. BUN and creatinine will decrease as a result of an increase in glomerular filtration rate
(GFR)
e. Nausea and vomiting that should be treated aggressively with antiemetics and
intravenous hydration

94. A woman with a previous stillbirth and postpartum DVT is found to have lupus anticoagulant
and medium-titre immunoglobulin M (IgM) anticardiolipin (aCL) on two occasions. In a
subsequent pregnancy:
a. Warfarin should be discontinued
b. She has an increase risk of miscarriage
c. She requires antibiotic prophylaxis to cover delivery
d. Low dose aspirin should be discontinued at 34 weeks
e. She does not require postpartum heparin if she has vaginal delivery

A 28-year-old G1P1 woman is being discharged from the hospital on postoperative day 4 after
having received a primary low transverse cesarean section for breech presentation, with an
estimated blood loss of 700 ml. Her pregnancy was otherwise uncomplicated and her hospital
course was also uncomplicated.
Ten days after Caesarean section, the patient came complaining of abdominal pain and fever. Fundal
height 2 fingers below navel.

95. What is the most likely probable diagnosis of the


patient? a. Metritis
b. Mastitis
c. Typhoid fever
d. Urinary tract infection
e. Breast engorgement

96. Lower urinary tract symptoms with pyuria but a sterile urine culture are likely due to which
pathogen?
a. Candida
b. Escherichia coli
c. Proteus mirabilis
d. Klebsiella pneumonia
e. Chlamydia
trachomatis

97. A patient calls your clinic complaining of continued heavy vaginal bleeding. She had an
“uncomplicated” vaginal birth 2 weeks ago of her second child. What is the most likely
diagnosis from the following differentials?
a. Uterine atony
b. Uterine rupture
c. Coagulopathies
d. Vaginal lacerations
e. Retained placental fragments

A 29-year-old G2P1 woman came to outpatient clinic with obesity, a history of GDM in the prior
pregnancy, and a strong family history for type 2 diabetes mellitus (T2DM) presents at 7 weeks’
gestation. In her previous pregnancy, she required insulin therapy. She delivered at 39 weeks and
her baby boy weighed 4.300 g.
98. In addition to the routine prenatal laboratory tests, what other testing will you obtain at this
point?
a. HbA1c
b. No other testing needed
c. An ultrasound to estimate gestational age
d. Perform a glucose challenge test at first visit
e. Perform a glucose challenge test at 24 weeks’ gestation

99. Laboratory test results return, and her fasting blood glucose is 145 mg/dL. An ultrasound
reveals the pregnancy to be 7 weeks and 2 days, consistent with LMP, Hemoglobin A1c is
7,5%. Her diagnosis is:
a. GDM
b. T1DM
c. T2DM
d. No diabetes in pregnancy
e. Impaired glucose tolerance
100. A 41-year-old women present at 36 weeks of gestation in active labour. The ultrasound
scan at 32 weeks showed low lying placenta. Immediately after rupture of the membrane,
she started bleeding vaginally with associated cardiotocography (CTG) of non reassuring.
What is the most likely diagnosis?
a. Vasa previa
b. Placenta previa
c. Placenta accrete
d. Abruptio placenta
e. Low lying placenta

UNAS JULI 2018

A 25 y.o woman in her first pregnancy is noted to have prolonged first and second stage of labor.
She was induced at 38 weeks pregnancy. The baby was delivered by forceps. After delivery the
placenta she is noted to have heavy vaginal bleeding. Abdominal examination demonstrated a
relaxed uterus.
1. What is the most likely cause of bleeding?
a. Uterine atony
b. Uterine rupture
c. Retained placenta
d. Genital tract laceration
e. DIC

2. What should we do if the fundus not firm after placenta delivery?


a. Methylergonevine (Methergine)
b. Carboprost (Hemabate, PF 2- alpha)
c. Fundal massage
d. Misoprostol
e. Dinoprostoneprostaglandine E2

3. Which of the following medications would be contraindicated in the treatment for this patient?
a. Methylergonevine (Methergine)
b. Carboprost (Hemabate, PF 2- alpha)
c. Intramuscular pitocin
d. Misoprostol (PgEs)
e. Dinoprostoneprostaglandine E2
A 33-year-old G2P0101 at 34 weeks’ gestation is evaluated in fetal ultrasound for fetal size less
than dating by fundal height. The fetus has been lagging in growth throughout the pregnancy,
but recently the obstetrician did not notice interval growth in the fundal height. Fundal height
today measures 31 weeks. She had an ultrasound at 18 weeks, which showed normal anatomy
and growth in the 30th percentile. The patient’s medical history is complicated by a history of
oxycodone abuse, and she is currently taking methadone in pregnancy. She has had negative
urine drug screening at each prenatal visit. She delivered her first child vaginally at 36 weeks
secondary to PPROM. You are covering for her provider today and are looking at the ultrasound
report, which shows that her fetus is small with head and abdominal circumferences at the 5th
percentile and femur length less that the 10th percentile. There are abnormal findings with the
placenta.

4. Which of the following placenta conditions do not increase a fetus’s risk of IUGR?
a. Chronic placental abruption
b. Placenta previa
c. Thrombosis
d. Chorioamnionitis
e. Marginal cord insertion

A 36-year-old G2P1001 woman at 12 weeks’ gestation presents to clinic for routine prenatal
visit. She reports that her nausea has resolved and denies vaginal bleeding. Her pregnancy has
been uncomplicated. Her prior pregnancy 2 years ago was complicated by the diagnosis of GH
that led to an induction of labor and cesarean delivery. She has no other medical history. On
examination, her BP is 138/84 mm Hg, her body mass index (BMI) is 36 kg/m, and a urine dip
shows trace protein.

5. Which of the following laboratory test should be ordered today?


a. Quad screen
b. 24-hour urine protein collection
c. HgbA1c
d. Low-density lipoprotein (LDL)
e. Herpes simplex virus (HSV) titer

A 17-year-old G1P0 woman present to labor and delivery complaining of contractions at 38


weeks’ gestation. Her BP at the beginning of pregnancy was 90/60 mm Hg. She has gained 46 lb
throughout pregnancy (10 lb in the past 4 weeks). Her BP on presentation was 145/80 mmHg. A
urine dip is +1 protein. On examination, her cervix is 3 cm dilated, 90% effaced, 1 station.

6. What is the next step in her management?


a. MgSO4
b. Betamethasone
c. Send laboratory tests
d. Artificial rupture of the membranes
e. Send home until in active labor

A 30-year-old African American G0 woman comes to your office for the Annual examination.
During the history assessment, she reports increased pelvic pressure, constipation, and
progressively heavy and prolonged menses. During menstrual cycles, she is using about 10 large
pads per day with her heaviest flow and passing “quarter sized” blood clots. On physical
examination, you palpate a nontender, irregularly enlarge uterus with a lumpy-bumpy, firm
contour. Her cervix appears normal, and he has no evidence of ascites or other abnormal
physical findings. You suspect that she has uterine fibroids.

7. Which of the following tests is most commonly used for diagnosis of uterine fibroids?
a. Computerized tomography (CT) scan
b. Pelvic X-ray
c. Pelvic ultrasound
d. MRI
e. Hysterosalpingogram (HSG)

8. All of the following medical therapies can be used to treat heavy and prolonged menses in
women with uterine fibroids except:
a. Combined oral contraceptive pills (OCPs)
b. Antifibrinolytic agent (tranexamic acid)
c. Nonsteroidal anti-inflammatory drugs
d. Progestin only
pills e. Opioid
agonists

9. Which of the following fibroid locations is most commonly associated with abnormal uterine
bleeding?
a. Submucosal
b. Intramural
c. Subserosal
d. Pedunculated
e. Parasitic

An 18-year-old young woman presents to your office with a complaint of amenorrhea. She notes
that she has never had a menstrual period, but that she has mild cyclic abdominal bloating. She
reports normal breast development starting at age 12. She reports she has become sexually
active, but she finds intercourse painful. Her past medical and surgical history is unremarkable.
On physical examination, you note normal appearing axillary and pubic hair. Her breast
development is normal. Pelvic examination reveals normal appearing external genitalia, and a
shortened vagina ending in a blind pouch.
10. Which of the following tests would be your first step in determining the diagnosis?
a. Karyotype
b. Pelvic ultrasound
c. Serum FSH
d. Diagnostic laparoscopy

11. You perform a bedside ultrasound and find normal appearing bilateral ovaries as well as an
absent uterus and fallopian tubes. What is your most likely diagnosis?
a. Imperforate hymen
b. Transverse vaginal
septum c. Mullerian agenesis
d. Androgen insensitivity syndrome

12. Which additional organ system should you be evaluating in a patient with this disorder?
a. Pancreas and duodenum
b. Cerebral circulation
c. Olfactory system
d. Renal and urinary collecting system

13. Your patient’s concerned mother calls you later that afternoon. “How will my daughter be able
to start a family?” You wisely counsel her about her options, which include:
a. Intrauterine insemination
b. Uterine transplant
c. In vitro fertilization with gestational carrier
d. The patient will not be able to reproduce using her genetic material

A 23-year-old G1P0 woman at 30 weeks 3 day presents to clinic for routine prenatal care. Her
pregnancy is dated by last menstrual period (LMP) consistent with a 10-week ultrasound. She
has had three prenatal visits, and her pregnancy has been complicated by vaginal bleeding in the
first trimester and the development of heartburn at 25 weeks. She has no complaints today. She
continues to smoke half a pack of cigarettes daily, which has decreased from one pack per day at
the beginning of her pregnancy. Her medical history is significant for asthma. Ultrasound at 20
weeks’ gestation showed no evidence of fetal abnormality, posterior placenta, AFI of 10.6 cm,
and fetal growth in the 20th percentile. Her current weight is 130 lbs, and her height is 5 ft 6 in.
She has gained 10 lbs so far in pregnancy. Urine dip is negative for protein, glucose, ketones and
leukocytes. BP is 112/64 mm Hg and heart rate is 80 beats per minute. Fetal heart tones are in
the 130s. Fundal height measures 25 weeks. Of note, at her last visit at 25 weeks, fundal height
measured 23 weeks, and she had a normal glucose tolerance test and complete blood count.

14. Which of the following is the next best step in managing this patient’s pregnancy?
a. Nonstress test (NST)
b. Fetal ultrasound
c. Group B streptococcus culture
d. TORCH titers
e. Amniocentesis

15. Which of the following is the most likely risk factor for this patient’s small for gestational age
(SGA) fetus?
a. Congenital anomaly such as cardiac anomaly
b. Congenital cytomegalovirus (CMV)
infection c. Tobacco abuse
d. History of chemotherapy exposure as a child
e. Genetic potential

16. Ultrasound demonstrates that the fetus measures less than the 10th percentile for head
circumference, femur length, and abdominal circumference. Doppler velocimetry of the fetal
umbilical artery was normal. Amniotic fluid index (AFI) was 11.2 cm. Which of the following is the
most appropriate component of the treatment strategy at this time?
a. Induction of labor
b. Continue routine prenatal care
c. Fetal ultrasound every 2 to 3 weeks
d. Admission to the hospital for daily NST and biophysical profile (BPP)
e. Daily Doppler velocimetry

17. The patient has a repeat ultrasound performed at 33 weeks of gestation. Fetal growth is noted
to be at the 4th percentile with intermittently elevated umbilical cord Dopplers and an AFI of 8.4
cm. What do you recommend at this time?
a. Repeat growth ultrasound in 4 to 6 weeks
b. Amniocentesis for fetal lung maturity and delivery if mature
c. Admission to hospital for continuous fetal monitoring until delivery
d. Induction of labor
e. Betamethasone administration

18. What neonatal outcome must be anticipated in macrosomic fetuses?


a. Hypoglycemia
b. Hyperglycemia
c. Hypocalcemia
d. Asthma
e. None of the above

A 35-year-old woman with a history of primary infertility present with her partner for initial
prenatal visit. They had been attempting pregnancy for the past 3 years and have now conceived
through IVF. She had one embryo transferred 6 weeks ago. The patient’s medical history is
significant for rheumatoid arthritis and polycystic ovarian syndrome. She is overweight. In
reviewing their family history, she tells you that her grandmother was a twin and there is no
family
history of congenital abmormalities or known genetic disorders in their family. Today, she
reports nausea and vomiting, which is worse in the morning. She has not had any vaginal
bleeding or cramping. You perform a transvaginal ultrasound and notice not one but two
embryos. There appears to be a thin dividing membrane between the two. Heart rate for each
embryo is around 150 beats per minute.

19. At what stage of division in the embryonic disk does monochorionic-diamniotic twining occur?
a. Before the differentiation of the trophoblast
b. After trophoblast differentiation and before amnion formation
c. After amnion formation
d. Not until after day 15 of development
e. After the stage of blastocyst

20. Postterm pregnancy is associated with all the following except


a. Transient tachypnea of the newborn
b. Oligohydramnios
c. Macrosomia
d. Meconium aspiration
e. Intrauterine fetal demise

A 32 years old woman, G3P2 37 wga, came to ER with contractions 5 minutes apart. On
examination you found that the fetus is breech, and she’s already fully dilated with breech is on
hodge IV. You asked the mother to bear down, after the umbilicus, the baby won’t decent even
after 2 times bearing down, you found nuchal arm.

21. Which is the maneuver to help release the nuchal arm?


a. Classic maneuver
b. Mauriceau-smellie-veit maneuver
c. Lovsett maneuver
d. Bracht maneuver
e. Muller maneuver

22. If after coming head happened, what should we do?


a. Naegel forceps
b. Downward traction
c. Lovsett maneuver
d. Modified Prague maneuver
e. Hyperextension of fetal head

23. Lung surfactant is critical to pulmonary functioning by keeping surface tension in the alveoli low
and thereby decreasing the occurrence of atelectasis and atrioventricular (AV) shunting.
Surfactant is formed in which of the following?
a. Epithelium of the respiratory bronchi
b. Hilum of the lung
c. Placental syncytiotrophoblasts
d. Type I pneumocytes of the lung alveoli
e. Type II pneumocytes of the lung alveoli

24. A 33-year-old patient has been diagnosed as having adenomyosis. Which of the following
symptoms is most consistent with this diagnosis?
a. Dyspareunia
b. Mood swings
c. Painful defecation
d. Secondary dysmenorrhea
e. Infertility

25. A 53-year-old woman is diagnosis with anovulatory dysfunctional bleeding. Which of the
following is the most appropriate medical therapy?
a. Orally administered estrogen for the first 25 day of each month
b. Vaginal estrogen cream two to three times per week
c. Orally administered progesterone 5 to 10 mg daily for 10 days each month
d. Testosterone tablets 10 mg/d
e. Estrogen 20 mg administered intravenously

26. When performing a hysterectomy, the surgeon must be aware that at its closest portion to the
cervix, the ureter is normally separated from the cervix by which of the following distances?
a. 0.5 mm
b. 1.2 mm
c. 12 mm
d. 3 cm
e. 5 cm

A 32-year-old G3P2002 woman presents for routinr prenatal care at 37 weeks. Her pregnancy is
complicated by Rh-negative status, depression, and a history of LSIL Pap smear with normal
colposcopy in the first trimester. Today she reports good fetal movement and denies leaking
fluid or contrarctions. During your examination you measure the fundal height at an appropriate
37 weeks, and find fetal hearts tones located in the upper aspect of uterus. A bedside ultrasound
reveals frank breech presentation

27. Which of the following is the next best step in management of this patient ?
a. Schedule a cesarean delivery at 39 weeks
b. Return visit in 1 week to reseassess fetal
position c. Schedule an external cephalic version
d. Offer a trial of vaginal breech delivery
e. Offer emergency cesarean delivery

28. Prior to discharging the patient from labor and delivery triage after her successful external
cephalic version, which of the following should you do first ?
a. Schedule induction for 39 weeks
b. Palce abdominal binder to help hold fetus in cephalic presentation
c. Prescribe
tocolytic d. Give
RhoGAM
e. Check fetal position with ultrasound

29. Which of the following findings would deter you from offering this patient a trial of breech
delivery
?
a. Frank breech presentation
b. Fetal weight of 3200 g
c. Complete breech
presentation d. Fetal weight of
4100 g
e. Footling presentation

30. Preterm rupture of the membranes is most strictly defined as spontaneous rupture at any time
prior to which of the following ?
a. A stage of fetal viability
b. Second stage of labor
c. 32 weeks of gestation
d. 37 weeks of gestation
e. Onset of labor

31. You determine her membranes have ruptured and admit her for active management of labor.
The first stage of labor
a. Includes an active and latent phase
b. Begins when the cervix has completely dilated
c. In considered prolonged if its duration is longer than 2 hours in a nulliparous woman
d. Begins with the onset of Braxton Hicks contractions
e. Is commonly associated with repetitive early and variable decelerations

32. On examination you attempt to determine the presentation of the fetus. Which of the following
presentations and position would be the most favorable to achieve vaginal delivery ?
a. Breech
b. Transverse
c. Vertex with occiput
posterior d. Vertex with occiput
anterior
e. Vertex with occiput transverse
33. The patient is 5 cm dilated with regular contactions. However on the CTG you find several
variable decelarations. What is the pathophysiology of variable declarations ?
a. Normal labor
b. Head compression
c. Cord compression
d. Maternal chronic anemia
e. Uteroplacental insufficiency

34. She pushes the head to the perineum and your deliver the head and the shoulders without
complication. The cord is clamped and the placenta delivered. You examined her for lacerations.
A second-degree laceration
a. Involves the anal mucosa
b. Is commonly associated with buttonhole lacerations
c. Involves the mucosa or the skin only
d. Will heal well without repair
e. Extends into the perineal body, but does not involve the anal sphingter

You are providing prenatal care to a 22-year-old G1P0 woman at 16 weeks GA by LMP. She has
had a relatively smooth pregnancy without complication thus far. At 5950 and 215 lb she has
obese BMI, otherwise without medical or surgical history. She presented to prenatal care at 14
weeks and so missed first-trimester screening. She undergoes the quad screen and has an
elevated level of maternal serum alpha-fetoprotein (MSAFP)

35. Given the elevation in MSAFP her pregnancy is at increased RISK for which the following ?
a. Gestational
diabetes b.
Gastroschisis
c. Down syndrome
d. Klinefelter syndrome
e. All of the above

36. You discuss the potential meaning of the elevated MSAFP. After a long conversation, the patient
decides to undergo her second trimester ultrasound. The ultrasound reveals a myelomeningcele.
Which of the following is true and may be used in counseling ?
a. This is generally a lethal anomaly
b. Delivery must be by cesarean to protect the baby
c. Fetal surgery includes laser therapy
d. Fetal surgery has been shown to improve some outcomes
e. Fetal surgery is experimental and has no known benefits

37. The increased incidence of this finding is a associated with which of the following medications
when used in pregnancy ?
a. Valproic acid
b. Lithium
c. Fluoxetine
d. Prednisone
e. Acetaminophen

38. In a subsequent pregnancy, prevention of reccurence would include :


a. Low-dose aspirin
b. Low molecular weight heparin
c. Prenatal vitamin taken twice per
day d. 4 mg folic acid
e. Increased dietary calcium

A 32-years-old woman is 36 weeks pregnant in first pregnancy with DCDA (dichorionic


diamniotic) twins and is being reviewed in the antenatal clinic. A recent obstetric growth scan
confirm both fetuses are normally grown. Both twins are longitudinal lie and cephalic
presentation. She has no other complicating medical or obstetric disorders. She is deciding
between planned vaginal or elective cesarean modes of delivery.

39. Which one of the following is correct in relation to the counseling she will receive ?
a. About 10% of twin pregnancies result in spontaneous birth before 37 weeks
b. Continuing twin pregnancies beyond 38 weeks increases the risk of fetal death
c. Maternal antenatal corticosteroids are routinely recommended for all twin pregnancies
d. Offer elective birth from 37 weeks after a course of maternal corticosteroids has been
administered
e. There is strong evidence to show cesarean delivery as safer for mother and fetuses than
vaginal mode of delivery

40. Which of the following statements about twinning is true ?


a. The frequencies of monozygosity and dizygosity are the same
b. Division after formation of the embryonic disk results in conjoined twins
c. A dichorionic twin pregnancy always denotes dizygosity
d. Twinning causes no appreciable increase in maternal morbidity and mortality over singleton
pregnancy

41. You are following a 38-year-old G2P1 at 39 weeks in labor she has had one prior vaginal delivery
of a 3800-g infant. One week ago, the estimated fetal weight was 3200 g by ultrasound . over the
past 3 hours her cervical examination remains unchanged at 6 cm. fetal heart rate tracing is
reactive. An intrauterine pressure catheter (IUPC) reveals two contractions in 10 minutes with
amplitude of 40 mm Hg each. Which of the following the best management for this patient ?
a. Ambulation
b. Sedation
c. Administration of oxytocin
d. Cesarean section
e. Expectant

42. A primipara is in labor and an episiotomy is about to be cut. Compared with a midline
episiotomy, which of the following is an advantage of mediolateral episiotomy.
a. Ease of repair
b. Fewer breakdowns
c. Less blood loss
d. Less dyspareunia
e. Less extension of the incision

43. A 24-year-old primigravid woman, at term, has been in labor for 16 hours and has been dilated
to 8 cm for 3 hours. The fetal vertex is in the right occiput posterior position, at +1 station, and
molded. There have been mild late decelarations for the pas 30 minutes. Twenty minutes ago,
the fetal scalp pH 7.27;it is now 7.20
For above clinical description, select the most appropriate procedure
a. External version
b. Internal version
c. Midforceps relation
d. Low transverse cesarean section
e. Classic cesarean section

44. A 27-year-old G2P1 at 38 weeks gestation was admitted in active labor at 4 cm dilated;
spontaneous rupture of membranes occurred prior to admission. She has had one prior
uncomplicated vaginal delivery and denies any medical problems or past surgery. She reports an
allergy to sulfa drugs. Currently, her vital signs are normal and the fetal heart rate tracing is
reactive. Her prenatal record indicates that her Group B streptococcus (GBS) culture at 36 weeks
was positives. What is the recommended antibiotic for prophylaxis during labor ?
a. Cefazolin
b. Clindamycin
c. Erythromycin
d. Penicillin
e. Vancomycin

45. Variable decelerations are caused by which fetal situation ?


a. Hypoxia
b. Academia
c. Cord compression
d. Head compression

46. With increasing gestational age, the fetal heart rate baseline undergoes which of the following
trends ?
a. Increase
b. Remains unchanged
c. Variability changes become more closely tied to activity changer
d. Decreases

47. Fetal bradycardia typically may result from which of the following ?
a. Maternal fever
b. Fetal head compression
c. Maternal atropine use
d. None of the above

48. A 20-year-old G1 at 38 weeks gestation presents with regular painful contactions every 3 to 4
minutes lasting 60 seconds. On pelvic examination, she is 3 cm dilated and 90% effaced; an
amniotomy is performed and clear fluid is noted. The patient receives epidural analgesia for pain
management, the fetal heart rate tracing is reactive. One hour later on repeat examination, her
cervix is 5 cm dilated and 100% effaced. Which of the following is the best next step in her
management ?
a. Begin pushing
b. Initiate Pitocin augmentation for protracted labor
c. No intervention; labor is progressing normally
d. Perform cesarean delivery for inadequate cervical effacement
e. Stop epidural infusion to enhance contractions and cervical change

A 63-year-old G4 P4 woman presents to your office with a chief complaint of vaginal spotting.
She reports an isolated episode 1 week prior to presentation that consisted of scant vaginal
bleeding. She denies any associated symptoms including pelvic pain, pressure, or early satiety.
She also denies any family history of gynecologic malignancy. Her past medical history in
significant for morbid obesity, hypertension and inflammatory bowel disease.

49. What is the most likely diagnosis


? a. Athrophic endometrium
b. Endometrial cancer
c. Endometrial polyp
d. Ovarian cancer
e. Adenomyosis

50. After obtaining a thorough history and performing a physical examination (including a pelvic
exam), what is the next best step in evaluation ?
a. CA 125
b. MRI
c. Cervical cytology
d. Transvaginal ultrasound and possible endometrial
e. Mammography

51. What is the patients most significant risk factor for endometrial cancer ?
a. Multiparity
b. History of prior tobacco use
c. Remote history of oral contraceptive use
d. Inflammatory bowel disease
e. Morbid obesity

52. The use of tamoxifen is associated with increased risk of which of the following ?
a. Endometrial intraepithelial neoplasia
b. Invasive carcinoma
c. Uterine sarcoma
d. Endometrial
polyps e. All of the
above

A 16-year-old girl presents to the physician for annual examination. She denies any current
symptoms or concerns. She has been sexually active for 1 year and is using Depo-Provera for
contraception. She does not use condoms. On urogenital examination, she has a moderate
amount of yellow mucopurulent discharge from the endocervix. There is no cervical motion,
adnexal, or uterine tenderness. Microscopy of the vaginal discharge was normal except greater
than 10 WBCs per high power field. Vaginal pH is normal.

53. What is the most appropriate next step in the management ?


a. Perform a Pap smear with reflex HPV testing
b. Order a pelvic ultrasound
c. Treat for presumptive chlamydial infection and gonorrhea
d. Send vaginal discharge for culture and Gram satin
e. Perform nucleic acid amplification test (NAAT) for chlamydial infection and gonorrhea

54. Test result confirm the diagnosis of gonorrhea. What is the best initial treatment for this patient
?
a. Ceftriaxone 125 mg IM once
b. Ceftriaxone 125 mg IM once plus azithromycin 1 g orally once
c. Doxycycline 100 mg orally twice daily for 7 days
d. Ciprofloxacin 500 mg orally once
e. Clindamycin 300 mg orally twice time daily for 7 days

A 22-year-old G3P1021 woman recently delivered and is now attempting to breastfeed her
baby. Her pregnancy and delivery were uncomplicated. She denies any medical history or social
history significant for drug use. She is frustrated but her lack of volume, worried that her son will
not gain weight, and is now requesting a bottle and prepared formula.
55. Which of the following statements about the benefits of breastfeeding is false ?
a. Breastfed children are more resistant to disease and infection early in life than formula-fed
children
b. Breastfeeding women have a lower risk of breast, uterine, and ovarian cancer if they have
breastfed for at least 2 years cumulatively
c. Children who are breastfed are significantly less likely to become obese later in childhood
d. Oxytocin released during breastfeeding causes the uterus to return to its normal size more
quickly
e. None. All of the above are true

56. Your patient is convinced of the benefits of breastfeeding and continues to try, successfully
breastfeeding by the end of postpartum day 1. The following morning however, she develops a
low-grade fever of 38.0˚C, for which your nurse alerts you. She complains about the pain
associated with the engorgement of her breasts bilaterally and very sharp. Recurrent pelvic
pains. Her vital signs are otherwise normal. What is likely explanation for these finding ?
a. Lactation fever
b. Mastitis
c. Breast abscess
d. Endometritis
e. Chorioamnionitis

57. Which of the following would be an appropriate form of contraception for this breastfeeding
patient ?
a. Progesterone-eluting intrauterine device (IUD)
b. Combined oral contraceptive pills
c. Contraceptive vaginal ring
d. Contraceptive patch
e. Non of the above

58. Your patient is admitted to the hospital 2 weeks later with rogors and chills and complaint of a
swollen and reddened right breast. She has been breastfeeding throughout the last 2 weeks. Her
vitals are significant for a fever up to 38.4˚C and tachycardia with pulse of 112; all other vital
signs are normal. Her physical examination is significant for cracked nipples and engorged
breasts bilaterally; her right breast is particularly tense, notable for erythema and increased
temperature compared with the left breast without masses. What is the appropriate therapy for
above condition ?
a. Dicloxacillin 3,10 to 14 days
b. Dicloxacillin until afebrile for 48 hours
c. Reassurance, ice, breast support, and breast pumping
d. Protective nipple shield and shooting ointments
e. Ultrasound-guided localization of abscess an aspiration
You are seeing a 20-year-old single G1P1 woman who is postpartum day 2 after a normal
spontaneous vaginal delivery of a healthy female infant. Her pregnancy and delivery were
uncomplicated. She is notable for being teary and anxious when you begin providing her
discharge instructions. She explains that she has been unable to get any sleep between her baby
crying, breastfeeding every 3 hours and her constant worries about whether she will be able to
handle a baby at home by herself. She is particularly pervasive thoughts that her daughter could
roll onto her stomach and be unable to breathe or that she could choke while breastfeeding
without her mother recognizing it because of her inexperience and sleep deprivation. She is
concerned about the overwhelming responsibility of raising a baby by herself, and that she may
never be able to o back to school; she expresses “this may all have been a bad idea” she denies
any history of depression.

59. Which of the following is the most important next step in evaluating the patient ?
a. Reassure the patient that she is likely experiencing a common condition called “the baby
blues”
b. Contact the father of the baby to ensure that the patient will have an alternative source of
childcare when she needs to care for herself
c. Offer a prescription sleep aid to help the patient get a full night’s rest
d. Prescribe the patient an SSRI for a new diagnosis of postpartum depression
e. Tell the patient that she likely has postpartum depression and should be seen by a counselor
while in the hospital

60. The patient sees you again 6 weeks later for her postpartum appointment and still reports
difficulty coping with her new baby. She is still having difficulty with sleep but is now unable to
stay asleep even when the baby has been sound asleep. She has been avoiding phone calls from
her friends because she does not want them to see her in this state. She has a limited appetite,
decreased interest in her normal sources of entertainment, and she reports jus generally being
sad since the baby was born. Although she has taken her baby to the pediatrician as needed and
notes interval weight gain, she reports having ignored her crying baby on more than one
occasion over the last few weeks. Which of the following is the most important next step in
evaluating the patient?
a. Tell the patient that she likely has postpartum depression and should be seen by a counselor
as soon as can be arranged
b. Prescribe the patient an SSRI for a new diagnosis of postpartum depression
c. Provide careful reassurance and arrange for follow-up appointment in 2 weeks to assess for
resolutions of symptoms
d. Contact the Department of Human Services for your concern regarding child neglect
e. Assess the patient for any current or historical thoughts of harming herself of her baby

61. After extensive counseling, the patient agrees to pharmacologic therapy for her postpartum
depression. In which of the following cases would an SSRI not be recommended for her?
a. The patient is currently breastfeeding
b. She reports a family history of bipolar disease
c. The patient is unlikely to be compliant with a daily medcation
d. She desires to becomes pregnant again in the near future
e. She reports having a glass of wine every other day

You see a patient for preconception counseling. She is a 24-year-old G3P0020 woman, and her
medical history is significant for IgA glomerulonephritis and recent ceratinine of 2.0. She is
otherwise healthy.

62. What can you tell her to expect regarding her kidney function if she were to get pregnant?
a. It will improve
b. It will stay at
home c. It will get
worse
d. It is unlikely will need dialysis by the end pregnancy
e. She will need kidney transplant in order to have a successful pregnancy

63. She return 6 months later and is 8 weeks pregnant. Her creatinine at this time is 1.8. Her
baseline 24-hour urine is 1,200 mg, and she has a mildly impaired creatinine clearance. You
again counsel that this is a high-risk pregnancy and will need clode monitoring. Which of the
following is she not at risk for during this pregnancy?
a. Preeclampsia
b. Preterm delivery
c. IUGR
d. Fetal cardiac malformations
e. Worsening renal disease

64. A patient present with amenorrhea and galactorrhea. Her PRL level are elevated. She is not has
never been pregnant. In addiction to evaluating her for a prolactinoma, one also needs to
evaluated for other causes that would increase PR such as elevated level of which of following?
a. Corticotropin-releasing hormone (CRH)
b. Dopamine
c. Gamma-aminobutyric acid (GABA)
d. Histamine type II receptor activation
e. Thyrotropin releasing hormone (TRH)

65. A 24-years-old G1P1 female comes to the office colposcopy due to LSIL pap 1 month prior. He
pap was normal 3 years ago. She began sexual activity at age 16 and had five partners. She has
had chlamydia and cigarettes per day since she was 15. She uses depo Provera for contraception.
Her mother died last year of cervical cancer at age 44. What are her risk factors for CIN?
a. Age 16 at onset of sexual activity
b. Five sex partners
c. Her history of STDs
d. Smoking 15 cigarettes a day
e. All of the above

66. A 67-years-old woman present with abdominal discomfort and bloating, 10Kgs weight loss,
decreased appetite, and fatigue. Vital signs are stable. Physical demonstrates a menopausal
woman with large left adnexal mass detected on bimanual examination. You perform a
transvaginal ultrasound that shows an 8 cm complex mass with solid and cystic examination,
thick irregular septations, and an irregular surface. Based on your clinical assumptions, what is
the most likely course of management for this patient?
a. Chemotherapy
b. Abdominal exploration with surgical resection
c. Neoadjuvant chemotherapy, abdominal exploration, and surgical resection
d. Abdominal exploration with surgical resection followed by chemotherapy
e. Abdominal exploration with surgical resection followed by radiation

67. A 15-years-old girls is seen in the emergency department. She has a sudden onset of heavy
vaginal bleeding. She has noted irregular, painless vaginal bleeding of 6 months duration. Her
medical history is unremarkable, and she is sexually active. Physical and pelvic examinations are
normal. But blood is coming through the cervical os. A serum pregnancy test is negative, and
complete blood cell count has hematocrit of 37% and normal white blood cell and platelet
counts. Which of the following is the course of immediate action?
a. Observations
b. Estrogen therapy
c. Progesterone therapy
d. Nonsteroidal anti-inflammatory therapy
e. D&C

68. A 27-year-old P3AI return to your clinic to follow up on the result of her annual pap smear. She
has had a full annual examination including pap smear since the age of 21 and has never had
abnormal cervical cytology. Her STD screening at her last examination was normal. The cytology
of her pap smear showed high-grade squamous intraepithelial lesion (HGSIL). You discuss the
management options to the patient, and she decides to have a colposcopy. The biopsy result of
the colposcopy you perform are insufficient to give a histology report. Which of the following
options is the next best step in management for this patient?
a. Repeat Pap smear at 6-month interval for a year
b. Repeat colposcopy to obtain better sample
c. Perform diagnostic excisional procedure such as Loop electrical excision procedure (LEEP)
c. Request that the repeats their test on the biopsy samples given
d. Conization
69. A 44-years-old P5 patient who is currently using oral contraceptive pills to control menorrhagia
had a hysterectomy for uterine enlargement. You suspect adenomyosis by history. Which
histological description supports the diagnosis of adenomyosis?
a. The metaplastic change of glandular epithelium to muscle fibers in the uterus
b. The same pattern and location as endometriosis
c. The presence of endometrial glands and stroma deep within uterine muscle
d. A premalignant change of the endometrium
e. A premalignant change of the uterine muscle

70. A 38-year-old African American woman present with heavy menses and an enlarged uterus.
After an examination the clinical diagnosis is leiomyoma of the uterus. Which of the following
best describes this finding?
a. A soft, interdigitating mass of the uterine wall
b. A premalignant papule of the uterine wal
c. A rapidly dividing necrotic malignancy
d. A rounded, smooth, firm, well-circumscribed mass
e. Erythematous, tender, and hereditary

71. A 45-years-old woman endergoes an abdominal hysterectomy for a large fibroid uterus. She is
found to have a fibroid in broad ligament, and there is a concern that her ureter may have been
damage during the difficult surgery. Which if the following is the least possible site of ureteric
injury in this surgery?
a. At the level of the uterosacral
b. Lateral to the uterine vessels
c. Renal pelviureteric junction
d. The area of the ureterovesical junction close the cardinal
e. The base of the infundibulopelvic ligament as the ureters cross the pelvic brim at the ovarian
fossa

72. A mother brings her 2-month-old daughter to pediatrician because she feels a lump in her
abdomen. On pelvic ultrasounds the infant has an ovarian mass. When counseling the mother,
you inform her that the most common ovarian lesion associated with the transient elevated
gonadotropins in a female newborn during the first 6 to 12 months of life is which of the
following?
a. Granulosa cell tumor
b. Leimyoma
c. Serous cystadenoma
d. Single large follicular cyst
e. Theca cell tumor

73. A woman has had a previous child with renal agenesis. She is a middle school biology teacher
and wants to understand more about development of kidneys. Which of the following best
describe the function of pronephros?
a. The begin the developmental sequence that forms the permanent excretory ducts and kidneys
b. They are primitive kidney and ureter that will mature into the adult urinary tract
c. They develop as the primitive kidney and migrate caudally and laterally to form the
mesonephros
d. They will serve as the fetal kidney until 16 weeks development of metanephos
e. They form primitive kidney and primitive upper genital ducts.

74. A 13-years-old adolescent girl who presents with cyclic pelvic pain. She has never had menstrual
cycle. She denies any history of intercourse. She is afebrile and her vital signs are stable. On
physical examinations, she age-appropriate breast and public hair development and normal
external genitalia. However, you are unable to locate a vaginal introitus. Instead, there is a tense
bulge where the introitus would be expected. You obtain a transabdominal ultrasound, which
reveals a hematocolpos and hematometra. What is the most likely diganosis?
a. Transverse vaginal septum
b. Longitudinal vaginal
septum c. Imperforate hymen
d. Vaginal atresia (MRKH)
e. Bicormuate uterus

75. A 7-years-old girls is brought in for evaluation. In examination, she has well-developed hair and
breast and she is 99% of height for her age. Her mother recently noted some bloodstains on her
underwear. Which of the following conditions is most likely the cause of these findings?
a. Estrogen-producing ovarian cyst
b. Hepatoma
c. Hypothalamic tumor
d. Sex streoid-containing medication
e. Thecal / Leydig cell tumor

76. A 57-years-old, postmenopausal woman complain of something coming out of her vagina, and
difficulty in defecating. She had four children uneventfully with vagina births. Her last child birth
was 18 years back/the most likely diagnosis is?
a. Anterior vaginal wall prolapse
b. Genuine stress incontinence
c. Overflow incontinence
d. Posterior vaginal wall prolapse
e. Stress ncontinence

77. After trying behavioral and lifestyle modifications, the patient continues to be symptomatic.
Urodynamic studies are done to further evaluate the incontinence. Cystometry reveals leakage
of urine with increase in intra-abdominal pressure during valsava effort and coughing. No
involuntary detrusor contractions are seen. Bladder compliance is normal. The pressure flow
pattern is continuous and normal with a void initiated by urethral relaxation and normal
detrusor
contraction. What type of urinary incontinence does this relaxation and normal detrusor
contraction. What type of urinary incontinence does this patient most likely have?
a. Stress incontinence
b. Urgency incontinence
c. Overflow incontinence
d. Mixed incontinence
e. Functional incontinence

A 20-years-old nulligravid young woman present to the emergency department 4 hours after
the onset of nausea, vomiting, and moderate lower abdominal pain. Her last menstrual period
was 2 weeks ago. She reports three new sexual partners in the last 6 months and uses condoms
intermittently. She denies any history of sexually transmitted infections. Her temperature is
38.0oC (100.4oF), pulse rate is 96/min, respirations are 20/min, and BP is 110/60 mm Hg,
Examinations shows a soft abdomen and lower quadrant tenderness without guarding or
rebound. On pelvic examinations, there is a mucopurulent cervical discharge, moderate cervical
motion tenderness, and uterine tenderness. Bilateral adnexa are nontender and without
enlargement.

78. What is the most likely diganosis?


a. Ectopic pregnancy
b. Pelvic inflammatory disease
c. Ovarian torsion
d. Pyelonephritis
e. Cervicitis

79. What is this patient’s future risk of infertility?


a. 1%
b. 12%
c. 20%
d. 40%
e. 60%

80. Which method of contraception is not recommended in this patient?


a. Combined oral contraceptive pills
b. Injectibles
c. Condoms
d. Intrauterine contraceptive device
e. Progestin implants

A 28-years-old G2P1 at 36 weeks was referred to your clinic from a midwife, due to previous
cesarean section. She had a cesarean section due to breech presentation. She asks if she could
deliver vaginally.
81. What is a contraindication to a trial of labor after cesarean (TOLAC) ?
a. Prior low transverse uterine
incision b. Prior intra mural
myomectomy
c. Small-for-gestational age fetus
d. Oligohydroamnions
e. Prior median abdominal skin incision

82. You performed trial of labor after cesarean in this patient. When the cervix was approximately
7- cm dilated and the fetus was at-1 station, she developed tachycardia to 120 and a drop in
blood pressure from 115/60 to 70/30. She has become dizzy upon sitting up. FHTs are 130. No
bleeding is evident. The hematocrit is reported at 35%. What is your primary diagnosis ?
a. Vasa previa
b. Pulmonary embulos
c. Chorioamnionitis
d. Supine hypotensive
syndrome e. Ruptured uterus

A 30 years-old G3P2 at 38 weeks came to the emergency department complaining of regular


uterine contractions. She only had 1x antenatal care and 1x ultrasound at 20weeks stating she
had twins. You performed an examination and found two fetal hearts tones, contractions 3-
4x/10’/40”, and 6 cm dilatation

83. In which of the following conditions would you allow to proceed for vaginal delivery ?
a. Breech 3000 g – transverse 2800 g
b. Cephalic 2800 g – transverse 2600 g
c. Cephalic 2800 g – breech 3400 g
d. Breech 2800 g – cephalic 2600 g
e. Transverse 2600 g – breech 3000 g

84. In which of the following cases might internal podalic version be indicate
? a. Vertex delivery of the first twin and transverse lie of the second twin
b. Term transverse lie with cervix completely dilated and membranes intact
c. Double footling breech
d. Impacted shoulder presentation
e. Compound presentation

85. A 19 years old has been bleeding intermittently for 26 weeks. She presents with a change to dark
bleeding and minimal cramping for 4 days. Your clinic ultrasound notes an intrauterine fetal
demise. She is distraught. She would prefer to let “nature take its course”. She ask if there is any
risk to her. What do you explain as the only significant risk ?
a. Positive human chorionic gonadotropin (hCG) titer
b. Systemic allergies
c. Bone marrow
depression d. Coagulopathy
e. Toxemia

A 28-years-old G3P0AB2 has a quantitative hCG of 2,850. She has spotting and abdominal pain.
An ultrasound shows fluid in the cul de sac and intrauterine pregnancy.

86. What is the most common site of an ectopic pregnancy ?


a. External fallopian tube
b. Ovarian surface
c. Mesosalpinx
d. Ampulla of the fallopian tube
e. Interstitial portion of the fallopian tube

87. An intrastitial (cornual) pregnancy is discovered on ultrasound and it is verified by laparoscopy.


Which of the following best described the condition ?
a. It rarely exceeds week’ gestation
b. It generally more dangerous than an ampullary ectopic pregnancy
c. It requires hysterectomy
d. It is quite common
e. It is extrauterine

88. Which of the following condition is suitable for medical management of ectopic pregnancy with
methotrexate ?
a. Unstable hemodynamics
b. Gestational sac 4 cm
c. Negative embryonic cardiac motion
d. Thrombocytopenia
e. Creatinine level 2.8 mg/dl

89. A 15-years old patient delivered a term infant yesterday. She ask you about breastfeeding.
Which of the following condition is breastfeeding not recommended ?
a. Hepatitis B
b. HIV
c. Tuberculosis
d. Mastitis
e. Postpartum blues

90. After your explanation, she refuses to breast-feed because she wants to place the child for
adoption. What is simplest and safest method of lactation suppression ?
a. Breast binding, ice packs, and analgesics
b. Bromocriptine
c. Domperidone
d. Depo-Provera
e. Oral contraceptive pills

A 42-years old G4 P3 woman presents to your emergency department with a 6 month history of
irregular bleeding and a new onset of coughing up blood. Her history reveals three term vaginal
deliveries, her last approximately 6 month ago. That delivery was uncomplicated. On physical
examination, vital signs stable, her uterus is approximately 10 to 12 weeks size, and the is
moderate amount of blood in the vaginal vault. CXR show a new single nodule in the left lower
lobe suspicious for a metastatic lesion for unknown location.

91. Which a following laboratory tests will most likely assist in her diagnosis ?
a. CA 125
b. Serum β – hCG
c. CBC
d. Prothrombin time
e. Fibrinogen

92. The quantitative serum β – hCG is 108,000 mUI/ml. which of the following is the most likely
diagnosis ?
a. Incomplete molar pregnancy
b. Complete molar pregnancy
c. Persistent molar pregnancy
d. Gestational choriocarcinoma
e. Placental site trophoblastic tumor

93. The pelvic ultrasound reveals bilateral multicystic ovarian masses along with an enlarged uterus.
What is the most likely diagnosis and most appropriate management of this finding ?
a. Metastatic lesions/surgical intervention
b. Primary epithelila ovarian carcinoma/surgicall intervention
c. Theca lutein cysts/percutaneous drainage
d. Metastatic lesions/chemotherapy
e. Theca lutein cysts/conservative surveillance

94. You refer the patient to a gynecologic oncologist for evaluation and management of
choriocarcinoma. What is the most likely intervention to be recommended ?
a. Total abdominal hysterectomy
b. Serum β- hCG
surveillance c. Chemotherapy
d. Whole pelvis radiation
e. Pulmonary wedge resection
A 24-years-old G1P1 female comes to the office for colposcopy due to LSIL pap 1 month prior.
Her pap war normal 3 years ago. She began sexual activity at age 16 and has had five partners.
She has had Chlamydia and vulvar warts in the past both of which have been treated an d
resolved. She is smoker of approximately 15 cigarettes per day since she was 15. She uses Depo
Provera for contraception. Her mother died last year of the cervical cancer at age 44

95. What are her risk factors for CIN ?


a. Age 16 at onset of sexual activity
b. Five sex partners
c. Her history of STDs
d. Smoking 15 cigarettes a day
e. All of the above

A 26-years-old woman G1P0 22 weeks gestation, comes to your clinic to do her antenatal care
without specific condition on the nutrition and hydration status.

96. How much the additional water for daily consumption to this patient ?
a. 200 mL
b. 250 mL
c. 300 mL
c. 350 mL
e. No need additional water from standard recommendation

97. If dehydration condition, which the correction information :


a. Dehydration and increased osmolalitas serum maternal (>30 mOsm/kg) can cause
oligohidramnion
b. Dehydration and increased osmolalitas serum maternal (>30 mOsm/kg) can cause
polihidramnion
c. Decreased level of osmolalitas maternal (>20 mOsm/kg) can cause increasing volume of
amniotic
d. A+C are correct
e. B+C are correct

98. During pregnancy, recommendation intake water increase due to :


a. Support fetus circulation
b. Amniotic fluid production
c. Increased blood
volume d. A, B, C, are
correct
e. B, C, are correct
99. Based on Flack et al. Study (2009) :
a. Excess water consumption will not impact to amniotic fluid volume, and less of water
consumption not impact to amniotic fluid
b. Excess water consumption will impact to amniotic fluid
c. Excess water consumption will not impact to amniotic fluid volume, and less of water
consumption will impact on decreased of amniotic fluid until 8%
d. Normal water consumption will not impact to amniotic fluid volume
e. 75% pregnant mother with oligohidramnion with 2 L will improve amniotic fluid index

100. During pregnancy, role of water as nutrient


a. As main medium to translate macronutrient into energy
b. Gives energy
c. As sweetener
d. As antioxidant
e. Part of carbohydrate

UNAS MARET 2018

1. A 32-year-old primigravida commences a planned delivery in a rural stand-alone midwifery


unit. After 8 hours of established labour, the second stage of labour is diagnosed. The woman
develops an urge to push 1 hour later dan she commences pushing. After 30 minutes late
deceleration are heard on intermittent auscultation. On examination the fetus is cephalic, 2/5
palpable per abdomen, fully dilated, direct OP and at station spines-1. There is 3+ caput and 3+
moulding. A decision is made for transfer to hospital, although this is delayed because of
treacherous snow-covered and ice-covered roads. On arrival at hospital 3 hours later the CTG is
severely abnormal, with examination findings unchanged and a category 1 caesarean section is
performed. Ten minutes after delivery resuscitation is stopped for a few seconds while the
neonate is reassessed. The fetus is still extremely floppy, pulse 45 bpm, no response to stimulation
and no spontaneous breathing. It is blue. What is the infant’s 10-minute Apgar score?
a. 0
b. 1
c. 2
d. 3
e. 4
2. You see a 16 yo female who presented with primary amenorrhea. Breast development was
noted at 13 years but there has been no increase in breast size. Pubic and axillary hair was noted
within one year of referral. An outside ultrasound showed no uterus or ovaries. Physical exam
reveals a normal vaginal introitus with hymen present. Breasts are Tanner 3, but seem to be more
fatty than mammary tissue. Lab test were significant for absent estradiol, elevated gonadotropins
and mildly elevated DHEA with normal testosterone. A karyotipe was requested by the
endocrinoilogist and was found to be 46, XY (SRY gene +). The MRI showed an infantile uterus with
no discernible gonads. During a laparoscopic evaluation, two dysplastic gonads attached to small
falopian tubes and a rudimentary uterus was visualized. The final diagnosis is:
a. Swyer Syndrome
b. Androgen insensitivity
c. Mullerian Agenesis
d. Partial gonadal dysgenesis
e. Testicular Regression Syndrome

3. A 28-year-old amenorrhoeic woman who wishes to become pregnant attends the fertility
clinic complaining of galactorrhoea and mild visual disturbance. Her serum prolactin level was
found to be elevated. An MRI scan of the head is performed, which showed the presence of a
macroprolactinoma, but without supracellar extension. What is the most appropriate first line
management?
a. Bromocriptine
b. Cabergoline
c. Quinagolide
d. Radiotherapy
e. Trans-sphenoidal surgical excision of the prolactinoma -> tergantung ukuran supracellar > 5
cm

4. A 31-year-old patient infertile for 4 years and preferred for start in vitro fertilization (IVF)
because of obstructed fallopian tubes. On hysterosalpingogram (HSG), it is noted that she has large
dilated hydrosalpinges present bilaterally. What should be your next step?
a. The patient should begin her IVF treatment cycle
b. The patient should repeat the HSG to confirm the result
c. The patient should not be offered the opportunity to have IVF
d. Her hydrosalpinges should be drained via transvaginal aspiration prior to starting IVF
e. Bilateral salpingectomies should be done prior to starting IVF

5. A 24-year-old G1P0 at 13 weeks’ gestation who has sickle-cell anemia requests genetic
counseling to learn about the risk of transmission to her fetus. If her partner is a heterozygous
carrier for this condition, what is the risk that their offspring will be affected?
a. 25%
b. 50%
c. 75%
d. 80%
e. 100%

6. A 23-year-old G2P0 1 00 at 25 weeks presents with a complaint of loss of fluid vaginally. The
fluid was copious and clear. Assessment by sterile speculum examination reveals vaginal pooling;
ferning was noted under the microscope and nitrazine paper turned blue. She is very worried
because she lost her first son about 4 weeks after he was born at 26 weeks due largely in part to
severe respiratory distress. To reassure her, you discuss using antenatal corticosteroids. Which of
the following is the most correct statement with respect to antenatal corticosteroids?
a. They are only helpful if given at least 24 hours to delivery
b. Men have better outcomes than women.
c. Maternal infections are increased
d. Surfactant alone is more effective than in combination with corticosteroids
e. They are less effective in cases of ruptured compared with intact membranes

7. A 41-year-old primigravida presents to your office during the ninth week of pregnancy.
Bacause of her age, so they are extremely anxious and would like prenatal diagnosis as early as
possible. They have many questions about chorionic villus sampling (CVS). You explain to the
couple that complications of CVS include which of the following?
a. Amniotic band syndrome
b. Vaginal bleeding
c. VATER syndrome
d. IUGR
e. Rupture of the umbilical cord

8. A 33-year-old G2P1 woman at 10 weeks’ gestation presents to clinic for routine prenatal
visit. Her pregnancy has been uncomplicated. Her prior pregnancy 2 years ago was complicated by
the diagnosis of gestational hypertension that led to an induction of labor. She has no other
medical history. On examination her BP is 138/88 mmHg, her body mass index (BMI) is 37 kg/m,
and a urine dip shows trace protein. The patient’s 24-hour urine shows 100 mg of protein. You
counsel the patient on the pregnancy complications associated with chronic hypertension as well
as management. What is the next best step in her management?
a. Recommend termination of pregnancy
b. Start antihypertension therapy
c. Bed rest
d. Hospitalization for further maternal and fetal testing
e. Expectant management with close observation for early signs of preeclampsia and fetal
growth restriction

9. A 24-year-old G2P1A1 woman referred from primary health care due to HIV. Her pregnancy
is at 10 weeks’ GA. She had never been on HAART medication before this pregnancy. You order a
viral
load and it returns at 12.000. Her CD4 count is normal (600). She is otherwise healthy and has no
other significant medical history. You start her on a three-drug HAART regimen of zidovudine,
lamivudine, and lopinavir/ritonavir at 14 weeks. Her viral load was decreased in the second anf
third trimester, 5.000 and 2.250. at 39 weeks she is admitted to labor and delivery in active labor.
What would you recommend to her in order to further decrease neonatal transmission?
CD4 < 350: mulai pengobatan ARV
Pervaginam: viral load < 1000, pengobatan ARV > 6 bulan
a. Cesarean delivery
b. Early artificial ROM for labor augmentation
c. Stopping all antiviral therapy to decrease risk of neonatal resistance
d. Assisted vaginal delivery with vacuum of forceps
e. Placement of a fetal scalp electrode to monitor for signs of fetal distress

10. Mrs. Y, 37 yo, G2P1 37 wga desire VBAC (vaginal birth after cesarean section), but she
worries about its complication. Her baby is in a vertex presentation and her previous low
transverse procedure was for postterm and failed of induction. In providing informed consent, in
which of the following ways do you explain the risk of uterine rupture?
a. Less than 1%
b. Between 2% and 5%
c. Dependent on the length of her labor
d. 15-20%
e. Dependent on the location and proximity of the scar site to the placental
implantation.

11. A 27-year-old G2P1 woman at 8 weeks’ GA comes to your clinic for an initial prenatal visit. A
medical history reveals a diagnosis of lupus 2 years ago. She is in normal condition right now. What
is the most significant neonatal complication of maternal lupus?
a. Congenital abnormalities
b. Neonatal thrombosis
c. Neonatal heart block
d. Acute respiratory distress syndrome (ARDS)
e. Feeding difficulties

12. A 31-year-old G4P0A3 woman presents to the hospital with vaginal bleeding and abdominal
pain. She appears pale and states that she feels lightheaded when sitting up or standing. She
reports that she is currently 8 weeks’ pregnant. On arrival, her temperature is 37°C, BP is 80/50,
pulse rate is 115 beats per minute, and respiratory rate is 20 breath per minute. Abdominal
examination reveals a rigid abdomen with rebound tenderness to palpation. Pelvic examination
reveals a small amount of vaginal bleeding, a 6-week-size uterus, and fullness at the right adnexa.
A urine β-hCG confirms that she is pregnant. A pelvic ultrasound reveals a right-sided ectopic
pregnancy as well as large amounts of fluid, thought to be blood in the abdomen. She now has IV
access and a bolus of IV
fluids is being given. Her BP is now 75/45 and her pulse rate is 120 beats per minute. Her hematocrit
returns as 25,2%. How is the next management for this patient?
a. Proceed with emergency laparoscopic salpingectomy
b. Proceed with emergency laparotomy
c. Administer IM methotrexate
d. Transfuse the patient with two units of packed RBCs and transfer her to the ICU
e. Start vasopressors and transfer the patient to the ICU

13. A patient presents for her routine prenatal visit at 30 weeks’ EGA. Her pregnancy up to now
has been uncomplicated. Her BMI is 25. Her laboratory testing is normal including a glucose
challenge screen. Anatomic ultrasound done at 12 weeks was normal and confirmed her dating.
Her fundal height is 37 cm today. A brief bedside ultrasound reveals an amniotic fluid index (AFI) of
30 cm. Which of the following situations is most likely to be the etiology of polyhydramnions in this
case?
a. Renal atresia
b. Pulmonary hypoplasia
c. Duodenal atresia
d. Gestational diabetes
e. Anencephaly

14. A 37-year-old G6P3205 at 34 weeks of gestational age with gestational diabetes moderately
controlled with insulin presents for her scheduled obstetric (OB) appointment. During the course
of the visit, the patient notes that the fetus is moving, but “sleeps a lot” her previous pregnancy
was also complicated by gestational diabetes and fetal macrosomia. Her BP is 138/85. Fetal heart
tones (FHTs) are in 130s. Fundal height measures 36 cm. Her last ultrasound was at 28 weeks and
showed a fetus in the 93rd percentile.
What is the next step in the management of this patient for this visit?
a. Schedule biweekly nonstress tests (NSTs)
b. Schedule a growth ultrasound
c. Draw preeclampsia labs and send her to triage for evaluations
d. Reassure her and return to clinic in 2 weeks
e. Refer to a maternal-fetal specialist for further management

15. Your patient is a 21-year-old nulligravida currently desiring pregnancy and attempting
conception. Her last menstrual period was 6 weeks ago. She presents with complaints of vaginal
spotting. Urine β-hCG positive, vital signs are normal, hematocrit is 36 volume percent. From
sonographic findings found the trilaminar endometrial lining. Other findings include normal
myometrium, a cul-de-sac without free fluid, and normal adnexa. Her serum β-hCG level is
1200miU/mL. She is planned to have another β-hCG measurement in 48 hours. She returns in 48
hours with minimal right lower quadrant pain. Her vital signs are normal, and spotting has
decrease. Her clinical and laboratory findings are unchanged except for a serum β-hCG level now
measuring 2300 miU/mL. What is the next best clinical step for this patient?
a. Administer methotrexate
b. Repeat transvaginal sonography
c. Perform dilatation and curettage
d. Schedule repeat serum β-hCG level in 48 hours
e. Emergency laparotomy

16. A 30-year-old woman has a twin pregnancy at 12 weeks gestation. During her initial prenatal
care visit, you review risks for multifetal pregnancies. Which of the following statements reflects
the most frequent risks in twin pregnancies?
a. Cesarean delivery is necessary in all of twin deliveries
b. Shoulder dystocia occurs more in the aftercoming vertex twin, as compared to a singleton
c. Pregnancy-induced hypertension occurs at a higher rate than in singleton
d. Perinatal death rate is less than that of singleton
e. Congenital anomalies occur at the same rate as in singletons

17. A 30-year-old G2P1 woman at 28 weeks’ GA comes to your office for a routine prenatal visit.
Her child was recently send home with a rash and fever. She states that the child had a rash on
both cheeks and the pediatrician said it was a viral infection called fifth disease. Her baby is moving
well and denies any vaginal bleeding, abnormal vaginal discharge, or contractions but she wonders
if she needs any more testing to see if she has been affected. What is the most likely causative
organism of the child’s infection?
a. Varicella
b. CMV
c. Parvovirus
d. Toxoplasmosis
e. Listeriosis

18. A pair of monochorionic twins presents at 21 weeks’ gestation with sonographic findings that
suggest twin-twin transfusion syndrome. This is significant growth discordance, no bladder is
visualized in the smaller twin, neither twin has ascites or hydrops, and umbilical Doppler studies
are normal. What should be the assigned Quintero stage?
a. Stage I
b. Stage II
c. Stage III
d. Stage IV
e. Stage V

19. You are providing prenatal care to a 24-year-old G1P0 woman at 17 weeks GA by LMP. She
has had a relatively normal pregnancy thus far. She has an obese BMI, otherwise without medical
or surgical history. She presented to prenatal care at 15 weeks, and so missed first-trimester
screening. She undergoes the quad screen and has an elevated level of maternal serum alpha-
fetoprotein (MSAFP). Given the elevation in MSAFP, her pregnancy is at increased risk for which of
the following?
a. Gestational diabetes
b. Gastroschisis
c. Down syndrome
d. Klinefelter syndrome
e. Potter syndrome

20. A 27 years old woman, 32 weeks’ gestation complains of shortness of breath during her
pregnancy, especially with physical exertion. She has no prior medical history. Her respiratory rate
is 16x/m. Lung auscultation are clear and her oxygen saturation to be 98% on room air. You
reassure her that this sensation is normal and explain which of the following?
a. Because of enlarging uterus pushing up on the diaphragm, her vital capacity
is decreased by 20%
b. Airway conductance is decreased during pregnancy
c. Pulmonary resistance increases during pregnancy
d. Small amniotic fluid emboli are shed throughout pregnancy
e. Maximal breathing capacity is not altered by pregnancy

21. A G3P2A0 woman at 35 weeks is seen in your office for her prenatal visit. She is concerned
because she has not felt her baby moving as much as she used to. Her pregnancy has been
uncomplicated and her past two pregnancies ended in full term, normal spontaneous vaginal
deliveries. When formal antenatal testing is done, which of the following is most reassuring?
a. Late decelerations on fetal monitoring
b. A contraction stress test (CST) with variable fetal heart rate (FHR) decelerations with
contractions, but moderate variability
c. A nonstress test (NST) with two accelerations of the FHR in 20 minutes that are at least
15 beats above baseline and last for at least 15 seconds
d. An increase in the systolic to diastolic ratio in the umbilical artery blood flow
e. A score of 6 on a BP

22. A 24-year-old woman presents to clinic with an atypical squamous cells of undetermined
significance (ASCUS) Pap. She had a Pap about 2 to 3 years ago, and it was normal. She became
sexually active at age 17 and has had two lifetime partners. She and her male partners have been
together for over a year. She started having periods at age 13 and has regular cycles on her own.
Her last period was 1 week ago on oral contraceptives. She has a history of herpes simplex virus
well controlled with daily acyclovir. Her physical examination is normal. what is the next step?
a. Repeat the Pap
b. Get high-risk human papillomavirus (HPV) typing
c. Get low-risk HPV typing
d. Perform colposcopy
e. Perform a conization
23. A 26-year-old P2A0 presents for her annual gynecologic examination and would like to
discuss her risk of ovarian cancer. Menarche occured at age 14. She used oral contraceptive pills
(OCPs) for 3 years prior to the birth of her first child when she was 23. She breastfed both of her
children for 1 year each. Her mother is 46 and currently is undergoing chemotherapy for ovarian
cancer, and she had a maternal aunt who passed away in her 50 from breast cancer. What is this
woman’s greatest potential risk factor developing ovarian cancer?
a. BRCA (breast cancer gene) mutation
b. Hereditary nonpolyposis colorectal cancer (HNPCC) mutations
c. Family history
d. Early menarche
e. Late childbearing

24. A 27-year-old, P1, woman comes to see you to discuss a recent decrease in sexual desire. The
patient is in a 7-year monogamous sexual relationship with her husband and states that otherwise
they have a great relationship. You start by discussing the four phases of the female sexual
response: desire, arousal, orgasm, and resolution.
What is the most likely problem noted in evaluating sexual dysfunction?
a. The sequence stated above is out of order
b. There is a missing step in the sequence
c. There is a lack of satisfaction with the sexual encounter
d. The patient reports pain with intercourse

25. An 18-year-old nulligravid woman presents to the student health clinic with a 4-week history
of yellow vaginal discharge. She also reports vulvar itching and irritation. She is sexually active and
monogamous with her boyfriend. They use condoms inconsistenly. On physical examination, she is
found to be nontoxic and afebrile. On genitourinary examination, vulvar and vaginal erythema is
noted along with a yellow, frothy, malodorous discharge with a pH of 6.5. the cervix appears to
have erythematous punctuations. There is no cervical, uterine, or adnexal tenderness. The addition
of 10% KOH to the vaginal discharge does not produce an amine odor. Wet prep microscopic
examination of the vaginal swabs in performed. What would you expect to see under microscopy?
a. Brancing hyphae
b. Multinucleated giant cells
c. Scant WBC
d. Flagellated, motile organisms
e. Epithelial cells covered with bacteria

26. A 35-year-old G3P2 woman at 39 weeks and 4 days was admitted to labor and delivery unit
for active management of labor. After her membranes had ruptured, she is very uncomfortable
and finds her contractions very painful. You perform cervix examination is at 6 cm. Which of the
following presentations and positions would you expected be most favorable to achieve a vaginal
delivery? a. Vertex with occiput anterior
b. Vertex with occiput posterior
c. Vertex with occiput transverse
d. Vertex with sacrum anterior
e. Vertex with sacrum posterior

27. Mr and Mrs. S counseling about infertility. They had been married for 5 years. They are
planning to have an IVF. In your discussion about conception, tubal disease, and implantation, you
explain to them that implantation in the uterus occurs at which stage of development?
a. Eight-cell embryo
b. Blastocyst
c. Zygote
d. Morula formation
e. Embryonic disk

28. A 41-year-old P2 woman comes to see you with a complaint of 14 months of “irregular
bleeding” She previously had regular menses but over the past 2 years they have become totally
unpredictable. The doctor said that it might becaused by hormonal imbalance. Which of the
following hormones can have a negative as well as a positive feedback at the level of the pituitary?
a. Cortisol
b. Oxytocin
c. Estradiol
d. Progesterone
e. Insulin

29. A 23 year old G1 woman at 8 weeks’ GA comes to your clinic for an initial prenatal visit. She
had a history of cardiac disease since 5 years ago, but now she didn’t have any complaint and not
taking any medications since 1 year ago. She asks if she could continue her pregnancy, since many
people said that pregnancy could put her in danger. You explain that it may be due to the increase
of blood volume and cardiac changes in pregnancy. The increase in blood volume in normal
pregnancy is made up of which of the following?
a. Erythrocytes
b. More erythrocytes than plasma
c. More plasma than erythrocytes
d. Neither plasma nor erythrocytes
e. Plasma only

30. A woman presents for her new obstetrical visit at 12 weeks’ EGA. Her medical history is
complicated by Graves thyroiditis that has been treated with radioactive iodine a few years prior.
The patients is currently being maintaned on thyroid replacement. She is worried that this will
compromise the fetus. She is told that the interaction between maternal and fetal physiology
relative to thyroid function is complex. Which of the following is an accurate description of this
interaction?
a. Maternal thyroid hormones (T4 and T3) readily cross the placenta
b. Maternal thyrotropin easily crosses the placenta
c. The athyroid fetus is growth retarded at birth
d. The fetal thyroid concentrates iodide
e. The placenta serves as a barrier to maternal iodine crossing to the fetus

31. A 26-year-old G3P2A0 woman come to your clinic for preconception counseling. She has a
medical history of significant for IgA glomerulonephritis. Creatinine result is 2.0. She is otherwise
healthy. Which of the following you tell her to expect regarding her kidney function if she were to
get pregnant?
a. It will stay the same
b. It will improve
c. She will need kidney transplant in order to have a successful pregnancy
d. It will get worse
e. It is unlikely she will need dialysis by the end of pregnancy

32. You are asked to see a 28-year-old woman who is day 2 following a caesarean section for
unsuccessful induction of labour at 37 weeks’ gestation. She was diagnosed with mild
preeclampsia at 36 weeks’ gestation. Her blood pressure has been 155/100 mmHg on two
occasions today, although she is asymptomatic and her deep tendon reflexes are normal. she is
breast feeding. You decide to start her on an antihypertensive agent. Which of the following
antihypertensive agents has insufficient evidence on infant safety to recommend or use in breast
feeding mothers?
a. Amlodipine
b. Atenolol
c. Captopril
d. Enalapril
e. Metoprolol

33. A 28-year-old G1P0A0 woman at 38 weeks and 5 days presents to labor and delivery unit,
with regular contractions occuring 3-4x/10 minutes. Her contractions last 30 to 90 seconds. On
vaginal examination you found 10 cm dilatation and now in the second stage of labor. She is
pushing effectively, but during contraction you notice deceleration on fetal heart tracings. Which
of the following would be most dangerous?
a. Absence of decelerations
b. Isolated early decelerations
c. Repetitive variable decelerations that resolve quickly after each contraction
d. Repetitive early decelerations and variable decelerations
e. Repetitive late decelerations and loss of variability between contractions

34. A 40-year-old para 3 is delivered by spontaneous vaginal delivery and oxytocin 10 IU is given
intramuscularly. During cord traction the woman screams in severe pain, the uterus is no longer
palpable abdominally and the uterine fundus can be felt inverted in the vagina. The emergency
buzzer is pressed. What is the next immediate step that should be performed?
a. Administer tocolytic
b. Hydrostatic pressure with warm sodium chloride
c. Immediate manual replacement and simultaneous resuscitation
d. Immediate transfer to theatre for general anaesthetic and manual replacement in theatre
e. Verbal consent for Huntingdon’s procedure

35. After a prolonged second stage, a 28-year-old woman delivers the vertex with an immediate
turtle sign with the head retracting against the perineum. McRobert’s maneuver does not affect
delivery. Which of the following would be a helpful maneuver in managing this shoulder dystocia?
a. Fundal pressure
b. Internal podalic version
c. Increased maternal pushing effort
d. Wood’s screw maneuver
e. Ritgen maneuver

36. A 36-year-old G2P1 with one prior cesarean delivery presents at 36 weeks’ gestation with
active vaginal bleeding and now requires emergency cesarean hysterectomy due to placenta previa
with accreta. Compared with patients who have elective surgery, this woman is at increased risk or
which of the following complications?
a. Bowel injury
b. Urinary tract injury
c. Venous thromboembolism
d. Exterbal iliac vessels injury
e. Hypogastric nerve injury

37. A 30-year-old G1P0 woman at 40 weeks and 4 days presents to labor and delivery unit with
second stage of labor. She pushes the head to the perineum and you deliver the baby without
complication. You examine her for lacerations. Classification of perineal tear caused by either
tearing or episiotomy in which injury to the perineum involves less than 50% of the external anal
sphincter thickness torn, is classified as
a. Second degree
b. Third A degree (3A)
c. Third B degree (3B)
d. Third C degree (3C)
e. Fourth degree

38. A 28-year-old G1P0 woman at 39 weeks and 6 days presents to labor and delivery unit, with
regular contraction occuring every 3 minutes. Her contractions last 30 to 50 seconds. She not sure
if she’s been leaking any fluid from her vagina. You take her history and conduct a physical
examination. Rupture of membrane would be best supported by which of the following?
a. Nitrazine paper remaining orange when exposed to fluid in the vagina
b. A negative fern test
c. An ultrasound with a normal AFI
d. A negative tampon test
e. Speculum examination with evidence of pooling in the vagina

39. A 22-year-old primiparous woman develops postpartum hemorrhage unresponsive to


oxytocin and uterine massage. The uterine contraction was good. Her infant was 3800 grams. She
has bled 750 cc. What is the most possible diagnosis?
a. Laceration of cervix or vagina
b. Placenta acreta
c. Uterine inversion
d. Ruptured uterus
e. Coagulopathy

40. A 38-year-old Asian mother has delivered her fourth baby normally. She is a known Type 2
diabetic and was taking metformin prior to pregnancy for glycemic control. From 32 weeks
gestation, Isophane insulin was added twice daily in addition to Metformin to achieve glycemic
control. The woman is planning to breast feed. What advise should be given with regard to a
hypoglycemic agent in the postnatal period?
a. Continue all the medications for the first 24 hours after delivery and then resume
Metformin as pre prepregnancy with monitoring of blood sugar
b. Stop all medications and follow diet control with monitoring of blood sugar
c. Stop Insulin and advise Metformin as per prepregnancy with monitoring of blood sugar
d. Stop Metformin and continue Isophane Insuline twice daily until breast feeding has
stopped
e. Stop Metformin and continue Isophane Insuline at half the dose used during pregnancy
until breast feeding is stopped

41. A 28-year-old nulliparous patient attended for routine cervical screening which has shown
borderline changes and HPV inadequate results. She is not currrently sexually active and has had
normal smear results before this test. What is the next appropiate step in management?
a. Refer to colposcopy
b. Repeat cytology only in 3 months
c. Repeat cytology and HPV in 3 months
d. Repeat cytology and HPV in 6 months (CIN I atau borderline atau HPV inadekuat)
e. Repeat cytology and HPV in 12 months (CIN 1 dengan HPV +)

42. An 18-year-old nulliparous patient attends the gynaecology clinic for heavy painful menstrual
bleeding. She is in a sexual relationship with a new partner and both have tested negative for STIs,
though she has had a previously treated chlamydia infection. She does not want any children in
near future. She takes lamotrigine for epilepsy control and has not suffered any seizures for 3
years. General and pelvic examinations are normal, and her BMI is 30. What is the best appropiate
method of contraception in her situation?
Orang yang konsumsi Lamotigrin tidak boleh diberikan COC, Patch, ring karena dapat menurunkan
efek obat Lamotigrin
a. Combined oral contraceptive pill
b. Depot medroxyprogesterone acetate
c. Combined estrogen and progestin patch
d. Levonorgestrel-releasing intrauterine system
e. Etynilestradiol ring

43. A 28-year-old P2 woman comes to emergency departement with 6 hours of increasing left
lower quadrant pain, inability to tolerate orals, and nausea. She is sexually active and uses Depo-
Provera for contraception. She was treated for gonorrhea and reports compliance with treatment.
Her temperature is 38,5oC, pulse rate is 112/min, respirations are 22/min, and BP is 100/70 mmHg.
On examination, her abdomen is soft with leftlower quadrant tenderness. Voluntary guarding is
present present without rebound. Pelvic examination shows no cervical motion tenderness or
uterine tenderness. The left adnexa is exquisitely tender and fullnes is appreciated. Her WBC count
is 21.000 cells/µL. Urine HCG is negative. Which of the following is the most appropiate next step
in management?
a. Pelvic ultrasound
b. Abdominal X-Ray
c. Measurement of scrum β-hCG concentration
d. Dilation and curettage
e. Discharge home on oral antibiotics

44. A 30-year-old recently diagnosed with acquired immunodeficiency syndrome (AIDS). She has
not consume any medications yet. She presents with genital lesions that have been present for 1
week. She reports having had a single sore 4 months that was not painful and subsided without
treatment after approximately 2 weeks. Which of the following test specific to these lesions do you
expect to be positive? Sifilis
a. PCR
b. Wright-Giemsa stain
c. Herpes simplex antibody assay
d. Positive skin punch biopsy staining for Candida
e. VDRL-RPR

45. A 45-year-old P0A0 patient with prolonged menstruation and dysmenorrhea. From pelvic
examination found enlarged uterus, both adnexa within normal limit. Which study listed below
would best differentiate between adenomyosis and uterine fibroids?
a. Pelvic ultrasound
b. Pelvic MRI
c. Pelvic CY
d. Sonohysterogram
e. Hysterosalpingogram
46. A 39-year-old woman is reviewed in the gynecology clinic with symptoms of pelvic pain for
the last 2 years. Her ultrasound scan 2 months ago revealed a left-sided ovarian cyst (6 cm) with
diffuse low-level internal echoes with one thin internal septae. After performed laparoscopy,
lesions are biopsied that are thought to represent endometriosis. The diagnosis of endometriosis is
confirmed histologically by identifying extragenital implants containing which of the following ?
a. Endometrial glands and stroma
b. Hypertrophic smooth muscle
c. Hemorrhage and iron pigment deposits
d. Fibrosis
e. Stromal decidualization

47. A 27-year-old nulligravida presents with 6 months of amenorrhea and diagnosed with
hyperprolactinemia. Laboratory test measuring which of the following should also be obtained?
a. Total Testosterone
b. 24-hour urinary free cortisol
c. Insuline-like growth factor II
d. Thyroid Stimulating Hormon
e. Oxytocin

48. A 51-year-old woman presents to your office due to amenorrhea for the past year but she
denied had any hot flashes, she is wondering if she is menopausal. She asking you how to make
sure, what blood test would confim the diagnosis of menopause?
a. Progesteron
b. Testosterone
c. FSH
d. Human chorionic gonadotropin (hCG)
e. Prolactin

49. A 18-year-old girl present to your office with chief complain never had menstruation before.
Which of the following conditions is considered the most frequent cause of primary amenorrhea?
a. Asherman syndrome (Riwayat kuret, amenore sekunder)
b. Androgen insensitivity syndrome (ada payudara, bulu halus tipis-tipis)
c. Mayer-Rokitansky-Kuster-Hauser syndrome (tidak ada uterus, ada vagina, ovarium hypoplasia)
d. Turner syndrome
e. Congenital Adrenal Hyperplasia (seperti menopause dini)

50. A 23-year-old women present to your clinic with complain of LLQ pain in every menstruation
period since 6 months ago. She has not married. From physical examination found enlargement on
LLQ. Ultrasound examination found a 8 cm cystic mass on left adnexa with internal echo. She was
given dienogest for treatment. The treatment effecct of progestins on endometriosis is secondary
to which of the following?
a. Estrogen antagonism (primary efeknya CC, secondary effect : progestin)
b. Androgen antagonism
c. Aromatase inhibiton
d. Glucocorticoid stimulatin
e. Gonadrotophin agonist

51. A your patient is a 30-year-old G0 with a long history of irregular cycles, hirsutism, and an
ultrasound appearance consistent with polycystic ovarian syndrome (PCOS) who presents for
preconception counseling. She is a nonsmoker and is already taking prenatal vitamins. Her
husband has fathered a child with a previous partner. She understands the pathophysiology of
PCOS and her ovaries. You discuss the use of medication to help her conceive (Clomphifen Citratre)
What is mechanism of action of this medication ?
a. It is an antiestrogen that results in increased production of follicle stimulating hormone
(FSH) and luteinizing hormone (LH)
b. It decreases the conversion of androgens into estrogens which reduces the negative
feedback loop on the hypothalamus which increases FSH (mekanisme kerja Letrozole,
menghambat Androstenedion menjadi estron, dan Testosteron menjadi estradiol, sehingg
Estrogen dianggap dalam kadar rendah terus  FSH naik)
c. It inhibits gluconeogenesis and therefore allows ovary the ovary to respond to normal
gonadotropin signals (Mekanisme kerja Metformin)
d. It interferes with the pulsatile release of FSH and LH from the anterior pituitary (GnRH Agonist)

52. A 42-year-old woman with oligomenorrhoea and hirsutism presents to the gynaecology clinic.
She recently had prolonged episode of vaginal bleeding, but an ultrasound scan and endometrial
biopsy performed in primary care both reported normal results. She is obese with a BMI of 40
kg/m2. She has mild hypertension but does not require anthypertensive therapy. She has no other
medical problems. Her father suffered from type 2 diabetes mellitus. What further investigation is
required
?
a. LH FSH ratio
b. MRI scan of pelvis
c. Oral glucose tolerance test
d. Pregnancy test
e. Serum cholesterol

53. A 33 year old primigravida with asthma delivers a live infant at term. There was no
meconium. The infant makes no spontaneous attempt at breathing and is floppy. It is dried,
covered and assessed. Five inflation breaths are performed. There is good chest movement on
inflation. The neonatus is then reassessed, there is a heart rate of around 50 bpm although still no
breathing. Senior assisstance is summoned and en route. What is the next immediate step ?
a. Chest compressions and ventilation breaths in ratio 1:1
b. Chest compressions and ventilation breaths in ratio 3:1
c. Chest compressions and ventilation breaths in ratio 5:1
d. Chest compressions and ventilation breaths in ratio 15:1
e. Chest compressions and ventilation breaths in ratio 30:2

54. A 32 year old presents for an infertility workup. She and her partner have been trying to
conceive for 2 years without success. She has regular menstruation, though she mentions she has
severe cramping during her cycles. She also notes she experiences pelvic pain during sex. On
examination, she is a thin, well developed woman. She is afebrile and she experiences a great deal
of pain during the pelvic examination. You do not discharge on examination. Which of the
following tests is required for diagnosis of the patien’s infertility ? suspek endometriosis
a. Ultrasound
b. Β-hCG level
c. Pap smear
d. Laparascopy
e. Hysterosalpingogram

55. A 28 yo woman, GI 36 weeks of gestational age, went to your clinic to do routine antenatal
care. During ultrasound, the doctor told that she will be expecting baby boy with estimated fetal
weight 2500 g, however, amniotic fluid considered to be less than normal. Then you askes the
patient to drink minimal of 2L of water a day and get herself another ultrasound within 3 days to
evaluate the amniotic fluid. Amniotic fluid volume is a balance between production and resorption.
What is the primary mechanism of fluid resorption ?
a. Fetal breathing
b. Fetal swallowing
c. Absorption across fetal skin
d. Absorption by fetal kidneys
e. Filtration by fetal kidneys

56. A 33 year old G0P0 woman comes to your office for her initial prenatal visit. She tested
positive with two home pregnancy tests and has been experiencing breast tenderness and mild
nausea for a few weeks. She has a history of regular menstrual periods occuring every 28 to 30
days. This was plannned pregnancy and is the first child for her and for her partner. As her
pregnancy continues, you would expect her cardiac output to increase by which of the following
mechanisms
a. First an increase in stroke volume, then a increase in heart rate
b. A decrease in systemic vascular resistance
c. Cardiac output would not change significantly until the third trimester
d. An increase in systemic vascular resistance facilitated by elevated progesterone levels
e. Increased heart rate alone

57. A 29 year old G1P0A0 presents to her obstetricians office for a routine OB visit at 32 weeks
gestation. Her pregnancy has been complicated by gestational diabetes requiring insulin for
control. She has been noncompliant with diet and insulin therapy. She has has two prior normal
ultrasound at 20 and 28 weeks gestation. She has no other significant past medical or surgical
history. During
the visit, her fundal height measures 38 cm. Which of the following is the most likely explanation for
the discrepancy between the fundal height and the gestational age ?
a. Fetal hydrocephaly
b. Uterine fibroids
c. Polyhydramnions
d. Breech presentation
e. Undiagnosed twin gestation

58. Your patient complains of decreased fetal movement at term. You recommend a modified
BPP test. Nonstress testing (NST) in your office was reactive. The next part of the modified BPP is
which of the following ?
a. Contraction stress testing
b. Amniotic fluid index evaluation
c. Ultrasound assessment of fetal movement
d. Ultrasound assessment of fetal breathing movements
e. Ultrasound assessment of of fetal tone

59. While working in the emergency department, a 25 year old female patient arrives with
severe acute abdominal pain. Before the start of her abdominal pain, the patient recalls having
some fever and chils. She reports that her menses is regular and that she is sexually active. She
recently started having intercourse with a new partner. Pregnancy test is negative and urinalysis is
normal. On physical examination, the patient has muscular guarding and rebound tenderness. On
pelvic examination, patient has cervical motion tenderness. Vital signs are significant for
tachycardia and fever (T 40 derajat celcius) which of the following is the most likely diagnosis ?
a. Ovarian torsion
b. Endometriosis
c. PID
d. Kidney stone
e. Ruptured ovarian cyst

60. A 43 year old woman, G4P3 at 37 weeks gestation presented in hospital with a ten day
history of low estremities edema, with idiopathic hypertension for 1 year. At presentation, she has
a blood pressure of 170/100 mmHg. Laboratory findings were normal except urinalysis (protein 2+).
She was diagnosed with superimposed severe preeclampsia. It was decided to deliver the fetus by
means of C-section by indication transverse lie. Blood pressure measurement was 150/100 mmHg.
She lost consciousness for 30 seconds five hours after operation. The laboratory studies gave the
following results : serum aspartate aminotransferase (AST), 225 IU/L; serum alanine
aminotransferase (ALT), 140 IU/L; serum lactate dehydrogenase (LDH), 1017 IU/L; serum urea and
creatinine were normal; hemoglobin 10,6 mg/dL; platelet count 50x10 3 µ/mL, a brain computed
tomography (CT) scan was performed on patient which revealed the left frontal lobe lacunar
infarction. The patient was transferred to intensive care unit
What is the most appropriate diagnosis ?
a. DIC
b. Acute fatty liver in pregnancy
c. HELLP syndrome
d. Severe puerpural infection
e. Thromboticthrombositopenic purpura

61. A 24 year old G2P1 woman at 39 weeks and 3 days is seen in clinic. She has been
experiencing more frequent contractions and thins she might be in labor. Her last pregnancy
ended with a caesarean delivery after a stage 1 arresr. There was no evidence of cephalopelvic
disproportion. Earlier in the course of her current pregnancy she had desired a scheduled repeat
caesarean, but now that she might be in labor she would like to try and delovery vaginally. What
would be a contra indication to trial of labor after caesarean (TOLAC)?
a. Prior classical hysterotomy
b. Prior Kerr hysterotomy
c. Small for gestational age fetus
d. Oligohydramnion
e. GBS + mother

62. An 18 year old G1 sees you in your office and tells you she missed her last period and had a
positive home urine pregnancy test. You perform your normal first obstetrical examination and
obtain basic prenatal labs as well as a first trimester viability ultrasound. The radiologist calls you
to discuss the findings, which demonstrate a gestational sac with a fetus measuring approximately
8 weeks gestation. There is no fetal heartbeat demonstrated. The placenta is markedly thickened
and echogenic; more than would be expected in the first trimester. There are multiple areas of
small cystic spaces within the placenta as well. The most likely diagnosis and coressponding
karyotype is
a. Complete mole, 46 XX
b. Incomplete mole, 46 XY
c. Incomplete mole , 69 XXY
d. Complete mole, 69 XXX
e. Incomplete mole, 69 XYY

63. During routine surveilance of a twin pregnancy, twin A weighs 1200 g and twin B weighs 750
g. Hydramnion is noted around twin A, while twin B has oligohydramnions. Which statement
concerning findings in this twin pregnancy is true ?
a. The donor twin develops hydramnions more often than does the recipient twin
b. Gross differences may be observed between donor and recipient placentae
c. The donor twin usually suffers from a hemolytic anemia
d. The donor twin is more likely to develop widespread thromboses
e. The donor twin often develops polycycthemia

64. A 14 year old adolescent presents with a history of primary amenorrhea. Her past medical
history is unremarkable as is her family history. Her vital signs are as follow : P 90 beats / min,
respiratory rate 14 breaths/min, BP 124/76 mmHg and BMI 21. General physical examination
reveals no evidence of acne. She has no axillary or pubic hair. External genitalia are of normal
female. Breast are tanner stage 4. She is not sexually active so a three dimensional 3D pelvic
ultrasound was performed revealing no evidence of a uterus. What is next step in evaluation ?
Adrenal insufisiency syndrome
a. Chromosomal analysis
b. Estrogen level
c. Follicle stimulating hormone (FSH) level
d. Speculum and bimanual examination
e. Testosterone level

65. A 28 year old woman presents to the emergency department with hemoptysis. She reports
that she has had increasing cough and shortness of breath over the past 8 weeks and that she
coughed up a dime-sized blood clot this morning. On review of systems, the patient endorses
heavy and irregular vaginal bleeding, she says that she had a spontaneous abortion 6 months ago
and that she started having increasingly irregular and heavy periods about 4 months ago. On
examination, her uterus is enlarged to 12 week size. Serum β-hCG is elevated, hemoglobin is 10
mg/dL and chest X-Ray reveals two dense areas in her lungs, one in the right upper lobe and one in
the left lower lobe. Which of the following is the most likely diagnosis ?
a. Missed abortion
b. Incomplete abortion
c. Choriocarsinoma
d. Molar pregnancy
e. Ectopic pregnancy

66. A young couple comes in with a chief complaint of infertility. The patient is a 30 year old GO
who has not undergone any evaluation. Her husband is a 33 year old who has had a semen
analysis, which was reported as normal. He has never fathered a child. The couple reports having
unprotected intercourse for the past 14 months. On further history, the patient reports that her
periods have been quiet irregular over the last year and that she has not had period in the last 3
months. She also reports hot flashes, vaginal dryness and decreased libido. The most likely
diagnosis for this patient based on her history is
a. Polycystic ovarian syndrome
b. Primary ovarian insufficiency
c. Endometriosis
d. Kalmann syndrome
e. Spontaneous pregnancy

67. A 35 year old G2P1001 woman at 40 weeks 6 days presents to labor and delivery triage with
a 5 hour history of painful contractions. Monitoring reveals contractions every 3 minutes and
cervical examination on arrival is 3 cm dilated, 50% effaced and-2 station. Her pregnancy has been
complicated by A1 gestational diabetes and 12 kg weight gain in pregnancy (BMI 24). In her
pregnancy, she presented at 2 cm dilation and 90% effacement. Her labor progressed slowly with
dilation of 1 cm every 3 hours until labor arrested at 7 cm. A healty baby girl weighing 4.200 g was
delivered by caesarean section after arrest at 7 cm for 4 hours. Today your patient requests a trial
of labor after cesarean section. You recheck her cervix 2 hours later and find that it is 4 cm dilated,
90% effaced, and -1 station. The estimated fetal weight by Leopold’s is 4.5 kg. A recent ultrasound
performed at 38 weeks estimated the fetal weight at 3900 g. You admit the patient to labor and
delivery for expectant management. Which of the following factors was least likely to result in her
prior failure to progress?
a. Obstetric conjugate diameter of the pelvis greater than 11.5 cm
b. Inadequate strength of uterine contractions
c. Fetal size or position
d. Maternal pelvis shape
e. Spina ischiadica less than 11 cm

68. You are providing prenatal care to a 22 year old G1P0 women at 16 weeks GA by LMP. She
has had a reletively smooth pregnancy without complications thus far. She has an obese BMI,
otherwise without medical or surgical history. She presented to prenatal care at 14 1/2 weeks, and
so missed first trimester screening. She undergoes the quad screen and has an elevated level of
maternal serum alpha fetoprotein (MSAFP). The ultrasound reveals a myelomeningocele. The
increased incidence of this finding is associated with which of the following medications when used
in pregnancy?
a. Valproic acid
b. Lithium
c. Fluoxetin
d. Prednisone
e. Acetaminophen

69. A 45 year old presents for evaluation because her primary care physician has diagnosed her
with pelvic organ prolapse while performing annual care. She denies any pelvic pressure, bulge, or
difficulity with urination. Her only medical comorbidity is obesity. For symptomatic grade 1 pelvic
organ prolapse, what do you recommended?
a. Conservative management with pelvic floor muscle exercises and weight loss
b. Colpocleisis obliterative procedure (usia tua tidak mau berhubungan lagi, tapi tidak mau
histerektomi)
c. Gelhorn pessary (pesarium)
d. Round ligament suspension (sacrospinosum histeroraphy untuk menggantung cervix, SSF :
Sacro Spinosun ligamentum Fixation dilakukan kalo pasien tidak mau di TVH)
e. Hysterectomy

70. A woman was found to be 10 weeks pregnant when she attended for her colposcopy
appointment for high grade dyskaryosis. The colposcopy examination suspected C1N 1. What will
be your futher management?
a. Repeat colposcopy at 20 weeks gestation
b. Repeat colposcopy at 6 weeks postpartum
c. Repeat colposcopy at 12 weeks postpartum
d. Repeat colposcopy at 16 weeks postpartum
e. Repeat colposcopy at term

71. A 36 year old woman has had a LETZ procedure for cervical glandular intraepithelial neoplasia
(CGIN). The histopathology result showed negative margins. Six months later the repeat cervical
smear is negative but the HPV test of cure is positive. What will be your next management?
a. Cervical biopsy
b. Colposcopy
c. Re excision
d. Smear and HPV test of cure in six months
e. Smear and HPV test of cure in 12 months

72. A 43 year old woman has had her cervical smear taken on day 14 of her menstrual period.
The result shows no abnormality but there were normal endometrial cells in the sample. She has a
history of bilateral tubal ligation. What will be your further management?
a. Cervical biopsy
b. Reassure the woman
c. Transvaginal scan and endometrial sampling
d. Repeat smear in six months
e. Urgent referral for colposcopy

73. A 28 year old women present to the antenatal clinic at 14 weeks pregnant with mild lower
abdominal pain and frequency in micturition. An ultrasound scan notes a solid adnexal mass. Her
serum lactate dehydrogenase (LDH) and human chorionic gonadotropin (hCG) levels are raised.
Which of the following is the most likely cause of her symptoms?
a. Dysgerminoma
b. Endodermal sinus tumor
c. Germ cell tumor
d. Immature teratoma
e. Mature teratoma

74. A 30-year-old G0 comes in for her annual examination and tells you that she plans to become
pregnant sometime in the next year. She had a LEEP procedure 7 years ago at another facility for
moderate dysplasia, CIN II. She states her Pap testing has been negative since the LEEP and her last
pap was 3 years ago, but you do not have any records documenting these results.
What are the complications of surgical excisional procedures of the cervix?
a. Cervical stenosis, cervical insufficiency, infection, bleeding
b. Cervical stenosis, infertility, infection, bleeding
c. Cervical insufficiency, cervical polyp infection and amenorrhea
d. Vaginitis and bleeding
e. There are no knows complications

75. A baby presents with ambiguous genitalia. A full chromosome count is sent and will return in
72 hours. Your laboratory can perform a test for Barr body so you can provide a preliminary
answer sooner. What is the Barr body?
a. The condensed, non functioning X chromosome
b. The darkest, widest band found on chromosome
c. An extra lobe on the female polymorphonuclear leukocytes
d. Found only in the female
e. The largest chromosome in the female genotype

76. A 22 year old G3P2002 who had a hematocrit of 36% at her initial obstetrical examination at
12 weeks is found to have hematocrit of 30% at 28 weeks when checked along with her 1 hour
glucosa. Based on the indices of the red blood cells on the CBC, you diagnose iron deficiency. She
asks why that occured since she has been taking her prenatal vitamins. As part of the explanation,
you note that which of the following maternal measurements or findings is first decreased by the
iron requirements of pregnancy?
a. Bone marrow iron
b. Hemoglobin
c. Jejunal absorption iron
d. Red cell size
e. Serum iron binding capacity

77. A patient returns for a postoperative checkup 2 weeks after a total abdominal hysterectomy
for fibroids. She is distressed because she is having continuous leakage of urine from the vagina.
Her leakage is essentially continuous and worsens with coughing, laughing, or movement. Given
her history and physical, you perform both a methylene blue dye test, which is negative and an
indigo carmine test, which is positive. The most likely diagnosis is :
a. Rectovaginal fistula
b. Uretrovaginal fistula
c. Vesicovaginal fistula
d. Ureterovaginal fistula
e. Imppossible to distinguish

78. Mrs. XY is a primigravida who is 34 weeks pregnant. Her last two serial scans have shown a
small gestational age fetus growing on the 9th centile. Her last scan shows positive end diastolic
flow with a normal PI. She reports having good fetal movements. How should further fetal
surveillance be undertaken?
a. Fortnightly umbilical artery Doppler
b. Twice weekly CTG
c. Twice weekly umbilical artery Doppler
d. Weekly CTG
e. Weekly umbilical artery Doppler+CTG

79. A 19 year old college student presents with amenorrhea for the past 6 months. Her menses
began at age 12 and were regular until then. She denies sexual activity, and a pregnancy test is
negative. Her physical examination is unremarkable except for the milky white discharge she
expresses from her nipples. What is the next step?
a. Transvaginal ultrasound
b. Measure serum prolactin
c. Order brain MRI
d. Get thyroid ultrasound
e. Prescribe bromocriptine

80. A 36 year old G2P2 woman presents to her gynecologist with a 3 week history of vaginal
irritation and fish smelling vaginal discharge. She recently tried an over the counter antifungal
treatment without any improvement in her symptom. She is sexually active in a monogamus
relationship with a male partner of 5 years and she uses a contraceptive ring (NuvaRing).
Genitourinary examination shows a thick white discharge. The remainder of her examination is
normal. Microscopic evaluation of saline “wet prep” of the vaginal secretions reveals decreased
lactobacilli, a few WBCs, and a vaginal epithelial cells with a stippled appearance. Risk factors for
developing this condition include all of the following except :
a. Multiple sexual partners
b. Cigarette smoking
c. Douching
d. Contraceptive ring (NuvaRing)
e. Lack of vaginal lactobacilli

81. It is now possible to administer GnRH in either a brief pulse or continuously. This allows
diagnostic and therapeutic interventions in the hypothalamic pitutary axis. To anticipate a normal
response to GnRH stimulus one must understand how GnRH controls LH and FSH release. Which of
the following is true concerning GnRH-stimulated LH secretion?
a. Associated with steady LH release
b. Enhanced by gonadotrope exposure to continuous GnRH
c. Enhanced by gonadotrope exposure to estrogen
d. Enhanced by gonadotrope exposure to progesterone
e. Enhanced by gonadotrope exposure to testosterone

82. A 46 year old P2 obese woman is referred from her primary care physician because of
increasingly heavy and painful mensesover the last 18 months. She has tried an oral contraceptive
with some improvement of her bleeding but no improvement in her pain. She reports no other
history of pelvic pain or abnormal bleeding in the past. She has never had an abnormal Pap smear
and states she has never had any infections, “down there.” Her only medical problem are her
obesity, hypertensionand gastroesophageal reflux disease. On examination, you note normal
external genitalia, vagina and cervix. However, her uterus is slightly enlarged, mildly tender, and
softer than you expected. She has no adnexal mass or tenderness. Which of these diagnoses is the
least likely choice to keep in your differential?
a. Leiomyoma
b. Adenomyosis
c. Irritable bowel syndrome
d. Endometrial hyperplasia
e. Endometriosis

83. A 38 year old African American woman presents with heavy menses and an enlarged uterus.
After an examination the clinical diagnosis is leiomyoma of the uterus. Which of the following best
describes this finding?
a. A soft, interdigitating mass of the uterine wall
b. A premalignant papule of the uterine wall
c. A rapidly dividing necrotic malignancy
d. A rounded, smooth, firm, well circumscribed mass
e. Erythematous, tender, and hereditary

84. If the fetal crown rump length is disparate in twins at the 12 weeks scan, select the best
method of dating the pregnancy. Gestation age can be allotted according to:
a. Average CRL of the two foetuses
b. CRL of the bigger fetus
c. CRL of the smaller fetus
d. CRL of the smaller fetus added to half the CRL of the bigger fetus
e. Dating is best done by LMP in such cases

85. A 30 year old G1P0A0 with twin gestation at 28 weeks is being evaluated for vaginal bleeding
and uterine contractions. A bedside ultrasound examination rules out the presence of a placenta
previa. Fetal heart rate tracing is reactive on both twins, and the uterine contractions are every 2
to 3 minutes and last 60 seconds. A sterile speculum examination is negative for rupture
membranes. A digital examination indicates that the cervix is 2 to 3 cm dilated and 50% effaced,
and the presenting part is at -3 station. Tocolysis with magnesium sulfate is initiated and
intravenous antibiotics are started for group B streptococcus prophylaxis. Betamethasone, a
corticosteroid is also administered. Which of the following statements regarding the use of
betamethasone in the treatment of preterm labor is true?
a. Betamethasone enhances the tocolytic effect of magnesium sulfate and decreases the
risk of preterm delivery
b. Betamethasone has been shown to decrease intramniotic infections
c. Betamethasone promotes fetal lung maturity and decreases the risk of respiratory
distress syndrome
d. The anti inflammatory effect of betamethasone decreases the risk of GBS sepsis in the
newborn
e. Betamethasone is the only corticosteroid proven to cross the placenta

86. A 49 year old woman experiences irreguler vaginal bleeding for 3 months duration. You
performed endometrial biopsy, which obtains copious tissue with a velvety, lobulated texture. The
pathologist reort shows proliferation of glandular and stromal elements with dilated endometrial
glands, consistent with simple hyperplasia. Cytologic atypia is absent. Which of the following is the
best way to advise patient?
a. She should be treated to estrogen and progestine hormone therapy
b. The tissue will progress to cancer in approximately 10% cases
c. The tissue maybe weakly premalignant and progress to cancer in approximately 1% of
cases
d. She requires a hysterectomy
e. No further therapy is needed

87. A 27 year old woman presents to her gynecologist for evaluation of new vulvar lesion. She
first noticed several ‘bumps’ on her outer labia 3 weeks ago. She is othewise without any
associated symptoms. On pelvic examination there are six flesh colored, non tender, 1- to 3- mm
verrucous papules. Which of the following is the most likely cause of the patient’s vulvar lesions?
a. Human papillomavirus (HPV) serotype 16
b. Human simplex virus (HSV) serotype 1
c. Pox virus
d. Human papillomavirus (HPV) serotype 6
e. Haemophilusducreyi

88. A 7 year old girl presents to her pediatrician with her parents who are concerned about her
early sexual development. She is developing breasts, axillary hair, and pubic hair, and they are
noticing body odor. A thorough clinical workup reveals the child has an irreguler echogenic, thickly
septated ovarian mass on her left ovary. What type of tumor is responsible for this child’s clinical
presentation?
a. Dysgerminoma
b. Embryonal carcinoma
c. Sertoli Leydig cell tumor
d. Endodermal sinus tumor
e. Granulosa theca cell tumor

89. A 36 year old, P2A0 presents to her primary care physician complaining of several months’
poor sleep, heat intolerance, newly irreguler menses, and decreased libido. She denies recent
changes in weight, diet or activity, and does not use alcohol or illicit drugs. Her obstetric history
includes two uncomplicated, full term vaginal deliveries and one first trimester spontaneous
abortion. On examination, she is thin and tired appearing without exophtalmos or thyromegaly.
Vital signs show the following t 36.8oC, respiratory rate of 17 breaths/min, P 77 beats/min, and BP
142/88
mmHg. Her breasts are non tender, without masses or nipple discharge. Pelvic examination reveals
moderately well estrogenized vaginal mucosa and a small, non tender uterus with reguler contours
and no palpable adnexa masses. A urinary pregnancy test is negative. What laboratory values are
appropriate to check at this time?
a. Follicle stimulating hormone (FSH)/ luteinizing hormone (LH), prolactin, thyroid
stimulating hormone (TSH), complete blood count (CBC), basic metabolic panel
(BMP)
b. CBC, BMP, serum ferritin/total iron binding capacity (TIBC)
c. Endometrial biopsy and CT abdomen/pelvis
d. Adrenocorticotropic hormone (ACTH), dehydroepiandrosterone (DHEA), 24
hour cortisol, and memantine challenge
e. β-hCG serum

90. A 34 year old G3P2 woman at 38 weeks and 6 days was admitted to labor and delivery unit
for active management of labor after it was determined that her membranes had ruptured and she
wa dilated to 3 cm. Her cervix has been steadily dilating and now she is at 6 cm. She is very
uncomfortable and finds her contractions very painful. Her partner is also very concerned that she
needs pain relief. With adequate pain control she dilates 10 cm and second state begins. Which of
the following is the correct order of the cardinal movements of labor?
a. Internal rotation, engagement, descent, flexion, external rotation
b. Engagement, descent, internal rotation, flexion, external rotation
c. Internal rotation, descent, engagement, flexion, external rotation
d. Engagement, descent, flexion, internal rotation, external rotation
e. Engagement, descent, internal rotation, flexion, external rotation

91. A 22 year old G1P0 woman at 36 weeks by LMP consistent with 12 weeks ultrasound with
limited prenatal care presents via ambulance to the L&D triage unit complaining of severe
abdominal pain and profuse vaginal bleeding. The patient is unstable and unable to communicate
coherently. The EMT reports that initially her BP was 180/100 mmHg and pulse rate was 110 bpm,
but she has lost at least 500 ml blood in route. On examination her BP is 90/50 mmHg, pulse rate is
120 bpm, she appears to be in significant pain, is unable to answer questions, and her abdomen
feels rigid. What is the most important laboratory test to order for this patient emergently?
a. Blood type and cross
b. CBC
c. Urine drug screen
d. Chemistry
e. PT, PTT, INR

92. A 39 year old woman undergoes vaginal hysterectomy and anteroposterior repair for uterine
prolapse. Which of the following is a complication of this procedure that often develops within 2
weeks of surgery?
a. Dyspareunia
b. Non fistulous fecal incontinence
c. Enterocele
d. Vaginal voult prolapse
e. Stress urinary incontinence

93. A 62 yo woman present for annual examination. Her last spontaneous menstrual period war
9 year ago, and she has been reluctant to use postmenopausal hormone replacement therapy
because if strong family history of breast cancer. She now complain of diminished interest in sexual
activity. Which of the following of the most likely cause her complaint?
a. Decreased vaginal length
b. Alienation from her partner
c. Untreatable sexual dysfunction
d. Decreasen ovarian function
e. Physiology anorgasmia

94. A 39 year old patient presents with symptoms of leakage of urine upon coughing, sneezing,
and during exercise. The symptoms started following the birth of her second child 18 months ago.
What would be the first line of management?
a. Biofeedback/electrical stimulation
b. Bladder retraining
c. Insertion of a mid urethral retropubic tape
d. Pharmacotherapy with duloxetine
e. Supervised pelvic floor muscle training

95. You were asked to perform a total abdominal hysterectomy because of a large right broad
ligament fibroid. How best should you avoid ureteric injury?
a. Ask a urologic surgeon to assist you to help prevent injury
b. Dissection and visualization of the ureter
c. Intraoperative insertion of ureteric catheter to identify the ureter
d. Intraoperative insertion of illuminating ureteric catheters
e. Preoperative intravenous urography to identify the course of the ureter

96. Which of the following vulval skin disorders is associated with the higher risk of developing
malignant disease?
a. Contact irritant dermatitis
b. Lichen planus
c. Lichen sclerosus
d. Squamous cell hyperplasia
e. Psoriasis

97. A woman is found to have an unilateral invasive vulvar carcinoma that is 2 cm in diameter
but not associated with evidence of lymph node spread. Initial management should consist of
a. Chemotherapy
b. Radiation therapy
c. Simple vulvectomy
d. Radical vulvectomy
e. Radical vulvectomy and bilateral lymphadenectomy

98. An 18-year-old girl present with a large abdominal mass with abdominal pain. She claims the
mass has increased in size within the last three months. A laparotomy and unilateral salpingo-
oophorectomy is performed. The histology report shows a mesodermal core with acentral capillary
(Schiller-Duval body). Which ovarian tumor is the most likely to be ?
a. Choriocarcinoma of the ovary (B-HCG)
b. Dysgerminoma (LDH)
c. Embryonal carcinoma (testis, AFP meningkat)
d. Endodermal sinus tumor / yolk sac tumor , B-HCG
UNAS NOVEMBER 2017 (SURABAYA)

1. A 32 -year-old presents for an infertility workup. She and her partner have been trying to
conceive for 2 years without success. She has regular menstruation, though she mentions
she has severe cramping during her cycles. She also notes she experiences pelvic pain during
sex. On examination, she is a thin, well-developed woman. She is afebrile, and she
experiences a great deal of pain during the pelvic examination. You do not note discharge
on examination. Which of the following tests is required for diagnosis of the patient's
infertility?
(A) Ultrasound
(B) ß-hCG level
(C) Pap smear
(D) Laparoscopy
(E) Hysterosalpingogram

2. A 45 -year-old patient post-op day 6 from total abdominal hysterectomy presents to the
emergency room (ER) complaining of serous fluid that is oozing from her incision. The
patient's BMI is 33 . She has diabetes and required an open laparotomy for her surgery. On
her examination, the skin incision is separated, and a Q-tip can be placed past the fascial
layer. What is the most likely cause of this patient's condition?
(A) Fascial dehiscence
(B) Hematoma formation
(C) Normal healing
(D) Infection development
(E) Seroma formation

3. A 32-year-old P0A0 presents to the clinic with worsening pelvic pain for the last several
years. Physical examination is grossly benign with the exception of some mild diffuse lower
abdomen tenderness on deep palpation. Which of the following is the most common
gynecologic diagnosis associated with chronic pelvic pain (CPP)?
(A) Pelvic inflammatory disease (PID)
(B) Adhesions
(C) Gynecologic malignancy
(D) Dysmenorrhea
(E) Endometriosis

4. A 70-year-old woman presents with complaints of leaking of urine with a cough, sneeze, or
while walking daily. It is significantly impacting her life. Medical history is complicated by
hypertension controlled on hydrochlorothiazide. Vital signs are BP 149/90 mmHg and pulse
87 beats/min. Physical examination is remarkable for loss ofher mid-urethral angle and
minimal cystocele. There is no uterine prolapse or rectocele. The patient undergoes
urodynamic testing confirming your suspected diagnosis of her urinary incontinence. What
is the next best step in therapy for this patient?
(A) Oxybutynin
(B) Mid-urethral sling
(C) Trimethoprim/sulfamethoxazole
(D) Tolterodine
(E) Anterior colporraphy

5. A 49-year-old P5A0 woman comes to see you because of intermittent pelvic pain and
constipation. The patient saw her primary carephysician 1 month ago forthe same
complaints. A colonoscopy performed at that time showed no abnormalities. Today, the
patient also confides that she has experienced increasing postcoital spotting over the past
year. The patient is afebrile with a BP of 125/85 mmHg and an HR of 90 beats/min. Cardiac,
pulmonary, and abdominal examinations are unremarkable. Bimanual pelvic examination
reveals a nontender, 15 –weeksized uterus with several dense cobblestone-like protrusions.
Pelvic ultrasound shows at least seven 2 to 5 cm areas of hypogenicity. What is the next best
step in the care of this patient?
(A) Saline infusion sonogram
(B) Endometrial biopsy
(C) Medroxyprogesterone
(D) Hysterectomy
(E) MRI-guided thermoablation

6. A 58-year-old postmenopausal woman is seen in clinic for a routine examination. At the end
of the visit, she sheepishly admits to mild urinary incontinence over the past several years .
She is P4A0; all her children were normal spontaneous vaginal deliveries, and the largest
weighed 4000 g. She has hypertension treated with metoprolol and has been using a topical
estrogen cream for vaginal dryness since menopause at age 54. Further questioning reveals
that she occasionally leaks small amounts of urine, particularly with laughing, sneezing, or
coughing. She denies large volume loss, increased urinary frequency, or nocturia. Based
upon this history, which of the following is a good initial treatment option for this woman's
urinary incontinence?
(A) Pelvic floor strengthening exercise
(B) Urethral bulking injections
(C) Imipramine
(D) Discontinue topical estrogen cream
(E) Urethral sling procedure

7. A 17-year-old girl complains of painful periods. She says that she began menstruating at 14
years of age and has had significant pain and cramping associated with menstruation since
then. Her pain is worst on the first and second days of menstruation and is often associated
with nausea, vomiting, and headache. Her pain is so severe that she often has to miss school
on the first day of her period. She has tried heating pads and baths, which provide some
minor relief. Physical examination reveals no abnormalities. Which of the following
interventions is considered first-line therapy for this patient's condition?
(A) Acetaminophen
(B) Nonsteroidal anti-infl ammatory drugs (NSAIDs)
(C) Opiate analgesics
(D) Oral contraceptive pills (O CPs)
(E) Gonadotropin antagonists

8. A 24-year-old P0A1 presents for a preconception consultation. She wishes to conceive


inthe next year. Three years ago, she has a termination of pregnancy at 10 weeks via
suction curettage. Until 18 months ago, she used depot medroxyprogesterone acetate
(DMPA) but has used no contraception since. She resumed having very regular menses 5
months after her last DMPA injection, but they are very scant and associated with severe
cramping. She regularly checks an ovulation predictor kit that reveals a luteinizing
hormone (LH) surge consistently on day 16 of her cycle. She has remained sexually active.
Her general medical examination is unremarkable. Her thyroid is of normal size. Gender
specific examination reveals normal internal and external genitalia. Uterus is small and is at
mid-position. Ovaries are palpable bilaterally about 2 cm each. Pregnancy test is negative.
Which of the following will be the most beneficial in evaluating this patient's differential
diagnosis?
(A) Diagnostic laparoscopy
(B) Hysteroscopy
(C) Progesterone challenge
(D) Thyroid function testing
(E) Ultrasound

9. A 19-year- old college student presents with amenorrhea for the past 6 months. Her menses
began at age 1 2 and were regular until then. She denies sexual activity, and a pregnancy
test is negative. Her physical examination is unremarkable except for the milky white
discharge she expresses from her nipples. What is the next step?
(A) Transvaginal ultrasound
(B) Measure serum prolactin
(C) Order brain MRI
(D) Get thyroid ultrasound
(E) Prescribe bromocriptine

10. The following is true regarding chemotherapy for breast cancer during pregnancy?
(A) It is generally only recommended after delivery
(B) It must exclude all Taxanes
(C) It is contraindicated only below 14 weeks of gestation
(D) It must be given with an increased dose because of changes to maternal
pharmacokinetics
(E) Altered maternal pharmacodynamics reduce the drug effect for many
chemotherapeutic agents

11. The following statement is true regarding the management of CIN in pregnancy:
(A) Colposcopic directed biopsies are mandatory in the management of CIN
in pregnancy
(B) Punch biopsies are adequate to safely confirm or exclude invasive disease
(C) Standard practice in the UK involves deferring definitive treatment until
3- 4 months postpartum if invasive cancer has been excluded
(D) Colposcopy should be repeated at 3-monthly intervals during pregnancyif CIN
2/3 is suspected
(E) Haemorrhage is more likely to occur if biopsies are taken in pregnancy

12. Regarding fetal asphyxia which of the following is true?


a) Fetal asphyxia almost always occurs as a result of a gradual insufficiency in the umbilical blood
flow or insufficient uterine blood flow
b) Occlusion of one or more of the vessels in the umbilical cord impedes the circulation to
and from the fetus and during these events both the oxygen content of the fetal blood
may decrease and the CO2 content may increase
c) Excess fetal CO2 is initially removed by a large increase in fetal respiratory rate
d) Prolonged hypoxia leads to a further increase in CO2 content and further respiratory
acidosis
e) The accumulation of CO2 can deplete the buffer system causing failure of the ATP-
dependent sodiume potassium pump initiating a cascade that leads to cell injury and
death

13. Regarding the causes of intrapartum fetal hypoxia/acidosis, the following statement is true?
a) Uterine contractions may decrease placental perfusion and reduce umbilicalcord circulation.
b) Aorto-caval compression by the pregnant uterus may cause sudden maternalhypotension.
c) Maternal cardio-respiratory arrest is an irreversible cause of fetal hypoxia/acidosis.
d) Shoulder dystocia may cause umbilical cord compression.
e) Asking the mother not to push during contractions in the second stage of
labour may contribute to reducing fetal hypoxia/acidosis.

14. A 38-year-old, P2A0, presents with her new husband and is interested in trying to conceive.
She is in good health, has no history of sexually transmitted infections (STis) , and does not
smoke. Her husband has fathered a child in the past. What single test will assess her
chances of getting pregnant?
(A) Thyroid- stimulating hormone (TSH)
(B) Prolactin
(C) Hemoglobin
(D) Follicle-stimulating hormone (FSH)
(E) Progesterone level

15. A 36-year-old, P2A0, presents to her primary care physician complaining of several months'
poor sleep, heat intolerance, newly irregular menses, and decreased libido. She denies
recent changes in weight, diet, or activity, and does not use alcohol or illicit drugs. Her
obstetric history includes two uncomplicated, full-term vaginal deliveries and one
firsttrimester spontaneous abortion. On examination, she is thin and tired appearing,
without exophthalmos or thyromegaly. Vital signs show the following: T 36.8°C, respiratory
rate of 17 breaths/min, P 77 beats/min, and BP 142/88 mmHg. Her breasts are nontender,
without masses or nipple discharge. Pelvic examination reveals moderately well-
estrogenized vaginal mucosa and a small, nontender uterus with regular contours and no
palpable adnexal masses. A urinary pregnancy test is negative. What laboratory values are
appropriate to check at this time?
(A) Follicle-stimulating hormone (FSH)/ luteinizing hormone (LH), prolactin, Thyroid- stimulating
hormone (TSH), complete blood count (CBC), basic metabolic panel (BMP)
(B) CBC, BMP, serum ferritin/total iron-binding capacity (TIBC)
(C) Endometrial biopsy and CT abdomen/pelvis
(D) Adrenocorticotropic hormone (ACTH), dehydroepiandrosterone (DHEA) , 24-hour
cortisol, and memantine challenge
(E) ß-hCG serum

16. A 14-year-old adolescent presents with a history of primary amenorrhea. Her past medical
history is unremarkable as is her family history. Her vital signs are as follow: P 90 beats/min,
respiratory rate 14 breaths/min, BP 124/76 mmHg, and BMI 21. General physical
examination reveals no evidence of acne. She has no axillary or pubic hair. External genitalia
are of normal female. Breasts are Tanner stage 4. She is not sexually active so a three-
dimensional (3D) pelvic ultrasound was performed revealing no evidence of a uterus. What
is the next step in evaluation?
(A) Chromosomal analysis
(B) Estrogen level
(C) Follicle- stimulating hormone (FSH) level
(D) Speculum and bimanual examination
(E) Testosterone level

17. An otherwise healthy 24-year-old woman presents to her gynecologist because of irregular,
heavy vaginal bleeding for the past few days. She also complains of worsening nausea,
vomiting, headache, and dizziness over the past few weeks. She reports that her last regular
menstrual period before her current bleeding started was 6 weeks ago. On examination, she
has a slight tremor in both hands, an enlarged 10-weeksized uterus, and blood coming from
the cervical os. Urine ß-hCG is positive. Vital signs are temperature 36.8ᵒC, heart rate 100
beats/min, blood pressure 160/100 mmHg, and respiratory rate 16 breaths/min. What is the
most likely diagnosis?
(A) Missed abortion
(B) Incomplete abortion
(C) Gestational hypertension
(D) Molar pregnancy
(E) Ectopic pregnancy
18. A 54-year- old P2A1 presents to your office for the first time with a chief complaint of
"feminine itching:' She has not had a gynecologic examination since the birth of her last
child 25 years ago and underwent menopause 3 years ago. She says that she has had
significant pruritus "for years;' occasionally accompanied by cracking and bleeding of the
affected area. She is clearly nervous, saying she has not sought treatment for this issue
sooner because she is afraid that something is "really wrong:' On examination, a single 3 -
cm lobular mass is noted on the right labia majora. Careful examination also reveals an
enlarged, painless right inguinal lymph node. The patient denies any recent weight loss,
pain, or changes in bowel or bladder habits. Surgical staging confirms that this mass is
squamous cell carcinoma with unilateral nodal involvement. What is the recommended
course of treatment?
(A) Wide radical local excision with ipsilateral inguinal lymph node dissection
(B) Modified radical vulvectomy with bilateral inguinal lymph nodedissection
(C) Radical vulvectomy, bilateral inguinal lymph node dissection, and pelvic exenteration
(D) Modified radical vulvectomy, bilateral inguinal lymph node dissection, and pelvic
exenteration
Modified radical vulvectomy, bilateral inguinal lymph node dissection, and pelvic
exenteration

19. A 28 -year-old woman presents to the emergency department with hemoptysis. She reports
that she has had increasing cough and shortness of breath over the past 8 weeks and that
she coughed up a dime- sized blood clot this morning. On review of systems, the patient
endorses heavy and irregular vaginal bleeding. She says that she had a spontaneous
abortion 6 months ago and that she started having increasingly irregular and heavy periods
about 4 months ago. On examination, her uterus is enlarged to 12-week size. Serum ß-hCG is
elevated, hemoglobin is 10 mg/dL, and chest X-ray reveals two dense areas in her lungs, one
in the right upper lobe and one in the left lower lobe. Which of the following is the most
likely diagnosis?
(A) Missed abortion
(B) Incomplete abortion
(C) Choriocarcinoma
(D) Molar pregnancy
(E) Ectopic pregnancy

20. A 24-year-old woman presents to clinic with an atypical squamous cells of undetermined
significance (ASCUS) Pap. She had a Pap about 2 to 3 years ago, and it was normal. She
became sexually active at age 17 and has had two lifetime partners. She and her male
partner have been together for over a year. She started having periods at age 13 and has
regular cycles on her own. Her last period was 1 week ago on oral contraceptives. She has
a history of herpes simplex virus well controlled with daily acyclovir. Her physical
examination is normal. What is the next step?
(A) Repeat the Pap
(B) Get high-risk human papillomavirus (HPV) typing
(C) Get low-risk HPV typing
(D) Perform colposcopy
(E) Perform a conization

21. A 67-year-old woman presents with abdominal discomfort and bloating, 10 kgs weight loss,
decreased appetite, and fatigue. Vital signs are stable. Physical examination demonstrates a
menopausal woman witha large left adnexal mass detected on bimanual examination. You
perform a transvaginal ultrasound that shows an 8 - cm complex mass with solid and cystic
components, thick irregular septations, and an irregular surface. Based on your clinical
assumptions, what is the most likely course of management for this patient?
(A) Chemotherapy
(B) Abdominal exploration with surgical resection
(C) Neoadjuvant chemotherapy, abdominal exploration, and surgical resection
(D) Abdominal exploration with surgical resection followed by chemotherapy
(E) Abdominal exploration with surgical resection followed by radiation

22. A 26-year-old P2A0 presents for her annual gynecologic examination and would like to discuss
her risk of ovarian cancer. Menarche occurred at age 14. She used oralcontraceptive pills
(OCPs) for 3 years prior to the birth of her first child when she was 23. She breastfed both of
her children for 1 year each. Her mother is 46 and currently is undergoing chemotherapy for
ovarian cancer, and she had a maternal aunt who passed away in her 50 from breast cancer.
What is this woman's greatest potential risk factor for developing ovarian cancer?
(A) BRCA (breast cancer gene) mutation
(B) Hereditary nonpolyposis colorectal cancer (HNPCC) mutation
(C) Family history
(D) Early menarche
(E) Late childbearing

23. A 27-year-old P3A1 returns to your clinic to follow up on the results of her annual Pap
smear. She has had a full annual examination including Pap smear since the age of 21 and
has never had abnormal cervical cytology. Her STD screening at her last examination was
normal. The cytology of her Pap smear showed high-grade squamous intraepithelial lesion
(HGSIL). You discuss the management options to the patient, and she decides to have a
colposcopy. The biopsy results of the colposcopy you perform are insufficient to give a
histology report. Which of the following options is the next best step in management for
this patient?
(A) Repeat Pap smear at 6-month intervals for a year
(B) Repeat colposcopy to obtain better sample
(C) Perform diagnostic excisional procedure such as Loop electrical excision procedure (LEEP)
(D) Request that the lab repeats their tests on the biopsy samples given
(E) Conization
24. A 25 -year-old P2A0 woman presents to discuss birth control options. She denies any past
medical history and does not smoke tobacco. She is interested in taking a daily birth control
pill. You review the risks and benefits with her. You report one of the noncontraceptive
benefits to combined oral contraceptive pills (OCPs) is:
(A) Decreased risk of deep vein thrombosis
(B) Decreased risk of ovarian cancer
(C) Decreased incidence of migraine headaches
(D) Decreased risk of high blood pressure
(E) Decreased risk of breast cancer

25. A 34-year-old woman has recently become sexually active. She often experiences
mittelschmerz (midcycle ovulatory pain) and hopes to find a method of birth control that will
decrease this occurrence. Which of the following contraceptive methods would be
inappropriate to offer the patient?
(A) Contraceptive vaginal ring
(B) Subdermal contraceptive implant
(C) Depot medroxyprogesterone acetate (DMPA)
(D) Progestin- only oral contraception
(E) Low dose estrogen

26. During the routine examination of the umbilical cord and placenta after a spontaneous
vaginal delivery, you notice that the baby had only one umbilical artery. Which of the
following is true regarding the finding of a single umbilical artery?
a. It is a very common finding and is insignificant.
b. It is a rare finding in singleton pregnancies and is therefore not significant.
c. It is an indicator of an increased incidence of congenital anomalies of the fetus.
d. It is equally common in newborns of diabetic and nondiabetic mothers.
e. It is present in 5% of all births.

27. During a routine return OB visit, an 28-year-old G1P0A0 patient at 23 weeks gestational age
undergoes a urinalysis. The dipstick done by the nurse indicates the presence of trace
glucosuria. All other parameters of the urine test are normal. Which of the following is the
most likely etiology of the increased sugar detected in the urine?
a. The patient has diabetes.
b. The patient has a urine infection.
c. The patient’s urinalysis is consistent with normal pregnancy.
d. The patient’s urine sample is contaminated.
e. The patient has kidney disease.

28. A 28-year-old G1P0A0 presents to your office at 18 weeks gestational age for an
unscheduled visit secondary to right-sided groin pain. She describes the pain as sharp and
occurring with
movement and exercise. She denies any change in urinary or bowel habits. She also denies
any fever or chills. The application of a heating pad helps alleviate the discomfort. As her
obstetrician, what should you tell this patient is the most likely etiology of this pain?
a. Round ligament pain
b. Appendicitis
c. Preterm labor
d. Kidney stone
e. Urinary tract infection

29. A 29-year-old G1P0A0 presents to her obstetrician’s office for a routine OB visit at 32 weeks
gestation. Her pregnancy has been complicated by gestational diabetes requiring insulin for
control. She has been noncompliant withdiet and insulin therapy. She has had two prior
normal ultrasounds at 20 and 28 weeks gestation. She has no other significant past medical
or surgical history. During the visit, her fundal height measures 38 cm. Which of the
following is the most likely explanation for the discrepancy between the fundal height and
the gestational age?
a. Fetal hydrocephaly
b. Uterine fibroids
c. Polyhydramnios
d. Breech presentation
e. Undiagnosed twin gestation

30. A 29-year-old G1P0A0 presents to the obstetrician’s office at 41 weeks gestation. On


physical examination, her cervix is 1 centimeter dilated, 0% effaced, firm, and posterior in
position. The vertex is presenting at –3 station. Which of the following is the best next step
in the management of this patient?
a. Send the patient to the hospital for induction of labor since she has a favorable Bishop
score.
b. Teach the patient to measure fetal kick counts and deliver her if at any time there
are less than 20 perceived fetal movements in 3 hours.
c. Order BPP testing for the same or next day.
d. Schedule the patient for induction of labor at 43 weeks gestation.
e. Schedule cesarean delivery for the following day since it is unlikely that the patient
will go into labor.

31. Your patient complains of decreased fetal movement at term. You recommend a modified
BPP test. Nonstress testing (NST) in your office was reactive. The next part of the modified BPP is
which of the following?
a. Contraction stress testing
b. Amniotic fluid index evaluation
c. Ultrasound assessment of fetal movement
d. Ultrasound assessment of fetal breathing movements
e. Ultrasound assessment of fetal tone
32. A 29-year-old G3P2A0 presents to the emergency center with complaints of abdominal
discomfort for 2 weeks. Her vital signs are: blood pressure 120/70 mm Hg, pulse 90 beats
per minute, temperature 36.9°C, respiratory rate 18 breaths per minute. A pregnancy test is
positive and an ultrasound of the abdomen and pelvis reveals a viable 16-week gestation
located behind a normal-appearing 10 × 6 × 5.5 cm uterus. Both ovaries appear normal. No
free fluid is noted. Which of the following is the most likely cause of these findings?
a. Ectopic ovarian tissue
b. Fistula between the peritoneum and uterine cavity
c. Primary peritoneal implantation of the fertilized ovum
d. Tubal abortion
e. Uterine rupture of prior cesarean section scar

33. During routine ultrasound surveillance of a twin pregnancy, twin A weighs 1200 g and twin
B weighs 750 g. Hydramnios is noted around twin A, while twin B has oligohydramnios.
Which statement concerning the ultrasound findings in this twin pregnancy is true?
a. The donor twin develops hydramnios more often than does the recipient twin.
b. Gross differences may be observed between donor and recipient placentas.
c. The donor twin usually suffers from a hemolytic anemia.
d. The donor twin is more likely to develop widespread thromboses.
e. The donor twin often develops polycythemia.

34. A 30-year-old G1P0A0 with twin gestation at 28 weeks is being evaluated for vaginal
bleeding and uterine contractions. A bedside ultrasound examination rules out the presence
of a placenta previa. Fetal heart rate tracing is reactive on both twins, and the uterine
contractions are every 2 to 3 minutes and last 60 seconds. A sterile speculum examination is
negative for rupture membranes. A digital examination indicates that the cervix is 2 to 3 cm
dilated and 50% effaced, and the presenting part is at −3 station. Tocolysis with magnesium
sulfate is initiated and intravenous antibiotics are started for group B streptococcus
prophylaxis. Betamethasone, a corticosteroid, is also administered. Which of the following
statements regarding the use of betamethasone in the treatment of preterm labor is true?
a. Betamethasone enhances the tocolytic effect of magnesium sulfate and decreases the
risk of preterm delivery.
b. Betamethasone has been shown to decrease intraamniotic infections.
c. Betamethasone promotes fetal lung maturity and decreases the risk of respiratory distress
syndrome.
d. The anti-inflammatory effect of betamethasone decreases the risk of GBS sepsis in the
newborn.
e. Betamethasone is the only corticosteroid proven to cross the placenta.

35. A 30-year-old G1P0A0 at 28 weeks gestation with a twin pregnancy is admitted to the hospital
for preterm labor with regular painful contractions every 2 minutes. She is 3 cm dilated with
membranes intact and a small amount of bloody show. Ultrasound reveals growth restriction of
twin A and
oligohydramnios, otherwise normal anatomy. Twin B has normal anatomy and has appropriate-for-
gestational-age weight. Which of the following is a contraindication to the use of indomethacin as a
tocolytic in this patient?
a. Twin gestation
b. Gestational age greater than 26 weeks
c. Vaginal bleeding
d. Oligohydramnios
e. Fetal growth restriction

36. A 34-year-old woman, G1P0A0, presents for her second prenatal visit at 9 week's gestation.
A review of prenatal laboratory results reveals she is rubella nonimmune. The risk to the
fetus should she contract rubella includes which constellation of findings:
(A) Blindness, deafness, and microcephaly
(B) Cerebral palsy, sensorineural hearing loss, and musculoskeletal deformity
(C) Chorioretinitis, hydrocephalus, and intracranial calcifications
(D) Ergogenic liver foci, microcephaly, ventriculomegaly, and deafness
(E) Nonimmune hydrops, placentamegaly, and anemia

37. While working in the emergency department, a 25-year-old female patient arrives with
severe acute abdominal pain. Before the start of her abdominal pain, the patient recalls
having some fever and chills. She reports that her menses is regular and that she is sexually
active. She recently started having intercourse with a new partner. Pregnancy test is
negative and urinalysis is normal. On physical examination, the patient has muscular
guarding and rebound tenderness. On pelvic examination, patient has cervical motion
tenderness. Vital signs are significant for tachycardia and fever (T 40ᵒC) . Which of the
following is the most likely diagnosis?
(A) Ovarian torsion
(B) Endometriosis
(C) PID
(D) Kidney stone
(E) Ruptured ovarian cyst

38. A 7-year-old girl is brought in to see you by her mother because the girl has developed
breasts and has a few pubic hairs starting to show up. Which of the following is the best
treatment for the girl’s condition?
a. Exogenous gonadotropins
b. Ethinyl estradiol
c. GnRH agonists
d. Clomiphene citrate
e. No treatment; reassure the mother that pubertal symptoms at age 7 are normal

39. You have just performed diagnostic laparoscopy on a patient with chronic pelvic pain and
dyspareunia. The patient had multiple implants of endometriosis on the uterosacral
ligaments and ovaries and several on the rectosigmoid colon. At the time of the procedure,
you ablated all of the visible lesions on the peritoneal surfaces with the CO2 laser. But
because of the extent of the patient’s disease, you recommend postoperative medical
treatment. Which of the following medications is the best option for the treatment of this
patient’s endometriosis?
a. Continuous unopposed oral estrogen
b. Dexamethasone
c. Danazol
d. Gonadotropins
e. Parlodel

40. A 28-year-old nulligravid patient complains of bleeding between her periods and
increasingly heavy menses. Over the past 9 months, she has had two dilation and curettages
(D&Cs), which have failed to resolve her symptoms, and oral contraceptives and
antiprostaglandins have not decreased the abnormal bleeding. Which of the following
options is most appropriate at this time?
a. Perform a hysterectomy.
b. Perform hysteroscopy.
c. Perform endometrial ablation.
d. Treat with a GnRH agonist.
e. Start the patient on a high-dose progestational agent.

41. Which of the following pubertal events in girls is not estrogen dependent?
a. Menses
b. Vaginal cornification
c. Hair growth
d. Reaching adult height
e. Production of cervical mucus

42. You haverecommendeda postcoital test for your patient aspart of her evaluation for
infertility. She and her spouse should have sexual intercourse on which day of her menstrual
cycle as part of postcoital testing?
a. Day 3
b. Day 8
c. Day 14
d. Day 21
e. Day 26

43. If the fetal crown rump length is disparate in twins at the 12 weeks scan, select the
best method of dating the pregnancy. Gestation is age can be allotted according
to:
A. Average CRL of the two foetuses.
B. CRL of the bigger fetus.
C. CRL of the smaller fetus.
D. CRL of the smaller fetus added to half the CRL of the bigger fetus.
E. Dating is best done by LMP in such cases.

44. Twin to twin transfusion occurs in about 15 % of monochorionic pregnancies. When the
Doppler studies are critically abnormal in either twin and are characterised as abnormal or
reversed end-diastolic velocities in the umbilical artery, reverse flow in the ductus venosus
or pulsatile umbilical venous flow, this corresponds to the following stage in Quintero’s
system:
A. Stage 1
B. Stage 2
C. Stage 3
D. Stage 4
E. Stage 5

45. A 38-year-old pregnant woman meets you in the antenatal clinic with the report of her 20
weeks anomaly scan. The report states that there is excessive amniotic fluid and a ‘double
bubble’ sign in the fetal abdomen. The ‘double bubble’ sign seen in fetal ultrasound is
suggestive of:
A. Anorectal malformation
B. Arteriovenous fi stula in the fetal liver
C. Colonic perforation
D. Duodenal atresia
E. Tracheoesophageal fistula

46. Mrs X, primigravida at term is in second stage of labour. After delivery of the fatal head,
shoulder dystocia was diagnosed and the McRoberts manoeuvre has not effected the
delivery of the shoulders, which is the next method to be used:
A. All-fours position
B. Delivery of posterior arm
C. Suprapubic pressure
D. Internal rotation manoeuvres
E. Zavanelli manoeuvre

47. A 25 year old, who is 40 weeks pregnant in her first pregnancy, is in the second stage of
labour. She has been actively pushing for 2 h and is exhausted. CTG shows a baseline of 150
bpm, normal baseline variability, occasional accelerations and infrequent typical variable
decelerations. She is contracting 3–4 every 10 min. Vaginal examination reveals a fully
dilated cervix with the fetal head in a direct occipito-anterior position and at station +1
below spines. Which of the following is the most appropriate next management step?
A. Caesarean section delivery
B. Episiotomy
C. Fetal blood sampling
D. Instrumental delivery
E. Start IV oxytocin augmentation
48. Ms XY is a G3P2A0 and term undergoing an emergency caesarean section under
GA, as she presents in labour with previous 2 caesarean sections with an APH. During
the CS, an anterior low-lying placenta fails to separate after delivery of the baby. A clear
cleavage plane cannot be identified. The bleeding is minimal. She has consented to a
sterilisation, as her family is now complete. Which of the following treatment options
are best suited to her?
A. Attempt to separate placenta and caesarean hysterectomy if bleeding occurs
B. Elective caesarean hysterectomy
C. Leaving the placenta in situ with postoperative methotrexate
D. Removal of the bulk of the placenta and cord and closure
E. Removal of the placenta piecemeal and closure of bleeding points

49. A 45-year-old woman undergoes an abdominal hysterectomy for a large fibroid uterus.
She is found to have a fibroid in broad ligament, and there is a concern that her ureter
may have been damaged during the difficult surgery. Which of the following is the least
possible site of ureteric injury in this surgery?
A. At the level of the uterosacral ligament
B. Lateral to the uterine vessels
C. Renal pelviureteric junction
D. The area of the ureterovesical junction close to the cardinal ligaments
E. The base of the infundibulopelvic ligament as the ureters cross the pelvic brim
at the ovarian fossa

50. Ms. XY is in theatre for repair of a 3C perineal tear. She has been given spinal anaesthetic
and antibiotics. Which suturing technique is most suited for the torn IAS (internal anal
sphincter)?
A. Continuous locked 3–0 PDS
B. Continuous unlocked 3–0 PDS
C. End-to-end technique (interrupted)
D. Figure-of-8 stitches
E. Overlap technique

51. A 27-year-old G2P1A0 at 38 weeks gestation was admitted in active labor at 4 cm dilated;
spontaneous rupture of membranes occurred prior to admission. She has had one prior
uncomplicated vaginal delivery and denies any medical problems or past surgery.
Shereports an allergy to sulfa drugs. Currently, her vital signs are normal and the fetal
heart rate tracing is reactive. Her prenatal record indicates that her Group B streptococcus
(GBS) culture at 36 weeks was positive. What is the recommended antibiotic for
prophylaxis during labor?
a. Cefazolin
b. Clindamycin
c. Erythromycin
d. Penicillin
e. Vancomycin

52. As a front line agent for ovulation induction in polycystic ovary syndrome:
a) Metformin plus clomiphene is superior to either alone
b) Metformin is superior to clomiphene
c) Aromatase inhibitors are superior to both clomiphene and metformin
d) The combination of aromatase inhibitors, clomiphene and metformin is
the best currentoption.
e) Weight loss is superior to all drug options

53. The same woman described in the preceding question now undergoes a laparotomy for
her pelvic mass. Her pre-operative tumour markers were all in the normal range. Intra-
operatively there is a right ovarian solid-cystic mass 9-cm diameter with smooth intact
capsule and 6-cm left ovarian solid-cystic mass also with a smooth intact capsule. The tubes and
uterus are normal and the intraperitoneal survey is normal. What is your consideration for the
next step regarding her management?
a) Ovarian dysgerminomas are usually bilateral.
b) A contralateral dermoid cyst may be found in malignant ovarian germ-cell tumour
(MOGCT).
c) Bilateral oophorectomy and lymphadenectomy should be carried out.
d) Bilateral ovarian cystectomy should be carried out initially.
e) Chemotherapy may be required if the histology is malignant.

54. The pathophysiology of end-organ damage in pre-eclampsia, except :


a) In the kidney, proteinuria results from dysregulation of the glomerular endothelium,
which is induced by vascular endothelial growth factor deficiency.
b) In the liver, hepatic sinusoids may be blocked by intravascular fibrin deposition with
consequent obstructed blood flow and hepatic ischaemia.
c) In the brain, reversible posterior leucoencephalopathy (PRES) is thought to arise from
dysregulation of the cerebral vasculature as a consequence of rapid elevations in blood
pressure.
d) In the brain, areas of both vasoconstriction and forced vasodilation develop,
particularly in the posterior circulation.
e) In the lung, spontaneous pulmonary oedema most commonly develops intrapartum due to
decreased plasma colloid pressure from massive proteinuria and increased cardiac output
during labour.

55. An indication for early delivery is identified, but first a test for fetal lung maturity is done.
Which of the following is true?
a. Type I pneumocytes secrete surfactant
b. A lecithin to sphingomyelin (L/S) ratio greater than 2 is ideal if an early delivery is indicated
c. A low L/S ratio is associated with fewer cases of respiratory distress syndrome (RDS)
d. Typically, lecithin decreases as the lung matures
e. Sphingomyelin decreases beyond 24 weeks

56. When formal antenatal testing is done, which of the following is most reassuring?
a. Late decelerations on fetal monitoring
b. A contraction stress test (CST) with variable fetal heart rate (FHR) decelerations
with contractions, but moderate variability
c. A nonstress test (NST) with two accelerations of the FHR in 20 minutesthat are at least 15
beats above baseline and last for at least 15 seconds
d. An increase in the systolic to diastolic ratio in the umbilical artery blood flow
e. A score of 6 on a BPP

57. The patient chooses to delay prenatal diagnosis at this time and plans on obtaining
second- trimester screening and an obstetric ultrasound. Which of the following is
true about obstetric ultrasound and Down syndrome
a. Ultrasound has a higher sensitivity for Down syndrome than the first-
trimester screening
b. Ultrasound has higher sensitivity for Down syndrome than the quad screening
c. A finding of caudal regression syndrome on ultrasound is pathognomonic
for Down syndrome
d. Down syndrome fetuses with normal ultrasound findings have lower IQs
than those with major anomalies
e. Classic findings of Down syndrome on obstetric ultrasound are an AV canal and pyloric
stenosis

58. A 31-year-old G1P0 woman at 39 weeks and 4 days presents to labor and delivery unit,
with regular contractions occurring every 3 to 5 minutes. Her contractions last 30 to 90
seconds. She not sure if she’s been leaking any fluid from her vagina. You take her
history and conduct a physical examination. ROM would be supported by which of the
following?
a. Nitrazine paper remaining orange when exposed to fluid in the vagina
b. A negative fern test
c. An ultrasound with a normal AFI
d. A negative tampon test
e. Speculum examination with evidence of pooling in the vagina

59. On examination you attempt to determine the presentation of the fetus. Which of
the following presentations and positions would be most favorable to achieve a
vaginal delivery?
a. Breech
b. Transverse
c. Vertex with occiput posterior
d. Vertex with occiputanterior
e. Vertex with occiput transverse

60. The patient dilates without difficulty to 10 cm and the second stage of labor begins. She
is pushing effectively, but during contractions you notice decelerations on fetal heart
tracings. Which of the following would be most concerning?
a. Isolated early decelerations
b. Repetitive variable decelerations that resolve quickly after each contraction
c. Repetitive early decelerations and variable decel
d. Repetitive late decelerations and loss of variability between contractions
e. Absence of decelerations

61. You are working in the emergency department when an 18-year-old Caucasian woman
arrives via ambulance. EMS reports that she was found seizing in a local drug store
approximately 10 minutes ago. She appears to be 7 to 8 months pregnant. She had no
family or friends with her, but police have contacted family who are on the way to the
emergency department. Here vital signs on arrival are as follows: BP, 180/116 mm Hg; heart
rate, 76 bpm; respiratory rate, 16 bpm; oxygen saturation, 98%. Her pants are soiled and
she is not responding to questions at this time. Bedside ultrasound demonstrates fetal
cardiac activity in the 130s. Quick bedside biometry estimates gestation age to be 32 weeks
1 day. You begin empiric magnesiumsulfate therapy. What is the most appropriate next
step in management?
a. Intubation to protect airwa
b. IV labetalol
c. Head CT
d. Lumbar puncture to rule out infection
e. Delivery

62. A 38-year-old G3P2002 woman presents at 40 weeks 3 days with contractions to labor and
delivery triage. Contractions started 1 hour ago and are very painful. The patient denies
leaking fluid but did notice blood and mucus on her underwear. The baby has not been
particularly active since contractions started. Her pregnancy has been complicated by
A2GDM. Fasting blood glucose are usually between 80 and 90 mg/dL with 1-hour
postprandial values between 120 and 140 mg/dL. Her prepregnancy weight was 130 lb and
she is 5 ft 5 in (BMI 21.6). She has gained 30 lb this pregnancy (BMI 26.6). Hemoglobin A1C is
6.0%. Fetal ultrasound at 20 weeks demonstrated normal fetal anatomy. Repeat ultrasound
for growth at 38 weeks demonstrated fetus with weight in the 90th percentile and an EFW
of 4,350 g. Her last pregnancy was complicated by A1GDM and she delivered a 4,200 g
infant without complications. Initial cervical examination reveals dilation of 6 cm, 50%
effacement, and 21 station. Two hours later the nurse calls you to the room after the
patient’s water breaks. Examination shows complete dilation and effacement, and fetus at
11 station. The patient has a strong urge to push and begins pushing. The delivery is
complicated by a second degree perineal laceration and a postpartum hemorrhage of 600
mL. Fetal weight is 4,560 g
and the Apgar scores are 6, 8. Which of the following is most likely the cause of fetal
macrosomia in this case?
a. Maternal weight gain in pregnanc
b. Gestational diabetes
c. Poor glycemic control
d. Advanced maternal age
e. Postterm pregnancy

63. The patient returns 2 years later to your office. She is now 8 weeks 3 days pregnant by
certain LMP. She weighs 160 lb (BMI 26.6). She has been very nauseated this pregnancy
and is eating frequent small snacks. She has not seen a physician since her last
postpartum visit with you during her last pregnancy. What test in particular do you
recommend performing, in addition to, routine prenatal laboratory studies?
a. Preeclampsia panel (creatinine, AST, BUN, platelets, uric acid)
b. 24-hour urine protein collection
c. Referral for eye examination to evaluate for retinopathy
d. Glucose tolerance test
e. Hemoglobin A1C
64. A 26-year-old G1P0 woman presents for a prenatal visit at 34 weeks’ gestation. She
complains of some mild nausea and vomiting over the past 3 days. She has no headache
and no visual changes. Her BP is 142/83 mm Hg. On examination, she has 21 lower
extremity pitting edema, and 3+ reflexes bilaterally with four beats of clonus. A urinalysis
dip has 1+ protein. Which of the following laboratory tests would NOT be helpful at this
point?
a. Platelet
b. WBC
c. LFTs
d. LDH
e. Obstetric ultrasound

65. A 27-year-old G1P0 woman at 9 weeks’ GA presents to your office to initiate prenatalcare.
She has no significant medical or surgical history. She has mild nausea throughout the
day, but on review of systems, has no other concerns. You explain to her
recommendations for prenatal screening tests at the first visit. Which of the following are
not recommended as part of routine prenatal laboratory test screening?
a. Urine cultur
b. Herpes simplex antibody screen
c. Chlamydia PCR
d. HIV antibody screen
e. Rubella immune status

66. The screening urine culture returns with 100,000 colony forming units of E. coli. Which
of the following are not complications of ASB in pregnancy?
a. Pyelonephritis
b. Preterm labor
c. Low-birth-weight infants
d. UTI
e. Oligohydramnios

67. A 24-year-old G2P0010 woman at 8 weeks’ GA presents for an initial prenatal visit. Her
pregnancy is complicated by a history of IV drug use as a teenager and HIV. She had never
been on HAART medication prior to this pregnancy. You order a viral load and it returns
at 10,000. Her CD4 count is normal (.500). She is otherwise healthy and has no other
significant medial history. When would you recommend she start HAART?
a. At this visit
b. At 37 weeks in preparation for deliver
c. At the beginning of the second trimester
d. Immediately after delivery
e. Only if it is medically indicated for maternal health

68. A 28-year-old G2P1001 woman who is 18 weeks pregnant presents to the ED with
increasing left lower extremity swelling, redness, and pain. She first noticed these
symptoms 2 days ago and has tried elevation and warm compresses, which have not
helped. She has no personal or family history of blood clots. She is otherwise healthy
and her only medication is a prenatal vitamin. In the ED, her vital signs are normal.
Physical examination is significant for left lower extremity edema, calf tenderness,
and erythema. You suspect she has a lower extremity DVT and you plan to start her
on anticoagulation therapy. Which of the following is the most important test to
confirm your diagnosis?
a. Venography
b. D-dime
c. Left lower extremity venous Doppler ultrasound
d. MRI of the left lower extremity
e. No further testing is indicated

69. A 36-year-old G7P50015 woman has just delivered a 4,500 g female infant at 39 weeks
gestation. She underwent induction of labor with oxytocin for severe preeclampsia
diagnosed with systolic BPs elevated to 160 mm Hg. Her pregnancy was complicated by
uncontrolled gestational diabetes and resultant polyhydramnios. She was placed on
magnesium throughout her induction for seizure prophylaxis. She had an epidural placed
during the first stage of labor and remained on a normal labor curve throughout. Her
second stage of labor lasted 3½ hours; she was, however, able to deliver vaginally with
preemptive McRoberts maneuvers and steady traction. The third stage of labor lasted 10
minutes andthe placenta was delivered intact. Immediately after the third stage her
bleeding was significant with the expulsion of blood clots and a fundus that was notable
for bogginess. Which of the following are not risk factors for postpartum hemorrhage?
a. Advanced maternal age
b. Grand multiparity
c. Prolonged use of oxytocin during labor
d. Polyhydramnios
e. Prolonged exposure to magnesium during labor

70. The patient was noted to have a third-degree perineal laceration (affected the
external anal sphincter) that was repaired in normal standard fashion. Which of
the following considerations in the treatment and counseling of these patients is
false?
a. You should provide a rectal examination to ensure that the mucosa is intact
b. She should be on regular stool softeners throughout the postpartum period
c. She should be given narcotic medications for pain control on a PRN basis
d. She should be counseled about her risk of anal sphincter defect and incontinenc
e. She should undergo anal endosonography and/or anal manometry in 1 year to evaluate for
sphincter defects

71. A 26-year-old G0 patient comes in with a problem visit for a complaint of an


intermittent painless mass on her vulva near the introitus. It seems to be aggravated by
intercourse, but usually goes away on its own. She’s had two lifetime sexual partners
and has been withher last partner for 5 years. She has always had normal periods and
Pap smears and has never had an STI. You examine her and find a 3 cm nontender mass
in the area described. What type of abnormality is this most likely to be?
a. Skene’s gland cyst
b. Gartner’s duct cys
c. Bartholin’s duct cyst
d. Cystocele
e. Epidermal inclusion cyst

72. a 13-year-old adolescent girl who presents with cyclic pelvic pain. She has never had a
menstrual cycle. She denies any history of intercourse. She is afebrile and her vital signs
are stable. On physical examination, she has age-appropriate breast and pubic hair
development and normal external genitalia. However, you are unable to locate a vaginal
introitus.Instead, there is a tense bulge where the introitus would be expected. You obtain
a transabdominal ultrasound, which reveals a hematocolpos and hematometra. What is the
most likely diagnosis?
a. Transverse vaginal septum
b. Longitudinal vaginal septu
c. Imperforate hymen
d. Vaginal atresia (MRKH)
e. Bicornuate uterus
73. Which of the following is the most common mullerian anomaly?
a. Bicornuate uteru
b. Septate uterus
c. Uterine didelphys
d. Unicornuate uterus
e. Arcuate uterus

74. A couple presents because they have been trying to conceive for 18 months. During
the interview you learn that the man has fathered a child in a previous relationship and
is in good health. The woman is 28 and reports that she has had painful menses for the
past 5 or 6 years. You begin to suspect that she may have endometriosis. All of information
below would increase that suspicion except:
a. she reports that a maternal cousin has a history of endometriosis
b. she has experienced dyspareunia with deep penetration for several years
c. her ethnicity is Caucasian
d. she report the development of abnormal bleeding in the last year
e. her menarche began at age 9

75. After completing your history you explain to your patient that you need to perform an
examination before making any recommendations. You explain that women with
endometriosis often have a normal examination but that there are certain findings that
are associated with endometriosis. During your examination, which of the findings
listed below would NOT increase your suspicion that she has endometriosis.
a. A fixed deviated uterus
b. Uterosacral nodularity on rectovaginal examination
c. Tender adnex
d. An enlarged irregular uterus
e. A fixed adnexal mass

76. An 18-year-old nulligravid woman presents to the student health clinic with a 4-week
history of yellow vaginal discharge. She also reports vulvar itching and irritation. She is
sexually active and monogamous with her boyfriend. They use condoms inconsistently. On
physical examination, she is found to be nontoxic and afebrile. On genitourinary
examination, vulvar and vaginal erythema is noted along with a yellow, frothy, malodorous
discharge with a pH of 6.5. The cervix appears to have erythematous punctuations. There is
no cervical, uterine, or adnexal tenderness. The addition of 10% KOH to the vaginal
discharge does not produce an amine odor. Wet prep microscopic examination of the
vaginal swabs is performed. What would you expect to see under microscopy?
a. Branching hyphae
b. Multinucleated giant cells
c. Scant WB
d. Flagellated, motile organisms
e. Epithelial cells covered with bacteria

77. A 16-year-old girl presents to the physician for annual examination. She denies any
current symptoms or concerns. She has been sexually active for 1 year and is using Depo-
Provera for contraception. She does not use condoms. On urogenital examination, she
has a moderate amount of yellow mucopurulent discharge from the endocervix. There is
no cervical motion, adnexal, or uterine tenderness. Microscopy of the vaginal discharge
was normal except greater than 10 WBCs per high power field. Vaginal pH is normal. What
is the most appropriate next step in the management?
a. Perform a Pap smear with reflex HPV testing
b. Order a pelvic ultrasound
c. Treat for presumptive chlamydial infection and gonorrhea
d. Send vaginal discharge for culture and Gram stai
e. Perform nucleic acid amplification test (NAAT) for chlamydial infection and gonorrhea

78. A 28-year-old G1P1 presents to the emergency department 4 days after primary cesarean
section with complaints of fever, malaise, and increased lower abdominal pain for the
past 6 hours. Her labor course was complicated by prolonged rupture of membranes and
stage 2 arrest due to cephalopelvic disproportion resulting in a cesarean delivery. Her
postoperative course was uncomplicated and she had been discharged home stable the
day prior to presentation. Her temperature is 38.1°C (100.6°F), pulse rate is 102/min,
respirations are 20/min, and BP is 110/70 mm Hg. Abdominal examination shows fundal
tenderness. The incision is intact without erythema, warmth, or discharge. On pelvic
examination, there is foul-smelling lochia. Her WBC count is elevated and there is
moderate blood on urine analysis. Which of the following is the most appropriate next
step in management?
a. Dilation and curettage
b. Discharge home with oral doxycycline
c. Exploratory laparotomy
d. CT scan of abdomen and pelvis
e. Administer IV clindamycin and gentamicin

79. A 34-year-old G2P2 woman comes to the emergency department with 8 hours of increasing
right lower quadrant pain, inability to tolerate orals, and nausea. She is sexually active and
uses Depo-Proverafor contraception. She was treated for gonorrhea and reports
compliance with treatment. Her temperature is 38.5°C (101.3°F), pulse rate is 114/min,
respirations are 22/min, and BP is 110/70 mm Hg. On examination, her abdomen is soft
with right lower quadrant tenderness. Voluntary guarding is present without rebound.
Pelvic examination shows no cervical motion tenderness or uterine tenderness. The right
adnexa is exquisitely tender and fullness is appreciated. Her WBC count is 17,000 cells/μL
and there are 15% bands. Urine HCG is negative. What is the most likely diagnosis?
a. Ovarian torsion
b. Appendiciti
c. Tubo-ovarian abscess
d. Cervicitis
e. Ectopic pregnancy

80. A 69-year-old woman with pelvic pressure and palpable bulge presents for evaluation.
She recalls some mention of a cystocele diagnosis, given by her primary care provider.
Today, she requests formal evaluation by a gynecologist. When performing the physical
examination, what is one type of staging system to describe prolapse?
a. Pelvic organ prolapse quantification scale (POP-Q)
b. Gray scale
c. Visual analog scale
d. Breslow scale
e. Clark scale

81. In discussing her symptoms, the patient points out that her voiding function has changed as
the prolapse has grown in severity. Initially, the patient reported stress urinary
incontinence, but as the prolapse worsened, the incontinence improved. While she is
happy with the resolution of her incontinence, she currently experiences some incomplete
bladder emptying, which is improved upon manual reduction of the prolapse. How do you
counsel her about her risk of incontinence after an isolated anterior wall repair (with no
other concomitant surgery)?
a. High likelihood of de novo urgency and urge urinary incontinence
b. High likelihood of urinary frequency
c. High likelihood that her stress incontinence will be cured by anterior repai
d. High likelihood that an anterior repair will unmask and potentially “worsen” her stress urinary
incontinence symptoms
e. High likelihood of de novo fecal incontinence

82. An 82-year-old G3P2 woman is brought to your office by her caregiver from a local
retirement home. She has a diagnosis of dementia that has been worsening over the
past year. She is followed closely by her PCP and saw him recently for her general
conditions. She is communicative but has a poor memory. Her history is obtained from
her caregiver. She is ambulatory with minimal assistance and is able to follow
commands. Her BMI is 23.5. Her caregiver tells you that over the past year she has had an
increase in the number of urinary leakage episodes. She wears adult diapers (Depends), which
have to be changed at least two times per day due to leakage. In the morning she wakes up
with a wet pad. Her pelvic examination is normal except for atrophic vaginitis (consistent with
menopause). Urinalysis with culture and sensitivity are negative. What is the initial
management of this patient?
a. Pelvic floor muscle exercises, 3 sets of 10 per da
b. Bladder training, she should be instructed to empty her bladder every 2 to 3hours
c. Weight loss
d. Incontinence pessary
e. Expectant management

83. The patient comes back to your office and she is pregnant. What hormone does
the developing trophoblast produce?
a. Human chorionic gonadotropin (hCg)
b. Progesterone
c. Androstenedione
d. LH
e. Estrogen

84. A 32-year-old G1P1001 woman presents to your office with the chief complaint of
amenorrhea since her most recent vaginal delivery 1 year ago. She notes that she had an
uncomplicated pregnancy, followed by the delivery of a healthy baby boy. Her delivery was
complicated by an intra-amniotic infection as well as a postpartum hemorrhage requiring a
postpartum dilation and curettage. After her delivery, she breastfed for 6 months, and
during this time she had scant and irregular vaginal bleeding. After stopping breastfeeding 6
months ago, she notes the absence of menses, but instead has monthly painful cramping,
which seems to be getting worse. She remarks that prior to her pregnancy, she had normal,
regular menses, which were not too heavy or painful. She and her husband would like to
have another child, and have been having unprotected intercourse for the past 6 months
without achieving a pregnancy. Your review of systems is otherwise negative. You perform
a physical examination, which is normal other than a slightly enlarged, tender uterus. A
urine pregnancy test in the office is negative. What is the most likely diagnosis?
a. Sheehan’s syndrome
b. Lactational amenorrhea
c. Asherman’s syndrome
d. Premature ovarian failure

85. A 58-year-old G3P3003 Caucasian, postmenopausal woman comes to your office. She has
been menopausal since age 50. She has a negative past medical and surgical history. She
took hormone replacement for about 2 years but stopped due to concerns of an increased
risk of cancer that she heard about from friends. Prior to the onset of menopause, she had
a history of normal and regular menses. She has had annual GYN care with you, and has
never been diagnosed with cervical dysplasia. Her last Pap smear with HPV was obtained
last year and both were negative. She has recently become sexually active with a new
partner and has noted some spotting with intercourse as well as intermittent spotting that
she notices on wiping for the past 2 to 3 months associated with occasional mild lower
abdominal cramping. She complains of a general feeling of vaginal dryness and does have
pain and dryness with intercourse. She has no other complains what so ever. Her most likely
diagnosis is:
a. Endometrial cancer
b. cervical cance
c. urogenital atrophy
d. bleeding dyscrasia
e. uterine fibroids

86. A 24-year-old G2P1001 female is diagnosed with fetal anencephaly, a lethal anomaly, at
her first prenatal visit at 16 weeks. She is otherwise healthy. She had a previous term
vaginal delivery without complications. She wishes to terminate this pregnancy. The
patient is counseled on the risks and benefits of both D&E and labor induction. She
opts for D&E. Which is NOT used for cervical dilation prior to the procedure?
a. Osmotic dilators
b. Laminaria
c. Misoprostol
d. High dose IV oxytocin
e. Foley balloon catheter

87. A young couple comes in with a chief complaint of infertility. The patient is a 30-year-old G0
who has not undergone any evaluation. Her husband is a 33-year-old who has had a semen
analysis, which was reported as normal. He has never fathered a child. The couple reports
having unprotected intercourse for the past 14 months. On further history, the patient
reports that her periods have been quiet irregular over the last year and that she has not
had period in the last 3 months. She also reports hot flashes, vaginal dryness, and decreased
libido.
The most likely diagnosis for this patient based on her history is
a. Polycystic ovarian syndrome
b. Primary ovarian insufficiency/Premature ovarian failure
c. Endometriosis
d. Kallmann syndrome
e. Spontaneous pregnancy

88. To confirm your suspicion you decide to do some lab tests. All of the following would
be appropriate test of ovarian reserve, except:
a. Anti-Mullërian Hormone (AMH)
b. Day 3 FSH
c. Day 3 estrodiol leve
d. Progesterone
e. Clomid Citrate Challenge Test (CCCT)

89. A 24-year-old G1P1 female comes to the office for colposcopy due to LSIL pap 1 month
prior. Her pap was normal 3 years ago. She began sexual activity at age 16 and has had five
partners. She has had Chlamydia and vulvar warts in the past, both of which have been
treated and resolved. She is a smoker of approximately 15 cigarettes per day since she was
15. She uses Depo Provera for contraception. Her mother died last year of cervical cancer at
age 44. What are her risk factors for CIN?
a. Age 16 at onset of sexual activity
b. Five sex partners
c. Her history of STDs
d. Smoking 15 cigarettes a day
e. All of the above

90. A 17-year-old Asian teen presents to the ED with left lower quadrant (LLQ) pain.
Her pregnancy test, UA, and cervical cultures are all negative. She denies sexual
activity. On pelvic ultrasound, she is found to have a unilateral complex ovarian
mass. What tumor marker is associated with this type of tumor?
a. hCG
b. AFP
c. Ca-12
d. LDH
e. All of the above

91. Profuse vaginal bleeding. Her medical history is unremarkable. She has had two term
vaginal deliveries, her last being 10 years ago. An initial workup shows a solid
intrauterine mass and a serum β-hCG level of 220 mIU/mL. You decide to proceed with
endometrial sampling, which is uncomplicated. Final surgical pathological diagnosis is
consistent with PSTT. Which of the following elevated serum markers is most associated with
PSTT?
a. Prolactin
b. Serum β-hC
c. Human placental lactogen
d. CA-125
e. Inhibin-A

92. A 42-year-old female G4P3 presents to your office with complaint of a palpable breast
mass that she noticed while performing her breast self-examination. After taking her
history, you perform a clinical breast examination which reveals a solitary, 3-cm,mobile,
nonpainful, rubbery mass. On the basis of her clinical examination, what is most likely her
diagnosis?
a. Fibrocystic breast chang
b. Fibroadenoma
c. Cystosarcoma phyllodes
d. Intraductal papilloma
e. Invasive breast cancer
93. A 35-year-old G2P1001 woman at 40 weeks 6 days presents to labor and deliverytriage
with a 5-hour history of painful contractions. Monitoring reveals contractions every 3
minutes and cervical examination on arrival is 3 cm dilated, 50% effaced, and 22 station.
Her pregnancy has been complicated by A1 gestational diabetes and 45 lb weight gain in
pregnancy (BMI
24). In her first pregnancy, she presented at 2 cm dilation and 90% effacement. Her labor
progressed slowly with dilation of 1 cm every 3 hours until labor arrested at 7 cm. A healthy
baby girl weighing 4,200 g was delivered by cesarean section after arrest at 7 cm for 4
hours. Today your patient requests a trial of labor after cesarean section. You recheck her
cervix 2 hours later and find that it is 4 cm dilated, 90% effaced, and 21 station. The
estimated fetal weight by Leopold’s is 9 lb. A recent ultrasound preformed at 38 weeks
estimated the fetal weight at 3,900 g. You admit the patient to labor and delivery for
expectant management. Which of the following factors was least likely to result in her prior
failure to progress?
a. Obstetric conjugate diameter of the pelvis greater than 11.5 cm
b. Inadequate strength of uterine contractions
c. Fetal size or position
d. Maternal pelvis shape

94. A 33 -year-old G2P2 woman presents to your office with a complaint of nippledischarge.
She states it is milky in color, comes from both breasts, and is present even when she
doesn’t express it. You perform a breast examination, and express milky discharge from
both breasts. You diagnose her with galactorrhea. Which condition is not associated
with galactorrhea?
a. Pregnanc
b. Breast abscess
c. Pituitary adenoma
d. Psychotropic medications
e. Hypothyroidism

95. A 33-year-old G0P0 woman comes to your office for her initial prenatal visit. She tested
positive with two home pregnancy tests and has been experiencing breast tenderness and
mild nausea for a few weeks. She has a history of regular menstrual periods occurring every
28 to 30 days. This was a planned pregnancy and is the first child for her and for her
partner. As her pregnancy continues, you would expect her cardiac output to increase by
which of the following mechanisms:
(A) First an increase in stroke volume, then an increase in heart rate
(B) A decrease in systemic vascular resistance
(C) Cardiac output would not change significantly until the third trimester
(D) An increase in systemic vascular resistance facilitated by elevated progesterone levels
(E) Increased heart rate alone

96. A 36-year-old G1P0 is 31 weeks and 5 days by LMP and is sure of her dates. Her pregnancy
has been complicated by persistent nausea and vomiting, back pain, and lower extremity
swelling. She comes to you for a routine prenatal visit. She had a quad screen at 16 weeks
that was normal. She is having a girl. Her low back pain is no longer relieved with a heating
pad and she finds that she needs pain relief to make it through each work day. Which of
the
following options would be safest for her?
(A) Ibuprofen
(B) Aspirin
(C) Oxycodone
(D) Flexeril
(E) Tylenol

97. A G3P2002 woman at 35 weeks is seen in your office for her prenatal visit. She is concerned
because she has not felt her baby moving as much as she used to. Her pregnancy has been
uncomplicated and her past two pregnancies ended in full term, normal spontaneous
vaginal deliveries. When formal antenatal testing is done, which of the following is most
reassuring?
(A) Late decelerations on fetal monitoring
(B) A contraction stress test (CST) with variable fetal heart rate (FHR) decelerations with
contractions, but moderate variability
(C) A nonstress test (NST) with two accelerations of the FHR in 20 minutes that are at least 15
beats above baseline and last for at least 15 seconds
(D) An increase in the systolic to diastolic ratio in the umbilical artery blood flow
(E) A score of 6 on a BPP

98. A 28-year-old P0010 woman presents to the emergency department with abdominal
pain since the past day. She reports a 1-week history of nausea with occasional
vomiting. She has noticed some breast tenderness as well. She denies dysuria, vaginal
bleeding, or any bowel symptoms. She reports that her last period was 4 weeks ago, but
was lighter than normal. She has been using condoms for contraception. On arrival, her
vital signs include a temperature of 37°C, BP of 117/68, pulse rate of 78 beats per
minute, and respiratory rate of 16 breaths per minute. Cardiovascular and respiratory
examinations are normal. She notes some suprapubic abdominal discomfort with
palpation, but she does not have rebound tenderness or guarding. A speculum
examination reveals a closed cervix without bleeding. A pelvic examination is mildly
uncomfortable and reveals a normally sized, anteverted uterus, and palpably normal adnexa. A
urine pregnancy test is positive. The quantitative β-hCG level is 1,300 mIU/mL. The patient
reveals that this was an unplanned, but desired pregnancy. What follow-up recommendations
do you give this patient?
(A) Make an appointment with her primary OB/GYN for an initial prenatal visit
(B) This is likely an ectopic pregnancy and she should proceed with methotrexate therapy
(C) She should undergo urgent laparoscopy for evacuation of an ectopic pregnancy
(D) She should return in 48 hours for a repeat β-hCG
(E) She has likely had a SAB and does not need further follow-up

99. A 40-year-old G2P0 woman at 7 weeks GA by LMP presents for her first prenatal visit. She
spontaneously conceived after 18 months of trying. She is excited about the pregnancy, but
at the same time is concerned about potential risks for herself as well as the baby because
of her age. Her husband is 52 years old, healthy, and has fathered two children from a
prior
marriage. The week prior to the visit, she experienced spotting that lasted 3 days and then
resolved. Currently, she has no complaints. She has no past medical or surgical history
except for a miscarriage 3 years ago. She has regular periods every 30 days. You offer her
which of the following prenatal screening/diagnostic tests?
(A) CVS
(B) Amniocentesis
(C) First-trimester screening
(D) Quad screening
(E) All of the above

100. You are providing prenatal care to a 22-year-old G1P0 woman at 16 weeks GA by
LMP. She has had a relatively smooth pregnancy without complications thus far. At 5950
and 215 lb she has an obese BMI, otherwise without medical or surgical history. She
presented to prenatal care at 14½ weeks, and so missed first-trimester screening. She
undergoes the quad screen and has an elevated level of maternal serum alpha-fetoprotein
(MSAFP). The ultrasound reveals a myelomeningocele. The increased incidence of this
finding is associated with which of the following medications when used in pregnancy?
(A) Valproic acid
(B) Lithium
(C) Fluoxetine
(D) Prednisone
(E) Acetaminophen

101. A 24-year-old G1P0 woman presents for prenatal care at 8 weeks by LMP. She has
regular menses every 28 to 30 days and you confirm her gestational age with an ultrasound
today in the office. She has no past medical or surgical history. She and her husband of 6
months planned the pregnancy and they have both been reading about pregnancy and
prenatal care. You discuss the prenatal tests for the first visit as well as the plan throughout
the rest of the pregnancy. The patient opts to undergo first-trimester screening, which
returns with a risk for Down syndrome of 1 in 1,214 and risk of trisomy 18 of 1 in 987. At 18
weeks, she gets a quad screen, and her estriol, β-hCG, and α-fetoprotein (AFP) were all low.
She has an ultrasound, which shows a fetus consistent with 16 weeks’ size, increased amniotic
fluid, clubfoot, omphalocele, choroid plexus cyst, and possible heart defect. On the basis of the
patient’s history and data provided, what is the most likely diagnosis?
(A) Trisomy 21
(B) Trisomy 18
(C) Trisomy 13
(D) Turner syndrome
(E) Klinefelter syndrome

102. A 34-year-old G3P2002 woman at 38 weeks and 6 days was admitted to labor and
delivery unit for active management of labor after it was determined that her
membranes had
ruptured and she was dilated to 3 cm. Her cervix has been steadily dilating and now she is
at 6 cm. She is very uncomfortable and finds her contractions very painful. Her partner is
also very concerned that she needs pain relief. With adequate pain control she dilates to 10
cm and second stage begins. Which of the following is the correct order of the cardinal
movements of labor?
(A) Internal rotation, engagement, descent, flexion, externalrotation
(B) Engagement, descent, internal rotation, flexion, externalrotation
(C) Internal rotation, descent, engagement, flexion, externalrotation
(D) Engagement, descent, flexion, internal rotation, external rotation
(E) Engagement, descent, internal rotation, flexion, external rotation

103. A 22-year-old G1P0 African American woman at 36 weeks by LMP consistent with 12-
week ultrasound with limited prenatal care presents via ambulance to the L & D triage unit
complaining of severe abdominal pain and profuse vaginal bleeding. The patient is unstable
and unable to communicate coherently. The EMT reports that initially her BP was 180/100
mm Hg and pulse rate was 110 bpm, but she has lost at least 500 mL blood in route. On
examination her BP is 90/50 mm Hg, pulse rate is 120 bpm, she appears to be in significant
pain, is unable to answer questions, and her abdomen feels rigid. What is the most
important laboratory test to order for this patient emergently?
(A) Type and cross
(B) CBC
(C) Urine drug screen
(D) Chemistry
(E) PT, PTT, INR

104. A 29-year-old G3P1102 woman at 29 weeks 3 days presents to labor and delivery
triage for evaluation of abdominal pain. Her pain started 2 hours ago and comes and
goes every 5 minutes. She denies any leaking fluid, change in vaginal discharge, or
vaginal bleeding. Her baby has been active. Her pregnancy is complicated by a history of
a urinary tract infection at 10 weeks with GBS and a history of preterm birth at 31
weeks with her last child. She is currently taking progesterone injections weekly and a
prenatal vitamin. On vaginal examination, her cervix is closed, 25% effaced, and 23
station. Uterine contractions are noted every 4 to 5 minutes with a category 1 tracing. A
fetal fibronectin test returns positive. Repeat examination after 1 hour shows dilation of
1 cm, 50% effacement, and 22 station. The team decides to start her on magnesium
sulfate for tocolysis and administers the first dose of betamethasone. What side effects or
complications should you counsel your patient that she might experience from magnesium
sulfate?
(A) Flushing, diplopia, headache
(B) Flushing, dizziness, headache
(C) Headache, tachycardia, anxiety
(D) Headache, flushing, dizziness
(E) Constriction of the ductus arteriosis in the neonate
105. A 32-year-old G0 woman with type 1 diabetes mellitus (T1DM) presents for a
preconception visit. She was diagnosed with T1DM at age 4, and other than some
challenges with glucose control during her teen age years, she generally has good control
per her report. She uses a subcutaneous insulin pump. She has no history of retinopathy,
renal disease, heart disease, proteinuria, peripheral neuropathy, or any other medical
conditions. On examination, she is 5960 tall and weighs 122 lb. Her BP is 128/76 mm Hg.
You order a laboratory test and her HgbA1c returns at 11. You advise her which of the
following?
(A) She should go ahead and start trying to get pregnant
(B) She should aggressively try to lower her HgbA1c to less than 9 to reduce the
risk of preterm birth
(C) She should aggressively lower her HgbA1c to less than 5 in order to
reduce her risk of preeclampsia
(D) She should aggressively lower her HgbA1c to less than 7 to reduce her riskof
congenital anomalies
(E) She should go ahead and start trying to get pregnant, but also slowly
reduce her blood glucoses with a HgbA1c target of less than 9

106. A 29-year-old G2P1 woman with obesity, a history of GDM in the prior pregnancy,
and a strong family history for type 2 diabetes mellitus (T2DM) presents at 7 weeks’
gestation by LMP. In her previous pregnancy, she required insulin therapy. She delivered at
39 weeks and her baby boy weighed 4,300 g (or approximately 9½ lb). Her fasting blood
glucose is 145 mg/dL. An ultrasound reveals the pregnancy to be 7 weeks and 2 days,
consistent with LMP, and anti-insulin antibodies are negative. Her diagnosis is:
(A) A1 GDM
(B) A2 GDM
(C) T1DM
(D) T2DM
(E) no diabetes in pregnancy

107. A 36-year-old G2P2 woman presents to her gynecologist with a 3-week history of
vaginal irritation and fish-smelling vaginal discharge. She recently tried an over-the-counter
antifungal treatment without any improvement in her symptoms. She is sexually active in a
monogamous relationship with a male partner of 5 years and she uses a contraceptive ring
(NuvaRing). Genitourinary examination shows a thick white discharge. The remainder of
her examination is normal. Microscopic evaluation of a saline “wet prep” of the vaginal
secretions reveals decreased lactobacilli, a few WBCs, and vaginal epithelial cells with a
stippled appearance. Risk factors for developing this condition include all of the following except:
(A) multiple sexual partners
(B) cigarette smoking
(C) douching
(D) contraceptive ring (NuvaRing)
(E) lack of vaginal lactobacilli

108. Post operatively maintenance intravenous administered fluids are:


(A) Ringer solution or a similar crystalloid solution with 5-percentdextrose
(B) 10% dextrose
(C) Colloid solution
(D) Crystalloid solution with 40-percent dextrose
(E) Amino acid based fluid
109. A 21 years old G4P2 at 17 weeks gestation presents for her first prenatal care visit.
She has a history of prostitution, but she denies engaging in such activities for the past
month. During examination, a painless lesion is noted on the right labia. The most likely
diagnosis is which of the following?
(A) Chancroid
(B) Primary syphilis
(C) Bartholin gland duct abcess
(D) Herpes simplex virus infection
(E) Condiloma acuminate

110. Duringa physical examination myrtiformcaruncles may benoted. What are they?
(A) circumferential nodules in the areola of the breast
(B) healing Bartholin’s cysts
(C) remnants of the Wolffian duct
(D) remnants of the hymen
(E) remnants of the Müllerian duct

111. During delivery, which ofthe following muscles is most likely to beobviously torn?
(A) ischiocavernosus muscle
(B) bulbocavernosusmuscle
(C) superficial transverse perineal muscle
(D) levator ani muscle
(E) coccygeus

112. A38-year-old African American woman presents with heavy menses and an enlarged uterus. After
anexaminationtheclinicaldiagnosisisleiomyomaoftheuterus.Whichofthefollowingbestdescribes
this finding?
(A) a soft, interdigitating mass of the uterine wall
(B) a premalignant papule of the uterine wall
(C) a rapidly dividing necrotic malignancy
(D) a rounded, smooth, firm, well-circumscribed mass
(E) erythematous, tender, and hereditary

113.Anovary isremoved forfrozensectionpathologic examination. Theovary isenlarged,withsmall


surface excrescences. Pathologic examination reveals numerous cysts lined by serous
epithelium with six to eight cell layers piled on top of one another to form the cyst walls. The cells
show marked cytologic atypia,and nests ofsimilar cells are presentintheovarian stroma. Roundlaminated
calcium bodies are also seen. What diagnosis does this histologic description indicate?
(A) normal proliferative phase follicle
(B) corpus luteum cyst
(C) ovarian endometriosis
(D) borderline ovarian carcinoma
(E) cystadenocarcinoma

114. Histologicexaminationofthenormalbreast fromapostmenopausalwomanascomparedto


the breast froma premenopausal woman would show which of the following?
(A) a decrease in the number and size of acinar glands and ductal elements, with decreased density of
the breast parenchyma
(B) anincrease inbreast size and turgiditybecause ofanincrease inthedensity of theparenchyma
(C) increase in number and size of acinar cells and awidening of the ductal lumens
(D) significant atrophy of the adipose tissue of the breast with little change in the
actual breast parenchyma
(E) no significant change in histology

115. Anamniocentesis results show afetus with 45XO. Incounseling theparents, how would
you explain that the genetic sex is determined?
(A) at ovulation
(B) at conception
(C) by the presence orabsence of testosterone
(D) in the absence of Müllerian-inhibiting factor
(E) psychosocially after birth

116. Which ofthe following is the result of lack of fusion of the Müllerian duct system?
(A) uterine didelphys
(B) transverse vaginal septum
(C) unilateral renal agenesis
(D) imperforate hymen
(E) ovarian remnant syndrome

117. Acouple is concerned about thesafetyof antenatalultrasound.


Whatshouldyoucounsel them regarding the procedure?
(A) Ultrasound has been in use for almost 40years with nonoted side effects.
(B) The reason it is called ultrasound is that it is safe, as verified by the Food and
Drug Administration (FDA).
(C) Ultrasound is done only by trained sonographers toensure safe practice.
(D) Ultrasound maybeassociated with cataracts and hearing loss in animals, if used
continually, an thus it is used only when indicated.
(E) Antenatal ultrasound may be associated with heating in tissue and thus is
used only when indicated.

118. Ababy presents withambiguousgenitalia. Afullchromosome count is sent and will return in
72 hours. Your laboratory can perform a test for Barr body so you can provide a preliminary
answer sooner. What is the Barr body?
(A) the condensed, nonfunctioning Xchromosome
(B) the darkest, widest band found on chromosomes
(C) an extra lobe on the female polymorphonuclear leukocytes
(D) found only in the female
(E) the largest chromosome in the female genotype

119. A22-year-oldG3P2002whohadahematocritof36%atherinitialobstetricalexamination at 12
weeks is foundtohaveahematocrit of30% at 28weeks whencheckedalongwithher 1hour
glucola. Basedontheindices oftheredblood cellsontheCBC,youdiagnoseirondeficiency.She asks
why that occurred since she has been taking her prenatal vitamins. As part of the explanation,
you note that. which of the following maternal measurements or findings is first decreased by
the iron requirements of pregnancy?
(A) bone marrow iron
(B) hemoglobin
(C) jejunal absorption of iron
(D) red cell size
(E) serum iron-bindingcapacity

120. A poorly controlled class D diabetic patient desired a repeat cesarean section. An
amniocentesis to verify pulmonary maturity was done prior to scheduled surgery at 37 weeks’
gestation. The L/S ratio was 2:1 and phosphatidyl-glycerol was absent. An infant was delivered
who developed infant respiratory distress syndrome (IRDS). What was the most likely reason?
(A) Diabetic patients do not produce lecithin.
(B) Fetal lung maturation may be delayed in maternaldiabetes.
(C) Foam test was not done.
(D) The L/S test was done on fetal urine.
(E) Maternal blood was present in the specimen.

UNAS AGUSTUS 2017


1. A 21 y.o G0 present for her first gynecologic examination. She states that she became sexually
active 2 weeks ago for the first time. She has no significant medical history. She has regular
menses with some mild dysmenorrheal. During the speculum examination, you observe a small
raised 0,5 cm lesion 0,5 cm. It is smooth and light-bluish in color with the appearance of a bubble
under the epithelial surface and a blood vessel running over the top. What is your diagnosis?
a. Bartholini’s gland cyst
b. Cervical dysplasia
c. Nabothian cyst
d. Skene’s gland cyst
e. Cervical cancer

2. A 45 y.o woman complains of pelvic pressure and abnormal uterine bleeding. Ultrasound reveals
an enlarged uterus with an intramural 4 cm mass. Which of the following is the most common
uterine neoplasm?
a. Sarcoma
b. Adenocarcinoma
c. Adenomyosis
d. Choriocarcinoma
e. Leiomyoma

3. The following statement is true regarding the physiological adaptations to pregnancy?


a. The increase in maternal heart rate contributes to an increase in cardiac output during
pregnancy
b. Mean arterial blood pressure falls because of a rising in systemic vascular resistance
c. Anatomical and physiological changes in the lungs allow a pregnant woman to with stand
hypoxia better than a non-pregnant woman
d. The lower bicarbonate levels in pregnant women relect a state of metabolic acidosis
e. TSH (thyrotropin) levels fall in the first trimester but return fastly to normal by term

4. A 29 y.o woman with a positive pregnancy test present with a good history of passing tissue per
vagina. A transvaginal ultrasound scan shows an empty uterus with an endometrial thickness of
11 mm. Regarding her diagnosis, you consider that :
a. She has had a complete miscarriage and needs no further treatment
b. She has a pregnancy of unknown location and needs further investigations
c. She should be offered a hysteroscopy
d. She should be offered medical management of miscarriage
e. A laparoscopy should be performed to exclude an ectopic pregnancy

5. A patient who underwent an abdominal hysterectomy a few days ago complains of numbness
over the skin over the anterior aspect of the upper thigh. This could be attributed to neuropathy
of the
:
a. Femoral nerve
b. Obturator nerve
c. Perineal nerve
d. Podendal nerve
e. Sciatic nerve

6. A 45 y.o woman undergoes an abdominal hysterectomy for a large fibroid uterus. She is found to
have a fibroid in broad ligament, and there is a concern that her ureter may have been damaged
during the difficult surgery. Which of the following is the least possible site of ureteric injury in
this surgery?
a. At the level of the uterosacral ligament
b. Lateral to the uterine vessels
c. Renal pelviureteric junction
d. The area of the ureterovesical junction close to the cardinal ligaments
e. The base of the infundibulopelvic ligament as the ureters cross the pelvic brim at the ovarian
fossa

7. A 7 y.o girl presents to her pediatrician with her parents who are concern about her early sexual
development. She is developing breasts, axillary hair, and pubic hair, and they are noticing body
odor. A thorough clinical workup reveals the child has an irregular, echogenic, thickly septated
ovarian mass on her left ovary. What type of tumor is responsible for this child’s clinical
presentation?
a. Dysgerminoma
b. Embryonal carcinoma
c. Sertoli-leydig cell tumor
d. Endodermal sinus tumor
e. Granulosa-theca cell tumor

8. On prenatal ultrasound, which of the following feature characteristic gastroschisis?


a. The abdominal wall defect is superior to cord insertion
b. Ectopia cordis is present
c. The abdominal wall defect is lateral to cord insertion
d. The abdominal wall defect is lower than cord insertion
e. The bladder cannot be visualized

9. A 39 y.o female G2P1A0, 15 weeks pregnant present to your clinic for having routine ANC. On
physical examination, you found her fundal height equals umbilical point. You performed
ultrasound and saw a multilocular hypoechoic mass sized 10cm (in diameter) in her left adnexa.
No free fluid in her abdomen and pelvis. What is your consideration in this case?
a. The incidence of adnexal masses in pregnancy is 1%
b. The incidence of ovarian cancers in pregnancy is between 1:1000
c. The most common type of bening ovarian cyst in pregnancy is a marute teratoma
d. The most common histopathological subtype for malignant ovarian tumor in pregnancy is
epithelial ovarian tumor
e. The resolution rate of adnexal masses in the second trimester of pregnancy is 60-70%

10. Regarding to question above. The patient does not believe that she has ovarian cyst during
pregnancy. She really concern about the possibility of malignant cyst. What will you inform
regarding this to her?
a. The most common mode of presentation of an adnexal mass is pain
b. The sensitivity of detection of ovarian cyst on clinical examination aline is less than 5%
c. The size of ovarian cyst that should prompt investigation for malignancy is 10 cm
d. The validated sensitivity and specificity of IOTA rules on US evaluation of an ovarian cyst is
sensitivity : 78%, specificity 87%
e. The sensitivity and specificity of MRI in the diagnosis of a malignancy is 100 and 94%
respectively

11. Regarding to question number 9. You have checked her Ca125 serum level and the result was
350µ/ml. you performed conservative surgical staging per laparatomy because her frozen
section result revealed malignancy cyst. After 1 week, the pathology result comes with serous
papillary carcinoma of left ovary. She is palnned for chemotherapy. What will you inform to her
regarding chemotherapy for ovarian cancer during pregnancy?
a. In a patient with ovarian cancer in pregnancy receiving chemotherapy the delivery should be
planned at completion chemotherapy
b. Chemotherapy use in pregnancy is generally considered safe beyond 20 weeks of gestation
c. CNS and neural tube complications occur during the week 8-12 week in pregnancy
d. This percentage of patients receiving chemotherapy in pregnancy who develop major
congenital malformations in 30-40%
e. Cardiovascular defect are common congenital malformations in platin based chemotherapy
regimens

12. A 40 y.o G4P3A0, 18 weeks pregnant, has had post coital bleeding for the last month. Speculum
examination showed a vaginal discharge and a 1 cm exophitic lesion on the anterior cervical lip.
She does not remember when she last time had a smear test. Which of the following statements
is true regarding her further management?
a. She should have a pap smear tes
performed b. Punch biopsy of the lesion is
indicated
c. Prescribe a dose of antibiotics and reassess the cervix when the vaginal discharge has settled
down
d. In view of the findings and the gestational age a cone biopsy should be offered as treatment
of the lesion
e. Assuming that the lesion has been biopsied and histology result showed cancer cells, MRI
without contrast should be considered as next step
A 28 y.o woman, G1 36 weeks of gestational age, went to your clinic to do routine antenatal care.
During ultrasound, the doctor told that she will be expecting bay boy with estimated fetal weight of
2500 grams, however amniotic fluid considered to be less than normal. Then you asked the patient to
drink minimal 2 L of water a day and get herself another ultrasound within 3 days to evaluate the
amniotic fluid

13. Oligohydramnios is defined as which of the following?


a. Amniotic fluid index < 5 cm
b. Single deepest pocket < 2 cm
c. Amniotic fluid index < 90 percentile
d. All of the above
e. None of the above

14. Amniotic fluid volume is a balance between production and resorption. What is the primary
mechanism of fluid resorption?
a. Fetal breathing
b. Fetal swallowing
c. Absorption across fetal skin
d. Absorption by fetal kidneys
e. Filtration by fetal kidneys

15. In a normal fetus at term, what is the daily volume of fetal urine that contributes to the
amount of amniotic fluid present?
a. 200ml
b. 250 ml
c. 500 ml
d. 750 ml
e. 1000 ml

Mrs A, 26 y.o, G1P0A0, according to her last LMP is 34 weeks pregnant, came for her first antenatal
care. She admit to have 20 kg weight gain during pregnancy with swelling ankles for the past 4
weeks. She never took any iron or vitamin supplementation. From the physical findings, BP 145/95
mmHg, HR 86 x/min, RR 20 x/min, BMI 35 kg/m2. Ultrasound examination confirmed twins in
breech presentation. Result from urinalysis were as follows color cloudy yellow, specific gravity
1.013, albumin +2, RBC 0-1, WBC 2-5, bacteria negative

16. What is the most likely diagnosis?


a. Acute fatty necrosis of the liver
b. Chronic hypertension
c. Preeclampsia
d. Renal disease
e. Pyelonephritis
17. Given the history of this patient, several more laboratory and diagnostics tests were
obtained. She was stable and the fetuses have reassuring heart rate tracings. Which of the
following do you expect to see in the test results?
a. Chest x ray to show decreased pulmonary vascular markings
b. Urine to show infection
c. Creatinin clearance to be increased above normal pregnancy levels
d. Serum uric acid to be increased
e. A decreased hematocrit

Mrs B, 37 years old came to your office at 32 weeks of gestation according to menstrual period. She
has no ultrasound examination before and did not get antenatal care routinely. The vital signs is
within normal limits. She ahs body mass index 19 kg/m 2 . During physical examination, the uterine
fundal is 22 cm. from ultrasound examination, the fetus has biometric values that correlate with 30
weeks fetus.

18. Which of the following is the next best step in managing this patient?
a. Antenatal care routinely for the next 2 weeks
b. Evaluate maternal status and comorbidities
c. Consider deliver the baby
d. Repeat sonography for fetal growth in 2 weeks
e. Doppler velocimetry evaluation every 3 days

19. According to algorithm for management of fetal growth restriction, you evaluate the Doppler
Velocimetry then find a reserved end diastolic flow and oligohydramnios. What is the
appropriate management at this time?
f. Regular fetal testing
g. Weekly evaluation of amniotic fluid
h. Consider corticosteroids for fetal lung maturation
i. Deliver the baby
j. Reevaluate middle cerebral arteries and ductus venosus

20. Fetal growth restriction is associated with all of the following , EXCEPT
a. Anti phospolipid Antibody Syndrome
b. Inherited thrombophilias
c. Infertility
d. Immunosuppressive drugs
e. Social deprivation

21. In addition to monitoring her TSH, what other additional testing should you perform during her
pregnancy?
a. Amniocentesis to determine if the fetus is affected by Graves Disease
b. A detailed fetal ultrasound at 18 to 20 weeks and again in the third trimester should be
performed given the increased risk of fetal goiter
c. Fetal echocardiogram to evaluate for cardiac abnormalities
d. Umbilical Doppler to monitor for placental dysfunction
e. MCA Doppler to monitor for fetal anemia

22. Regarding TSH in pregnancy, which statement is true?


a. Serum TSH levels in early pregnancy decline
b. TSH crosses the placenta
c. Pituitary TSH secretion is suppressed when hCG level is decreasing
d. It cannot be used for diagnosis of many thyroid disorder sin pregnancy
e. TSH level is static at midpregnancy

23. Management of hypothyroidism in pregnancy is:


a. Women after thyroidectomy may require higher dose of levothyroxine
b. TSH levels measured weekly
c. Thyroxine dose is adjusted by 100 mcg increments until TSH values become normal
d. Increased thyroxine requirements begin as early as 12 weeks
e. None above

Mrs E, 32 y.o referred from midwife with antepartum hemorrhage, She is G3P2 term pregnancy. On
examination her blood pressure is 160/100 mmHg, HR 100 bpm. She looks anemic, not icteric.
Obstetrical examinations reveal contraction 4-5 x/ min, FHR 179 bpm, head presentation 3/5. After
thorough examination it is concluded that there is a placental abruption with retropacental
hematoma size 6 x 5 cm. This patient planned to do cesarean section.

24. If the patient above during cesarean section found to be in atonic condition, which of th
following are not a pre requisite for performing b-lynch suture?
a. Patient in lithotomy position during operation
b. Bimanual compression reduce the amount of bleeding
c. Availability of suture material
d. Patient in stable hemodynamic condition
e. None of the above

25. If during operation the uterus is couvelaire but with good contraction, how would you
manage the condition?
a. Perform prophylactic b-lynch suture
b. Ascending uterine artery ligation
c. Hypogastric artery ligation
d. Sub total hysterectomy
e. Uterotonic and observation
26. Postoperative period is very crucial in this patient. Which of the following is not included as
a parameter needed to be evaluated In early warning system?
a. Blood pressure
b. Heart rate
c. Urine production
d. Central venous pressure
e. All of the above

27. What is the best next step?


a. Obstetrical examination consist of sterile speculum examination followed by vaginal
examination
b. Abdominal ultrasound
c. Transvaginal ultrasound
d. Stabilized patient, obtain two large bore iv, and start iv fluid bolus with 30 cc/kgBB
e. Emergency cesarean section

28. What is the most important laboratory test to the patient?


a. CBC
b. Urinalysis
c. PT, APTT
d. AST/ALT
e. Blood type and cross match

A 26 years old woman, G1P0A0 was admitted to emergency room because she lost her
consciousness around 1 hour ago, according to the husband, she is 36 weeks pregnant. She
performed antenatal care at scheduled time, and never missed one. Her husband said, she never
had severe nausea and omitting. Physical examination reveals BP 120/80 mHg, pulse rate 86 x/min,
RR 18 x/min, temperature
36.5 C. You notice there is an icteric sclera. Other physical examination was remarkable. Obstetrical
examination reveals no heart beat was detected.
Laboratory examination reveals CBC 10.2/29.9/8900/253.000, Ur/C 18/0,8, AST/ALT 458/878, RBG
32, urinalysis was within normal limit

29. What is the best next management in this case?


f. Abdominal ultrasound
g. Induction of labor
h. Emergency cesarean section
i. Whole blood transfusion
j. Injection of 40 % dextrose

30. All EXCEPT which of the following are clinical characteristics that increase for acute fatty liver of
pregnancy?
a. Nulliparity
b. Female fetus
c. Male fetus
d. Twin gestation
e. Third trimester
31. What is the underlying pathophysiology of intrahepatic cholestasis of pregnancy?
f. Acute hepatocellular destruction
g. Incomplete clearance of bile acids
h. Microvascular thrombus accumulation
i. Eosinophil infiltration of the liver
j. Hepatocellular injury

A 17-year old G1P0 woman present at 25 weeks gestation complaining of headache for the past 36
hours. She has had regular prenatal visit going back to her first prenatal visit at 8 weeks gestation. A
20 weeks ultrasound redated her pregnancy by 2 weeks as it was 15 days earlier than her LMP
dating. She has a BP of 155/104 mmHg

32. You review her medical record and determine that she does not have chronic hypertension. The
patient denies having RUQ pain but because of your high suspicion of severe preeclampsia you
order a CBC, liver enzymes, renal function test and a 24-hour urine protein collection. Her
laboratory test result reveal a normal platelet count and liver enzymes but a slightly elevated
creatinine and proteinuria off 550 mg in 24 hours. Her headache has resolved after a dose of
acetaminophen. What is the next best step in her management?
a. Give her a prescription for labetalol and have her follow up in clinic in 2 weeks
b. (a) plus bed rest
c. Hospitalization for futher evaluation and treatment
d. immediate delivery
e. Begin Induction of labor

33. Over the next 12 hours, her SBPs rise above 16o mmHg on several occasions, most notably to
174/102 mmHg hours after admission and to 168/96 mmHg 9 hours after admission. Her
headache does not return and she has no RUQ pain or visual symptoms. A set of repeat
laboratory test results are unchanged and by increasing her labetalol dose to 400 mg TID, her
BPs decrease to 140s- 150s/70-90 mmHg. She is also started on magnesium sulfate. What
change in physical or laboratory examination do you observe that would indicate delivery?
a. Another BP of 174/102 mmHg
b. Headache returning
c. Double vision
d. Platelets of 108
e. AST of 265

A 43 y.o woman, G4P3, at 37 weeks gestation presented in hospital with a ten day history of low
extremities edema, with idiopathic hypertension for 1 year. At presentation, she had a blood
pressure of 170/100 mmHg. Laboratory findings were normal except urinalysis (protein 2+). She was
diagnosed with superimposed severe preeclampsia. It was decided to deliver the fetus by means of
C section by
indication of transverse lie. Blood pressure measurement was 150/100 mmHg. She lost
consciousness for 30 seconds five hours after operation. The laboratory studies gave the following
results serum aspartate aminotransaminase (AST) 225 IU/L, serum alanine aminotransaminase (ALT)
140 IU/L, serum lactated dehidrogenase (LDH) 1017 IU/L. serum urea and creatinine were normal,
hemoglobin
10.6 mg/dL, platelet count 50 x 103 µ,mL. A brain computed tomography (CT) scan was performed on
patient which revealed the left frontal lobe lacunar infarction. The patient was transferred to
intensive care unit.

34. What is the most appropriate diagnosis


a. DIC
b. Acute Fatty Liver in pregnancy
c. HELLP syndrome
d. Severe puerperal infection
e. Thrombotic thrombositopenic purpura

35. What is the best management after, for this case?


a. Fresh frozen plasma and thrombocytes concentrate
b. Anti-platelets
c. Anti-oxydant
d. Corticosteroid
e. Magnesium sulfate

36. Twelve hours observation showed urine production was 100 ml.
a. Immediately giving diuretics bolus iv
b. Immediately giving diuretics maintained by syringe-pump
c. Check albumin level, giving diuretic justified after hypoalbuminemia condition had been
distinguished
d. Renal failure due to micro thrombopathy suspected, heparin provision could be considered
e. Immediately step on fluid rescucitation

A 33 year old woman, G1P0A0, came to the hospital with major complaint, watery leakage. She was
on her 33 weeks of gestational age. Data from medical record showed that she came previously a
week ago, complaining vaginal discharge. Vaginal swab has done.

37. In case above, what kind of examination should you performed for establishing diagnosis?
a. Vaginal examination
b. Inspeculo
c. Blood test
d. Ultrasound
e. Simple urine test

38. You found on Leopold 1,hard, round, with ballottement +, contraction was infrequent and weak.
What was your plan?
a. Went to labor induction
b. Immediate C-section
c. Tocolytic and corticosteroid provision
d. Performed ultrasound
e. Performed external version

Woman refer from primary health care due considerably low maternal body weight. She is on her 35
weeks of gestation, height 150 cm, weight 32 kg. She had previous history of severe hyperemesis.
Other medical problems are denied

39. Ultrasound examination showed that trans-cerebellar diametes was proper to gestational age,
abdominal circumference was lower than 2.5 percentile and amniotic fluid deepest pocket was
1.2 cm. what is the most likely diagnosis?
a. Growth restriction with oligohydramnion
b. Normal growth with oligohydramnion
c. Growth restriction with normal amniotic fluid
d. Normal growth with normal amniotic fluid
e. Need another examination for establishing diagnosis

40. Lack of baby movement had been felt for two days, fetal heart rate was 146 bpm. What
was your next step?
f. Termination of pregnancy
g. Giving oxygenation and left lateral position
h. Ensuring fetal well being by Manning criteria
i. Fetal lung maturation
j. Giving intravenous fluid restriction

41. Cardiotocography showed low variability with checkmark pattern and no desceleration. What
was your interpretation and the best management through?
f. Category one, continued for fetal lung maturation
g. Category two, intrauterine resuscitation for 24 hours and reevaluation after
h. Category two, went for Doppler velocimetry
i. Category three, went for Doppler velocimetry ultrasound exam
j. Category three, delivered baby

42. A 24 y.o G2P1 woman at 39 weeks and 3 days is seen in the clinic. She has been experiencing
more frequent contractions and thinks she might be in labor. Her last pregnancy ended with a
cesarean delivery after a stage 1 arrest. There was no evidence of cephalopelvic disproportion.
Earlier in the course of her current pregnancy she had desired a scheduled repeat cesarean, but
now that she might be in labor she would like to try and deliver vaginally. What woud be a
contra indication to a trial of labor after cesarean (TOLAC)?
a. Prior classical hysterotomy
b. Prior Kerr hysterotomy
c. Small for gestational agefetus
d. Oligohydramnios
e. GBS + mother

43. A 29 y.o G3P2 A0 presents to the emergency center with complaints of abdominal discomfort for
2 weeks. Her vital signs are BP 120/70mmHg, pulse 90 beats/min, temperature 36 C, respiratory
rate 18 breath/minute. A pregnancy test was positive and an ultrasound of the abdomen and
pelvis reveals a visible 16 weeks gestation located behind a normal appearing 10x6x5.5 cm
uterus. Both ovaries appear normal. No free fluid is noted. Which of the following is the most
likely cause of these findings?
a. Ectopic ovarian tissue
b. Fistula between the peritoneum and the uterine cavity
c. Primary peritoneal implantation of the fertilized ovum
d. Tubal abortion
e. Uterine rupture of prior cesarean section scar

44. Invasive cervical cancer identified only microscopically, invasion is limited to measured stromal
invasion with a maximum depth of 3 mm and no wider than 7 mm should be treated with
a. Observation
b. Tissue ablation
c. LEEP/LLETZ
d. Simple total hysterectomy
e. Radical hysterectomy

45. The majority of vulvar, vaginal and cervical cancer appear to have a common cause and
usually caused by:
a. High risk types of herpes simplex virus (HSV) infection
b. High risk types of human papilloma virus (HPV) infection
c. Increased exposure to endogenous estrogen
d. Increased exposure to exogenous estrogen
e. Chronic bacterial and parasitic infection

46. Childhood neoplastic ovarian masses most commonly originate from


a. Gonadal epithelium
b. Gonadal stroma
c. Sex cord
d. Germ cell
e. Metastatic disease

47. A patient returns for a postoperative checkup 2 weeks after a total abdominal hysterectomy for
fibroids. She is distressed because she is having continuous leakage of urine from the vagina. Her
leakage is essentially continuous and worsens with coughing, laughing, or movement. Given her
history and physical, you perform both a methylene blue dye test, which is negative and indigo
carmine test, which is positive. The most likely diagnosis is
a. Rectovaginal fistula
b. Uretrovagina fistula
c. Vesicovaginal fistula
d. Ureterovagina fistula
e. Impossible to distinguish

48. A 38 years old multigravid woman complaints of the painless loss of urine, beginning
immediately with coughing, laughing, lifting, or straining. Immediate cessation of the activity
stops the urine loss after only a few drops. The history is most suggestive of
a. Fistula
b. Stress incontinence
c. Urge incontinence
d. Urethral diverticulum
e. UTI

49. Select the incorrect regarding guidelines for Nuchal Translucency (NT) measurement is
a. The margins of NT edges must be clear enough for proper caliper placement and the fetus
is not necessary in the midsagital plane
b. The image must be magnified so that is filled by the fatal head, neck and upper thorax
c. The fetal neck must be in a neutral position, not flexed and not hyperextended
d. The amnion must be seen as separate from NT line, and the calipers must be placed on the
inner borders of the nuchal space with none of the horizontal crossbar itself protruding
into the space
e. The calipers must be placed perpendicular to the long axis of the fetus and
the measurement must be obtained at the widest space of the NT

50. Choose the wrong statement about the Down Syndrome


a. An increased NT thickness itself is not a fetal abnormality
b. Nuchal translucency is a marker that confers increased risk of fetal abnormality
c. Approximately one third of the fetuses with increased nuchal translucency thickens
will have a chromosome abnormality, nearly half of which are Down Syndrome
d. Down syndrome is caused by an autosomal trisomy, is the most common non lethal trisomy
e. All cases of Down Syndrome is caused by trisomy 21

51. A 32 years old presets for an infertility work up. She and her partner have been trying to
conceive for 2 years without success. She has regular menstruation, though she mentions she
has severe cramping during her cycles. She also notes she experiences pelvic pain during sex. On
examination, she is a thin,well developed woman. She is afebrile, and she experiences a great
deal of pain during the pelvic examination. You do not note discharge on examination. Which of
the the following tests is required for diagnosis of the patient infertility?
a. Ultrasound
b. Beta hCG level
c. Pap smear
d. Laparoscopy
e. Hysterosalpingogram

52. Mrs XY is a primigravida who is 34 weeks pregnant. Her last two serial scans have shown an
small gestational age growing on the 9 th centile. Her last scan shows positive end diastolic flow
with a normal PI. She reports having good fetal movements. How should further fetal
surveillance be undertaken?
a. Fortnightly umbilical artery Doppler
b. Twice weekly CTG
c. Twice weekly umbilical artery Doppler
d. Weekly CTG
e. Weekly umbilical artery Doppler + CTG

A 25 y.o woman in her first pregnancy is noted to have prolonged first and second stage of labor.
She was induced at 38 weeks pregnancy. The baby was delivered by forceps. After delivery the
placenta she is noted to have heavy vaginal bleeding . Abdominal examination demonstrated a
relaxed uterus.
53. What is the most likely cause of bleeding?
a. Uterine atony
b. Uterine rupture
c. Retained placenta
d. Genital tract laceration
e. DIC

54. What should we do if the fundus not firm after placenta delivery?
a. Methylergonevine (Methergine)
b. Carboprost (Hemabate, PF 2- alpha)
c. Fundal massage
d. Misoprostol
e. Dinoprostone prostaglandine E2

55. Which of the following medications would be contraindicated in the treatment for this patient?
a. Methylergonevine (Methergine)
b. Carboprost (Hemabate, PF 2- alpha)
c. Intramuscular Pitocin
d. Misoprostol (PgEs)
e. Dinoprostone prostaglandine E2
A 30 y.o multiparous woman has rapid delivery soon after arriving in emergency room. After
delivery, the placenta she is noted to have heavy vaginal bleeding. Help has been summoned.
Abdominal examination demonstrates the fundus was soft.
57. What is the most appropriate next step?
a. Intravenous access for fluid resuscitation
b. Uterine packing
c. Balloon tamponade
d. Hecting laceration
e. Misoprostol administration

58. What is the most likely cause of bleeding


a. Uterine atony
b. Uterine rupture
c. Retained placenta
d. Genital tract laceration
e. DIC

59. After use of a 20 units of oxytocin in 1000 ml of cystaloid solution to increase the tone of her
uterus, stop the bleeding, however you can continue to notice a massive bleeding from the
vagina. What is the most appropriate next step in the evaluation of this patients’s bleeding?
a. Perform a bedside ultrasound for retained product of conception
b. Perform a bedside ultrasound to look for blood in the abdomen significant for
uterine rupture
c. Perform a manual exploration of the uterine fundus and exploration for retained clots
or product
d. Examine the perineum and vaginal for laceration during delivery
e. Consult interventional radiology for uterine artery exploration

A 32 years old woman comes to your clinics due to shortness of breath, that worsen since 2 days
ago. On history taking, she told you that she had ever diagnosed of having significant mitral stenosis.
She is 33 weeks pregnant. The fetus is size-date appropriate. She has had a recent echocardiography
showing ejection fraction of 54% with moderate-severe pulmonary hypertension

60. What is the best management for this patient currently?


a. Perform emergency C section
b. Lung maturation and C section
c. Conservative management until term pregnancy
d. Second stage acceleration
e. Induction of labor

61. What is the most common cause of heart failure during pregnancy and the puerperium?
a. Chronic hypertension with severe preeclampsia
b. Viral myocarditis
c. Obesity
d. Valvular heart disease
e. Pulmonary Artery Hypertension

62. For patients with congenital heart disease, what is the most common adverse cardiovascular
event encountered in pregnancy?
a. Heart failure
b. Arrhytmia
c. Thromboembolic event
d. Cerebrovascular hemorrhage
e. Heart axis changes

63. A 55 years old woman presents to your office for consultation regarding her symptoms of
menopause. She stopped having periods 8 months ago and is having severe hot flushes. The hot
flushes are a causing her considerable stress. What should you tell her regarding the
psychological symptoms of the climacteric?
a. They are not related to her changing levels of estrogen and progesterone
b. They commonly include insomnia, irritability frustration and malaise
c. They are related to a drop in gonadotropin levels
d. They are not affected by environmental factors
e. They are primarily a reaction to the cessation of menstrual flow

64. A 62 years old woman present for annual examination. Her last spontaneous menstrual period
was 9 years ago and she has been reluctant to use postmenopausal hormone replacement
because of a strong family history of breast cancer. She now complains of diminished interest in
sexual activity. Which of the following is the most likely cause of her complaint?
a. Decreased vaginal length
b. Decreased ovarian function
c. Alienation from her partner
d. Untreatable sexual dysfunction
e. physiologic anorgasmia

A 49 year old woman experiences irregular vaginal bleeding for 3 months duration. You performed
endometrial biopsy, which obtains copious tissue with a velvety, lobulated texture. The pathologist
reports shows proliferation of glandular and stromal elements with dilated endometrial glands,
consistent with simple hyperplasia. Cytologic atypia is absent.
65. Which of the following is the best way to advise the patient?
a. She should be treated to estrogen and progestin hormone therapy
b. The tissue will progress to cancer in approximately 10 % of cases
c. The tissue may be weakly premalignant and progress to cancer in approximately 1 % of
cases
d. She requires hysterectomy
e. No further therapy is needed
66. Which of the following factors is protective against endometrial hyperplasia?
a. Obesity
b. Tamoxifen
c. Oral contraceptive pills (OCP)
d. Early menarche or late menopause
e. Unopposed exogenous estrogen therapy

You see a 16 y.o female who presented with primary amenorrhea. Breast development was noted at
13 years but there has been no increase in breast size. Pubic and axillary hairs were noted within
one year of referral. An outside ultrasound showed no uterus or ovaries. Physical examination
reveals a normal vaginal introitus with hymen present. Breast are Tanner 3, but seem to be more
fatty than mammary tissue.

67. Lab test were significant for absent estradiol, elevated gonadotropins and mildly elevated
DHEAS with normal testosterone. A karyotype was requested by the endocrinologist and was
found to be 46 XY (SRY gene +). What is the working diagnosis following her initial work up?
a. Androgen insensitivity
b. Disorders of testicular development
c. Mullerian agenesis
d. A and B
e. B and C

68. The MRI showed an infantile uterus with no discernible gonads. During a laparoscopic
evaluation, two dysplastic gonads attached to small fallopian tubes and a rudimentary uterus
was visualized. The final diagnosis is:
a. Swyer syndrome
b. Androgen insensitivity
c. Mullerian agenesis
d. Partial gonadal dysgenesis
e. Testicular regression syndrome

69. What type of tumor is a significant for such condition?


a. Mature teratoma
b. Leydig cell tumors
c. Germ cell tumors
d. Stromal cell tumor
e. Granulose cell tumors

Miss 25 yo P3 comes to Gynecologic outpatient clinic with cytology result low grade SIL.She curious
about the result since her last cytology result was normal 3 years ago. She has no complaint
recently. She began sexually active since 10 years ago and has six partners. She smokes 10
cigarettes per day
since 4 years ago. Her mother was diagnosed for cervical cancer at 44 yars old and just died 3
months ago, her child is at 6 years old now.

70. What are patient risk factor for CIN?


a. Her smoking habits
b. Onset of sexual activity
c. Six sex partners
d. Early child nearing
e. All mention above

71. What is next proper management for patient?


a. IVA test
b. Colposcopy
c. No procedure need in her treatment
d. HPV DNA test
e. Endocervical curettage

72. The result showing a condylomatous acetowhite lesion with punctuation and atypical vessels.
Biopsy result confirms CIN 1 with HPV DNA test positive. What do you suggest for patient?
a. LEEP procedure
b. Reevaluation of HPV DNA
c. Cold knife conization
d. Repeat cytology in 12 months
e. Repeat cytology in 6 months

A 19 years old woman came to clinic with chief complaint of irregular menstruation. She had
menstruation once every every three months. She is also obese (BMI 32). Physical examination
reveals she has hirsutism (Ferriman Gallwey score 9), other physical examination within normal
limit. Gynecologic examination within normal limit.

73. Which of the following criteria diagnosis of polycystic ovarian syndrome (PCOS) is not part of
the Rotterdam Criteria?
a. Oligo/ anovulation
b. Appearance of polycystic ovaries by gynecologic ultrasound
c. Excess androgen activity
d. Ferriman Gallwey score > 8
e. All statement is true

74. In PCOS, increased testosterone production from the ovaries is secondary to stimulation by
which of the following hormones?
a. Inhibin
b. Estradiol
c. Prolactin
d. Follicle stimulation hormone (FSH)
e. Luteinizing hormone (LH)

A 55 years old nulliparous woman who underwent menopause at age 50 years complaint of a 1
month history of vaginal bleeding and smells. Her medical history reveals she has hypertension and
controlled with antihypertensive agent, and also she has diabetes mellitus controlled with an oral
hypoglycemic agent. On examination , she weights 89 kg and 152 cm tall. Her blood pressure is
150/90 mmHg. Heart and lung examination are normal. The abdomen is obese and no masses are
palpated. The external genitalia appear normal, and the uterus seems to be enlarged, without
adnexal masses palpated.

75. What is the probable diagnosis of this patient?


a. Cervical cancer
b. Hyperplasia endometrium
c. Endometrial carcinoma
d. Uterine fibroids
e. Ovarian cancer

76. Which of the following does not increase woman’s risk of developing endometrial cancer?
a. Obesity
b. Smoking
c. Diabetes mellitus
d. Tamoxifen
e. Unopposed estrogen

A 40 y.o G2P0 woman at 7 weeks GA by LMP present for her first prenatal visit. She spontaneously
conceid after 18 months of trying. She is ecited about the pregnancy. But at the same time is
concerned about potential risks for herself as well as the baby because of her age. Her husband
is 52 y.o, healthy and has fathered two children from a prior marriage. Thw week prior to the
visit, she experienced spotting that lasted 3 days and then resolved. Currently she has no past
medical or surgical history except for a miscarriage 3 years ago. She has regular periods every 30
days

77. Three second-trimester analyze level abnormalities in a pregnancy at increased risk for Down
Syndrome
a. Decreased MSAFP, increased unconjugated estriol, increases inhibin, increased beta hCG
b. Decreased MSAFP, decreased unconjugated estriol, increased inhibin, increased beta hCG
c. Increased MSAFP, increased unconjugated estriol, decreased inhibin, decreased beta hCG
d. Decreased MSAFP, decreased unconjugated estriol, decreased inhibin, increased beta hCG
e. Decreased hemoglobin

78. Patient undergo first trimester screening, which return with a risk for Down syndrome of 1 in
1,214 and risk of trisomy 18 of 1 in 987. At 18 weeks she gets a quad screen, and her estriol,
beta
hCG, and AFP were all low. She has an ultrasound which shows a fetus consistent with 16 weeks
size, increased amniotic fluid, clubfoot, omphalocele, choroid plexus cyst and possible heart
defect. On the basis of the patients history and data provide what is the most likely diagnosis?
a. Trisomy 21
b. Trisomy 18
c. Trisomy 13
d. Turner syndrome
e. Klinefelter syndrome

A 41-year-old primigravida with DCDA twins following a successful IVF treatment presents with a
booking BMI of 42 at 13 weeks of pregnancy and is seen in the antenatal clinic. What is the
most appropriate medication that you would recommend ?
A. Folic acid 5 mg/day
B. Vitamin D 10 mcg/day
C. Aspirin 75 mg/dya
D. Metformin 500 mg TDS
E. Omega 3 fatly acid

80. A 38-year-old woman presents to your emergency department with complaints of irregular
vaginal bleeding for the past 1 year. Upon evaluation of her surgical history, she fells you she
had a procedure outside of the country for an abnormal pregnancy about 1 year ago. She has no
follow- up since then. On review of systems. She tells you she has been coughing un blood for
the past 1 week. Physical examination reveals old blood in the vaginal vault. You lab data are
significant for a serum -hCG of 112.000 ml U/ml. the most likely diagnosis is :
A. Complete molar pregnancy
B. Metastatic persistent GTD
C. Placental site trophoblastic tumor
D. Dysfunctional uterine bleeding
E. Partial molar pregnancy

81. A 21-year-old woman undergoes hysteroscopy an currtage for persistent irregular uterine
bleeding after her term vaginal delivery 8 months ago. Final pathologic lindings form the D&C
are consistent with chonocarcinoma. Baseline serum -hCG is 200.2000 ml uteri. When of the
following is not currently indicated ?
A. Imaging for distant melastastic lesions
B. Surgical intervention (hysterectomy)
C. Chemotherapy
D. Clos surveillce of serum -hCG
E. Reliable contraception

82. A postmenopausal woman with symptomatic vaginal vault prolapsed elects to process with
surgical correction of the descensus. You determine that she is an appropriate candidate for an
uterosacral ligament suspension. After completion of the surgery. Successful anatomic
restoration is noted. Before you conclude the surgery, what additional procedure is indicated?
A. Hysteroscopy
B. Defecography
C. Cystoscopy
D. Episiotomy
E. Anterior colporrhaphy

83. A 37-year-old female G5P5 has come to your office to discuss long term contraception A
thorough history and office examination reveal a healthy, thin female, in no acute distress. She
reports seasonal allergies and has undergone one cesarean section in the past as well as four
spontaneous vaginal deliveries and no additional surgeries. She has never had an STI and is a
nonsmoker. Her Pap Smears and HPV screens have all been normal and are up to date. She asks
you to explain which methods will be effective for a long period of time, as she has trouble
remembering to take a pill daily with five small children at home you inform her that many
methods can be considered “long term” except which of the following?
A. 1,5 mg Levonorgestrel pill
B. Etonogestrel rod (Nexplanon)
C. Copper IUD (ParaGard)
D. Levonorgestrel IUD (Mirena)
E. Tubal Ligation/Occlusion

SAMPUN NGGIH….MATUR NEMBAH NUWUN..

UNAS MARET 2017

A 28 yo woman G1 36 weeks of gestasional age went to your clinic to do routine ANC. During
ultrasound the doctor told that she will be expecting baby boy with EFW 2500 gram, however
amniotic fluid considered to be less than normal. Then u asked the patient to drink minimal
2L of water a day and get herself another ultrasound within 3 days to evaluate the amniotic
fluid

1. Oligohydramnios is defined as which of the following ?


a. Amniotic fluid index < 5 cm
b. SDP < 2 cm
th
c. AFI < 90 percentile
d. All of the above
e. None of the above

2. Amniotic fuid volume is a balance between production and resorption. What is the
primary mechanism of fluid resorption?
a. Fetal breathing
b. Fetal swallowing
c. Absorption across fetal skin
d. Absorption by fetal kidney
e. Filtration by fetal kidney

3. In a normal fetus at term what is the daily volume of fetal urine that contributes to the
amount of AFI present?
a. 200mL
b. 250mL
c. 500mL
d. 750mL
e. 1000mL

rd
A patient came with chief complaint of spotting two hours ago. Her LMP was 3 January 2017
and she had not had any pregnancy test. She appears well and complaint of slight cramps on
her lower abdomen. On examination you found that she is normotensive and there is no
tachycardia, her general status appears normal. During gynecological exam her external
ostium is closed, her uterus is enlarged slighty with no active bleeding. By trans vaginal
ultrasound examination you found gestasional sac intra uterine with crown rump length
appropriate for 8 weights of gestation with FHR 186 bpm.

4. What is the best management for this patient?


a. Bed rest and progestogen
b. Bed rest and tocolitic
c. Long maturation
d. Bed rest only
e. Progesterone only

5. What is the following is least likely to be associated with preterm delivery?


a. Incompetent cervix
b. Hydramnios
c. Uterine fibroi
d. Abdominal pregnancy
e. Hydrops fetalis
6. Which one of the following is most likely to be contraindication for tocolysis at 28
weeks gestation
a. Suspected abruption
b. Group B Streptococcal bacteriuria
c. Recent laparotomy for appendicitis
d. Uterine fibroid
e. Preterm premature rupture of the membrane

7. Each of the following is a side effect of terbutaline, except for :


a. Fetal tachycardia
b. Hyperglycemia
c. Hyperkalemia
d. Hypokalemia
e. Increase pulse pressure

st
Mrs. A 26 years old G1P0A0 according to the her LMP is 34 weeks pregnant, came for her 1
ANC. She admit tohave 20 Kg weight gain during pregnancy with swelling ankles for the past 4
weeks. She never too any iron or vitamin supplementation. For the physical finding 145/95
mmHg. HRb86 x/m, RR 20x/m, BMI 35 kg/m2. Ultrasound examination confirm twins in breech
presentation. From urinalysis were as follows color cloudy yellow spesifik Gravity 1.013,
albumin 2+ RBC 0 -1, WBC 2-5, bacteria negatif

8. What is the most likely diagnosis?


a. Acute fatty necrosis
b. Chronic hypertension
c. Precclampsia
d. Renal disesase
e. Pyelonepriis
9. Given the history of this patient several more laboratory and diagnostic test were
obtained. She was stable and the fetuses have reassuring heart rate tracing. Which of
the following do you expect to see in the rest result?
a. Chest x ray to show decreased pulmonary vascular marking
b. Urine to show infection
c. Creatinin clearance to be increased above normal pregnancy level
d. Serum uric acid to be increased
e. A decreased hematocrit

Mrs B 37 yo came to you office at 32 minggu weeks of gestation accoriding to her last
menstrual period. She has no ultrasound examination before and didn”t get ANC routinely.
2
The vital sign is within normal limit. She has BMI 19 kg/m . During physical examination the
uterine fundal height is 22 cm. From ultrasound examination the fetus has biometric value tha
correlate with 30 weeks fetus

10. Which of the f is the next best step in managing this patient?
a. ANC routinely for the next 2 weeks
b. Evaluate maternal status and comorbidities
c. Consider deliver the baby
d. Repeat sonography for fetal growth in 2 weeks
e. Doppler velocimetry evaluation every 3 days

11. According to the algorithm for management of fetal growth restriction you evaluate
the Doppler velocimetry then find reversed end diastolic flow and oligohydramnios.
What is the appropriate management at this time?
a. Regular fetal testing
b. Weekly evaluation of amniotic fluid
c. Consider corticostetroid for lung maturation
d. Delivery the baby
e. Reevaluate middle cerebral arteries and ductus venosus

12. Fetal growth restriction is associated with all except which of the following?
a. APS antibody
b. Inherited trombhophilias
c. Infertility
d. Immunosuresive drugs
e. Social depriviation

A 29 yo G3P0A2 woman at 12 weeks gestation came to your office for prenatal visit. She has
st
history of miscarriage in the 1 trimester. She admitted fatigue arthralgia, and
photosensitivity.Her BP was 130.80 mmHg. On physical examination you found the malar rash
and anemic conjunctiva. Laboratory result from urinalysis revealed proteinuria 3+ and cellular
cast. Hemoglobin level was 7,2 gr/dL with peripheral blood smear showing spherocytes and
3 3
reticulocytosis, platelet count 98.000/mm and leucocyte cunt 2600/mm

13. ..Which of the following specific laboratory test to establish the diagnosis?
a. Antinuclear (ANA) nd anti cardiolipin antibodies (ACA)
b. High titer anti double stranded (ds) – DNA and anti SM (Smith)
c. IgG and IgM ß2 glicoprotein
d. Anti Ro (SS-A) and anti La (SS-B)
e. Antiplatelet and anti erythrocyte
14. According to the disorder above the pregnancy outcome is best in those woman in
which ?
a. Does not develop superimposed preeclampsia
b. No evidence of antiphospholipid syndrome
c. No nephritis manifest by proteinuria and renal disfunction
d. The disease has been quiescent for at least 6 months before conception
e. All of the above

15. ..The exacerbation flare is becoming the risk of the disorder. If there is no indication to
terminate the pregnancy the following treatment are true, EXCEPT :
a. Immunosupresive agent such as azathioprine can be used to control the active
disease with daily oral dose 2-3 mg/kg
b. Mycophenolate mofetil and MTX should be avoided
c . Antimalarias (hydrochloroquin) can’t be used due to association
malform
d. High dose glucocorticoid can be the alternative for severe disease by using
methylprednisolone 1000mg intravenously
e. Cyclopospamide can be used after 12 weeks gestation

You see a patient in you office who had a thyroidectomy from graves disease. She is now
hypothyroid on hormone replacement. You monitor her TSH and keep it between 0,5 and 2,5.
You increased her thyroid hormone replacement each semester and her tsh remains around
2.0 during the entire pregnancy.
16. In addition to monitoring her TSH, what other additional testing should you perform
during her pregnancy?
a. Amniocentesis to determine if the fetus is affected by Graves disease
b. A detailed fetal ultrasound at 18 to 20 weeks and again in the third
be performed given the increased risk of the The fetal goiter
c. Fetal echocardiogram to evaluate for cardiac abnormalities
d. Umbilical Doppler to monitor for placental dysfunction
e. MCA Doppler to monitor for fetal anemia

17. Regarding TSH in pregnancy, which statement is true?


a. . Serum TSH levels in early pregnancy decline
b. TSH crosses the placenta
c. Pituitary TSH Secretion is suppress ran HCG serum level is decreasing
d. It cannot be used for diagnosis of many thyroid disorder in pregnancy
e. TSH level is static at mid pregnancy

18. Management of hypothyroidism in pregnancy is :


f. Woman after thyroidectomy may require doses of levothyroxine
g. TSH levels measured weekly
h. Thyrosine dose is adjusted by 100mcg increment until TSH values become normal
i. Increased thyroxine requirement begin as early as 22 week
j. None above

Mrs. E 32 yo referred from midwife with antepartum hemorrhage. She is G3P2 term
pregnancy. On examination her blood pressure is 160/100 mmHg, HR 100 bpm. She looks
anemic not icteric. Obstetrical examination reveal contraction 4-5x/10minutes, FHR 170 bpm,
head presentation 3/5. After thorough examination it is concluded that there is a placenta
abruption with retroplacenta hematom size 6x5 cm. This patient planned to do cesarean
section
19. If the patient above during cesarean section found to be in atonic condition, which of the
following are not prerequisite for performing b lynch suture?
a. Patient in lithotomy position during operation
b. Bimanual compression reduce the amount of bleeding
c. Availability of suture material
d. Patient in stable hemodynamic condition
e. None of the above

20. If during operation the uterus is couvelaire but with good contraction, how would you
manage this condition?
a. Perform prophylactic b lynch suture
b. Ascending uterine arteri ligation
c. Hypogastric artery ligation
d. Sub total hysterectomy
e. Uterotonic and observation

21. Post operative period is very crucial in this patient. Which of the following is not included as
a parameter nedded to be evaluated in early warning system
a. Blood pressure
b. Heart rate
c. Urine production
d. Central venous pressure
e. All of the above

A 22 yo G1P0at 25 weeks gestation with HIV infection on ARV therapy has purified protein
derivative (PPD) has performed. Two days later the induration at the test site is noted to be 7
mm. She has no symptomsTBC and her chest radiography shows normal finding.

22. Your plan is which of the following


k. Treat post partum because the chest radiograph is normal
l. Star treatment now isoniazid because she has an 8 % annual risk for active disease
m. Start treatment now with 4 drugs therapy because she has an 8 % annual risk for
active disesase
n. No treatment is required for a reading of 7 mm
o. All of the above

23. You are taking care for a pregnant woman who has complaint of cough for a month ,
intermittent fever and night sweat, You are concerned about TB. Which of the following
statements is most accurate regarding TB in pregnancy?
p. All woman with those symptoms should have a chest x ray in pregnancy
q. If patient is treated adequately during pregnancy, TB generally has effect in mother
for child
r. . Mantoux test have unusually high false positive rate in pregnancy
s. The best single drug for therapy of TB in pregnancy is streptomycin
t. 2 drug anti TB therapy is pregnancy is contraindicated

A 21 years old at 37 weeks of gestasional age was referred to the hospital due to profuse
vaginal bleeding. Previously patient felt severe abdominal pain around 2 hours ago. On physical
examination patient was unstable and in delirium stage. Her initial BP was 130/80 mmHg and
pulse rate was 115 beat/minute. Her abdomen feels rigid

24. What is the best next step?


a. Obstetrical examination consist of sterile speculum followed by vaginal examination
b. Abdominal ultrasound
c. Transvaginal ultrasound
d. Stabilized patient, obtain two large bore IV and start fluid boluswith 30 cc/kgBB
e. Emergency SC

25. What is the most important laboratory test to do in this patient?


a. CBC
b. Urinalysis
c. PT, apttAST/ ALT
d. Blood type and cross match

26. These factors are precipitating or predisposing factors of placental abruption, EXCEPT?
u. Hypertension
v. Nullipara
w. Previous placental abruption
x. Abdominal trauma
y. Short umbilical cord
A 26 yo woman G1P0A0 was admitted to emergency room because she lost her consciousness
around 1 hour ago. According to her husband she is 36 weeks pregnant. She performed
antenatal care at scheduled time and never missed one. Her husband said, she never had any
hypertension or any other disease before. Three days prior hospitalization she had severe
nausea and vomiting. Physical examination reveals BP 120/80 mmHg, pulse rate 87x/mnt, RR
18x/mnt, temperature 36,5 C. You notice there is an icteric sclera. Other physical examination
was remarkable. Obstetrical examination reveals no fatal heart beat was detected. Laboratory
examination reveals CBC 10,2/ 29,9/8900/263000 Ur/Cr 18/0,8 AST/ALT 458/878, RBG 32,
urinalyisis was within normal limit

27. What is the best next management in this case?


a. Abdominal ultrasound
b. Induction of labour
c. Emergency SC
d. Whole blood transfusion
e. Injection of 40 % dextrose

28. All EXCEPT which of the following are clinical characteristic that increase the risk for
acute fatty liver of pregnancy
a. Nulliparity
b. Female fetus
c. Male fetus
d. Twin gestation
e. Third trimester

29. What is the underlying pathophysiology of intrahepatic cholestasis of pregnancy?


a. Acute hepato cellular destruction
b. Incomplete clearance of bile acid
c. Microvascular thrombus accumulation
d. Eosinophil infiltration of the liver
e. Hepatocellular injury

Mrs P, 30 yo G2P1A0 30 weeks gestation came to your out patient clinic for routine prenatal
visit. She complained of dizzy and fatique lately. No sign of contraction of fetal kick count was
12x/10 hours. Her BP 110/70 mmHg and pulse rate 92x/mnt. You asked for CBC panel and the
result was Hb 10gr/dL, Ht 30,5 %, Leucocyte 11.700, tromb 237.000/uL MCV 80fL, MCH 26,
6 st
MCHC 31gr/dL, RBC 5,3 10 /microliter. Her 1 child was delivered in 38 weeks gestation with
birth weight 2500 gram.
30. What is next management for this patient?
rd
a. Perform 3 trimester obstetrical ultrasonography screening
b. Asked for complete iron studies (FEP, serum iron, TIBC, ferritin)
c. Administer iron supplementation
d. Asked for peripheral blood smear
e. Perform bone marrow examination

31. Your diagnostic examination result showing normal FEP, plasma iron 95 ug/dL, TIBC
350ug/dL, and ferritin 50 ng/dL. What will be possible diagnosis
a. Iron and deficiency anemia
b. Sideroblastic anemia
c. Α thalassaemia
d. Sickle cell anemia
e. Anemia of chronic disease

32. What is the cause of this condition?


a. Deletion of one or all α genes on chromosomes 16
b. Deletion of one or all α genes on chromosomes 11
c. Deletion of one or all ᵦ genes on chromosomes 11
d. Deletion of one or all ᵦ genes on chromosomes 16
e. Deletion of one or all α genes on chromosomes 18

A 17 year old G1P0 woman present at 25 weeks gestation complaining of headache for the
st
past 36 hours. She has had regular prenatal visit going back to her 1 prenatal visit at 8 weeks
gestation. A 20 weeks ultrasound redated her pregnancy by two weeks as it was 15 days earlier
than her LMP dating. She has a BP of 155/104 mmHg.

33. You review her medical record and determine that she does not have chronic hypertension.
The patient denies having RUQ pain, but because of your high suspicion of severe
preeclampsia, you order a CBC, liver enzyme, renal function test, and a 24 hours urine
protein collection. Her laboratory test result reveal a normal platelet count and liver but a
slightly elevated creatinine and proteinuria of 550mg in 24 hours. Her headache has
resolved after a dose of acetaminophen. What is the next best step in her management.
a. Give her a prescription for labetolol and have her follow up in clinic in 2 weeks
b. (a) plus bed rest
c. Hospitalization for further evaluation and treatment
d. Immediate delivery
e. Begin induction of labor

34. Over the next 12 hours, her SBP rise above 160mmHg on several occasion, most notably
to 172/ 102 mmHg 2 hours after admission and to 168/96 mmHg 9 hours after
admission. Her head ache does not return and she had no RUQ or visual symptoms. A
set of repeat laboratory test result are unchanged and by increasing her labetolol dose
to 400 mg TID, her BPs decrease to 140s-150s/70-90 mmHg. She is also started on
magnesium sulfate. What change in physical or laboratory examination do you observe
that would indicate delivery?
a. Another BP of 174/102 mmHg
b. Headache returning
c. Double vision
d. Platelets of 108
e. AST of 265

A 43 years old woman G4P3 at 37 weeks gestation presented in hospital with a ten day history
of low extrimities edema, with idiopatic hypertension, for 1 year. At presentation she had a
blood pressure of 170/100 mmHg. Laboratory finding was normal except urinalysis (protein
2+). She was diagnosed with superimposed severe precclampsia. It was decided to deliver the
fetus by mean of a C section by indication of tranverse lie. Blood pressure measurement was
150/100 mmHg. She lost consciousness for 30 second five hours after operation. The
laboratory studies give the following result AST 225, ALT 140, LDH 1017, serum urea and
creatini were normal, Hb 10,6, plateet 50.000, CT scan was performed on patient which
revealed the left frontal lobe lacunar infarction. The patient was transferred to ICU
35. What is the most apporopriate diagnose?
a. DIC
b. Acute fatty liver in pregnancy
c. HELLP syndrome
d. Severe purpural infection
e. Trombotic trombositopenic purpura

36. What is the best management after, for this case?


a. Fresh-frozen plasma and thrombocytes concentrates
b. Anti-platelets
c. Anti-oxidant
d. Corticosteroid
e. Magnesium sulfate

37. Twelve hours observation showed urine production was 100 ml.
a. Immediately giving diuretics bolus IV
b. Immediately giving diuretics maintained by syringe pump
c. Check albumin level, giving diuretic justified after hypoalbuminemia condition had been
distinguished
d. Renal failure due to micro thrombopathy suspected, heparin provision could be
considered
e. Immediately step on fluid rescucitation
A 33 year old woman G1P0A0, came to hospital with major complaint, watery leakage. She
was on her 33 weeks of gestational age. Data from medical record showed that she came
previously a week ago, complaining vaginal discharge. Vaginal swab has done.
38. In case above, what kind of examination should you performed for eshtablishing diagnosis.
a. Vaginal examination
b. Inspeculo
c. Blood test
d. Ultrasound
e. Simple urine test

39. You found leopold 1, hard, round with ballottement (+). Contraction was infrequent and
weak. What was your next plan.
a. Went for labour induction
b. Immediate C-section
c. Tocolytic and costicosteroid provision
d. Performed ultrasound
e. Performed external version

Woman refer from primary health care due considerably low maternal body weight. She is on
her 35 weeks of gestation, height 150 cm weight 32 kg. She had previous history of severe
hyperemesis. Other medical problems were denied.

40. Ultrasound examination showed that trans-cerebellar diameter was proper to gestational
age, abdominal circumference was lower than 2.5 centile and amniotic fluid deepest pocket
was 1.2 cm.What is the most likely diagnosis?
a. Growth restriction with olygohydramnion
b. Normal growth with olygohydramnion
c. Growth restriction with normal amniotic fluid
d. Normal growth with normal amniotic fluid
e. Need another examination for establishing diagnosis

41. Lack of baby movement had been felt for two days, fetal heart rate was 146 bpm. What
was your next step?
a. Termination of pregnancy
b. Giving oxygenation and left lateral position
c. Ensuring fetal well-being by manning criteria
d. Fetal lung maturation
e. Giving intravenous fluid rehydration

42. Cardiotocography, showed low variability with checkmark pattern and no deceleration.
What was your interpretation and the best management through?
a. Category one, continued for fetal lung maturation
b. Category two, intrauterine resuscitation for 24 hours and reevaluation after
c. Category two, went for Doppler velocymetry
d. Category three, went for Doppler velocimetry, ultrasound exam
e. Category three, delivered the baby

A 24 year old G2P1 woman at 39 weeks and 3 days is seen in clinic. She has been experiencing
more frequent congtractions and thinks she might be in labour. Her last pregnancy ended with
a caesarean delivery after stage 1 arrest. There bwas no evidence of cephalopelvic
disproportion. Earlier in the course of her current pregnancy she had desired a scheduled
repeat caesarean, but now she might be in labour she would like to try and delivery vaginally.

43. What would be a contraindication to a trial of labour after caesarean (TOLAC)?


a. Prior classical hysterotomy
b. Prior Kerr hysterotomy
c. Small for gestational age fetus
d. Olygihydramnions
e. GBS + mother

44. A 33 year old G8P6A1 woman with fundal height of 39 cm presents for initial prenatal visit
in your clinic. She is currently an inmate at a local jail. She is imprisoned on charges of
marijuana possession. She has had no prenatal care. She is unsure of her last period but reports
that started feeling the baby move approximately 5 months ago. She denies vaginal bleeding,
leaking fluid, or contractions during the pregnancy. Ultrasound performed in your clinic shows
gestational age tobe approximately 40 weeks 3 days. What is the most common cause of
diagnosis of postterm pregnancy?

a. Inaccurate dating
b. Fetal anomaly
c. Delayed presentation to prenatal care
d. Advanced maternal age
e. Multiparity

45. A 27 year old, P1A0 and her husband desire to use natural family planning for
contraception. They decide to use the calendar method. Based on her regular 28-day cycle,
which of the following represents her fertile period?
a. Days 14 through 21
b. Days 10 through 17
c. Days 17 through 14
d. Days 12 through 19
e. Days 16 through 25

46. A 29 year old G3P2A0 presents to the emergency center with complains of abdominal
discomfort for 2 weeks. Her vital signs are : blood pressure 120/70mmHg, pulse 90 beats per
0
minute, temperature 36,9 C, respiratory rate 18 breaths per minute. A pregnancy test is
positive and an ultrasound of the abdomen and pelvis reveals a viable 16 weeks gestation
located behind a normal appearing 10x6x5.5 cm uterus. Both ovaries appear normal. No free
fluid is noted. Which of the following is the most likely cause of these findings?
a. Ectopic ovarian tissue
b. Fistula between the peritoneum and uterine cavity
c. Primary peritoneal implantation of the fertilized ovum
d. Tubal abortion
e. Uterine rupture of prior caesarean section

47. Obesity in pregnancy increase risk of which of the following in the baby?
a. An autism spectrum disorder
b. Intra-uterine growth restriction
c. Hydrocephaly
d. Type 1 diabetes in childhood
e. Childhood cancer

48. the stage of gestational development at which endometrial implantation occurs is


a. Eight cell embryo
b. Zygote
c. Morula formation
d. Blastocyst
e. Embryonic disk

49. During early pregnancy, which of the following replaces the ovary as the main source of
progesterone production?
a. Amnion
b. Chorion
c. Trophoblast
d. Myometrium
e. Yolksac

50. Anti D Immunoglobulin should not be given for Rh-negative women in which of the
following settings?
a. Threatened abortion
b. Following complete hydatidiform mole evacuation
c. Before external cephalic version
d. After first trimester elective pregnancy
e. Following complete abortion

51. To preserve the pneumoperitoneum when placing secondary trocars, it is important to :


a. Ensure the surgeon is in proper position
b. Close the stopcock on the secondary port before insertion
c. Ensure the stopcock on the secondary port is open before insertion
d. Ensure the blade on the trocar is sharp
e. Close the insufflation stopcock on the primary port

52. clinical features suggestive of hyperandrogenism include all of the following, except :
a. Facial acne
b. Seborrhea
c. Alopecia
d. Anovulation
e. Hirsutism

53. invasive cervical cancer identified only microscopically, invasion is limited to measured
stromal invasion with a maximum depth of 3mm and no wider than 7 mm should be treated
with :
a. Observation
b. Tissue ablation
c. LEEP/LETZ
d. Simple total hysterectomy
e. Radical hysterectomy

54. The majority of vulvar, vaginal dan cervical cancers appear to have a common cause and
usually caused by :
a. High risk of herpes simplex virus (HSV) infection
b. High risk of human papilloma virus (HPV) infection
c. Increased exposure to endogenous estrogen
d. Increased exposure to exogenous estrogen
e. Chronic bacterial and parasitic infection

55.Childhood neoplastik ovarian Mass most commonly originate from:


a. Gonadal epitelium
b. Gonadal stroma
c. Sex cords
d. Germ cells
e. Metastatic disease
56. A patient returns for postoperative checkup 2 weeks after a total abdominal hysterectomy
for fibroids. She distressed because she is having continuous leakage of urine from the vagina.
Her leakage is essentially continuous and worsen with coughing, laughing, or movement. Given
her history and physical, you perform both a methylene blue dye test, which is negative and
indigocarmine test, which is positive. The most likely diagnosis is:
a. Rectovaginal Fistula
b. Uretro vaginal fistula
c. Vesico vaginal fistula
d. Uretero vaginal fistula
e. Impossible to distinguish

57. A 38 years old multigravida woman complains of the painless loss of urine, beginning
immediately with coughing, laughing, lifting or straining. Immediate cessation of the activity
stops the urine loss after only a few drops. This history is most suggestive of…..
a. Fistula
b. Stress incontinence
c. Urge incontinence
d. Urethral diverticulum
e. UTI

58. Family planning services in Indonesia is important because it has a role in efforts to reduce
maternal mortality. Select the statement that is not true:
a. The maternal mortality rate (MMR) by IDHS in 2012 amounted to 259/100,000 births
b. Number of maternal death related to pregnancy or childbirth in Indonesia based on
the demographic and health survey in 2012 a number of 17,000 up to 18,000mothers
during one year
c. Estimated of number of maternal death related to pregnancy or childbirth in Indonesia
is about 50 deaths per day
d. Family planning services for mothers immediately after delivery of placenta s/d 2 days
postpartum included as postpartum family planning services.
e. In planning services we prioritize contraceptive long term contraception method (LTM),
the IUD, Implant or tubal ligation or vasectomy

A 40 years old G2P1A0 gestational age of 12 weeks. The first child 10 years of age. Never use
contraception. Weight 80 kg, height 156 cm. has never antenatal care.
59. the conclusion below is not true:
a. High risk pregnancy due to maternal age of 40 years
b. Including overweight
c. Have history of secondary infertility
d. There is possibility of pregnancy termination by caesarean section
e. Need for ultrasound screening

60. Select the incorrect regarding guidelines for nuchal translucency (NT) measurement is :
a. The margin of NT edges must be clear enough for proper calliper placement not
necessary in the midsagittal plane
b. The image must be magnified so that it is filled by fetal head, neck, and upper thorax
c. The fetal neck must be in neutral position, not flexed and not hyperextended
d. The amnion must be seen as separate from the NT line, and the + calipers must be
placed on the inner borders of the nuchal space with none of the horizontal crossbar
itself protruding into the space.
e. The callipers must be placed perpendicular to the long axis of the fetus and the
measurement must be obtained at the widest space of the NT

61. choose the wrong statement about Down syndrome is


a. An increased NT thickness itself is not a fetal abnormality
b. Nuchal translucency is a marker that confers increased risk of fetal abnormality
c. Approximately one third of fetishes with increased nuchal gtranslucency thickness will
have a chromosome abnormality, nearly half of which are Down syndrome
d. Down syndrome is caused by an autosomal trisomy, is the most common nonlethal
trisomy
e. All cases of down syndrome is caused by trisomy 21

One mother after normal delivery, P4 age 35 weight 160 kg. Having delivery one day ago, 3000
grams birth weight, breastfeeding, want injectable contraceptives before returning home.

62. if in the cases mentioned above, the mother wanted contraception progesterone-only
contraceptives (POCs), is the correct statement below is :
a. Breastfeeding woman who are <6 weeks postpartum should not use levonorgestrel
(LNG) and etonogestrel (ETG) implants.
b. Breastfeeding women who are <6 weeks postpartum generally should not use
progesterone-only injectables (DMPA)
c. There is no theoretical concern about the potential exposure of the neonate to
DMPA/NET-EN during the first 6 weeks postpartum
d. Breastfeeding women who are <48 hours postpartum can generally use LNG-IUDs
e. Progesterone-only contraceptives (POCs) consist of POPs, progestogen-only injectables
(DMPA and NET-EN) and LNG and ETG implants

63. In Indonesia, pregnancy related morbidity and mortality risk are high, and access to
services is limited, the least likely statement below is :
a. DMPA may be one of the few types of methods widely available and breastfeeding
woman immediately
b. In Indonesia DMPA injections should not be given one day after the delivery as early
postpartum contraception
c. Breastfeeding women who are 6 weeks to < 6 months postpartum can use without
restriction the following contraceptive methods : progestogen-only injectables
(DMPA), and LNG and ETG implants
d. Breastfeeding (and non-breastfeeding) women are generally should not have an LNG-
IUD inserted from 48 hours to<4 weeks postpartum.
e. Available data from clinical and observational trials do not suggest an increased risk
for either breastfeeding performance or infant health outcomes with use of
progestogen-only injectables compared to outcomes in studies using other progestogen-
only methods.

A 27 years old P3A2 returns to your clinic to follow up the result of her annual PAP-SMEAR.
She has had a full annual examination including Pap Smear since the age of 21 and has never
had abnormal cervical cytology. Her STD screening at her last examination was normal. The
cytology of her pap smear showed high-grade squamous intraepithelial lesion (HGSIL). You
discuss the management option for the patient, and she decides to have colposcopy. The
biopsy result of the colposcopy you perform are not insufficient to give a histology report.

63. Which of the following option is the next best step in management for this patient?
a. Repeat Pap smear at 6-month interval for a year
b. Repeat colposcopy to obtain better sample
c. Perform diagnostic excisional procedure such as look electrical excision procedure
(LEEP)
d. Request that the lab repeats their test on the biopsy samples given
e. Conisation

64. A 32 years old presents for an infertility workup. She and her partner have been trying to
conceive for 2 years without success. She has regular menstruation, though she mention she
has severe cramping during cycles. She also notes she experiences pelvic pain during sex. On
examination, she is a thin, well developed woman. She is afebrile and she experiences a great
deal of pain during pelvic examination. Which of the following tests is required for diagnosis
of the patients infertility?
a. Ultrasound
b. Β-hCG level
c. Pap smear
d. Laparoscopy.
e. Hysterosalpingogram

65. A 29 year old G2P1A0 woman is 22 weeks pregnant with reports of pelvic bulge and
bothersome pressure. Physical examination reveal cervical prolapse, approaching the opening
of vaginal introitus. She worried that the prolapse will worsen as the pregnancy progress. What
can you offer this patient for her symptomatic prolapse?
a. Pessary trial
b. Reassessment after delivery but nothing during the pregnancy
c. Cerclage
d. Hysteropexy
e. Uterosacral ligament fixation

66. Mrs. XY is a primigravida who is 34 weeks pregnant. Her last two serial scan have shown a
small gestational age fetus growing on the 9 th entile. Her last scan shows positive end diastolic
flow with a normal PI. She reports having good fetal movements. How should further fetal
surveillance be undertaken?
a. Forthnightly umbilical artery Doppler
b. Twice weekly CTG
c. Twice weekly umbilical artery Doppler
d. Weekly CTG
e. Weekly umbilical artery Doppler + CTG

A 25 year old woman in her first pregnancy is noted to have prolonged first and second stages
of labour. She was induced at 38 weeks pregnancy. The baby was delivered by forceps. After
delivery the placenta she is noted to have heavy vaginal bleeding. Abdominal examination
demonstrates as relaxed uterus.

67. what is the most likely cause of bleeding


a. Uterine atony
b. Uterine rupture
c. Retained placenta
d. Genital tract laceration
e. DIC

68. what should we do if the fundus not firm after placental delivery.
a. Methylergonovine (methergine)
b. Carboprost (hemabate,PGF2-alpha)
c. Fundal massage
d.
Misoprostol (PGE1)
e.
Dinoprostone-prostaglandin E2

69. which of the following medications would be contraindicated in the treatment for this
patient?
a. Methylergonovine (methergine)
b. Carboprost (hemabate,PGF2-alpha)
c. Intramuscular pitocin
d. Misoprostol (PGE1)
e. Dinoprostone-prostaglandin E2

30 year old multiparous woman has rapid delivery soon after arriving in emergency room.
After delivery the placenta she is noted to have heavy vaginal bleeding. Help has been
summoned. Abdominal examination demonstrates the fundus was soft.

70. What is the most appropriate next step ?


a. Intravenous access for fluid resuscitation
b. Uterine packing
c. Baloon tamponade
d. Hecting laceration
e. Misoprostol administration

71. What is the most likely cause of bleeding


a. Uterine atony
b. Uterine rupture
c. Retained placenta
d. Genital tract laceration
e. DIC

72. After use of a 20 units oxytocin in 1000 mL of crystalloid solution to increase the tone of
her uterus stop the bleeding, however, you continue to notice a massive bleeding from the
vagina. What is the most appropriate next step in the evaluation of this patient’s bleeding ?
a. Perform a bedside ultrasound for retained products of conception
b. Perform a bedside ultrasound to look for blood in the abdomen significant for
uterine rupture
c. Perform a manual exploration of the uterine fundus and exploration for retained
clots or products
d. Examine the perineum and vaginal for laceration during delivery
e. Consult interventional radiology for uterine artery embolization

A 31 year old woman came to the hospital with chief complaint of water broke. She is G3P2 at
38 weeks’ gestation with one prior caesarean delivery on second baby due to fetal distress.
She wants to try trial labor. On physical examination, you find 3 cm cervical dilatation, clear
amniotic fluid leakage with inadequate contraction and other examinations reveal no
contraindication. You will start the oxytocin infusion and you inform to the patient about the
risk.
73. According to a large study conducted by the Maternal-Fetal Medicine Units Network, the
use of oxytocin increases the risk for uterine rupture by what magnitude in women with a prior
caserean delivery ?
a. Threefold
b. Sixfold
c. Tenfold
d. Sixteenth fold
e. No difference with normal delivery

74. Labor induction and augmentation are NOT associated with which of the following risk ?
a. Postpartum hemorrhage from uterine atony is more common in women undergoing
induction or augmentation
b. Amniotic fluid emobolism in a laboring patient receiving oxytocin can be occurred
c. The increased risk for caesarean delivery undergoing induction is related with cervical
favorability (Bishop Score)
d. The uterine rupture risk is increased threefold for women in spontaneous labor with
uterine scar
e. Woman whose labor is managed with amniotomy have lower chorioamnionitis
compared with those in spontaneous labor

75. According to the patient above, what is the most favorable condition for succesfull trial
labour after cesarean birth?

a. Water broke with clear amniotic fluid


b. A previous vaginal delivery
c. In a hospital with available facility of anesthesia
d. Only one previous cesarean delivery
e. Induction given at term pregnancy

A 32 years old woman comes to your clinic due to shortness of breath, that worsen since 2
days ago on history taking, she told you that she had ever diagnosed of having significant mitral
stenosis. She is 33 weeks pregnant. The fetus size appropriate. She has had a recent
echocardiography showing ejection fraction of 54% with moderate-severe pulmonary
hypertension.

76. what is the best management for this patient currently?


a. Performed emergency C-section
b. Lung maturation and c-section
c. Conservative management until term pregnancy
d. Second stage acceleration
e. Induction of labour
77. what is the most common cause of heart failure during pregnancy and the puerperium?
a. Chronic hypertension with severe preeclampsia
b. Viral myocarditis
c. Obesity
d. Valvular heart disease
e. Pulmonary artery hypertension

78. for patients with congenital heart disease, what is the most common adverse
cardiovascular enent encountered in pregnancy?
a. Heart failure
b. Arrythmia
c. Thromboembolic event
d. Cerebrovascular hemorrhage
e. Heart axis changes

A 18 year-old G1P0 woman at 10 weeks GA presents for an initial prenatal visit. From history
taking she had a history of IV drugs use. You ordered a rapid test and showed reactive for HIV.
She had never been on HAART medication prior to this pregnancy. You prder a viral load and
it returns at 10,000. Her CD4 count is normal (500). She is otherwise healthy and has no other
significant medical history.

79. When is transmission of human immunodeficiency virus (HIV) from a woman to her baby
most likely to occur ?
a. Intrapartum
b. Before 36 weeks gestation
c. In the days before delivery
d. Transmission occurs equally throughout gestation
e. In the first trimester

80. When would you recommend she start HAART ?


a. At this visit
b. At 37 weeks in preparation for delivery
c. At the beginning of the second trimester
d. Immediately after delivery
e. Only if it is medically indicated for maternal health

81. Which of the following statements about the treatment of HIV pregnancy is true ?
a. Treatment is recommended for all HIV-infected pregnant women.
b. All medications used for the treatment of HIV are safe in pregnancy.
c. Treatment is only required if the HIV-infected woman would qualify for
treatment when not pregnant
d. Zidovudine must be added to whatever regimen the woman is already taking,
even if her viral load is adequately suppressed.
e. Treatment is recommended only for woman with high viral load

82. Which of the following statements regarding intrapartum management HIV is true ?
a. If cesarean delivery is planned, it should be scheduled at 36 weeks’ gestation
b. Cesarean delivery is recommended for women with a viral load > 1000 copies/mL
c. In labor, with a plan for vaginal delivery, amniotomy should be performed as soon as
possible to hasten delivery
d. In labor, internal monitors should be placed because fetuses of HIV-infected women
are at increased risk for distress
b. The HAART should be suspended during labor

83.A 55 year old woman presents to your office for consultation regarding her symptoms of
menopause. She stopped having periods 8 months ago and is having severe hot flushes. The
hot flushes are causing her considerable stress. What should you tell her regarding the
psychological symptoms of climacteric ?

a. They are not related to her changing levels of estrogen and progesterone
b. They commonly include insomnia, irritability, frustration and malaise
c. They are related to a drop in gonadotropin levels
d. They are not affected by environmental factors.
e. They are primarily a reaction to the cessation of menstrual flow

84. A 62 year old woman presents for annual examination. Her last spontaneous menstrual
period was 9 years ago and she has been reluctant to use postmenopousal hormone
replacement because of a strong family history of breast cancer. She now complains of
diminished interest in sexual activity. Which of the following is the most likely cause of her
complaint ?

a. Decreased vaginal length


b. Decreased ovarian function
c. Alienation from her partner
d. Untreatable sexual dysfunction
e. Physiologic anorgasmia

85. A 58 year old post menopause, Caucasian woman comes in to your office for advice
regarding her risk factors for developing osteoporosis. She is 5 ft 1 in tall and weighs 195 lb.
She stopped having periods at age 49. She is healthy but smokes one pack of cigarettes a day.
She does not take any medications. She has never taken hormone replacement for
menopause. Her mother died at age 71 after she suffered a spontaneous hip fracture. Which
of the following will have the least effect on this patient’s risk for developing osteoporosis ?
a. Her family history
b. Her race
c. Her history of smoking cigarettes
d. Her menopause status
e. Her obesity

A 49 year old woman experiences irregular vaginal bleeding for 3 months duration. You
performed endometrial biopsy, which copious tissue with a delivery, lobulated texture. The
pathologist report shows proliferation of glandular and stromal elements with dilated
endometrial glands, consistent with simple hyperplasia. Cytologic atypia is absent.

86. Which of the following is the best way to advised the patient ?
a. She should be treated to estrogen and progestin hormone therapy
b. The tissue will progress to cancer in approximately 10% of cases
c. The tissue may be weakly premalignant and progress to cancer in approximately 1 % of
cases
d. She requires a hysterectomy
b. No further therapy is needed

87. Which of the following factors is protective against endometrial hyperplasia ?


a. Obesity
b. Tamoxifen
c. Oral contraceptive pills (OCPs)
d. Early menarche or late menopause
e. Unopposed exogenous estrogen therapy

You see a 16 yo female who presented with primary amenorrhea. Breast development was
noted at 13 years but there has been no increase in breast size. Pubic and axillary hair were
noted within one year of referral. An outside ultrasound showed no uterus or ovaries. Physical
exam reveals a normal vagina introitus with hymen present. Breast are Tanner 3, but seem to
be more fatty than mammary tissue.

88. Lab test were significant for absent estradiol, elevated gonadotropins and mildly elevated
DHEAs with normal testosterone. A karyotype was requested by the endocrinologist and was
found to be 46 XY (SRY gene +). What is the working diagnosis following her initial workup ?
a. Androgen insensitivity
b. Disorders of testicular development
c. Mullerian Agenesis
d. A and B
e. B and C

89. The MRI showed an infantile uterus with no discernible gonads. During a laparoscopic
evaluation, two dysplastic gonads attached to small fallopian tubes and rudimentary uterus
was visualized. The final diagnosis is :
a. Swyer syndrome
b. Androgen Insensitivity
c. Mullerian Agenesis
d. Partial gonadal dysgenesis
e. Testicular Regression Syndrome

90. What type of tumor is a significant risk for such condition ?


a. Mature Teratoma
b. Leydig cell tumors
c. Germ cell tumors
d. Stromal Cell Tumors
e. Granulose Cell Tumors

Miss 25 years old P3 comes to Gynecolog outpatient clinic with cytology result low grade SIL.
She curious about the result since her last cytology result was normal 3 years ago. She has no
complaint recently. She began sexually active since 10 years ago and has had six partners. She
smokes 10 cigarrettes per day since 4 years ago. Her mother was diagnosed for cervical cancer
at 44 years old and just died 3 months ago. Her child now is 6 years old.

91. What are patient risc factor for CIN ?


a. Her smoking habits
b. Onset of sexual activity
c. Six sex partners
d. Early childbearing
e. All mention above

92. What is next proper management for patient ?


a. IVA test
b. Colposcopy
c. No procedur need in her treatment
d. HPV DNA test
e. Endocervical curettage
93. The result showing a condylomatous acetowhite lesion with punctuation and atypical
vessels. Biopsy result confirms CIN 1 and HPV DNA test positive. What do you suggest for
patient ?
a. LEEP procedure
b. Reevaluation of HPV DNA
c. Cold knife conization
d. Repeat cytology in 12 months
e. Repeat cytology in 6 months

Mrs. A 37 years old had just undergone laparoscopic procedures. Her chief complaint was
infertility for 6 years with history of severe dysmenorrhea. From hysterosalpingography, both
tubes were non patent. Pelvic ultrasound found bilateral cystic mass with echo interna sized
50 and 60 mm in diameter. Her husband sperm examination was within normal limit. From
laparoscopy findings, normal size uterus with adhesion of posterior part to colon and Douglas
pouch was also severely adhered. Both of ovaries were enlarged approximately 50-60 mm with
severe adhesion at the right and left adnexae. After adhesiolysis, both of tubes could be
identified and were blocked. Chocolate fluid was spilled from the cysts. The pathology of the
cysts was endometriosis cyst.

94. Which one is the TRUE statement ?


a. The patient should not undergo surgery because the best procedure to solve her pain
and infertility problem was giving GnRH agonist then performaned IVF
b. Laparoscopic procedure should be done if infertility problem already more than 3 years
c. Laparoscopic cystectomy was done because the cyst have diameter 50 and 60 mm
d. GnRH agonist 1 time prior to surgery will give better results
e. None of the above (atau cek male factor dan ovarian reserved dlu)

95. Which of the statement below IS NOT TRUE regarding the surgery ?
a. Bilateral cystectomy could be done without any precautions on ovarian reserved.
b. Patient has severe grade of endometriosis because its AFS score more than >40
c. Since she has not conceived yet we should conserve both of her tubes
d. A and C
e. B and C

A 19 years old woman came to clinic with chief complaint of irregular menstruation. She had
menstruation once every three months. She is also obese (BMI 32). Physical examination
reveals she has hirsutism (Ferriman Gallwey score 9), other physical examination within
normal limit. Gynecologic examination within normal limit.

96. Which of the following criteria diagnosis of polycystic ovarian syndrome (PCOS) is not part
of Rotterdam criteria ?
a. Oligo/anovulation
b. Appearance of polycystic ovaries by gynecologic ultrasound.
c. Excess androgen activity
d. Ferriman Gallwey score >8
e. All statement is true

97. In PCOS, increased testosterone production from the following hormones ?


a. Inhibin
b. Estradiol
c. Prolactin
d. FSH
e. LH

A 55 years old nulliparous woman who underwent menopause at age 50 years complaint of a
1 month history of vaginal bleeding and smells. Her medical history reveals she has
hypertension and controlled with anti hypertensive agent, and also she has diabetes mellitus
controlled with an oral hypoglycemic agent. On examination, she weighs 89 kg and 152 cm tall.
Her blood pressure is 150/90 mmHg. Heart and lung examination are normal. The abdomen is
obese and no masses are palpated. The external genitalia appear normal, and the uterus seems
to be enlarged, without adnexal masses palpated.

98. What is the probable diagnosis of this patient ?


a. Cervical cancer
b. Hyperplasia Endometrium
c. Endometrial carcinoma
d. Uterine Fibroid
e. Ovarian Cancer

99. Which of the the following does not increase a woman’s risk of developing endometrial
cancer ?
a. Obesity
b. Smoking
c. Diabetes Mellitus
d. Tamoxifen
e. Unopposed estrogen

100. Which of the following subtypes is NOT a type II endometrial cancer ?


a. Clear cell carcinoma
b. Papillary serous carcinoma
c. Carcinosarcoma
d. Endometrioid adenocarcinoma
e. None of the above

sampun nggih
matur nuwun…

UNAS NOVEMBER 2016

Mrs. A, 24 year primigravida, 39 weeks gestational age, undergoes caesarean delivery due to
breech presentation. She delivered female baby, with body weight 3600 grams, and APGAR Score
was 8/9.

1. Post operatively maintenance intravenous administered fluids are:


a. Ringer solution or a similar crystalloid solution with 5-percent dextrose
b. 10% dextrose
c. Colloid solution
d. Crystalloid solution with 40-percent dextrose
e. Amino acid based fluid

2. Solid food as one of the source of nutritional diet in post caesarean patient will be offered within
A. 2 hours post operative
B. 3 hours post operative
C. 8 hours post operative
D. 12 hours post operative
E. 24 hours post operative

3. Women undergoing cesarean delivery have an increased risk of venous thromboembolism


compared with those delivering vaginally. In order to decrease the risk, what is your
suggestion to these women:
A. Early ambulation
B. Early feeding
C. Administered analgesic
D. Inspected incision each day
E. Removed the skin closure on the fourth postoperative

Mrs. A, 24 years, G2P1A0 39 weeks gestational age, admitted to your emergency room with
complained abdominal cramping with bloody show. In examination, revealed normal vital sign,
contraction was 3x/10’/35”. Fetal heart rate was 146x/m. estimated fetal weight was 2900 gram.
Pervious baby was 3200 gram. Vaginal examination revealed: dilatation was 4 cm, effacement
100%, amniotic membrane was intake, lowest part was head with descent of the head was Hodge
II. Denominator was monir fontanella at the left side.

4. What is the condition of this patient?


A. Not in labour yet
B. In labour, stage 1 latent phase
C. In labour, stage 1 active phase
D. In labour, stage 2
E. In labour, stage 3

5. According the vignette above, what will be happened to that mother?


A. She will continue the progress of labour and will give birth normally
B. The progress of labour will be stucked in this phase of labour
C. She will underwent prolonged second stage
D. The second stage of labour must be terminated by vacuum or forcipal extraction
E. The mother will face the possibility of postpartum haemorrhage

Mrs. Selly 20 year-old primipara is 36 hours postpartum following cesarean delivery for failure to
progress. She is complaining of abdominal pain and has a fever of 38 C. She is not yet tolerating
oral intake because of nausea. You diagnose metritis.

6. Which of the following is the most important criterion for the diagnosis of postpartum metritis?
A. Uterine tenderness
B. Fever (1st)
C. Foul-smelling lochia (3rd)
D. Leukocytosis (2nd)
E. Leukopenia

7. Which of the following is the most frequent cause metritis?


A. Group A streptococcus
B. Group B streptococcus
C. Chlamydia trachomatis
D. Mycoplasma hominis
E. Ureaplasma urealyticum

8. A defect in the anterior abdominal wall that abdominal contents of the fetus covered only
by a two-layered sac of amnion and peritoneum
A. Omphalocele
B. Gastroschisis
C. Diaphragmatic hernia
D. Duodenal atresia
E. Posturethral value

9. In cases of severe hyperemesis gravidarum, all EXCEPT which of the following initial
complications are common?
A. Acidosis  seharusnya alkalosis
B. Dehydration
C. Hypokalemia
D. Hyponatremia
E. Mild transaminitis

10. Which of the following twin pregnancies would be candidate for fetoscopic laser
ablation therapy for twin-twin transfusion syndrome (TTTS)?
A. Monochorionic diamniotic twins at 23 weeks of gestation with stage I TTTS.
B. Dichorionic diamnionic twins at 19 weeks of gestation with stage II TTTS.
C. Monochorionic diamnionic twins at 15 weeks of gestation with stage IV TTTS.
D. Monochorionic diamnionic twins at 21 weeks of gestation with stage III TTTS.
E. Monochorionic diamnionic twins at 14 weeks of gestation with stage III TTTS. (Antara 16-
21 mgg , )

11. Which of the following symptoms of adenomyosis is correctly paired with its etiology?
A. Dysmenorrhea - increased prostaglandin production
B. Dysmenorrhea-hemorrhage within the ectopic glandular foci
C. Menorrhagia-increased and abnormal vascularization of the adenomyotic tissue
D. Infertility - increased and abnormal vascularization of the adenomyotic tissue
E. Its severity does not correlate with ectopic foci and degree of invasion

12. What is the most common site of metastatic spread of choriocarcinoma? (William obstet)
A. Brain
B. Lever
C. Lung
D. Vagina
E. Tuba

13. Regarding coagulation system in pregnancy, which of the following statements is true?
(William obstet)
A. Mean platelet count is 250.000/Ul.
B. Fibrinolytic is usually reduced.
C. Fibrinogen levels are increased.
D. Decreases in platelet concentration are solely due to hemodilution
E. Is responsible for dependent edema in the lower extremities.

14. A. 55-year-old healthy woman undergoes exploration for a large pelvic mass. Frozen section
analysis of her right ovary notes “mucinous low malignant potential, cannot exclude
invasion”. There is no other obvious disease. What surgical procedures should be performed,
in addition to total abdominal hysterectomy and bilateral salpingo-oophorectomy?
A. Pelvic washings, omentectomy, multiple peritoneal biopsies
B. Pelvic washings, omentectomy, multiple peritoneal biopsies, bilateral pelvic and para-
aortic lymph node dissection
C. Pelvic washings, omentectomy, multiple peritoneal biopsies, bilateral pelvic and para-
aortic lymph node dissection, appendectomy
D. The procedure done was completed
E. Omentectomy, multiple peritoneal biopsies, bilateral pelvic and para-aortic lymph node
dissection

15. A 32-year-old G1P1 woman present to your office with the chief complaint of amenorrhea since
her most recent vaginal delivery 1 year ago. She notes that she had an uncomplicated
pregnancy, followed by the delivery of a healthy baby boy. Her delivery was complicated by
an intra- amniotic infection as well as a postpartum hemorrhage requiring a postpartum
dilation and curettage. After her delivery, she breastfed for 6 months, and during this time she
had scant and irregular vaginal bleeding. After stopping breastfeeding 6 months ago, she
notes the absence of menses, but instead has monthly painful cramping, which seems to be
getting worse. She remarks that prior to her pregnancy, she had normal, regular menses,
which were not too heavy or painful. She and her husband would like to have another child,
and have been having unprotected intercourse for the past 6 months without achieving a
pregnancy. Your review of system is otherwise negative. You perform a physical examination,
which is normal other than a slightly enlarged, tender uterus. A urine pregnancy test in the
office is negative.

What is the most likely diagnosis?


A. Sheehan’s syndrome
B. Lactational amenorrhea
C. Asherman’s syndrome
D. Premature ovarian failure
E. Kallmann syndrome

16. From a hysterosalpingogram, you find out multiple synechiae within the uterus. What is
your next step in therapy of this patient?
A. Diagnostic and operative hysteroscopy
B. Provera 10 mg daily for 5 days in an attempt to achieve a withdrawal bleed
C. In vitro fertilization
D. Place an intrauterine device
E. Inform your patient that unfortunately, she is “barren” and will not be able to carry
a pregnancy again

17. The following are acceptable method for confirmation of ovulation


A. A drop in basal body temperature of at least 0.50 C on day 14
B. Blood progesterone level on day 21
C. Menstruation
D. Gonadotrophin level

A 27-year-old patient and her husband present to you with primary infertility. The patient reports
regular periods every 28 to 30 days. The patient has no significant medical history and does not
take any medications other than prenatal vitamins. Her husband is also in good health, is 30 year
of age, and has two children froma previous marriage. When you asked the patient how long they
have been trying to achieve a pregnancy, they tell you 6 months.
18. Your instructions to the couple are the following
A. They will likely need IVF to achieve a pregnancy
B. They will likely need IUI cycles
C. Continue trying appropriately timed intercourse for 6 months and if no pregnancy is
achieved, come back to see you
D. Consider donor egg
E. Consider adoption

19. The couple comes back to you after appropriately timed intercourse, not having achieved a
pregnancy. At this time you ambark on a workup that includes a semen analysis, an HSG, and
an endrocrine evaluation including FSH, E2, TSH, prolactin levels, and ovarian reserve testing.
All of the tests come back normal. Your next recommendation:
A. Have 6 more months of timed intercourse and if no pregnancy is achieved, come back to
see you
B. Clomiphene citrate with IUI
C. IVF
D. Donor egg
E. Human gonadotropin (HMG)

20. Which of the following defines heterotopic pregnancy?


A. One tubal and one abdominal pregnancy
B. One ectopic and one intrauterine pregnancy
C. Two pregnancies in one fallopian tube
D. Two pregnancies in one fallopian tube
E. Two tubal and one abdominal pregnancy

21. The American College of Obstetricians and Gynecologists consider which of the following
factors the most important in selecting a suitable candidate for trial of labor After Cesarean
Section (TOLAC)?
A. Prior classic uterine incision type
B. Infection at the time of the original surgery
C. Gestational age at the time of the original surgery
D. Degree of uterine distention during the current pregnancy
E. Cervical dilatation at the time of hospital admission

22. Post operatively, after the urinary catheter removal the most common problem in the
female bladder is
A. Urinary tract infection
B. Bladder atony caused by over distention
C. Overactive bladder
D. Bladder and urethral trauma
E. Urge incontinence

23. The incubation period of syphilis is which of the following? (Williams 24th)
A. 1-7 days
B. 10 days
C. 3-90 days
D. 120-180 days
E. 210 days

24. From pelvic examination findings: pelvic brim: round; diagonal conjugate 12 cm, symphisis
parallel to sacrum, subpubic angle is acute, convergent side walls, bituberous diameter is 7
cm. by analyzing your findings, which causes below is unlikely to be?
A. Android pelvic
B. Anthropoid pelvic
C. High assimilation pelvic
D. Platypelloid pelvic
E. Oblique pelvic

25. Which of the following is a factor predisposing to the development of malignant


mixed mullerian tumors?
A. Prenatal exposure to diethylstilbestrol (DES)  krn hormone testosterone meningkat
B. Exposure to mumps virus
C. Family of ovarian cancer
D. Previous pelvic irradiation
E. Perineal use of talc

Mrs. S, 32-year-old G4P3AO gravid with chronic hypertension had a normal labor that arrested in
second stage at +1 station. She complained of mild dyspnea and fatigue. The fetus had a left
occiput anterior presentation and was delivered by forceps. Completion if third stage followed
quickly, and the fundus was noted to be firm. The OBGYN was carefully examiner then noted the
lower uterine segment was boggy.

26. For Mrs. S, one dose uterotonic agent is given and the fundus is massaged. Despite this,
she continues to bleed. Which of the following is suitable treatmen in this situation?
A. Administer methergin 0,2 mg intramuscularly
B. Mobilize a team that include obstetricians, nurses, and anesthesiologists
C. Perform laparotomy to prepare postpartum hysterectomy
D. Apply ballon catheter
E. Uterine and ovarian arthery ligation

27. The patient continues to bleed and you have initiated whole blood transfusion. Which of the
following is suitable treatment in this situation?
A. Continue to administer Hermabate intramuscularly every 20 minutes
B. Insert Bakri postpartum Balloon or large Folley catheter balloon into the uterine
cavity and inflate the balloon
C. Consider laparotomy and utiren compression suture placement
D. All of the above
E. A and B
A 38-year-old G3PS002 woman present at 40 weeks 3 days with contraction to labor and delivery
triage. Contractions started 1 hour ago and are very painful. The patient denies leaking fluid but
did notice blood and mucus on her underwear. The baby has not been particularly active since
contractions started. Her pregnancy has been complicated by A2GDM. Fasting blood glucose are
usually between 80 and 90 mg/dL with 1-hour postprandial values between 120 and 140 mg/dL.
Her prepregnancy weight was 130 lb and she is 5 ft 5 in (BMI 21.6). She has gained 30 lb this
pregnancy (BMI 26.6). Hemoglobin A1C is 6.0% Fetal ultrasound at 20 weeks demonstrated
normal fetal anatomy. Repeat ultrasound for growth at 38 weeks demonstrated fetus with weight
in the 90th percentile and an EFW of 4,350 g. her last pregnancy was complicated by A1GDM and
she delivered a 4,200 g infant without complications. Initial cervical examination reveals dilation
of 6 cm, 50% effacement, and 21 station. Two hours later the nurse calls you to the room after the
patient’s water breaks. Examination shows complete dilation and effacement, and fetus at 11
station. The patient has a strong urge to push and begins pushing. The delivery is complicated by a
second-degree perineal laceration and a postpartum hemorrhage of 600 mL. fetal weight is 4,560
g and the Apgar score ore 6,8.

28. What neonatal risks are most commonly present in macrosomic fetuses?
1. Jaundice
2. Hypoglicemia
3. Hyperglicemia
4. Birth Trauma
5. Asthma
6. Hypocalcemia
A. 1, 2, 4, 6
B. 1, 3, 4, 5
C. 3, 4, 5
D. 1, 3 5, 6
E. 1, 2, 5, 6

29. Match the following fetal heart rate tracing (Figure 11-2) with the descriptive term that
best fits the situation.

Figure 11-2 (Reproduced with permission from Cunningham F, et al. Williams Obstretrics, 23 rd ed.
New York: McGraw-Hill 2010, p. 418).
A. Early decelerations
B. Late deceleration
C. Normal tracing
D. Poor variability
E. Sinusoidal pattern

30. A 49-year-old woman present complaining of vulvar pain that increases with ambulation and
intercourse. She also notes a lump on her right labia that has increased in size over the past
48 hours and quite painful. The patient has had these same symptoms in the past On
examination she has a 5-cm tender cyst on the medial aspect of her right labia, with a
surrounding erytherma of the labia that extends 1 to 2 c, away. What would be your first
step in management of this patient?
A. Sitz baths rendam duduk pada px , fisura anal, , keputihan, hemoroid
B. Insertion of word catheter
C. Antibiotics
D. Biopsy of the cyst and insertion of word catheter
E. Marsupialization of the cyst

31. A 68-year-old post menopausal G4 P4004 is seen in your clinic for her annual exam. She
complains of occasional urinary incontinence with sneezing or coughing, and also reports
heaviness in the lower Abdomen and mild bulging from the vagina that is more prominent at
the end of the day. She has no fecal incontinence and no vaginal dryness. Her PMH is
significant for obesity, COPD, CHF, and poorly controlled diabetes. She has smoked ½ PPD for
the last 40 years. Three of her children weighed more than 9 lbs at birth. On pelvic
examination, you note second-degree pelvic relaxation.
What treatment would you recommend for this patient ?
A. Placement of a mesh transobturaor sling for urethral support
B. Vaginal hysterectomy and McCall culdoplasty
C. Anterior colporrhaphy
D. Posterior colporrhaphy
E. Placement of a pessary and encourage Kegal exercises

32. A. 27-year-old G0P0 woman presents to your office with a history of amenorrhea. She has a
history of regular menstrual cycles in high school as well as while in college and medical
school, when she began oral contraceptives for birt control. She stopped her birth pills about
7 months ago and her period never resumed. She is sexually active with a male partner and
uses condoms for contraception. She has a history of seasonal allergies. She has not has a Pap
smear in 5 years, but has no history of dysplasia and no prior known STDs. When you ask her
about her general diet and activity, she states that she is a vegetarian. She is a resident, so
often eats on the run or skips meals but does eat one to two meals per day and has no prior
history of an eating disorder. She exercises when she can, about one two time per week. She
typically jogs or goes for a walk with her dog. Her BMI is elevated at 30. Which of the
following tests are most appropriate to order at this visit? (Blueprint)
A. TSH, prolactin, and β-bCG
B. 17-OH progesterone, DHEAS, and testosterone
C. Transvaginal ultrasound
D. Hysterosalpingogram to look for intrauterine adhesions
E. All of the above

33. A 24-year-old G1P0 at 28 weeks 5 days of gestation presents to routine prenatal care with
complaint of increased discharge today. She first noticed it after going to the bathroom.
When she stood up she felt as if a little urine continued to leak out. Throughout the
afternoon, she has continued to feel like water is leaking from the vagina. There is no vagina
bleeding or abdominal pain. The discharge is clear and odorless. Her pregnancy has been
otherwise uncomplicated. Which of the following is the first step in evaluating this patient?
A. Amnion dye test/tampon test
B. Ultrasound to check for Amniotic Fluid Index (AFI)
C. Sterile speculum examination
D. Amniocentesis
E. Amniocentesis to rule out chorioamnionitis

34. A 32-year-old woman comes to your clinic for preconception counseling. She was diagnosed
with epilepsy at age 12 and is currently taking phenytoin and carbamazepine. She has been
seizure-free for 1 ½ years. She and her husband are planning to conceive within the next
year. What should you advise to decrease the risks for the upcoming pregnancy?  bebas
kejang 2 th
A. Stop all seizure medications
B. Optimize her seizure regimen to include only one medication
C. Start taking a prenatal vitamin and 400 mcg of folic acid
D. Keep the same dose of both medications and start taking 4 mg of folic acid
E. Recommend she transition off both her current medications and start taking valproic
acid for monotherapy

35. You are called to see a 21-year-old G3P2002 at 28-weeks gestational age who has had limited
prenatal care. She has diffuse complaints of abdominal pain, cold sweats, anxiety, and
insomnia. Upon review of her history, she tells you that she usually has two to three vodka
drinks every day. For the past 3 days,she has not been able to afford to purchase any alcohol.
She denies any other significant medical or surgical history. You diagnose her with alcohol
withdrawal and admit her to the hospital for treatment. What is the most significant long-
term complication of alcohol dependence during pregnancy?
A. Fetal alcohol syndrome
B. Maternal withdrawal
C. Neonatal withdrawal
D. Low fetal birth weight
E. Neonatal admission to the NICU

36. A 45-year-old presents for evaluation because her primary care physician has diagnosed her
with pelvic organ prolapse while performing annual care. She denies any pelvic pressure,
bulge, or difficulty with urination. Her only medical comorbidity is obesity. For asymptomatic
grade 1 pelvic organ prolapse, what do you recommend?
A. Conservative management with pelvic floor muscle exercises and weight loss
B. Colpocleisis obliterative procedure
C. Gelhorn pessary
D. Round ligament suspension
E. Hsyterectomy

37. Micturition is voluntary and occurs with relaxation of the urethra and sustained contraction of
the bladder until emptying is complete. Sustained contraction of the detrusor muscle of the
bladder requires parasympathetic stimulation. Parasympathetic control of the detrusor is
supplied by which of the following nerves?
A. Hypogastric nerve
B. Pudendal nerve
C. Peroneal nerve
D. Pelvic nerve
E. Sciatic nerve

38. A 53-year-old G3P3 comes in to see you for management of the hot flashes and night sweats.
She stopped having menses 1.5 years ago and still has her ovaries and uterus in situ. She has
been suffering with recurrent hot flashes and night sweats, which interfere with her quality of
life. She had hoped to avoid taking hormones but her symptoms have not improved over the
past 1.5 years and she is now ready for treatment. In counseling her about treatment options
for her hot flashes and night sweats, which of the following would not be an appropriate
option?
A. Oral estrogen and progesterone
B. Topical estrogen and progesterone patch
C. Low dose vaginal estrogen cream
D. A selective serotonin reuptake inhibitor (SSRI) such as paroxetine (Paxil) or
fluoxetine (Prozac)
E. Topical clonidine (Catapres) pacth

39. A 27-year-old G1P1001 woman comes to your office 8 months following an uncomplicated
vaginal delivery of a healthy male newborn. She and her husband have been trying to
conceive a second pregnancy for 3 months, but she has been unsuccessful and is very anxious
about this. After a thorough history, you find that she is still breastfeeding her newborn
routinely and is not menstruating. Assuming that she has not yet ovulated, what is the likely
underlying cause of the findings in his patient ?
A. Thyroid hormone suppression of the anterior pitiutary
B. Abnormal endometrial regeneration causing failure of implantation
C. Prolactin-induced inhibition of pulsatile GnRH from the hypothalamus
D. Reduced tubal motility secondary to elevated prolactin levels
E. Pathologically decreased sperm count of the male partner

40. A 28 year-old G2P1 woman at 30 weeks of gestational age presents to the clinic for a routine
OB visit. She was diagnosed with gestational diabetes 4 weeks ago and was started on a
diabetic diet and exercise regimen. She adhered to the recommendations, measured her
glucose levels four times a day, and recorded results for the past week. Her average fasting
glucose value is 84 mg/dl and 1-hour postprandial values after all three meals range from 135
to 165 mg/dl. What is the best next step in the management of this patient?
A. Continue diabetic diet plus exercise
B. Start the patient on Lispro and NPH in the morning and Lispro at dinner
C. Start the patient on Lispro in the morning, and Lispro and NPH at dinner
D. Start the patient on Metformin 500 mg PO daily
E. Start the patient on NPH in the morning and NPH at dinner

41. Laceration of abdominal wall vessels can increase blood loss and risk of postoperative
hematoma formation. The superficial epigastric, superficial circumflex iliac, and superficial
external pudendal arteries all arise from which of the following ?
A. Femoral artery
B. External iliac artery
C. Deep circumflex artery
D. Internal thoracic artery

42. Early treatment of hidradenitis suppurativa involves which of the following?


A. Topical corticosteroid ointment
B. Infliximab, a monoclonal antibody
C. Surgical excision of apocrine gland sinus tracts
D. Warm compresses, topical antiseptics, and systemic antibiotics

43. Which of the following is a painful, self limited mucosal lesion? (William gynec)
A. Vitiligo
B. Apthous ulcer  di bibir
C. Pemphigus vulgaris
D. All of the above

44. By definition, patients with severe oligospermia have sperm counts less than which of
the following per mililiter of semen ?
A. <5 milllion/mL
B. <15 million/mL
C. <25 million/mL
D. <35 million/mL

45. Mrs. P, a 45-year-old pregnant woman at term collapses in front of the nursing staff while
waiting in the delivery room. She is unresponsive and has no pulse. The midwife activates the
emergency responses system and begins chest compressions. A team of 2 doctors and 2
nurses arrive with the emergency equipment.
A. The first thing one should do is displace the uterus to the left while chest compressions is
ongoing
B. One should follow BLS guidelines and perform chest compressions on the sternum at
the inter- nipple line
C. One should do 2 minutes of CPR before considering defibrillation in this patient
D. The doctors should transfer the patient to theatre for an urgent Caesarean delivery
E. Intubation should only be considered after return of spontaneous circulation

46. A 28-year-old woman, with monochorionic twin pregnancy showed a discordant crown-lump
length (CRL) at 12 weeks of gestation, and a discordant amniotic fluid volume at 15 weeks
with a deepest vertical pocket of 1.8 cm in the smaller twin and 6.3 cm in the larger twin.
Both bladders are visible. Which of the following is/are likely diagnosis?
A. Unequal placental sharing
B. Twin to twin transfusion syndrome quintero stage I
C. Complicated monochorionic twin pregnancy, suspected to develop either selective fetal
growth restriction or twin-twin transfusion syndrome (TTTS)
D. Velamentous cord insertion in the one twin
E. Physiological variation

47. A woman, 25 -year-old, comes to your practice for preconception counselling, she plans to
married next week, and has a complaint of hyper secretive mucus from her genital. Different
sex hormones have different effects on the cervical mucus. Which of the following statements
accurately describes the effect of the estrogen?
A. It decreases the water content of cervical mucus
B. It decreases the palm-leaf crystallization pattern of mucus upon drying (ferning)
C. It decreases formation of glycoprotein channels, which favor sperm penetration
D. It increases cervical mucus strecthability (spinnbarkeit)
E. It increases the amount of potassium chloride in the cervical mucus

A 34-year-old G3P2A0 woman at 38 weeks and 6 days was admitted to labor and delivery unit for
active management of labor after it was determined that her membranes had ruptured and she
was dilated to 3 cm. Her cervix has been steadily dilating and now she is at 6 cm. She is very
uncomfortable and finds her contractions very painful. Her partner is also very concerned tha she
needs pain relief. ( blue print chapter 2)

48. You advise your patient that


A. Narcotics are available, but should be reserved for closer to the time of delivery when
her pain will be greatest
B. If she continues with natural childbirth and eventually needs a cesarean section she will
require general anethesia
C. Spinal anesthesia is her best option because it gives a constant infusion of medicine over
a long period of time
D. She cannot have an epidural yet because she is not yet in the active phase in labor
E. A variety of relaxation techniques can be incorporated into her labor in addition to pain
medication

49. With adequate pain control she dilates to into 10 cm and second stage begins. Which of
the following is the correct order of the cardinal movements of labor ?
A. Internal rotation, engagement, descent, flexion, external rotation
B. Engagement, descent, internal rotation, flexion, external rotation
C. Internal rotation, descent, engagement, flexion, external rotation
D. Engagement, descent, flexion, internal rotation, external rotation
E. Engagement, descent, external rotation,flexion, internal rotation

50. An uncomplicated vaginal delivery typically includes which maneuver?


A. Perineal support to decrease perineal trauma
B. An episiotomy to hasten delivery
C. Vacuum extraction if the fetal station is low
D. Forceps to aid maternal efforts
E. The McRoberts manuever

51. Stage 3 begins following the delivery of the infant and typically involves which of the following?
A. Placental separation
B. Stopping oxytocin drips if they were used during stage 2
C. An abrupt increase in the size of the intrauterine cavity
D. Uterine prolapse
E. A delay of 60 minules before the placenta is delivered

52. Which of the following is the most common cause of first trimester pregnancy loss?
A. Uterine anomalies
B. Incompetent cervix
C. Intrauterine infection
D. Fetal chromosomal abnormalities
E. Placenta adhesive

53. Which of the following NOT clinical indicator of heart disease during pregnancy?
A. Cyanosis
B. Clubbing of fingers
C. Systolic murmur grade 2/6
D. Diastolic murmur
E. Cardiomegaly

54. Fetal hyperinsulinemia in the second half of pregnancy is NOT associated with which of the
following?

A. Macrosomia
B. Neonatal hypoglicemia
C. Maternal hyperglicemia
D. Increased maternal weight
E. Increased chance of neonatal acidosis

55. Which of the following statements regarding intrapartum management of HIV is true?
A. If cesarean delivery is planned, it should be scheduled at 36 weeks’ gestation
B. Cesarean delivery is recommended for women with a viral load > 1000 copies/ml
C. In labor, with a plan for vaginal delivery, amniotomy should be performed as soon
as possible to hasten the delivery
D. In labor, internal monitor should be placed because fetuses of HIV-infected women are at
increased risk for distress.
E. No vertical transmission risk increased if there is PROM > 38 weeks

56 Increased serum free thyroxine levels in women with hydatidiform moles stem from increases
in which of the following? (Williams obt 24th ch 20 , nomor 15)

A. Maternal estrogen levels


B. Fetal thyroxine production
C. Maternal progesterone levels
D. Maternal beta-human chorionic gonadotropin levels
E. Maternal estriol levels

A 33-year-old G0P0 woman comes to your office for her initial prenatal visit. She tested positive
with two home pregnancy tests and has been experiencing breast tenderness and mild nausea
for a few weeks. She has a history of regular menstrual periods occurring every 28 to 30 days. This
was a planned pregnancy and is the first child for her and for her partner.
57. Your patient was actively tracking her menstrual cycle and is certain that the first day of
last menstrual period (LMP) was 12/2/11. Using Naegel rule, estimate her date ofdelivery:
A. 5/12/11
B. 2/11/11
C. 19/12/11
D. 19/11/11
E. 19/10/11

58. As her pregnancy continues, you would expect her cardiac output to increase by which of the
following mechanisms:
A. First an increase in stroke volume, then an increase in heart rate
B. A decrease in systemic vascular resistance
C. Cardiac output would not change significantly until the third trimester
D. An increase in systemic vascular resistance facilitated by elevated progesterone levels
E. Increased heart rate alone

59. Which of the following is true regarding the physiologic changes she might expect during her
pregnancy?
A. Gastric emptying and large bowel motiity are increased in pregnancy
B. BUN and creatinine will decrease by 25% as a result of an increase in glomerular filtration
rate (GFR), which will be maintained until delivery
C. An overall decrease in the number of WBC and platelets
D. Nausea and vomiting that should be treated aggressively with antiemetics
and intravenous hydration
E. An increase in tidal volume along with an increase in total lung capacity (TLC)

60. Miscarriage can be diagnosed with certainty on ultrasound in which of the following situations?
A. Gestasional age greater than 9 weeks
B. The presence of an embryo measuring 10 mm with no evidence of cardiac activity
C. A gestasional sac with yolk sac measuring more than 12 mm
D. An empty uterus on the follow up scan in a woman diagnosed with an
intrauterine pregnancy on a previous scan
E. A falling serum bHCG measurement over 48 hours in a pregnancy of unknownlocation

61. This woman is also a long-distance runner and wants to continue to train during her planned
pregnancy. She wants to know if there are any potential adverse effects to her fetus if she
pursues a program of regular exercise throughout gestation. You advice her to
A. During pregnancy, women should stop exercising because such actvity is
commonly associated wilh intrauterine growth retardation in the fetus
B. Exercise is best performed in the supine position to maximize venous return and
cardiac output
C. It is acceptable to continue to exercise throughout pregnancy as long as the
maternal pulse does not exceed 160
D. Non-weight-bearing exercises are optimal because they minimize the risks of maternal
and fetal injuries
E. Immediately following delivery, patients can cnontinue to exercise at prepregnancy levels

62. A 10 year old woman comes to antenatal visit with a history of trace glucosuria and today,
she undergoes a urinalysis. The dipstick done by the nurse indicates the presence of trace
glucosuria . Allof other parameters oftho urine test are normal. What is the most likely etiology of
the increased sugar detected in the urine?

A. The patient has diabetes


B. The patient has a urine infection
C. The patient's urinalysis is consistent with normal pregnancy
D. The patient's urine sample is contaminated
E. The patient has kidney disease

63. A 22-year-old G2P0 at 33 weeks' gestation is found to have a BP of 166/114 mmHg on a


routine office visit. Her BP at her first prenatal visit at 7 weeks was 124/72. Her urine dip at this
most recent visit shows 3+ protein although it previously had 0 to trace protein. The patient is
also complaining of a persistent headache although she has no history or migraines. Her ALT and
AST are elevated at 92 and 105, respectively. After starting an antihypertensive agent,
magnesium sulfate, and betamethasone, what do you do next?
A. Order bed rest until week 37, then amniocentesis for fetal lung maturity, then deliver
B. Order bed rest for the remainder of the pregnancy
C. Immediate induction of labor for anticipated vaginal delivery
D. Immediate delivery via cesarean section
E. Continue expectant management
64. A 19 year old G3P1 patient with a history of miscarriage in her last pregnancy presents to the
emergency department with some vaginal spotting. She reports that her last menstrual period
occurred 6 weeks earlier. She has had no vaginal discharge other than the spotting, no cramping,
and no abdominal pain. Her physical examination reveals a slightly enlarged uterus, no
tenderness, and a closed cervical os. A serum b-hCG level is sent off and returns 346. A pelvic
ultrasound shows no intrauterine pregnancy, a 2-cm left ovarian cyst, and no free fluid. Your
diagnosis of this patient is which of the following?

A. Threatened abortion/rule out ectopic pregnancy


B. Ectopic pregnancy
C. Inevitable abortion
D. Missed abortion
E. Normal pregnancy

65. In the patient described above, when should a repeat b-hCG be drawn?
A. 24 hours
B. 48 hours
C. 72 hours
D. 1 week
E. It does not need to be drawn again

66. . A 25-year-old G1 PO al 9 weeks' gestation presents for her initial prenatal visit. She has no
medical or surgical history and has a certain LMP that is consistent withher examination. The
patient has donated blood in the past and knows that she is Rh negative. In which of the following
situations would the patient be cared for without being treated with RhoGAM?

A. First trimester bleeding


B. Second trimester beeding
C. Routinely at the beginning of the third trimester
D. Contractions at 34 weeks' gestation
E. At the time of an amniocentesis

A woman 45 years old with 2 children came to the specialist clinic in the international hospital
with the complaint of post coital bleeding since 3 months ago. The lady had the first
menstruation at the age or 13 years old and got married at the age of 17 years old. From the
vaginal examination. 0,5 cm mass was seen in the upper lip of the vagina without involvement of
the vaginal mucosa

67. What is the most likely initial procedure has to be taken after seeing the condition of the cervix
A. VIA
B. Biopsy
C. Leep
D. Pap smear
E. Endocervicat curettage
68. What physical diagnostic examination is important in that situation?
A. Palpation or the cervical mass
B. Rectal examination
C. Rectovaginal examination
D. Palpation of the vagina
E. Vaginal examination

69. What is the most likely stage of the cases?

A. Stage lA1
B. Stage IA2
C. Stage IIIA
D. Stage IB1
E. Stage IB2

A 69 years-old woman with pelvic prcssure und palpable bulge presents for evaluation. She
recalls some mention of a cystocele diagnosis, given by her primary care provider. Today, she
requests format evaluation by a gynecologist.

70. When performing the physical examination, what is one type of staging system to describe
prolapse?
A. Pelvic organ prolapse quantification scale (POP-Q)
B. Gray scale
C. Visual analog scale
D. Breslow scale
E. Clark scale

71. A 25-year-old lady G1 term pregnancy comes to delivery room in active phase of labor. You
perform CTG, and you find the CTG as the following.

A. Head compression
B. Cord compression
C. Maternal chronic anemia
D. Severe preeclampsia
E. Uteroplacental insufficiency
72. The sonographic appearance of the endometrium during the menstrual cycle correlates with
the phasic changes in its histologic anatomy. Which phase of the cycle is depicted with the
classic trilaminar appearance as shown below?

A. Menstrual
B. Secretory
C. Proliferative
D. Periovulatory
E. Lutheal phase

73. A 28-year-old G2P0 at 39 weeks is in early labor. She is 2 cm dilated and 90% effaced, with
contractions every 4 to 5 minutes. The fetal heart tones are reassuring. Her nurse steps out for
a moment and returns to find her having a seizure. This patient is most at risk for mortality
fromwhich of the following complications? (LANGE)

A. Infection
B. Uremia
C. Congestive heart failure
D. Fever
E. Cerebral hemorrhage

74. Which of the following is the characteristic histologic renal lesion of preeclampsia cases?
A. Glomerular endotheliosis / glomerular endothelial swelling
B. Interstitial fibrosis
C. Crescent formation
D. renal cortical sclerosis
E. glomerular sclerosis

75. Which of the following test would be the most effective in identifying an underlying cause of
recurrent miscarriage?
a. Antithrombin III assay
b. serum progesterone level
c. Lupus anticoagulant assay
d. luteinizing hormone assay
e. serum estrogen level
76.A 29 yo G3P1 present complaining of no menses for 4 months after stopping her birth control
pills. She is concerned that the use of oral birth control pills (OCPs) has left her with
amenorrhea. The patient has had no recent changes weight, exercise two to three times a
week, and notes no particular changes in either work or home life. Her obstetric history
includes a therapeutic abortion at age 21, a normal spontaneous delivery at age 25, and a
miscarriage at age 27. After dilatationand curettage at the time of miscarriage, the patient
was hospitalized with infection of her uterus. Since that day, she has taken OCPs, given this
history, which of the following is the most likely etiology of this patients amenorrhea?
a. Vaginal agenesis
b. Asherman syndrome
c. Mayer Rokitansky-Kuster-Hauser syndrome
d. testicular feminization
e. hypogonadotropic hypogonadism

77. A 65 yo G2P2002 presents to the urogynecology clinic with complaints of urinary


incontinence. She leaks urine occasionally when she coughs, but also leak urine without
provocation. She often has difficult making it to the rest room in time. She has never even leaked
urine shortly after normal emptying her bladder. She normally gets up at least 2 to 3 times per
night to urinate. Urinalysis and urine culture performed last week at her PCPs office are both
negative. What is the most likely diagnosis and treatment for this type of incontinence?
a. Stress urinary incontinence, Ditropan (Oxybutynin chloride) mixed incontinence
b. Detrusor overactivity, Detrol (Tolterodine)
c. overflow incontinence, cholinergic agents frekuensi, stress, tanpa urgensi
d. detrusor overactivity, suburethral sling
e. stress urinary incontinence, suburethral sling

78. A 18 yo G0 present to your clinic with primary amenorrhea. She has normal breast
development, but has limited development of pubic and axillary hair. On pelvic examination, you
note that she has a forshortened vagina transvaginal and transabdominal ultrasound unable to
identify a uterus. Chromosomal analysis shows that the patient is 46 XY. What is this patient
diagnosis?
a. testicular feminization
b. gonadal agenesis
c. Complete androgen insensitizing (AIS)

Sudah dibahas………..
UNAS MARET 2016 (JAKARTA)

1. Which of the following is a disadvantage of uterine exteriorization to repair the


hysterotomy during caesarean delivery? (usulan unsri maret 2016)
a. Increased blood loss
b. Increased operative injury rate
c. Increased nausea and vomiting
d. Increased postoperative infection rate
e. Increased the operative time

A 28-year-old G1P0 woman at 28 week’s GA comes to your office for a routine prenatal visit. She
works as a kindergarten teacher and one of her students was recently sent home with a rash and
fever. She states that the child had a rash on both cheeks and paediatrician said was a viral
infection called fifth disease. She relates the baby is moving well and denies any vaginal bleeding,
abnormal vaginal discharge, or contractions. She wonder if she needs any more testing to see if
she has been affected.

2. What is the most likely causative organism of the chil’s infection? (blueprint ed 6)
a. Parvovirus
b. Varicella
c. CMV
d. Toxoplasmosis
e. Listeriosis

3. You sent serologies for the agent above and they come back showing the patient has a
positive IgM and negative IgG consistent with an acute infection. What is the most
common fetal/neonatal complication of this infection during pregnancy? (blueprint ed 6)
a. Fetal anemia
b. Preterm labor
c. Premature preterm ROM
d. Fetal anomalies
e. Oligohydramnios

A 34-year-old G3P2002 woman at 38 week and 6 day was admitted to labor and delivery unit for
active management of labor after it was determined that her membranes had ruptured and she
was dilated to 3 cm. Her cervix has been steadily dilating and now she is at 6 cm. She is very
uncomfortable and findings her contractions very painful. Her partner is also very concerned that
she needs pain relief. BLUE PRINT
4. You advise your patient that
a. Narcotics are available, but should be reserved for closer to the time of delivery when
her pain will be greatest
b. If she continues with natural childbirth and eventually needs a cesarean section she will
require general anesthesia
c. Spinal anesthesia is her best option because it gives a constant infusion of medicine over
a long period of time
d. She cannot have an epidural yet because she is not yet in the active phase of labor
e. A variety of relaxation techniques can be incorporated into her labor in addition to pain
medication

5. With adequate pain control she dilates to 10 cm and second stage begins. Which of the
following is the correct order of the cardinal movements of labor?
a. Internal rotation, engagement, descent, flexion, external rotation
b. Engagement, descent, internal rotation, flexion, external rotation
c. Internal rotation, descent, Engagement, flexion, external rotation
d. Engagement, descent, flexion, internal rotation, external rotation
e. Engagement, descent, internal rotation, flexion, external rotation

A 38-year-old G3P2002 woman present at 40 weeks 3 days withcontractions to labor and delivery
triage. Contractions started 1 hour ago and are very painful. The patients denies leaking fluid but
did notice blood and mucus on her underwear. The baby has not been particularly active since
contractions started. Her prenancy has been complicated by A2GDM. Fasting blood glucose are
usually between 80 and 90 mg/Dl with 1-hour postprandial values between 120 and 140 mg/Dl.
Her prenancy weight was 130 lb-and she is 5 ft 5 in (BMI 21.6). She has gained 30 lb this pregnancy
(BMI 26.6). Hemoglobin A1C 1s 6.0%. Fetal ultrasound at 20 weeks demonstrated normal fetal
anatomy. Repeat ultrasound for growth at 38 weeks demonstrated fetus with weight in the 90 th
percentile and an EFW of 4,350g. Her last pregnancy was complicated by A1GDM and she
delivered a 4,200 g infnt without complications. Initial cervical examination reveals dilation of 6
cm. 50% effacement and 21 station. Two hours later the nurse calls you to the room after the
patient’s water break. Examination shows complete dilation nd effacement and fetus at 11station.
The patient has a strong urge to push and begins pushing. The selivery is complicated by a
second5degree perineal laceration and begins pushing. The delivery is complicated by a
second5degree perineal laceration and a postparfum hemorhage of 600 mL. Fetal weight is 4,560
g and the Apgar scores are 6,8.

6. What neonatal risks are most commonly present in macrosomic fetuses?


• Jaundice
• Hypogglycemia
• Hyperglycemia
• Birth trauma
• Asthma
• Hypocalcemia
a. 1,2,4,6
b. 1,3,4,5
c. 3,4,5
d. 1,3,5,6
e. 2,4,5,6

A 36-year-old G7P50015 woman has just delivered a 4,500 g femaleinfant at 39 weeks gestation.
She underwent induction of labor withoxytocin for severe preeclampsia diagnosed with systolic
BPs elevated to 160 mm Hg. Her pregnancy was complicated by uncontrolledgestational diabetes
and resultant polyhydramnios. She was placed on magnesium throught her induction for seizure
prophylaxis. Shehad en epidural placed during the firts stage of labor and remainedon a normal
labor curve throughout. Her second stage of labor lasted 3 ½ hours; she was, however, able to
delivery
vaginally with preemtive McRoberts maneuvers and steady traction. The third stage of laborlasted
10 minutes and the placenta was delivered intact. Immediatelyafter the third stage her bleeding
was significant with the expulsion ofblood clots and a fundus that was notable for bogginess.

• Which of the following are not risk factors for postpartum hemorrhage?
a. Advanced maternal age
b. Grand multiparity
c. Prolonged use of oxytocin during labor
d. Polyhydramnios
e. Prolonged exposure to magnesium during labor

• Which of the following medications would be contraindicatioin the treament of uterine atony
in this patient?
a. Methylergonovine (Methergine)
b. Carboprost (Hemabate, PGF2-alpha)
c. Intramuscular Pitocin
d. Misoprostol (PGEI)
e. Calcium gluconate

Your next patient is a 13-year-old adolescent girl who present withcyclic pelvic pain. She has never
had a menstrual cycle. She denies anyhistory of intercourse. She is afebrile and her vital sign are
stable. Onphysical examination, she has age-appropriate breast and pubic hairdevelopment and
normal external genitalia. However, you are unableto locate a vaginal introitus. Instead, there is a
tense bulge where the introitus would be expected. You obtain a transabdominal ultrasound,
which reveals a hematocolpos and hematometra.

• What is the most likely diagnosis?


a. Transverse vaginal septum
b. Longitudinal vaginal septum
c. Imperforate hymen
d. Vaginal atresi (MRKH)
e. Bicornuate uterus

A 23-year-old G0 woman present complaining of increasing pelvicpam with her menses over the
last year since she stopped her OCPs. In particular, she has noticed more pain on her left side in the
last couple of months. She denies any changes in her bladder or bowel habits but reports that she
has begun to have pain with deep penetration during intercourse. She started OCPs when she was
17 for painful irregular cycles but stopped them a year ago when her insurance changed. She hs
had only one lifetime sexual partner and no history of sexually transmitted infections. She would
like to preserve fertility. On examination, she has no abnormal discharge but her uterus is tender
as well as her left adnexa. You appreciate fullness that you suspect maybe a mass. On pelvic
ultrasound she has a 5 cm cystic ovarian mass thought to be an endometrioma. It persists in
repeat ultrasound 8 weeks later and the patient is still symptomatic.

• What whould be the most appropriate next step in her care?


a. Resume an oral contraceptive
b. Schedule diagnostic laparoscopy with left ovarian cystectomy
c. Prescribe an NSAID for her pain and repeat the ultrasound in 6 to 8 weeks
d. Prescribe a GNRH agonist (i.e, Depo-Lupron)
e. Refer her to a gynecologic oncologist

11. You perform a laparoscopic left ovarian cystectomy and notethat the cyst is a “chocolate
cyst.” She alo has other superficial implats of endometriosis on uterosacral ligaments. The
final pathology report is consistent with an endometrioma. At your patient’s postoperative
visit 2 weeks after surgery she tells you that her pain is resolved and she is feeling well. What
do you recommend for the continued postoperative management of her endometriosis?
a. Because endometriosis cannot be cured medically, she should undergo total
hysterectomy with bilateral salpingo-oophorectomy
b. You were able to completely remove the cyst, so she does not need ny further therapy at
this time
c. Wait 6 months and then schedule a repeat laparoscopy to make sure there is no
further endometriosis that needs to betreated
d. Initiate therapy with a combined oral contraceptive or a progestin to delay the return of
her previous symptoms
e. Endometrial ablation because that will destroy her endometrium and decrease the risk
of new implants developing from retrograde menstruation.

A 21-year-old nulligravid woman present to her gynecologist with a 3 day history of painful genital
ulcer. Last week she had a low-grade fever And generalized malaise, which has since resolved. She
denies any history of genital ulcers. She has had four new sexual partners in the inconsistently. On
genitourinary examination, several 1 to 2 painful vesicles are noted on the left labia minora. There
is no ingunal lymphadenopathy.

12. Which of the following is the most likely causal organism?


a. Treponema pallidum
b. Herpes simplex virus
c. Trichomonas vaginalis
d. Chlamydia trachomatis L1, L2, or L3
e. Haemophilus ducreyi

13. Assuming that additional testing was performed, which of the following results would
you most likely expect to find?
a. T. pallidum particle agglunation assay (TPPA), positive
b. H. ducreyi culture, positive
c. HSV-2 IgG, positive
d. HSV-1 IgG, POSITIVE
e. None of the above. Diagnostic testing is not reliable and treatmentshould be based
on clinical suspicion

14. What is the bet initial treatment for this patient?


a.Ceftriaxone 250 mg IM once  GONOREA
b.Imiwuimod (Aldara) applied to affected area three times perweek GENITAL WARD
c. Acyclovir 200 mg orally five times daily for 7 days  HERPES
d.Benzathine penicilin G 2.4 million units IM once  SIFILIS
e.Dixycyline 100 mg orally twice daily for 21 day  CLAMYDIA T

A 69-year-old woman with pelvic pressure and palpable bulge presents for evaluation. She recalls
some mention of a cystocele diagnosis, given by her primary care provider. Today, she requests
formal evaluation by a gynecologist
15. When performing the physical examination, what is one type of staging system to
describe prolapse?
a. Pelvic organ prolapse quantification scale (POP-Q)
b. Gray Scale
c. Visual analog scale --. NYERI
d. Breslow scale
e. Clark scale

16. In discussing her symptoms, the patient points out that her voiding functions has changed as
the prolapse has grown in severity, Initially, the patient reported stress urinary incontinence,
but as the prolapse worsened, the incontinence improved. While she is happy with the
resolution of her incontinence, she currently experiences some incomplete bladder emptying,
which is improved upon manual reduction of the prolapse. How do you counsel her about her
risk of incontinence after an isolated anterior wall repair (with no other concomitant
surgery)?
a. High likelihood of de novo urgency and urge urinary incontinence
b. High likelihood of urinary frequency
c. High likelihood that her stress incontinence will be cured by anterior repair
d. High likelihood than an anterior repair will unmask and potentially “worsen” her stress
urinary incontinence symptoms
e. High likelihood of de novo fecal incontinence

18. In assessing the above patient, you also find a posterior vaginal wall defect. What is a
common symptom that is associated withrectoceles?
a. Urinary urgency
b. Hematuria
c. Incomplete evacuation of stool that may require splinting
d. Vaginal bleeding
e. Vaginal wall erosion

19. On pelvic examination the vagina is well-healed, and you do not see any lesions or
active leaking of urine into the vagina. Further testing at this time may include:
a. Cystourethroscopy
b. Dye testing with retrograde filling of the bladder with methylene blue or indigo carmine
c. IV dye testing with indigo carmine
d. All of the above
e. None of the above

A 63-year-old G3P2 woman is referred to the urogynecology clinic for evaluation of urinary
incontinence. Urinalysis and urine culture done by her PCP 1 week earlier were negative. Her
medical history’s positive for hypertension and osteoarthritis. She is complaining of leakage
of urine following an overwhelming need to void. She runs to the bathroom, but leaks a large
amount before she makes it to the toilet. She also has urinary frequency and empties her
bladder every 1 to 1.5 hours during the day and gets up four times at night to void. She denies
loss of urine with cough, sneeze, and exercise.
20. From this patient history, what is your initial diagnosis?
a. Urgency incontinence, URGENCY TANPA STRES
b. Overflow incontinence STRES, FREQUENSI TANPA URGENCY
c. Stress incontinence, STRES AJA
d. Mixed incontinence, STRES, URGENCY, FREQUENCY
e. Continuous incontinence secondary to a urinary fistula

21. As her pregnancy continues, you would expect her cardiac output to increase by which
of the following mechanism:
a. First an increase in stroke volume, then an increase in heart rate
b. A decrease in system vascular resistance
c. Cardiac output would not change significantly until the third trimester
d. An increase in systemic vascular resistance facilitated by elevated progesterone
levels
e. Increased heart rate alone

22. Which of the following is true regarding the physiologic changes she might expect
during her pregnancy?
a. Gastric emptying and large bowel motility are increased in pregnancy
b. BUN and creatine will decrease by 25% as a result of an increase in glomerular
filtration rate (GFR), which will be maintained until delivery
c. An overall decrease in the number of WBC and platelets
d. An increase in the tidal volume along with an increase in total lung capacity (TLC)

A year-old G2P1001 women present to your office with the chief complain Of amenorrhea since
her most recent vaginal delivery 1 years ago. She notes that she had an uncomplicated
pregnancy, followed by the delivery of a healthy baby boy. Her delivery was complicated by
an intra-amniotic infection as well as a post partum haemorrhage requiring a postpartum
dilation and curettage. After her delivery, she breastfeed for 6 months, and during this time
she had scant and irregular vaginal bleeding. After stopping breastfeeding 6 months ago, she
notes that sence of mense, but instead has monthly painful cramping, which seems to be
getting worse. She remarks that prior to her pregnancy, she had normal regular menses, which
were not too heavy or painful. She and her husband would like to have another child, and have
been having unprotected intercourse four the past 6 months without achieving a pregnancy.
You r review of system is otherwise negative. Your perform a physical examinations, which is
normal other than a slightly enlarged, tender uterus. A urine pregnancy test in the office is
negative.
23. What is the most likely diagnosis?
a. Sheehan’s syndrome
b. Lactational amenorrhea
c. Asherman’s syndrome
d. Premature ovarian failure

24. You suspect Asherman’s syndrome and perform a hysterosalpingogram, which reveals
multiple synechiae within the uterus, confirming your suspicions. Your next step in
therapy is which of the following
a. Diagnostic and operative hysteroscopy
b. Provera 10 mg daily for 5 days in an attempt to achieve a withdrawal bleed
c. In vitro fertilization
d. Place an intrauterine device
e. Inform your patient that unfortunately, she is “barren” and will not be able to carry
a pregnancy gain

An 18- year-old G0 F present to your office for contraceptive counseling. She has never used
any method of contraception before and is engaged in a monogamous sexual relationship.
Gynecologic history is significant for regular, heavy menstrual cycles using up to eight pads
per day, lasting up to 7 days t a time, with severe pain (dysmenorrhea). She smokes one-half
pack of cigarettes per day and tells your hat her mother and aunt both have factor V Leiden
disease, but that she has never been tested herself or had a thromboembolic event. She will
attend college soon and has no plats for a pregnancy in the near future. She indicates her
desire for the “most reliable” method of contraception that you can offer.
25. Which of the following methods of contraception has the lesser efficacy?
a. Ortho Evra patch
b. Combined oral contraceptive pills
c. Mirena IUD
d. Condoms with spermicide
e. Coitus interruptus

26. Of course, during your discussion at this visit, you could encourage smoking cessation
and recommend weight loss to help improve her overall health. You and the patient
have decided to proceed with IUD placement. Prior to placement, it is important to
perform which of the following test?
a. FSH level
b. Prolactin level
c. Urine pregnancy test
d. Gonorrhea/Chlamydia testing
e. Both c and d

A 30-year-old GO comes in for her annual examination and tells you that she plans to become
pregnant sometime in the next year. She had a LEEP procedure 5 years ago at another facility
for moderate dysplasia. You verify her pap tests have all been negative since the LEEP, but
her last pap was 2 years ago.
27. What is the recommended cervical cancer screening for this patient ?
a. Pap testing every 6 months  salah usia > 35tahun
b. Pap testing and colposcopy every 6 months
c. Pap testing every 3 years  salah krn usia > 30 thn
d. Pap testing and high-risk HPV testing every 5 years  usia > 29 th
e. She has been treated with the LEEP, so she no longer needs pap smear

28. Her Pap test returns HSIL and she is high-risk HPV positive. You have her return for
colposcopy. After the application of acetic acid, you see a large, dense, white area with
mosaic vessels encompassing the entire anterior cervix and extending into the
endocervical canal. You obtain a biopsy of this area and perform an endocervical
curettage. The pathology reports for both biopsies are CIN III. What treatment do you
recommend? (Blueprint)
a. Leep in office
b. Cryotherapy
c. Cold-knife cone or two stage LEEP in OR
d. Simple hysterectomy
e. Radical hysterectomy

A 62-year-old woman present to the office complaining of watery vaginal discharge and
bleeding for the past 2 months. She has not had a Pap test in 14 year. She states she had a
mildly abnormal pain her 3Os, but that was treated with cryotherapy. She states she went
through menopause at age 50 and has never been on hormone replacement therapy. She
does admit to smoking one half pack a day for 40 year. Her husband is decease, and she has
not been sexually active in 10 years. Her examination reveals cervical necrotic mass
approximately 5 cm in size. Rectovaginal examination reveals a cervical necrotic. There is no
evidence of adnexal masses, but examination of the uterus and adnexa is limited by the
patient/s body habitus. You suspect this may be cervical cancer. You obtain a Pap smear and
take a biopsy of her cervical abnormality. The Pap test returns with a reading of SCC, and the
biopsy confirms this diagnosis. She also received a cystocopy for hematuria with positive
urine cytology. The biopsy also shows SCC. You order a CT scan, which shows a cervical mass
measuring 7.7 3 5 0 cm as well as an avid left internal iliac lymph node consistent with locally
metastatic disease.

29. What is the International Fedration of Gynecology and Obstetrics (FIGO) stage for her
cancer?
a. Stage I
b. Stage II
c. Stage III
d. Stage IV

30. Mrs. S, 34 years, G4P3 A I 39 weeks gestational age, second stage of labour. The head of
the baby was delivered but the shoulder was stuck. Turtle sign (+). The mother has an
uncontroled diabetes for 5 years. Estimated fetal weight by ultrasound was 4200 gram.
What is this condition called?
a. After coming head
b. Compaction
c. Retention of the baby
d. Shoulder dystocia
e. Collision

Mrs. S, 32-year-old G4P3 Ao gravid with chronic hypertension had a normal labor that arrested in
second stage at + 1 station. She complained of mild dyspneu and fatigue. The fetus had a left
occiput anterior presentation and was delivered by forceps. Completion of third stage followed
quickly, and the fundus was noted to be firm. The Obgyn was carefully examined, and no laceration
was noted. The examiner then noted the lower uterine segment was boggy.--> lembek
31. Which of the following is suitable treatment in situation above? (Williams obstet
24 th 41-19)
a. Hemabate, 250 ug intramuscularly
b. Methergin, 0,2 mg intramuscularly
c. A 20-unit oxytocin intravenous bolus
d. Misoprostol 400 mg, intrarectally
e. Carboprost 0,25 mg intramuscularly

32. For Mrs. S, one dose uterotonic agent is given and the fundus is massaged. Despite this
she continues to bleed. Which of the following is suitable treatment in this situation?
(Williams obstet 41-10)
a. Administer methergin 0,2 mg intramuscularly
b. Mobilize a team that include obstetricians, nurses and anesthesiologist
c. Perform laparotomy to prepare postpartum hysterectomy
d. Apply ballon catheter
e. Uterine and ovarian artery ligation

33. The patient continues to bleed and you have initiated whole blood transfusion. Which
of the following is suitable treatment in this situation?
a. Continue to administer Hemabate intramuscularly every 20 minutes
b. Insert Bakri postpartum Balloon or large Foley catheter balloon into the uterine
cavity and inflate the balloon
c. Consider laparotomy and uterine compression suture placement
d. All of the above
e. a and b

34.A A pelvic ultrasound reveals a right sided ectopic pregnancy as well as large amounts
of fluid, thought to be blood in the abdomen. She now has IV access and a bolus of IV
fluids is being given. Her BP is now 78/45 and her pulse rate is 112 beats per
minute. Her hematocrit returs as 27,2%. How will you proceed?
a. Administer IM methorexate
b. Transfuse the patient with two units of packed RBCs and transfer her to the ICU
c. Proceed with a laparoscopic salpingectomy
d. Proceed with emergent laparotomy
e. Start vasopressor and transfer the ptient to the ICU

35. An 89-year-old female patient with multiple, serious medical comorbidities present to
discuss options for treatment of her high-grade prolapse. The prolapse is externalized
and becoming ulcerated from friction against her undergarments. She cannot tolerate
a pesarry. Her main priority is to ‘fix or get rid of this thing,’ but her primary care
provider has cautioned against a lengthy or open abdominal procedure. She is not
interested in future intercourse. What can you offer this patient?
a. Nothing can be done
b. Open abdominal sacral colpopexy
c. Robot-assisted laparoscopic sacral calpopexy
d. Hysterectonmy with anterior and posterior colporrhappy, vault suspension
e. Colpocleisis
36. The drug that blocks the conversion of arachidonic acid to thromboxane A2 while
sparing prostacyclin production in treatment APAs;
a. Unfractionated heparin
b. Aspirin
c. Azathioprine
d. Cyclosporine
e. Cyclophosphamid

37. A 28-year-old G2P0 at 39 weeks in in early labor. She is 2 cm dilated and 90% effaced,
with contractions every 4 to 5 minutes. The fetal heart tones are reassuring. Her nurse
steps out for a moment and returns to find her having a seizure. The nurse administered
a 4-g magnesium bolus. The seizure stops. The fetal heart tone variability is flat, but
there are no decelerations. This patient is most at risk for mortality from which of the
following complications?
a. Infection
b. Uremia
c. Congestive heart failure
d. Fever
e. Cerebral hemorrhage

38. A 40- year-old woman is seen for a routine examination. Her menses have been regular,
and she has no complains. Findings, including those on pelvic examination, are normal.
Ten days later, her Pap smear is returned as ‘ high-grade squamous intraepithelial
lesion”. Which of the following options is the best course of action?
a. immediate wide-cuff hysterectomy
b. repeated Pap smear at 3-month intervals
c. Fractional dilation and curettage (D&C)
d. punch biopsy of anterior cervical lip
e. Colposcopy with biopsy

39. A pelvic ultrasound reveals a right-sided ectopic pregnancy as well a large amounts of
fluid, thought to be blood in the abdomen. She now has IV access and a bolus of IV fluids
is being given. Her BP is now 78/45 and her pulse rate is 112 beats per minute. Her
hematocrit returns as 27,2%. How will you proceed?
a. Administer IM methotrexate
b. Transfuse the patient with two units of packed RBCs and transfer her to the ICU
c. Proceed with a laparoscopic salpingectomy
d. Proceed with emergent laparotomy
e. Start vasopressors and transfer the patient to the ICU

40. G3P2A0, 32 years old, referred from primary health care with term pregnancy and post
cesarean section. She has a history of cesarean delivery of her first child due to placenta
previa. Factors affecting the likelihood of successful trial of labor in this patient is:
a. Prior cesarean for malpresentation
b. Spontaneous labor with advanced cervical dilation on admission.
c. Prior vaginal delivery
d. Increased maternal age
e. Recurrent indication for initial cesarean delivery

41. With regard to HIV in Pregnancy: (MRCOG MCQ Arnold 44)


a. A positive HIV blood test in pregnancy is not reliable
b. A high maternal HIV RNA load decreases the mother-to-child transmission of HIV
c. Use of antiretroviral agents is always commenced in the first trimester of
pregnancy
d. HIV infection increases the mother-to-child transmission of the hepatitis C virus
e. If are ruptured membrane for 6 hours, there is no advantage to delivery baby boy
Section

42.A A One woman, married, G3P3A0, 43 yo, no history of abdominal surgery, has been
getting a diagnosis as cystic ovarian neoplasm with a size of 6 months pregnant. Limited
mobility mass, not pain. Hb 11,5 gr o /dl, Ca-125 1U. Other laboratory
examinations, Investigations and cytology smears perioperative smear within -normal
limits. Choose the correct answer:

a. Vertical midline incision offers inferior access to the upper abdomen if greater space or
access is needed
b. Transverse incision is used commonly in gynecologic malignancy surgery.
c. Vertical midline incision was not recommended in patients who have coagulopathy,
declined transfusion, or are administered systemic anticoagulation
d. Nerve injury to the iliiohypogastric, illoingumal and femoral nerves in Pfannenstiel incision
was more commonly involve sensory function and typically are transient

43. Tile couple came to the clinic with complaint : to get pregnant. This couple has been
married 3 years. Current wife age 36 years, height 151 cm and weight 73 kg, a history of
sexual intercourse 2-3 time a week- From anamnesis we found a history of menstrual
pain since the age of 20 years and intermittent treatment for vaginal discharge odour
complaints and itchy since before marriage. In this couple:
a. Infertility services can be given by qualified practitioners who can provide consultation,
education and advice to both partners and have knowledge of the terms of
reproductive success and problems
b. Given infertility service by health workers who have experience and documented certified
to perform the procedure endocrine, gynaecologic and urology, have extensive knowledge
about the effectiveness, side effects, the cost of doing the diagnosis and treatment
of infertility.
c. Treatment is aimed to be able to determine the cause of infertility from both sides as well
as determine whether the couple need to get service at a higher level of service
d. Infertility service that require special expertise because it includes the action assisted
reproductive
e. Having practioner qualification certification and experienced in TRB, Urology or
andrology and qualifies as an infertility counsellor. You are going to make critical skill
training. There are some problems that you are worried to get in worried to get in
relation of training approach

Sampun nggihh
Matur nuwun ……

UNAS MARET 2015

1. A 20-year-old woman, in her first trimester come to the ante-natal clinic (ANC) for
routine care. After counseling she volunteered for HIV testing and was found to be HIV
postive. During pregnancy HIV transmission occurs mostly during:
a. 1st trimester
b. 2nd trimester
c. 3rd trimester
d. During labour
e. During lactation

2. With regard to HIV in pregnancy:


a. A positive HIV blood test in pregnancy is not reliable
b. A high maternal HIV RNA load decreases the mother-to-child transmission of HIV
c. Use of antiretroviral agents is always recommeneed in the first trimester of pregnancy
d. HIV infection increases the mother-to-child transmission of the hepatitis C virus
e. If there are ruptured membranes for 6 hours, there is no advantage to delivery baby by
caesarean section

3. A woman with a previous stillbirth and postpartum DVT is found to have lupus anticoagulant
and medium titer Immunoglobulin M (IgM) anticardiolipin antibodies (aCL) on two occasions.
In a subsequent pregnancy
a. She has an increased risk of miscarriage
b. Low dose aspirin shoud be discontinued at 34 weeks
c. Warfarin should be continued  meningkatkan intracranial hemorrage
d. She does not require postpartum heparin if she has vaginal delivery
e. She requires antibiotic prophylaxis to cover delivery

4. The following drugs IS NOT appropriate for a woman with SLE who is 35 weeks pregnant:
a. Diclofenac
b. Prednisone
c. Hydroxychloroquine
d. Sulfasalazine
e. Azanthioprine  gol D pada sedmua trimester

A 69 year-old-woman with pelvic pressure and palpable buldge presents for evaluation. She recalls
some mention ofa cystocele diagnosis, given by her primary care provider. Today, she request
formal evaluation by a gynecologist. (Blueprints OG Lippincott chapter 18)  vignette 1 no 2

5. In discussing her symptoms, the patient points out that her voiding function has changed as
the prolapse has grown in seventy. Initially, the patient reported stress urinary incontinence,
but as the prolapse worsened, the incontinence improved. While she is happy with the
resoulution of her incontinence, she currently experiences some incomplete bladder
emptying, which is improved upon manual reduction of the prolapse. How do you counsel her
about her risk of incontinence after an isolated anterior wall repair (with no other
concomitant surgery)?
a. 90 % of de novo urgency and urge urinary incontinence
b. 90 % of urinary frequency
c. 95 % that her stress incontinence will be cured by anterior repair
d. 95 % that an anterior repair could "worsen" her stress urinary incontinence symptoms
e. 90 % of de novo fecal incontinence

6. In assesing the above patient. you also find a posterior vaginal wall defect. What is a
common symptom that is associate with rectoceles?
a. Urinary urgency
b. Hematuria
c. Incomplete evacuation of stool that may require splinting
d. Vaginal bleeding
e. Vaginal wall erosion

A 37 y.o woman has just delivered her second baby, she had a previous premature delivery at 34
weeks. After the placenta was delivered the midwife noticed a perinea) tear. The tear extended
from introitus in the midline and she could see torn muscle fibers suggestive of the torn end of the
external anal spinchter. She has called you to review the patient.

7. What is the likely diagnosis?


a. Second degree tear
b. 3a degree tear
c. 3b degree tear
d. 3c degree tear
e. 4 degree tear

8. What factors predispose to this condition?  First stage of labour more than 18 hours Didn't
performed an episiotomy (jawaban)
a. First stage of labour more than 18 hours
b. Didn’t performed an episiotomy
c. Nulliparity
d. Persistent occipito anterior position
e. Ventouse delivery

9. How would you manage this patient?


a. Primary sutured
b. Consultation to the colorectal surgeon
c. Should be transferred to operating theatre
d. Elective C-section should be performed for subsequent delivery
e. Should be discharged in two days

Ny. X 24 years came to the ER with complaints of headaches since the last day of examinations
obtained expecting her first child, gestational age 37-38 weeks blurred vision denied heartburn (-)
on physical examination found BP 190/120 mmHg pulse 90 x/m breathing 16 x/m at Leopold found
the lower left back head FHR 140 x/m contraction irregularity in the examination pelvic score of 1
was found, pelvis size wide laboratory investigation hb obtained 11.5 g%.. platelets 9000/
mm3..LDH 510 iu IL..Proteinuh +2.. SGOT 10 u/L SGPT 15 u/L
(usulanUNAND)

10. What is the best diagnose for ny.X


a. Chronic hypertension
b. HELLP syndrome
c. Gestational hypertension
d. Severe pre eclampsia
e. Superimposed pre eclampsia

11. What is the most clinically effective antihypertensive agent for ny.x
a. Magnesium sulfat
b. Diazepam
c. Hidralazin  seharusnya jawaban nifedipin
d. Hydrochlorothiazide
e. Clonidine

12. What is the best management for ny. X


a. Induction of labour
b. Cervical ripening
c. Caesarean section
d. Forcep extraction
e. Vaccum ectraction

13. If ny. X breastfeeding what is the best anti-hypertensive agent should you give
a. Hydrochlorothiazide  nifedipin gol C untuk ibu hamil
b. Furosemide
c. Spirolactone
d. Labetalol
e. Dopamine

Mrs. Ani 38 year-old-grande multiparous, post vaginal delivery with oxytocin drips and spinal
anesthesia 2 hours ago. Baby born wight 4200 gr, alive. The happen early HPP with blood pressure
80/40 mmHg and the mother is shock.

14. Etiology early HPP Mrs. Ani is:


a. Uterine atony
b. Laceration
c. Retained placenta
d. Coagulopathy
e. Rupture uterine

15. It NOT risk factors early HPP Mrs. Ani is:


a. Grande multiparous
b. oxytocin drips
c. Makrosomia
d. Spinal anesthesia  significant reduce risk of HPP
e. Prolonged labor
Mrs. Selly 20 years-old primipara is 36 hours postpartum following cesarean delivery for failure to
progress. She is complaining of abdominal pain and has a fever of 38 C. She is not yet tolerating oral
intake because of nausea. You diagnose metritis.

16. Which of the following is the most important criterion for the diagnosis of postpartum
metritis: (William 24 hal 684)
a. Uterine tenderness
b. Fever
c. Foul-smelling lochia
d. Leukocytosis
e. Leukopenia

17. Which of the following is the most frequent cause metrtis:(William24 hal 684)
a. Group A streptococcus
b. Group B streptococcus
c. Chlamydia trachomatis
d. Mycoplasma hominis
e. Ureaplasma urealyticum

A woman, married, G3P3AO, 43 years, no history of abdominal surgery, has gained as myoma uteri
diagnosis. Uterine size of 4 months pregnant, good mobility. Hb 12,5gr 0/6 and other laboratory
examinations, investigations and cytology smears perioperative within normal limits

18. Choose the correct answer below:(William Gynhal 1020)


a. Incision of the abdominal wall can be selected midline vertical incision or Pfannenstiel
incision.
b. Vertical midline incision offers a quick entry, but the resulting extensive blood loss
than Pfannenstiel minimal incision.
c. Performed Pfannenstiel incision was excellent cosmetic results can not be achieved.
d. Pfannenstiel incision decreased rates of postoperative pain.
e. Pfannenstiel increased rates of incisional hernia.

19. Choose the correct answer below:(William Gynhal 1045


a. No difference in sexual satisfaction after hysterectomy among the subtotal and total
hysterectomy.
b. Vaginal prolapse is lower rates in the subtotal hysterectomy.
c. There are differences in bladder function in postoperative total hysterectomy
and subtotal hysterectomy.
d. Post subtotal hysterectomy lower risk of bowel disfunction ecents rather than post-
hysterectomy.
e. After subtotal hysterectomy patients do not need to do a pap smears again.

One women, married, G3P3A0, 43, no history of abdominal surgery, has beengetting a diagnosis as
cystic ovarian neoplasms with a size of 6 months pregnant. Limited mobility mass, not pain, Hb
11,5grO/o, Ca-125 IU, Other laboratory examinations, investigations and cytology smears
perioperative Smir within normal limit.

20. Choose the correct answer: (William Gynhal 1022)


a. Vertical midline incision offers inferior access to the upper abdomen if greater space or
access in needed (William Gynhal 1020 -->superior access)
b. Transverse incisions are used commonly in gynecologic malignancy surgery  Iinea
mediana
c. Vertical midline incision was not recommended in patiens who have coagulopathy,
decline transfusion, or are administered systemic anticoagulation pfanenstiel
d. Nerve injury to the iliohypogastric, ilioninguinal and femoral nerves in Pfannensiel
incusion was more commonly involve sensory function and typically are transient.

A 30-year-old, gravida 4, pars 3 at 37 weeks' gestation by last menstrual period in delivery room,
with preexist of cardiac is comfortable a rest, but cannot stand up to brush her teeth without
experiencing chest pain. Adequate and Regular uterine contraction.

21. According of the above base on clinical classification of the New York Heart Association
(NYHA):
a. Class I
b. Class II
c. Class III
d. Class IV
e. Unclassified

22. Plan:
a. Observation and vaginal delivery
b. Induction deliver
c. Cesarean delivery
d. Spinal blockade is the recommended anesthetic
e. Invasive monitoring with pulmonary artery catheter is required

23. Accurate and safe non examination in diagnosis heart disease in pregnancy:
a. Electrocardiography
b. Echocardiography
c. Chest radiography
d. CT angiography
e. Ultrasonography

A P1, 24-year old, Miranda gave birth to 4.3 kg at home and she intended to breast feed her baby.
She was doing fine during the first two hours, but then there was a sudden onset of massive
vaginal bleeding and she was bought to the nearest emergency unit. There she fell unconscious,
her blood pressure fell markedly until unpalpable. RR. 16x/m. Heart rate 120 bpm. Lung were
normal. Uterine fundus was palpable at the level of umbilicus, contraction was poor. Bladder was
empty.
She was put into Trendelengburg position, her head was turned to one side, and an oxygen mask
was used to deliver 5 liters of oxygen per minute. Two intravenous lines were accessible.
At this time bed side clot test showing blood clot in 1 minute and haemoglobin level was found 5 gr
%. Exploration of birth canal revealed no injury requiring further intervention.
24. The statement which fits best for the case above is:
a. Internal bimanual uterine compression is less preferable from external bimanual
uterine compression.
b. Early administration of blood pack was actually inappropriate.
c. The use of colloid to restore intravascular volume might reduce the need for
other intravenous fluid in this case.
d. Colloid alone should be able to prevent hemorrhagic loss in this case.
e. The use of oxytocin is preferable in this case.

25. The most likely birth canal injury requiring intervention in such a case is:
a. Extension of episiotomy wound
b. Obstetric Anal sphincter Injuries
c. Cervical tear
d. Symphysiolysis
e. Inversion uteri

26. The Tredelenburg position helped this case in one of the following mechanism:
a. Preventation of fluid redistribution from intravascular to interstitial space
b. Assuring optimal venous return
c. Prevention of blood overflow to the head
d. Reduction of blood flow to the uterus
e. Preventation of increasing heart rate

P1A0 was delivered by forcep Extraction, born female baby 3600 grams. After placenta delivered,
there was Obstetric Anal Spinchter injury Grade 3C
27. What is the suturing technique to repair the internal anal spinchter needed?
a. Continue suture
b. Interrupted suture
c. Vertical Matrass Suture
d. Horizontal Matrass Suture
e. Overlapping Suture

28. What is the type og ligature to repair the internal anal spinchter?
a. Poligelactin acid 2.0
b. Chromic cut gut 2.0
c. Poliglactin Acid 3.0 anus (vicryl 3.0)
d. Polidiaxon 3.0
e. Chromic gut 3.0

The couple came to the private practice with complaints want to get pregnant. This couple has
been married 2 years. Current wife age 30 years. Height 151 cm and weight 55 kg, a history of
sexual intercourse 2-3 times a week. From anamnesis we found a history of menstrual was regular,
and no history of pain during Menstruation.

29. The wife of the couple asked, overall cause of infertility according to female factors
a. Ovulatory disfunction 40%.
b. Tubal and pelvic pathology 20%
c. Unexplained infertility 25%
d. Unusual problems 5%
e. Social-economic problems 10%

30. The wife of the couple also asked about assessing ovulation disorders, and the right
statement is?
a. Women with regular monthly menstrual cycles should be informed that they are likely
to be ovulating.
b. Women with regular menstrual cycles and more than 2 years infertility no need a
blood test to measure serum progresterone.  cek progesterone hr ke 23
c. The use of basal body temperature charts to confirm ovulation is recommended
and reliably predict ovulation.
d. Women with regular menstrual cycles should be offered a blood test to measure
serum gonadotrophins.
e. Prolactin test does not need to be offered to women who have an ovulatory disorder,
galactorrhea or a pituitary tumour because it was wasting charge.

31. If this sub fertile woman presents with amenorrhea and galactorrhea and her prolactin levels
are elevated. In additional to evaluating her for a prolactinoma, one also needs to evaluate for
other causes that would increase prolactin such as elevated.
a. Corticotrophin-releasing hormone.
b. Dopamine
c. Gamma-aminobutyric acid (GAGA)
d. Histamine type II receptor activation
e. Thyrotropin-releasing hormone (TRH).

32. During the evaluation for infertility, a woman may have an endometrial biopsy to evaluate the
quality of her ovulation since the development of the corpus luteum is most closely associated
with :
a. Fertilization of an ovum
b. Follicular phase of the endometrium
c. Proliferative phase of the endometrium
d. Secretory phase of the endometrium
e. Sheding phase of the endometrium

The couple came to the clinic with complaints: want to get pregnant This couple has been married
years 3 years. Current wife age 36 years. Height 151 cm and weight 73kg, a history of sexual
intercourse 2-3 times a week. From' anamnesis we found a history of menstrual pain since the age
of 20 years and intermittent treatment for vagina discharge odor complaints and itchy since
before marriage. (konsens hiferi)

33. In this case, the couple should get treatment at facilities: -


a. Primary level infertility services
b. Secondary level infertility services
c. Tertiary level infertility services
d. Basic level of infertility services
e. Advanced infertility services

34. In this couple:


a. Infertility services can be given by qualified practitioners who can provide consultation,
education and advice to both partners and have knowledge of the term of reproductive
success and problems.
b. Given infertility services by health workers who have experience and documented
certified to perform the procedure endocrine, gynecology and urology, have extensive
knowledge about the effectiveness, side effects, the cost of doing the diagnosis and
treatment of infertility.
c. Treatment is aimed to be able to determine the cause of infertility from both sides as well
as determine whether the couple needs to get service at a higher level of service.
d. Infertility service that required special expertise because it includes the actions assisted
reproductive technology that can only be performed in specialized infertility clinic
e. Having practitioner qualification certification and experience in TRB, Urology Or
andrology, and qualifies as an infertility counselor.

You are going to make Clinical skill training. There are some problems that you are worried to get,
in relation of successfully of that training approach.

35. The training approach described in this chap ter is guided by principles of adult learning. These
principles are based on the assumption that people participate in training courses because
a. They are interested in the topic. Wish to improve their knowledge or skills, and thus their job
performance. Desire to be actively involved in course activities.
b. Uses behavior modeling, Is competency-based, Incorporates humanistic training techniques.
c. Competency-based, which means assessment is keyed to the course objectives and
emphasizes acquiring the essential knowledge, attitudinal concepts and skills needed to
perform a job, not simply acquiring new knowledge.
d. Less stressful, because from the outset participants, both individually and as a group, know
what they are expected to learn and where to find the information, and have ample
opportunity for discussion with the clinical trainer.

A 28 years old woman comes to outpatient clinic as she feels that the fetal movement was
decreased. She states that this is the first pregnancy and she was forgotten about her last
menstrual period because she had irregular menstruation. During this pregnancy, she has already
performed US in first and second trimester.

36. To confirm the gestational age, which the following tools that can be used
a. Last menstrual period
b. Quickening
c. Fundal height
d. USG in first trimester
e. Fetal weight

37. Ralated to USG result in first trimester, now she has 42 weeks and 3 days of gestational age.
Because she feels that the fetal movement was decreased, which the first step should be
performed
a. USG
b. NST
c. Amniocentesis
d. Reffered to the delivery room
e. Admitted to the hospital

A 36 years old woman comes to the emergency room because of vaginal bleeding. She had four
living child and her last menstrual period about 8 month ago. Examination shows that vital
signwas in normal limit, no contraction or abdominal pain, presenting part was floating, and
there's no heavy bleeding from vagina.

38. The appropriate diagnosis for this case was


a. Placental abruption
b. Placenta previa
c. Heavy vaginal discharge
d. Ca cervix
e. Vaginal laceration

39. To confirm the diagnosis, we could performed the following step


a. Transabdominal USG
b. Transvaginal USG
c. X-ray
d. CT scan
e. MRI

A woman aged 29 years G1P0A0 late period of 3 months with complaints of bleeding a little
through the birth canal since 1 week ago. KU good. Gynceological examination found inscpeculo
bleeding (+). VN not, portio slippery. Vaginal examination positive bleeding. V/v is normal, covered
ostium, uterine 12-14 weeks gestation, tenderness (-), adnexa parametrial sin./dex. Normal, CD
normal

40. Based on the complaints and symptoms in the above cases can be diagnosed as the following
a. Early pregnancy with complications bleeding
b. Early pregnancy with experiencing signs iminens abortus.
c. Early pregnancy with signs to abortion insipiens
d. Early pregnancy initial accompanied with ruptured ectopic pregnancy
e. Early pregnancy with impaired coagulation function.

41. Appropriate diagnosis is made in the question no. 1, another examination must to be
performed is
a. Examination hormonal
b. BHcg serum examination
c. Ultrasound examination
d. Doptone examination
e. Rontgen foto pelvic

42. Treatment in this case the recommended to the mother is


a. No change the activity
b. Activites reduced and give progesterone hormone therapy
c. Allow the usual activities coupled with the administration of progresterone
d. Progesterone hormone therapy alone
e. Total bed rest

A 35 years old lady with 2 children came to specialist clinic in the international hospital with
complain of vaginal discharge with her pap smear result that showed : CIN 3+ HPV infection, she
ask for the appropriate treatment.

43. What is the next diagnostic procedure should be offered to the patient :
a. HPV DNA test
b. Liquid Based Cytology Pap smear
c. Ultrasound
d. VIA
e. Colposcopy

44. Based on the Bethesda system, the pap smear result was categorized as
a. ASCUS
b. LSIL
c. HSIL
d. AGUS
e. Ca in situ

45. What is the appropriate treatment for the patient:


a. Cryotherapy
b. Hysterectomy
c. Just Observation / Follow up
d. LEEP
e. Cauterization

A woman 55 years old with 1 children came to the specialist clinic in the international hospital
with the complain of vaginal bleeding menopause for 3 years. She had bleeding since 2 months
ago, sometime with heavy bleeding. Her height was 152 cm with the weight of 85 kgs. She had
also diabetes since she was 40 years old

46. What is the most likely diagnosis of the patient


a. Cervical cancer
b. Endometrial cancer
c. Ovarian cancer
d. Vaginal cancer
e. Sarcoma

47. What is the most suitable initial diagnostic procedure for the patient
a. Curettage
b. Transvaginal Ultrasound
c. MRI
d. CT scan
e. PET scan

48. If the patient ask you the most probable therapeutic procedure after knowing the history of
the disease ?
a. Hysterectomy
b. Inserting mirena
c. Oral Pill
d. Endometrial ablation
e. Tranexamic acid

A woman 45 years old with 2 children came to the specialist clinic in the international hospital
with the complain of the post coital bleeding since 3 months ago. The lady had the first
menstruation at the age of 13 years old and got married at the age of 17 years old. From the
vaginal examination, 0,5 cm mass was seen in the upperlip of the vagina without involvement of
the vaginal mucosa.

49. What is the most likely initial procedure has to be taken after seeing the condition of the cervic
a. VIA
b. Biopsy
c. Leep
d. Pap smear
e. Endocervical curettage

50. What physical diagnostic examination is important in that situation


a. Palpation of the cervical mass
b. Rectal examination
c. Rectovaginal examination
d. Palpation of the vagina
e. Vaginal examination

51. What is the most likely stage of the cases


a. Stage la1
b. Stage la2
c. Stage Ilia
d. Stage Ibl  < 4 cm
e. Stage Ib2  > 4 cm

A 55 years old lady come to the specialist clinic with the enlarged abdomen. She was unmarried
and menarche at the age of 11 years old. Decrease appetite since 2 months ago, no vaginal
bleeding nor pain. Lost 8 kilograms within 4 months.

52. What is the most likely diagnosis


a. Ovarian cancer
b. Endometrial cancer
c. Ovarian cyst
d. Ascites
e. Intrabdominal bleeding
53. What may prevent her of having the diagnosis
a. Mirena
b. Combined oral contraception  menghambat ovarial cancer
c. Low dose aspirin
d. Norethisteron
e. Injection contraception

54. What tumor marker may help the diagnosis


a. Alpha fetoprotein (AFP)
b. Ca 125
c. LDH
d. Nor ethisteron
e. Injection contraception

A 21 year old unmarried woman has conducted a general check-up, from abdominal and
gynecological ultrasound examination found a cystic mass originating from the left adnexa with a
diameter of 4 cm, she did feel pain on lower abdominal and bit longer of menstrual cycle. Physical
examination and other laboratory within normal limits.

55. Which of the following is the most likely diagnosis


a. Follicular Cyst
b. Benign ovarian cyst
c. Malignant ovarian Cyst
d. Ovarian Fibroma
e. Hydosalphynx

56. Management in this case is :


a. biopsy guided ultrasound
b. MRI for definitive diagnosis
c. Observation 2-3 months without any treatment
d. Operative Laparascopy
e. Laparatomy Cystectomy

57. If the cyst removal surgery her then expected her histopatological types are
a. Neoplastic Cells
b. Malignant Cell
c. Normal Cells
d. Theca lutein cells pada kehamilan mola
e. Germ cell tumors  usia muda

A 63-year-old woman has bloating associated with tightening of her clothing around her abdomen.
She recently has developed dyspepsia and has lost 15 pounds unintentionally. She is short of
breath. Pulmonary auscultaion shows loss of breathsound abdominal percussioncause a wavelike
movement of fluid around a central tympanitic area. Pelvic examination demonstrates a fixed,
irregular nodular adnexal mass with cul-de-sac nodularity. A chest radiogram shows bilateral
pleural effusions. Which of the following is the most likely diagnosis
58. Which of the following is the most likely diagnosis ?
a. Fallopian tube carcinoma
b. Ovarian carcinoma
c. Uterine leimyoma
d. Uterine sarcoma
e. Endometrial carcinoma

59. What kind further examinations should be done for diagnosis


a. RMI test
b. Pap smear
c. Laparatomy
d. Consult to Hemato-oncology Medicine
e. Surgical staging

60. Factors that may help increase the risk of ovarian cancer include
a. Birth control pills
b. Childbirth and breastfeeding
c. Tubal ligation (tying fallopian tubes) or hysterectomy (removal of uterus) after childbearing
d. Obesity
e. Bilateral salpingo-oopherectomy surgery (removal of both ovaries and fallopian tubes) in
women those who have BRCA 1 and BRCA 2 genetic mutations

(61-70 :usulan UNSRI) (William obsed 24)


Mrs. A, 24 years, G2P1A0 39 weeks gestational age, admitted to your emergency room with
complained abdominal cramping with bloody show. In examination, revealed normal vital sign,
contraction was 3x/10'/35". Fetal heart rate was 146 x/m. Estimated fetal weight was 2900 gram.
Previous baby was 3200 gram. Vaginal examination revealed: dilatation was 4 cm, effacement 100
%, amniotic membrane was intake, lowest part was head with descent of the head was Hodge + 1.
Denominator was minor fontanelle at the left side.

61. What is the condition of this patient ?


a. Not in labour yet
b. in labour, stage 1 latent phase
c. in labour, stage 1 active phase
d. in labour, stage 2
e. in labour, stage 3

62. According the vignette above, what wil be happen to that mother
a. She will continue the progress of labour and will give birth normally
b. The progress of labour will be stucked in this phase of labour
c. she will underwent prolonged second stage
d. The second stage of labour must be terminated by vaccum of forcipal extraction
e. The mother will face the possibility of postpartum haemorrhage

Mrs. S, 29 years, G3P1A1 39 weeks gestational age, reffered by midwife with prolonged second
stage. In examination, revealed normal vital sign Obstetric examination revealed contraction was
3.10.25". Fetal heart rate was 160 x/m. Estimated fetal weight 3100 gram. Previous baby was 3000
gram. Vaginal examination revealed : full dilatation, amniotic membrane was absent thick and
greenish, lowest part was head with descent of the head was Hodge IV. Denominator was minor
fontanelle at the left anterior.

63. What will you choose to terminate this condition?


a. Spontaneous delivery
b. Augmentation
c. Embriotomy
d. Forcep extraction
e. Cesarean section

64. What are indication for assisted vaginal delivery?


a. Uterine rupture
b. PPROM
c. Post partum haemorrhage
d. Fetal anomaly
e. Fetal or maternal distress

65. Frequent complication of the action above is


a. Uterine rupture
b. Cephalhematoma (kunci UNSRI) paling banyak pada VE
c. Parese n. VII (hal 576 William Obs 24)
d. Erb's paralysis  brachial plexus 10 % permanen, yg lain balik normal
e. Fracture of os femur

Mrs. S, 34 years, G4P3A1 40 weeks gestational age, second stage of labour. The head of the baby
was delivered but the shoulder was stucks. Turttle sign N. The mother has an uncontrolled
diabetes for 5 years. Estimated fetal weight by ultrasound was 4200 gram.

66. What is this condition called ?


a. After coming head
b. Compaction
c. Retention of the baby
d. Shoulder dystocia
e. Collision

67. What is predisposing factor for this condition


a. Preterm
b. PPROM
c. Macrosomia
d. Acrania
e. Breech presentation

Mrs. Y, 31 years, G5P2A2 39 weeks gestational age, with history of 2x cesarean section and 2x
curettages. She asked for elective C-section.
68. What is the possibility complication of this case?
a. PROM
b. Placenta accreta
c. Preterm delivery
d. Shoulder dystocia
e. After coming head

Mrs. S, 21 years, G I PO send by midwife with failed of induction. Examination revealed: active
bleeding from vagina, and shock. Vital sign : BP 80/60 mmRg—,-HR 134x/m, skin was wet and acral
was cold. Part of baby wasn't palpable. Fetal heart rate (-).

69. What is the diagnosis?


a. Placental abruption
b. Placenta Previa
c. Vasa previa
d. Embolism
e. Uterine rupture

70. What is the management?


a. Cesarean section
b. Laparatomy -
c. Forcep extraction
d. Embriotomy
e. Spontaneuous delivery

An 17 year old women with her husband ' and he had just got married, is requesting a consultation
for the appropriate contraceptive method, as the couple want to be pregnant after she is 20 years.

71. Which of the following method is appropriate for the couple? (Atlas of
contraception. Contraception for special group, Adolescent. hal 85)
a. Condom
b. Oral pills
c. Implant
d. IUD
e. Monthly injectable contraception

No single method of contraception can be considered suitable for adolescents as a group, but
factors such as age, parity, sexual habits, risk of infection, risk of pregnancy, the need to conceal
the sexual activity, and contraceptive use, should be considered prior to providing a method. The
combined oral contraceptive is the most popular and most requested method of contraception by
teenagers. This is appropriate because oral contraceptives are almost never medically
contraindicated in healthy adolescents. Added advantages for them would be improvement of
acne, reduction of menstrual flow and dysmenorrhea, and reduced risk of pelvic inflammatory
disease. Other hormonal methods such as injectables and implants can be used in older
adolescents. Although their long-acting feature improves the continuity of use of these methods,
there are concerns about depletion of bone density with depot medroxy progesterone acetate,
which may result in adolescents not achieving their peak bone mineral density. Intrauterine
devices (IUDs) are not the method of first choice for nulliparous women or a woman who is likely
to have more than one partner.
72. After anamnesis the doctor predict that the woman will not more appropiate for them ?
a. Condom
b. Oral pills
c. Implan
d. IUD
e. Monthly injectable contraception

Your patient delivered a healthy baby 2 weeks ago and wishes to use contraception method after
her puerperium. She is breastfeeding exclusively. (Atlas of contraception. Contraception for special
group, Nursing mother. hal 87)

73. For which of the following is there strong evidence that use decrease the quantity and
quality of breast milk?
a. Progestin-only pills  boleh
b. Depo medroxyprogesterone acetate  boleh
c. Combination hormonal contraception -> estrogen itu menurunkan prod ASI
d. IUDs
e. Implant

74. Your patient has diabetes mellitus and hypertension but she prefer to use "pills" for
contraception. She is considering progestin-only pill and combination oral contraception
(COC). You give counseling to her about the advantage and disadvantage of progestin-only pill
compare with COCs.(Atlas of contraception)
a. More appropiate for diabetic and hypertension patient
b. Lower failure rate  kerugian progestin only pill
c. Low rate of irregular bleeding
d. Low relative ectopic pregnancy rate
e. Relative more nausea and vomiting

75. Which of the following is an advantage of progestin-only emergency contraception regimens


compare with estrogen-progestin combination for this purpose?
a. More effective in pregnancy preventation (William gynhal 163)
b. Effective if taken beyond 5 days after intercourse
c. Provides better protection againts STD
d. More side effect--->COC
e. More simple use

76. Which of the following is not an absolute contraindication for using of Combination of
Oral Contraceptive?(William gynhal 149-156, Atlas contraception hal 46)
a. Thrombotic disorders
b. Cholestatic Jaundice
c. Migrainesfoca withl neurologic deficits
d. Uncomplicated SLE (including negative test for APA)--> kontraindikasi pada SLE dgn test
APA (+)
e. Uncontrol hypertension

A 30 years woman complain of colorless vaginal discharge with she feel that the smell very bad odor
then she feel low self esteem

77. What is the possible infection of this woman?(William gynhal 83)


a. Candidiasis itching burning, cottage cheese like
b. Gonorrhoe
c. Tricomoniasis frothy discharge, green-yellow
d. Bacterial vaginosis thin,gray,white,colorless discharge, KOH whiff test (+)
e. Clamidia Trachomatis

78. What is the antibiotics which is usually use to threat this symptoms?(William gynhal 67)
a. Ampicillin
b. Doxycyclin
c. Amoxycillin
d. Metronidazole (2x500 mg po 7 hari)
e. Trimethoprim

79. What is the typical range of normal vaginal pH?,Nilliam gynhai


65) a. 3.0-3.5
b. 4,0-4.5
c. 5.0-5.5
d. 6.0-6.5
e. 7.0-7.5

Primigravida full term pregnancy is reffered to hospital by midwife due to unprogressed labor.
General condition is weak, normal blood pressure with heart rate 100 bpm. In clininal examination
you find uterine contraction is weak and rare, with midline measurement of 30 cm, and only two
fifth of the head is palpated. Fetal heart rate is 144 bpm.

80. By using johnson's formula, what is estimated fetal weight?


a. 90 gram
b. 2945 gram
c. 3000 gram
d. 3200 gram
e. Could not be measured due to unfullfilled criteria

81. You are doing internal examination and found cervix is already dilated 8 cm, no
amniotic membranes palpated with mentum on 6 O'clock, what would the diagnosis
likely to be?
a. Occipito posterior persistent ket mentum anterior bisa lahir, emtum posterior g
bisa lahir
b. Deep transverse arrest
c. Brow presentation
d. Vertex presentation
e. Face presentation

82. From pelvic examination findings, pelvic brim : round, diagonal conjugate 12 cm, symphisis
parallel to sacrum, subpubic angle si acute, convergent side walls, bituberous diameter is 7
cm. By analyzing your findings, which causes bellow is unlikely to be?
a. Android pelvis
b. Anthropoid pelvic
c. High assimilation pelvis
d. Platypelloid pelvis
e. Oblique pelvis

83. Which of the following is the most appropriate etiology of case above?
a. Uterine inertia
b. Malpresentation
c. Cephalopelvic disproportion
d. Macrosomia
e. Uterine anomaly

84. What is the most appropriate management for this case:


a. Augmentation of labor
b. Advising cardiotocography examination
c. Forceps extraction
d. Vacuum extraction
e. Caesarean Section

Patient reffered from midwife's private practice with poor generalcondition, somnolent. She
has been conducted on delivery for two hours. Vital signs by 70/palpable blood pressure, 120
bpm heart rate.

85. What is your initial management?


a. Performing holistic clinical obstetrics examination
b. Performing ultrasound
c. Ask for help
d. Giving oxygen
e. Put IV line

86. In your examination you find that there isn't any contraction, distended abdomen, shifting
dullnes. In vaginal examination, you find cervix is not fully dialted, and head could be pushed
upward. What is the most appropriate diagnosis?
a. Threatened uterine rupture
b. Uterine rupture
c. Unprogressed labor with ascites
d. Incordinate uterine
e. Prolonged second stage

87. Which additional examination is mainly vital for preparation of next management
a. Ultrasound exam
b. Routine blood exam
c. Coagulation factor exam
d. Blood gas analysis
e. Urine analysis
88. One of the parameter that could be easily measured for prognosting acute kidney injury is
a. Urine output
b. Serum creatinine levels
c. Thrombocyte count
d. Amount of blood loss
e. Hemoglobin levels

You are attending delivery, patient had already bearing down with good contraction for half hour
but there is no further descent of head. Occiput is at left posterior.  POP  bisa lahir tapi
terlentang

89. What is the diagnosis ?


a. Prolonged latent phase
b. Incoordinate uterine action
c. Unprogressed second stage
d. Deep transverse arrest
e. Uterine rupture

90. Fetal heart rate shows declining arises from beginning of contraction and goes to normal
baseline as soon as the contraction ends. This findings could refer to
a. Fetal distress
b. Fetal head compression
c. Fetal hypoxia
d. Umbilical cord compression
e. Threatened uterine rupture

91. You find that descent of the head is on station -1, what action is most appropriate.
a. Giving oxytocin drip
b. Waiting for internal rotation by left lateral position
c. Vacuum extraction
d. Forceps extraction
e. Caesarean section

(92-99 :usulan UB)


A 32-year-old woman (gravida3, pars 1, abortus 1) at term is admitted in labor with an initial
cervical examination of 6-cm dilatation, complete effacement, and the vertex at -1 station.
Estimated fetal weight is 8 lb, and her first pregnancy resulted in an uncomplicated vaginal delivery
of an 8-lb infant. After 2 hours there is no cervical change. An intrauterine pressure catheter is
placed. This shows three contraction in a 10-minute periods, each with a strength of 40 mmHg. 
seharusnya 200 mmhg

92. What is this abnormality of labor termed? (Iange 9 bab10 no 27)


a. prolonged latent phase
b. active-phase arrest
c. failure of descent
d. arrest of latent phase
e. protraction of descent
93. What is the best course of action at this time?(Iange 9 bab 10 no 28)
a. wait 2 more hours and repeat the cervical examination
b. start oxytocin augmentation
c. perform a cesarean section
d. discharge the patient, instructing her to return when contractions become stronger
e. therapeutic rest with analgesia and short-acting anti anxiety medication

94. The routine use of midline episiotomy during delivery has been shown to do which of the
following?
a. prevent urinary stress incontinence in the fourth decade of life
b. decrease the incidence of fetal cranial molding
c. decrease maternal blood loss
d. increase the incidence of third and fourth degree lacerations
e. prevent the development of a rectocele and uterine prolapsed
post menopausal (kunci UB don Lange 9 bab 10 no 29, tapi pembahasan mengacu jawaban D,
pada lange 8 bab 10 no 39, soal sama pembahasan sama, dgn jawaban E. expedite delivery when
the head is crowning)

95. Which of the following statements most accurately describes postpartum hemorrhage?
a. It is prevented primarily by the increased concentration of clotting factors in
maternal blood
b. Grand multiparity is a risk factor
c. Women with severe pre-eclampsia are more tolerant of heavy blood loss.
d. Changes in pulse and blood pressure are good early indications of excessive blood loss
e. Placenta accreta is the most frequent cause.

96. A relative contraindication for induction of labor includes which of the following?
a. prolonged pregnancy
b. severe pre-eclampsia
c. intrauterine growth restriction
d. previous myomectomy entering the ute
e. rine cavity at the fundus
f. prolonged rupture of membranes without labor

97. What is the maximum normal time for the second stage of labor in a primigravida without
anesthesia?(Iange 9 bab 11 no V)  klo pakai anastesia 3 jam
a. 20 minutes
b. 60 minutes
c. 120 Minutes
d. 240 Minutes
e. no normal maximum

98. A 21-years-old G1 now P1 just delivered after a prolonged induction of labor due to being
postdates. After the placental delivery she continues to bleed excessively. Your initial
intervention to address this bleeding is to active the normal physiologic mechanisms. Which
of the following is the most important hemostatic mechanism in combating postpartum
hemorrhage? (lange 9 bab 6 no 4)
a. Contraction of interlacting uterine muscle bundles
b. fibrinolysis inhibition
c. increased blood-clotting factors in pregnancy
d. intramyometrial vascular coagulation due to vasoconstriction
e. markedly decreased blood pressure in the uterine venules

99. Worldwide, which of the following is the most common problem during pregnancy?(Iange
9 bab 8 no 1)
a. diabetes
b. preeclampsia
c. heart disease
d. urinary tract infection (UTI)
e. iron deficiency anemia
Sampun nggih Matur Nuwun ….

You might also like